Practice Questions

अब Quizwiz के साथ अपने होमवर्क और परीक्षाओं को एस करें!

f. Second degree heart block Conduction abnormalities can produce first-degree, second-degree, or third- degree heart block. In a second-degree heart block, a P wave is not always followed by a QRS complex as in the above EKG, where the second P wave is not followed by a QRS complex.

A 32-year-old female presented to her PCP for an annual physical exam. During the exam, an EKG is obtained and is displayed below. Her EKG tracing is most consistent with which of the following conditions? a. Normal sinus rhythm b. Atrial fibrillation c. Ventricular fibrillation d. Myocardial infarction e. First degree heart block f. Second degree heart block

a. A The frequency of slow waves is fixed in various parts of the gut. The maximum frequency of smooth m The frequency of slow waves is fixed in various parts of the gut. The maximum frequency of smooth muscle contractions cannot exceed the slow wave frequency. The slow wave frequency averages about 3 per minute in the stomach, 12 per minute in the duodenum, 10 per minute in the jejunum, and 8 per minute in the ileum. Therefore, the duodenum is most likely to have the highest frequency of smooth muscle contraction.

A 48-year-old woman consumes a healthy meal. At which location are smooth muscle contractions most likely to have the highest frequency in the diagrams shown? a. A b. B c. C d. D e. E

f. Titin Titin is a large protein that connects the Z lines to the M lines, thereby providing a scaffold for the sarcomere. Titin contains two types of folded domains that provide muscle with its elasticity. The resistance to stretch increases throughout a contraction, which protects the structure of the sarcomere and prevents excess stretch.

A 64yearold male was admitted to the hospital with edema and congestive heart failure. He was found to have diastolic dysfunction characterized by inadequate filling of the heart during diastole. The decrease in ventricular filling is due to a decrease in ventricular muscle compliance. Which of the following proteins determines the normal stiffness of ventricular muscle? a. Calmodulin b. Troponin c. Tropomyosin d. Actin e. Myosin light chain kinase f. Titin

D. Brain Insulin is not required for glucose uptake into brain cells.

Insulin stimulates facilitated diffusion of glucose into cells of all of the following tissues EXCEPT: a. Fat b. Lymphatic c. Muscle d. Brain e. Kidney

c. G protein coupled channels Activation of Gproteins usually changes the longterm response characteristics of the neurons

Prolonged changes in neuronal activity are usually achieved through the activation of: a. voltagegated chloride channels b. transmitter gated sodium channels c. G protein coupled channels d. voltage gated potassium channels e. transmitter gated chloride channels f. voltage gated sodium channels

c. Glutamate

The most prevalent excitatory neurotransmitter in the Central Nervous System is: a. Norepinephrine/Noradrenaline b. Dopamine c. Glutamate d. Glutamine e. GABA

c. Free bilirubin Jaundice denotes a yellowish discoloration of body tissues, most notable in the skin and sclera of the eyes. The condition is caused by an accumulation of bilirubin in extracellular fluid, either in free form or after conjugation.

A 4-week-old newborn in the neonatal unit presents with jaundice and clay white stools. On liver biopsy, giant cells with ballooning degeneration of the cytoplasm are seen. When the bile duct is blocked, jaundice develops. This jaundice is most likely to be associated with a rise in the plasma concentrations of a. Glucuronic acid b. Glucuronyl transferase c. Free bilirubin d. Heme e. Urobilinogens f. Free fatty acids g. Albumin

d. Inhibit synthesis of prostaglandins Nonsteroidal anti-inflammatory drugs (NSAIDs) such as aspirin, acetaminophen, ibuprofen, indomethacin, and naproxen directly target and inhibit the synthesis of cyclooxygenase products such as prostaglandins, prostacyclins, and thromboxanes.

A 42-year-old woman with chronic back pain takes aspirin on a daily basis to relieve her pain. Which of the following is the primary benefit for using aspirin? a. Inhibit chemotaxis of leukocytes b. Inhibit synthesis of bradykinin c. Inhibit synthesis of leukotrienes d. Inhibit synthesis of prostaglandins e. Inhibit transmigration of leukocytes f. Inhibit chemotaxis of cyclooxygenases

c. Decrease central venous pressure and decrease cardiac output The patient's acute blood loss decreased the driving force for venous return, decreased central venous pressure, and decreased venous return, which means cardiac output decreased. Acute blood loss would decrease peripheral venous pressure, decrease driving pressure (δP) for venous return, and decrease venous return. Thus, central venous pressure would be decreased, not increased.

A 44-year-old woman with severe cardiac failure caused by coronary artery disease was treated by cardiac transplantation. Three months after surgery, she developed a bleeding duodenal ulcer and lost approximately 600 ml of blood in one hour. She was treated with dietary changes and antibiotics and the ulcer was healed in about two weeks. The acute blood loss from the patient's duodenal ulcer would be expected to: a. Decrease central venous pressure and increase cardiac output b. Increase central venous pressure and decrease mean arterial pressure c. Decrease central venous pressure and decrease cardiac output d. Increase mean arterial pressure and decrease cardiac output e. Decrease central venous pressure and increase aortic pulse pressure f. Constant central venous pressure and decreased cardiac output

e. Loop diuretic This patient is significantly fluid overloaded as indicated by his shortness of breath, oliguria, and elevated level of ANP. ANP is released in response to increased atrial stretch which occurs with hypervolemia. To best manage the fluid overloaded state of the patient, a drug will need to be administered that rapidly improves fluid status, the loop diuretic is the most potent of the medications listed to alleviate this issue.

A 71-year-old man recovering from a myocardial infarction two days ago complains of severe shortness of breath. Urine output is low at 125 mL/day, and plasma levels of atrial natriuretic factor are increased. Which of the following is the most appropriate therapy for his acute symptoms? a. Thiazide diuretic b. ACE inhibitor c. Potassium-sparing diuretic d. beta blocker e. Loop diuretic f. Calcium channel blocker

a. Secretion is stimulated when the plasma volume decreases Antidiuretic hormone (ADH) or vasopressin is secreted in response to low blood volume and decreased blood pressure. ADH increases renal water reabsorption to expand the extracellular fluid volume and causes arterial vasoconstriction in response to the blood loss due to the described injury.

A 9-year-old child has suffered severe lacerations and blood loss in an automobile accident. What is the most likely cause of increased vasopressin secretion in this child? a. Secretion is stimulated when the plasma volume decreases b. Vasopressin decreases fluid loss by inhibiting aquaporin-2 (AQP2) water channel activity c. Secretion increases as plasma osmolality decreases d. Decreased K+ transport by increasing protein kinase A (PKA) activity in kidney cells e. Secretion is increased when blood pressure increases f. Increased stretch of the right atria

g. Surfactant Infant respiratory distress syndrome (IRDS), which affects ~30,000 newborns annually in the United States, is characterized by increased work of breathing (nasal flaring, the use of accessory musculature, intercostal and subcostal retractions, tachypnea, and grunting) and impaired gas exchange (cyanosis). Retractions occur because the lungs—with collapsed, fluid-filled, and poorly expanded alveoli—are less compliant than the chest wall, with non-ossified ribs. The increased inspiratory effort to expand the noncompliant lungs creates a very negative intrapleural pressure. As a result, the chest wall becomes distorted, caving in between the ribs or beneath or above the rib cage, so that the increased inspiratory work does not much improve tidal volume. The more immature the infant, the more severe and life-threatening the lung disease and the more likely that signs of respiratory distress become apparent immediately after birth or within a few minutes. IRDS is usually caused by a deficiency of pulmonary surfactant owing to prematurity. Prematurity is by far the single most important risk factor for development of IRDS. Surfactant insufficiency can result from abnormalities of surfactant synthesis, secretion, or recycling. Reduced surfactant lowers compliance directly (see pp. 615-616Links to an external site.) and further lowers compliance indirectly because alveoli—with their propensity to collapse—tend to be on the lowest, flat part of the lung pressure-volume curve (stage 1 in Fig. 27-4BLinks to an external site.).

A male newborn delivered at 26 weeks of gestation develops respiratory distress immediately after a spontaneous vaginal delivery. His respirations are 40/min. Physical examination shows cyanosis and lower rib retractions with respiration. Heart sounds are normal. Bilateral breath sounds are heard on auscultation. A chest x-ray shows bilateral lung opacities. The most likely cause of this patient's breathing difficulties is insufficient production of which of the following substances? a. Collagen, type III b. α-Fetoprotein c. Meconium d. Retinoic acid e. Alpha-1 antitrypsin f. Cortisol g. Surfactant

a. Decreasing opening of a voltage-dependent K+ channel. The binding of acetylcholine to muscarinic receptors decreases the opening of voltage-gated K+ channels to prolong the depolarization of the sympathetic neuron.

Acetylcholine, acting through muscarinic receptors, increases the number of action potentials evoked by a maintained depolarization of a sympathetic neuron, by a. Decreasing opening of a voltage-dependent K+ channel. b. Increasing inactivation of a voltage-dependent Na+ channel. c. Increasing opening of a voltage-dependent K+ channel. d. Increasing the number of open 'leak' K+ channels. e. Opening ligand-gated nonselective cation channels. f. Increasing the activity of the Na+/K+-ATPase g. Increasing the insertion of ligand-gated Na+ channels into the post-synaptic membrane

d. Preferentially uses lactate for energy Astrocytes preferentially use glucose for energy they provide lactate to the neuron.

All the statements about the astrocyte are correct EXCEPT: a. Isolates neurons from blood vessels b. Metabolizes the neurotransmitter glutamate c. Maintains the pH and osmolality of the extracellular fluid within normal physiological limits d. Preferentially uses lactate for energy e. Capacity to regenerate following injury f. Have higher anaerobic metabolic rate than neurons g. Predominantly permeable to K+

b. 0.25 The filtration fraction represents the proportion of the fluid reaching the kidneys which passes into the renal tubules. The fraction of creatinine that is reabsorbed is ¾ that of the amount of creatinine that is filtered. This indicates that 25% of the creatinine is filtered and excreted.

Creatinine, a freely filterable substance that is neither secreted nor reabsorbed, is often used as a clinical marker of glomerular filtration rate. If a patient has a creatinine concentration of 12 mg/mL in the renal artery and a creatinine concentration of 9 mg/mL in the renal vein, then which of the following is the best approximation of the creatinine filtration fraction in this patient? a. 0 b. 0.25 c. 0.33 d. 0.75 e. 1.00 f. 1.50

a. Decreased blood volume A 3-week period of bed rest in a supine position with little activity results in elevated central blood volume, decreased peripheral blood volume, and elevated peripheral venous pressures (due to lack of skeletal muscle pump activity). Elevated peripheral venous blood volume and pressure leads to a net filtration out of capillaries and a net loss of blood volume.

In a patient with a fractured pelvis, requiring a 3-week period of bed rest which of the following physiologic changes would be expected to occur? a. Decreased blood volume b. Increased plasma concentration of aldosterone c. Increased plasma volume d. Increased activity of the sympathetic nervous system e. Decreased plasma concentration of sodium f. Decreased activity of the parasympathetic nervous system

c. The regenerative depolarization responsible for the upstroke of the action potential The upstroke of the action potential is caused by sodium influx through voltage-gated sodium channels.

K+ channels contribute to all of the following EXCEPT: a. The after-hyperpolarization following an action potential b. The pattern of action potential firing (e. g., tonic firing vs. adaptation) c. The regenerative depolarization responsible for the upstroke of the action potential d. Repolarization of the action potential e. The resting potential

b. Intrinsic pacemaker wave activity

Smooth muscle that exhibits rhythmical contraction in the absence of external stimuli also necessarily exhibits which of the following? a. "Slow" voltage-sensitive Ca2+ channels b. Intrinsic pacemaker wave activity c. Higher resting cytosolic Ca2+ concentration d. Hyperpolarized membrane potential e. Action potentials with plateaus

b. 70 The heart rate can be calculated in two way, by dividing 300 by the number of large boxes in the R-R interval (300/~4-4.5 boxes= ~70 BPM) or by dividing 60 by the duration of the R-R interval (60/0.86 seconds = 70 BPM)

What is the heart rate (BPM) in the following EKG? a. 55 b. 70 c. 88 d. 94 e. 104 f. 115 g. 130

e. Potassium efflux

Which of the following events is most closely related to the major mechanism of repolarization of excitable cells? a. Sodium influx b. Sodium efflux c. Decreasing excitability of the cell d. Potassium influx e. Potassium efflux

e. The amount of sodium entering the nerve with each action potential increases Cold delays inactivation of Na+ channels allowing more Na+ to enter the cell. Increased duration does not equal increase membrane potential. No change to capacitance or refractory period. Duration of the AP depends on Na+ not K+.

. A 19-year-old woman with a history of diplopia and paresthesia is diagnosed with multiple sclerosis (MS). Immersion of an affected limb in a cold bath restores nerve conduction in many MS patients. The explanation often cited for this effect is that cold increases the duration of the action potential. Which of the following best explains why increasing the duration of the action potential can restore nerve conduction in patients with MS? a. The membrane potential becomes more positive b. The capacitance of the nerve fiber membrane is increased c. The duration of the refractory period is increased d. The potassium conductance of the membrane is increased e. The amount of sodium entering the nerve with each action potential increases

a. Decreased renal HCO3- excretion, increased NH4+ excretion, increased plasma anion gap DKA results in a metabolic acidosis that is characterized by a decrease in plasma bicarbonate concentration, increased anion gap (due to the addition of unmeasured anions to the ECF along with the keto acids), and a renal compensatory response that includes increased secretion of NH4+. There is also an increased respiratory rate with a reduction in arterial PCO2, as well as decreased urine pH and renal bicarbonate excretion.

. In a patient who has chronic diabetic ketoacidosis, you would expect to find which of the following? a. Decreased renal HCO3- excretion, increased NH4+ excretion, increased plasma anion gap b. Increased respiration rate, decreased arterial PCO2, decreased plasma anion gap c. Increased NH4+ excretion, increased plasma anion gap, increased urine pH d. Increased renal HCO3- production, increased NH4+ excretion, decreased plasma anion gap e. Decreased urine pH, decreased renal HCO3- excretion, increased arterial PCO2 f. Decreased respiration rate, decreased NH4+ excretion, constant plasma anion gap

b. False In the PNS, a single Schwann cell provides a single myelin segment to a single axon of a myelinated nerve (see Fig. 11-13B). This situation stands in contrast to that in the CNS, where one oligodendrocyte myelinates many axons.

15. In the CNS a single oligodendrocyte myelinates a single axon. a. True b. False

c. K+ into the hair cell A positive deflection—toward the tallest stereovilli—further opens the apical channels, leading to influx of K+ and thus depolarization.

24. The morning after a rock concert, a 20-year-old college student notices difficulty hearing his professor during lecture. The physician at the student health center suspects possible damage to his hair cells by the loud music. Depolarization of the hair cells in the cochlea is caused primarily by the flow of which of the following? a. Ca2+ into the hair cell b. Cl- out of the hair cell c. K+ into the hair cell d. K+ out of the hair cell e. Na+ into the hair cell

b. Botulinum toxin Botulinum toxin inhibits the release of acetylcholine from the nerve terminal, resulting in blurred vision, ptosis, unreactive pupils, paralysis, and respiratory failure. Botulinum infection is food poisoning caused by a bacterium (botulinum) growing on improperly sterilized canned meats and other preserved foods.

45-year-old woman experiences blurred vision and difficulty swallowing after eating some home-canned vegetables. These symptoms are followed by respiratory distress and flaccid paralysis. The symptoms of her illness are most associated with which of the following? a. Black widow spider toxin b. Botulinum toxin c. Organophosphate poisoning d. Benzodiazepine ingestion e. alpha-Bungarotoxins f. Cholera toxin

c. RA 75%, RV 75%, PA 80%, PV 95%, LV 95%, A 95% Because the neonatal patient has hypoxemia (low arterial PO2) secondary to ARDS, the ductus arteriosus has no stimulus to close. The patient, therefore, exhibits the continuous murmur of a patent ductus arteriosus (PDA). Neonates with congenital heart diseases often have shunts between chambers or vessels. When oxygenated blood (SaO2 95%) is shunted into a vessel with venous blood (SaO2 75%), there is a step-up of SaO2 (∼80%) in the venous blood. This is known as a left-to-right shunt. PDA is an example of left-to-right shunt because blood initially flows from the aorta (where pressure is high), through the PDA, and into the pulmonary artery (where pressure is low) causing a step-up of the SaO2 in the pulmonary artery seen in option C.

A 1-day-old neonate with acute respiratory distress syndrome (ARDS) has moderately severe hypoxemia. Physical examination reveals, dry inspiratory crackles in the lungs and a continuous harsh murmur heard over the entire precordium. Which of the following sets of oxygen saturation (SaO2) values in the cardiac chambers and vessels most likely match those of this patient? a. RA 75%, RV 80%, PA 80%, PV 95%, LV 95%, A 95% b. RA 75%, RV 80%, PA 80%, PV 95%, LV 95%, A 95% c. RA 75%, RV 75%, PA 80%, PV 95%, LV 95%, A 95% d. RA 75%, RV 75%, PA 75%, PV 95%, LV 80%, A 80% e. RA 75%, RV 75%, PA 75%, PV 95%, LV 95%, A 95% f. RA 80%, RV 80%, PA 80%, PV 80%, LV 95%, A 80%

d. Oligodendrocyte

A new drug is developed that prevents the demyelinization occurring in the progress of multiple sclerosis. The drug protects the cells responsible for the synthesis and maintenance of myelin in the central nervous system. These cells are most likely which of the following? a. Astrocyte b. Ependymal cell c. Microglial cell d. Oligodendrocyte e. Schwann cell

d. Malabsorption of vitamin B12 in gastrectomized patients due to removal of intrinsic-factor secreting tissue Patients with pancreatic insufficiency, as well as patients with Crohn's dis- ease, bacterial overgrowth, or who have undergone ileal resection, may exhibit vitamin B12 deficiency. Cobalamin, also known as vitamin B12, is an essential vitamin found in such foods as liver, fish, and dairy products. Absorption of cobalamin occurs exclusively from the ileum, where specific receptors on ileal enterocytes bind a complex of cobalamin and intrinsic factor. Although intrinsic factor is secreted by gastric parietal cells, binding of the vitamin to intrinsic factor occurs primarily in the proximal small intestine. The acidic environment of the gastric lumen favors the binding of cobalamin to R protein-type binding proteins that originate from salivary and gastric secretions. Pancreatic proteases in the small intestine degrade the R proteins, and the rise in pH favors rapid and complete transfer of the vitamin to intrinsic factor.

An 18-year-old male presents with symptoms of vitamin B12 deficiency. Further diagnostic tests reveal that he has pernicious anemia. The underlying problem in pernicious anemia is which of the following? a. Iron deficiency b. Cobalt deficiency c. Inadequate dietary intake of cyanocobalamin d. Malabsorption of vitamin B12 in gastrectomized patients due to removal of intrinsic-factor secreting tissue e. Autoimmune destruction of parietal cells in the gastric mucosa f. Erythropoietin deficiency

c. Arterioles The energy imparted to the blood by ventricular systole is dissipated as the blood flows through the circulation. The greatest energy loss occurs where the resistance to blood flow is greatest. This would also be the site of the greatest pressure change. The arteriolar vessels produce the largest resistance to blood, and thus the greatest energy loss and pressure drop occur as the blood passes through them.

At which of the following sites in the cardiovascular system does the blood flow lose the greatest amount of energy? a. Mitral valve b. Large arteries c. Arterioles d. Capillaries e. Venules f. Vena cava

f. F Dissolved CO2 combines with water in red blood cells to form carbonic acid, which dissociates to form bicarbonate and hydrogen ions. Many of the bicarbonate ions diffuse out of the red blood cells while chloride ions diffuse into the red blood cells to maintain electrical neutrality. The phenomenon, called the chloride shift, is made possible by a special bicarbonate-chloride carrier protein in the red cell membrane that shuttles the ions in opposite directions. Water moves into the blood cells to maintain osmotic equilibrium which results in slight swelling of the red blood cells in the venous blood.

Carbon dioxide is transported from the tissues to the lungs predominantly in the form of bicarbonate ion. Compared to arterial red blood cells, which of the following best describes venous red blood cells? a. A b. B c. C d. D e. E f. F g. G

d. Helicobacter pylori pylori is a gram-negative bacterium with high urease activity, it catalyzes the formation of ammonia from urea. The ammonia is converted to ammonium in the acid environment of the stomach. The ammonium damages the gastric mucosal barrier because it damages the epithelial cells. H. pylori also increases gastric acid secretion, possibly by increasing parietal cell mass. The combination of damage to the mucosal barrier and increased gastric acid secretion promotes the development of gastric ulcer. Bile salts can damage the gastric mucosal barrier but do not effect acid secretion. Epidermal growth factor, gastrin, and mucous strengthen the gastric mucosal barrier. Somatostatin inhibits gastric acid secretion. Intrinsic factor depends on gastric acid secretion to function.

Damage to the gastric mucosal barrier is a forerunner of gastric ulcer. Which of the following can both damage the gastric mucosal barrier and stimulate gastric acid secretion? a. Bile salts b. Epidermal growth factor c. Gastrin d. Helicobacter pylori e. Mucous f. Somatostatin g. Intrinsic factor

d. Increase in the rate of voltage-dependent changes in K+ permeability

Drug X applied to a nerve axon decreases the duration of the action potential without affecting the resting potential or peak amplitude of the action potential. Which of the following is the most likely mechanism of action of Drug X? a. Block of voltage-dependent Na+ permeability b. Decrease in the rate of Na+ inactivation c. Decrease in voltage-dependent Na+ permeability d. Increase in the rate of voltage-dependent changes in K+ permeability e. Inhibition of the Na+ -K + pump

a. Large motor neurons that innervate large motor units require a larger depolarizing stimulus Muscle fibers involved in fine motor control are generally innervated by small motor neurons with relatively small motor units, including those that innervate single fibers. These neurons fire in response to a smaller depolarizing stimulus compared with motor neurons with larger motor units. As a result, during weak contractions, increases in muscle contraction can occur in small steps, allowing for fin motor control. This concept is called the size principle.

During a demonstration for medical students, a neurologist uses magnetic cortical stimulation to trigger firing of the ulnar nerve in a volunteer. At relatively low-amplitude stimulation, action potentials are only recorded only from muscle fibers in the index finger. As the amplitude of the stimulation is increased, action potentials are recorded from muscle fibers in both the index finger and the biceps muscle. What is the fundamental principle underlying this amplitude-dependent response? a. Large motor neurons that innervate large motor units require a larger depolarizing stimulus b. Recruitment of multiple motor units requires a larger depolarizing stimulus c. The biceps muscle is innervated by more motor neurons d. The motor units in the biceps are smaller than those in the muscles of the fingers e. The muscles in the fingers are innervated only by the ulnar nerve f. Fast twitch muscle fibers are more rapidly innervated, requiring a smaller depolarizing stimulus g. Small motor units are made up mostly slow twitch fibers that are innervated by fewer motor neurons

e. Increase in nerve cell potassium conductance caused by membrane depolarization Electrically excitable gates are those that respond to a change in membrane potential. The most notable electrically excitable gates are those on the sodium and potassium channels that produce the nerve action potential. The potassium channel gate is opened by depolarization. Ventricular muscle SR releases its calcium in response to an increase in intra- cellular calcium. The gates opened by ACh are chemically excitable gates. In rods, sodium channels are closed when cGMP is hydrolyzed. Electrically excitable gates do not regulate the active transport of glucose.

Electrically excitable gates are normally involved in which of the following? a. The depolarization of the end-plate membrane by acetylcholine b. Hyperpolarization of rods by light c. Release of calcium from ventricular muscle sarcoplasmic reticulum d. Transport of glucose into cells by a sodium-dependent, secondary active transport system e. Increase in nerve cell potassium conductance caused by membrane depolarization

e. All of the above During erection, relaxation of the smooth muscles of the corpora cavernosa and the corpus spongiosum allows increased inflow of blood to fill the corporal interstices and results in an increase in volume and rigidity. Vascular actions of the smooth muscles of the corpora and the perineal striated muscles are coordinated. For example, contraction of the striated muscles overlying the vascular reservoirs of the penile bulb increases the pressure of the blood in the corpora and promotes increased rigidity. During erection, a decrease in this sympathetic tone allows relaxation of the corpora and thus contributes to tumescence.

Erection can occur when: a. Penile blood flow increases about 25fold b. Parasympathetic input to penile arterioles causes them to relax c. Sympathetic input to penile arterioles is blocked by stimulation of an inhibitory interneuron d. Some appropriate stimulus (such as tactile, visual, or psychological) reaches the erection center in the lower spinal cord e. All of the above f. None of the above

b. A decreased probability of action potential generation Lower potassium levels in the extracellular space cause hyperpolarization of the resting membrane potential. This hyperpolarization is caused by the effect of the altered potassium gradient on resting membrane potential as defined by the Goldman equation. As a result, a greater than normal stimulus is required for depolarization of the membrane to initiate an action potential.

Excessive vomiting in the short term can lead to hypokalemia with no change in intracellular [K+]. One possible result of this is: a. An increase in the Na/K ATPase activity b. A decreased probability of action potential generation c. A decrease in net K+ efflux d. An increase in the number of IPSPs at the neuromuscular junction e. An increase in the number of EPSPs at the neuromuscular junction

a. A During fluid deprivation, the plasma osmolarity will increase. This stimulates osmoreceptors in the hypothalamus which stimulate the release of ADH from the posterior pituitary. ADH promotes reabsorption of water from the distal renal tubule and also promotes increased thirst to encourage fluid intake.

Following 24 hours without any fluid intake, characterize the changes in plasma osmolarity, ADH secretion, and thirst. a. A b. B c. C d. D e. E

e. Constricted pupils Muscarine poisoning is characterized by miosis (pupil constriction), blurred vision, increased salivation, excessive sweating, lacrimation, bronchial secretions, bronchoconstriction, bradycardia, abdominal cramping, increased gastric acid secretion, diarrhea and polyuria. These symptoms are due to ACh acting on muscarinic receptors, think parasympathetic stimulation.

Following a feast of wild mushrooms, an individual arrives in the emergency room. The mushrooms were of the genus Inocybe, which has a very high muscarine content. One probable symptom is: a. An overresponsive or exaggerated patellar tendon reflex b. Increased blood glucose levels c. Fast heart rate d. Bronchodilation e. Constricted pupils f. Anuria

c. A G-protein-coupled increase in Adenylyl Cyclase activity Odor receptors signal through Golf a G-protein coupled receptor that stimulates adenylyl cyclase activity.

Odor receptors signal exclusively through the activation of: a. An ionotropic odorant receptor b. A G-protein-coupled increase in phospholipase C activity c. A G-protein-coupled increase in Adenylyl Cyclase activity d. A G-protein-coupled activation of endoplasmic reticulum IP3 receptors e. An adrenergic odorant receptor f. None of the above g. all of the above

b. Low serum concentrations of glucose Whereas both glucose and several amino acids stimulate insulin secretion by β cells, only amino acids stimulate glucagon secretion by α cells; glucose inhibits glucagon secretion. The signaling mechanism by which α cells recognize either amino acids or glucose is not known.

Glucagon is secreted by the alpha cells of the pancreatic islets. Which of the following is most likely to induce glucagon secretion? a. Low serum concentrations of amino acids b. Low serum concentrations of glucose c. High serum concentrations of glucose d. Secretion of somatostatin by the delta cells e. Parasympathetic stimulation f. Elevated serum levels of lipids

c. Arterial CO2 pressure (PaCO2) will double The reduction in ventilation means that less CO2 will be expelled from the lungs while production in the tissues remains constant. As a result, the arterial CO2 pressure doubles

If alveolar ventilation is halved (while breathing room air and if CO2 production remains unchanged), then: a. Alveolar CO2 pressure (PACO2) will be halved b. Arterial O2 pressure (PaO2) will double c. Arterial CO2 pressure (PaCO2) will double d. Alveolar O2 pressure (PAO2) will double e. Arterial O2 pressure and arterial CO2 pressure will remain constant f. Alveolar CO2 pressure will be halved while arterial O2 pressure will double g. Arterial CO2 pressure and arterial O2 pressure will double

d. all of the above Photoreceptors hyperpolarize in response to light to modulate rate of glutamate release from the photoreceptor onto its postsynaptic neuron. In the dark, ionic current flow steadily into the outer segment and out of the inner segment. In the dark, there is a continuous release of glutamate onto bipolar cells, photoreceptors continuously express a large Na+ conductance, and express a large K+ conductance.

In the dark, the photoreceptors: a. continuously release glutamate onto bipolar cells b. continuously express a large sodium conductance c. continuously express a large potassium conductance d. all of the above e. a and b

c. The regenerative depolarization responsible for the upstroke of the action potential The regenerative depolarization responsible for the upstroke of the action potential is determined by voltage-gated Na+ channels.

K+ channels contribute to all of the following EXCEPT: a. The after-hyperpolarization following an action potential b. The pattern of action potential firing (e. g., tonic firing vs. adaptation) c. The regenerative depolarization responsible for the upstroke of the action potential d. Repolarization of the action potential e. The resting potential

c. The release of ACh from the nerve terminal In LES, antibodies to voltage-gated calcium channels in the terminal bouton of presynaptic neurons result in impaired ACh release

Lambert-Eaton Syndrome (LES) is similar to myasthenia gravis. In LES, which of the following is most affected during neuromuscular transmission? a. The driving force for Ca2+ influx in the nerve terminal b. The synthesis of acetylcholine (ACh) in the nerve terminal c. The release of ACh from the nerve terminal d. The degradation of ACh in the synaptic cleft e. The response to ACh in the muscle membrane

f. When Na+ channels are activated repeatedly at a high frequency Lidocaine blocks activated and inactivated, but not resting channels. When the frequency of excitation is increased, Na+ channels tend to stay in their inactivated state giving lidocaine a preferential effect on arrhythmogenic tissue.

Lidocaine, a usedependent blocker of voltagegated Na+channels, is an effective antiarrhythmic because its blocking effect becomes greater: a. When the drug is repeatedly used b. When the drug is used sparingly c. When Na+ channels are activated repeatedly at a low frequency d. When Na+ channels are closed e. When the extracellular Na+ concentration is high f. When Na+ channels are activated repeatedly at a high frequency

a. interface between air and the anterior surface of the cornea The index of refraction for a substance is essentially a measure of the speed of light within it; for example, light travels faster through air (index of refraction, 1.0003) than through the denser substance of the cornea (index of refraction, 1.376). In the case of the eye, most of the focusing takes place at the interface between the air and the tear-covered anterior surface of the cornea because this region is where light encounters the greatest disparity in refractive indices on its path to the retina.

Light is focused as a result of the angle of incidence and the difference in refractive index between two media. Where does the majority of focusing happen in the eye? a. interface between air and the anterior surface of the cornea b. interface between the posterior surface of the cornea and the aqueous humor c. interface between the aqueous humor and the lens d. interface between the lens and the vitreous humor e. within the vitreous humor

a. Is primarily regulated by the autonomic nervous system Unlike the pancreas, in which hormones have an important role in stimulating secretion, the salivary glands are mostly controlled by the autonomic nervous system. The major agonists of salivary acinar secretion are ACh and norepinephrine, which are released from postganglionic parasympathetic and sympathetic nerve terminals, respectively. The cholinergic receptor on the salivary acinar cell is the muscarinic M3 glandular subtype. The adrenergic receptors identified on these cells include both the α and β subtypes. At low flow rates, the saliva is hypotonic and rich in K+, whereas at higher flow rates, its composition approaches that of plasma. The salivary duct cells are responsible for producing hypotonic fluid that is rich in potassium bicarbonate. Stimulation of fluid and electrolyte secretion by salivary acinar cells is largely mediated by cholinergic and α-adrenergic input.

Salivary secretion: a. Is primarily regulated by the autonomic nervous system b. Is usually hypertonic to plasma c. Is primarily regulated by hormones d. Is inhibited by sympathetic stimulation e. Has a low [K+] f. Depends primarily on the acinar cells to secrete bicarbonate rich fluid g. Is mediated largely by nicotinic input

d. angular acceleration Semicircular canals sense acceleration, but not the linear acceleration that the otolithic organs prefer. Angular acceleration generated by sudden head rotations is the primary stimulus for the semicircular canals.

Semicircular canals of the inner ear are mainly responsible for sensing a. linear acceleration b. sense of "place" c. angular orientation d. angular acceleration e. all of the above f. None of the above g. A & D

b. Dephosphoyrlation of myosin light chain Smooth muscle contraction is regulated by Ca2+ and myosin light chain phosphorylation. When the cytosolic Ca2+ concentration decreases following the initiation of contraction, myosin kinase becomes inactivated. However, cross-bridge formation continues, even in the absence of Ca2+, until the myosin light chains are dephosphorylated through the action of myosin light chain phosphatase

Smooth muscle contraction is terminated by which of the following? a. Dephosphorlyation of myosin kinase b. Dephosphoyrlation of myosin light chain c. Efflux of Ca2+ ions across the plasma membrane d. Inhibition of myosin phosphatase e. Uptake of Ca2+ into the sarcoplasmic reticulum f. Conformational change of tropomyosin

d. All of the above

Spinal reflexes can involve which of the following elements: a. Sensory afferent b. Alpha motor neuron c. Interneurons that can either inhibit or excite motor neurons d. All of the above

f. Granular (rough) endoplasmic reticulum While in the rough ER, newly synthesized secretory and membrane proteins undergo the first in a series of post-translational modifications, including glycosylation, disulfide bond formation, and the acquisition of tertiary structure. Enzymes called glucosidases and one called a mannosidase associate with the luminal face of the ER and remove the three glucose residues and one mannose. This trimming process is a critical step in the quality-control process through which the cell determines whether a protein is properly folded and ready to proceed to subsequent stations of the secretory pathway

The abnormal cleavage of mannose residues during the post-translational processing of glycoproteins results in the development of a lupus-like autoimmune disease in mice. The abnormal cleavage is due to a mutation of the enzyme alpha-mannosidase II. Which subcellular unit is the site of the mutant protein? a. Nucleolus b. Lyososome c. Agranular (smooth) endoplasmic reticulum d. Golgi apparatus e. Mitochondria f. Granular (rough) endoplasmic reticulum

c. Reflex inhibition of motor neurons

The stimulation of nerve endings in the Golgi tendon organs leads directly to a. Contraction of intrafusal muscle fibers b. Contraction of extrafusal muscle fibers c. Reflex inhibition of motor neurons d. Increased γ-efferent discharge e. increased activity in group II afferent fibers

c. An increase in arterial carbon dioxide tension Both the central chemoreceptors, located on or near the ventral surface of the medulla, and the peripheral chemoreceptors, in the carotid and aortic bodies, cause an increase in ventilation in response to an increase in PaCO2. The peripheral chemoreceptors also cause an increase in ventilation in response to a decrease in arterial pH and a decrease in PaO2, but the central chemoreceptors are unresponsive to hypoxemia and do not cause an increase in ventilation in response to a decrease in arterial pH because the blood-brain barrier is relatively impermeable to hydrogen ions. Neither the central chemoreceptors nor the carotid bodies are stimulated by a decrease in arterial blood pressure or O2 content.

The wife of a recently married 24-year-old medical student reports that even though her husband doesn't snore, he stops breathing for periods of 1 to 2 minutes while he is sleeping. His physician refers him for a polysomnography study and pulmonary function testing, including ventilatory response curves. The tests confirm the apneic episodes during sleep and show depressed ventilatory responsiveness of both the peripheral and central chemoreceptors. Peripheral and central chemoreceptors both contribute to the increased ventilation that occurs as a result of which of the following? a. A decrease in arterial oxygen content b. A decrease in arterial blood pressure c. An increase in arterial carbon dioxide tension d. A decrease in arterial oxygen tension e. An increase in arterial pH

b. Cpeptide in a GTT Insulin is rapidly taken up by the cells. Cpeptide, the resultant peptide from insulin activation remains in the blood. To measure the amount of insulin secreted, measuring the serum concentration of C peptide is most effective.

To evaluate whether the pancreas has recovered some beta cell secretory capability in an insulin requiring type2 diabetic, which of the following plasma measurements would be most useful? a. Insulin in a GTT b. C peptide in a GTT c. Proinsulin after injecting epinephrine d. Insulin after stimulation with amino acids e. Glucagon after inducing mild hypoglycemia f. Glucagon in a GTT

e. Replacement of the voltage-gated Na+ channels with "slow" Ca2+ channels The slow Ca2+ channels have a slower inactivation rate, thereby lengthening the time during which they are open. This, in turn delays the repolarization phase of the action potential, creating a plateau before the channels inactivate.

Trace A represents a typical action potential recorded under control conditions from a normal nerve cell in response to a depolarizing stimulus. Which of the following perturbations would explain the conversion of the response shown in trace A to the action potential shown in trace B a. Blockade of voltage-gated Na+ channels b. Blockade of voltage-gated K+ channels c. Blockade of Na-K leak channels d. Replacement of the voltage-gated K+ channels with "slow" Ca2+ channels e. Replacement of the voltage-gated Na+ channels with "slow" Ca2+ channels f. Hyperactivity of the Na+/K+-ATPase g. Complete inhibition of the Na+/K+-ATPase

c. C The cephalic phase of gastric secretion is mediated entirely by the vagus nerve: vagotomy abolishes the response. The vagus also mediates a significant portion of the gastric phase of gastric secretion. Vagal stimulation increases acid secretion by a direct action of parietal cells as well as by stimulating gastrin secretion. Vagal stimulation increases gastrin secretion by directly stimulating G cells, which secrete gastrin, and inhibiting somatostatin cells from secreting somatostatin, which would otherwise inhibit G cells from secreting gastrin. Gastrin releasing peptide (GRP) is the neurotransmitter released from interneurons that stimulate G cells to secrete gastrin.

Vagal stimulation plays an essential role during the cephalic and gastric phases of gastric secretion. Vagal stimulation tends to cause which of the following changes in the release of gastric releasing peptide (GRP) and somatostatin? a. A b. B c. C d. D e. E

c. Surgical resection of the jejunum Vitamin B12 reaches the stomach bound to proteins in ingested food. In the stomach, pepsin and the low gastric pH release the cobalamin from the ingested proteins. Achlorhydria, autoimmune destruction of the parietal cells, autoimmune destruction of the chief cells, and total gastrectomy inhibit the release of cobalamin from the proteins it is bound to during ingestion. The parietal cells are also responsible for secreting intrinsic factor which is essential for the absorption of Vitamin B12. Vitamin B12 is absorbed in the ileum. If the ileum is resected, then Vitamin B12 cannot be absorbed.

Vitamin B12 (cobalamin) is required for a number of metabolic processes. Which of the following lesions would not lead to a deficiency of this vitamin? a. Chronic gastritis resulting in achlorhydria b. Autoimmune destruction of the gastric parietal cells c. Surgical resection of the jejunum d. Surgical resection of the ileum e. Total gastrectomy f. Autoimmune destruction of the gastric chief cells

d. Potassium ions Although most cells in the nervous system depolarize in response to sodium entry, hair cells are one group of cells that depolarize in response to potassium entry.

Which of the following molecules moves from the endolymph into the stereocilia and depolarizes the hair cell? a. Calcium ions b. Chloride ions c. Hydrogen ions d. Potassium ions e. Sodium ions f. Bicarbonate ions

b. Heart

Which of the following organs has the highest arteriovenous O2 difference under normal resting conditions? a. Brain b. Heart c. Skeletal muscle d. Kidney e. Stomach f. Liver

c. Capillaries The capillaries have the largest total-cross sectional area of all of the vessels of the circulatory system. The venules also have a relatively large cross-sectional area but not as large as the capillaries.

Which of the following vessels has the greatest cross-sectional area in the circulatory system? a. Aorta b. Small arteries c. Capillaries d. Venules e. Vena cava f. Arterioles g. Coronary arteries

e. Alveolar pressure is positive during expiration During expiration, inspiratory muscles relax and the thorax and lungs recoil increasing alveolar pressure causing outward air flow. With no activity of abdominal or intercostal muscles the residual volume will be increased compared to normal lung volumes. Vital capacity is the amount of air that can be slowly exhaled after a maximal inspiration and will not equal total lung capacity, which is the amount of air in the lung at maximal inspiration. During expiration, the lungs recoil and the intrapleural pressure is negative. Transpulmonary pressure equals alveolar pressure minus pleural pressure and the net difference is positive during expiration causing the lungs to deflate.

Your patient has normal function of the diaphragm but has a spinal injury that completely prevents any activity of abdominal or intercostal muscles. In this patient you expect to find that: a. Residual volume is smaller than normal b. Vital capacity equals total lung capacity c. Intrapleural pressure is positive during expiration d. Transpulmonary pressure is negative during expiration e. Alveolar pressure is positive during expiration f. Tidal volume equals total lung capacity

c. Distal convoluted tubule The DCT reabsorbs approximately 5% of the filtered NaCl through an electrically neutral thiazide-sensitive Na+/Cl- cotrasnproter on the apical membrane. Loss-of-function mutation of the SLC12A3 gene causes Gitelman's syndrome, a salt-wasting disorder associated with hypokalemic alkalosis, hypomagnesemia, hypocalciuria, and decreased urine chloride. The thick ascending limb of the loop of Henle employs a carrier that binds one sodium, one potassium, and two chloride ions. It is also electrically neutral. Diffusion of Na+ through channels on the apical surface of principal cells of the cortical and medullary collecting ducts is electrogenic.

. A 17-year-old male presents with fatigue, muscle cramps, and joint pain. Blood analysis reveals hypokalemia, hypomagnesemia, and hypochloremic metabolic alkalosis, and urinalysis reveals decreased urinary chloride and calcium. The clinical findings suggest a loss-of-function mutation of the SLC12A3 gene encoding the thiazide-sensitive sodium-chloride cotransporter (NCCT). Electrically neutral active transport of sodium and chloride occurs in which of the following areas of the nephron? a. Cortical collecting duct b. Descending limb of the loop of Henle c. Distal convoluted tubule d. Medullary collecting duct e. Thin ascending limb of loop of Henle f. Proximal convoluted tubule

b. Binding of Ca2+ to calmodulin Excitationcontraction coupling in skeletal muscle begins with an excitatory depolarization of the muscle fiber membrane (sarcolemma). This depolarization triggers the allornone opening of the voltage sensitive Na+ channels and an action potential that travels deep into the muscle fiber via the T tubule. At the T tubulesarcoplasmic reticulum "triad", the depolarization of the T tubule causes a conformational change in the dihydropyridine receptor and subsequently in the ryanodine receptor on the SR. The latter causes the release of Ca2+ into the sarcoplasm and the binding of Ca2+ to troponin C (not to calmodulin) on the actin filament.

44. Excitationcontraction coupling in skeletal muscle involves all of the following events EXCEPT one. Which is the EXCEPTION? a. ATP hydrolysis b. Binding of Ca2+ to calmodulin c. Conformational change in dihydropyridine receptor d. Depolarization of the transverse tubule (T tubule) membrane e. Increased Na+ conductance of sarcolemma f. Conformational change to the ryanodine receptor

b. Vitamin K The gall bladder and bile duct are essential for the digestion and absorption of fat vile the secretion of bile. Without adequate fat digestion, fat-soluble vitamins including Vitamins A, D, E, and K become deficient.

A 43-year-old man is diagnosed with cholecystitis and biliary obstruction. A deficiency in which of the following is most likely? a. Vitamin C b. Vitamin K c. Iron d. Vitamin B12 e. Intrinsic Factor f. Folate g. Calcium

f. Toxin blocks release of acetylcholine at synapses botulinumis a Gram-positive, anaerobic bacillus found in poorly sterilized canned foods as well as in soil or marine environments worldwide. The toxin blocks the release of acetylcholine at the synaptic ganglia and parasympathetic motor end plates, leading to paralysis of the skeletal muscles and cranial nerves. Infant botulism occurs with the ingestion of contaminated spores, commonly from canned foods or honey. The incubation period lasts between 3 days to 1 month, and is followed by constipation, lethargy, hypotonia, poor feeding, and inability to handle oral secretions. Treatment includes supportive care as well as antitoxin, which binds botulinum toxin that has not yet attached to the neuromuscular junctions.

A 1-year-old infant is brought to the pediatrician with lethargy, a poor suck, and feeding reflux days after eating home-canned peaches. You suspect Clostridium botulinumpoisoning. What is the mechanism of the toxin responsible for this child's symptoms? a. Toxin causes convulsive contractions of skeletal muscle b. Toxin stimulates influx of chloride at the apical membrane of the epithelial cell c. Exfoliative toxin causes separation of granular epidermal cells d. Enterotoxin causes stimulation of ADP ribosylation e. Protein causes pore formation in epithelial cell membranes f. Toxin blocks release of acetylcholine at synapses

a. Type 1 vitamin D-dependent rickets As treatment with vitamin D3 did not elicit improvement, but treatment with 1,25(OH)2-vitamin D3 did increase ionized Ca2+ levels, the absence of 1-alpha-hydroxylase enzyme in the kidney is indicated or type 1 vitamin D-dependent rickets.

A 10-year-old boy has a low plasma ionized Ca2+and an increased parathyroid hormone level. He has a high plasma alkaline phosphatase, exhibits muscle weakness, and has skeletal areas that are poorly calcified. You prescribe vitamin D3, but see no improvement. However, treatment with 1,25(OH)2-vitamin D3causes the patient's ionized Ca2+ level to increase. What would the most likely diagnosis be? a. Type 1 vitamin D-dependent rickets b. Rickets due to vitamin D deficiency c. Inability to form 24(OH)-vitamin D3 d. Receptors for dehydrocholesterol are absent e. Inability to form vitamin D3 in the skin

c. Decreased blood volume Anaphylactic shock occurs as a result of a severe allergic reaction to an antigen to which the patient has developed a sensitivity (e.g. insect bite). Compensatory mechanisms produced in shock involve strong arteriolar vasoconstriction by intense sympathetic activation, which significantly reduces blood volume. During shock sympathetic stimulation results in release of angiotensin II, which is a potent vasoconstrictor. In shock, the arteriolar vasoconstriction reduces capillary hydrostatic pressure, resulting in a net shift of fluid from the interstitial space into the vascular space.

A 10-year-old boy is admitted through the emergency department where he was brought after receiving a wasp sting while on a family outing. During a previous wasp sting, he experienced respiratory problems, itching, and urticaria, and he fainted. Upon admission he was obviously suffering from a generalized systemic inflammatory condition (anaphylaxis). What will anaphylactic shock cause? a. Increased renal fluid output b. Hypertension c. Decreased blood volume d. Decreased interstitial fluid pressure e. Increased level of atrial natriuretic peptide

A. A Amino acid reabsorption occurs primarily in the early portion of the proximal tubule utilizing co-transport with Na+.

A 10-year-old boy is brought to the emergency department with a brief history of severe, excruciating left flank pain, nausea, vomiting, and hematuria. The patient has had one similar episode in the past that brought him to the emergency department a few months ago. The boy also has a family history of recurrent kidney stones. Urinalysis showed hematuria and microscopic evaluation of the urinary sediment showed hexagonal crystals. An abdominal radiograph of the patient demonstrated the appearance of a fairly opaque ground glass stone in the left ureter. The patient's recurring clinical presentation is likely due to a defect in amino acid reabsorption in the kidney. What part of the kidney is responsible for the majority of amino acid reabsorption? A. A B. B C. C D. D E. E

c. Gluten free diet Malabsorption syndrome refers to the inability to adequately absorb nutrients and vitamins from the intestinal tract. One example of a malabsorption syndrome is the autoimmune disease, celiac sprue, which is also called gluten enteropathy. The disease is characterized by a deficiency in MHC class II antigen HLADQ2, which causes an allergic response to ingestion of gluten and related proteins. Elimination of these proteins, which are found in wheat, rye, barley, and oats, can restore normal bowel function in these patients.

A 10yearold boy presents with below average body weight and height, signs of vitamin K deficiency, steatorrhea, and bloating. He is found to have the MHC class II antigen HLADQ2. Which of the following is the most appropriate dietary treatment for malabsorption in this condition? a. Fatfree diet b. Lactose free diet c. Gluten free diet d. Highfiber diet e. Lowsalt diet f. Low protein diet

d. Celiac sprue This patient has celiac sprue or gluten enteropathy and presents with weight loss and characteristic diarrhea. The appropriate serologic testing includes endomysial antibody and antigliadin antibody testing. The diagnosis can be clinched by mucosal biopsy of the small intestine, revealing loss of intestinal villi, hypertrophy of intestinal crypts, and infiltration of the lamina propria with lymphocytes and plasma cells.

A 12-year-old girl is seen in your practice for weight loss and diarrhea, with bowel movements described as large, soft, greasy and foul smelling, occurring about ten times a day. After serologic testing and mucosal biopsy of the duodenum, you confirm your diagnosis and the patient's condition improves on a gluten-free diet. What is the most likely diagnosis of this patient's condition? a. Cystic Fibrosis b. Diverticulosis c. Gastrinoma d. Celiac sprue e. Chagas disease f. Gastroesophageal reflux disease

d. Juvenile myasthenia gravis Myasthenia gravis is an acquired autoimmune disease causing skeletal muscle fatigue and weakness. The disease is associated with (caused by) IgG antibodies to acetylcholine receptors at postsynaptic membranes of neuromuscular junctions. The major symptoms is muscle weakness, which gets worse with activity. Patients often feel well in the morning, but become weaker as the day goes on. The muscle weakness usually causes symptoms of double vision (diplopia) and drooping eyelids (ptosis). The presence of anti acetylcholine antibodies is specific for myasthenia gravis and this rules out the other answer choices. The normal CT rules out the possibility of an astrocytoma. The normal thyroid function test rules out the possibility of Graves and Hashimoto thyroiditis. The presence of antiacetylcholine antibodies is specific for myasthenia gravis and is not consistent with MS.

A 12yearold boy presents with a 4month history of diminished vision and diplopia. He also experiences tiredness toward the end of the day. There are no other symptoms. On examination, the patient has ptosis of the left eye that improves after a period of sleep. Clinical examination is otherwise normal. There is no evidence of weakness of any other muscles. Additional testing indicates the presence of antiacetylcholine antibodies in the plasma, a normal thyroid function test, and a normal CT scan of the brain and orbit. What is the initial diagnosis? a. Astrocytoma b. Graves disease c. Hashimoto thyroiditis d. Juvenile myasthenia gravis e. Multiple sclerosis f. Conn syndrome

e. Venules and veins The venules and veins are capacitance vessels and store the bulk of the blood for the body. When hemorrhage occurs, blood is redistributed from the veins and venules to the rest of the body. This redistribution is promoted by an increase in sympathetic outflow which promotes vasoconstriction and the shunting of blood back to the core of the body.

A 19-year-old man severs an artery in a motorcycle accident. A bystander applies a tourniquet to stop the bleeding. When paramedics arrive, the blood pressure of the injured man was only slightly hypotensive and his pupils were reactive. The greatest percentage of the redistributed blood volume came from which of the following? a. Heart b. Aorta c. Arteries and arterioles d. Capillaries e. Venules and veins f. Splanchnic circulation g. Kidneys

b. Actin to bdystroglycan Dystrophin is a large protein that forms a rod, which connects the thin filaments of actin to the transmembrane protein bdystroglyan in the sarcolemma. bDystroglyan is connected to laminin in the extracellular matrix by adystroglyan. The dystroglycans are also associated with a complex of four transmembrane glycoproteins, called sarcoglycans. The dystrophinglycoprotein complex adds strength to the muscle by providing a scaffolding for the fibrils and connecting them to the extracellular environment. Muscular dystrophy is the term used for some 50 diseases that cause progressive skeletal muscle weakness. Duchenne's and Becker's muscular dystrophy are two types resulting from mutations in the dystrophin gene.

A 12yearold male with muscular dystrophy is found to have a mutation of the gene that encodes the protein dystrophin. Genetic alterations in dystrophin lead to progressive muscular weakness because dystrophin provides structural support to the sarcolemma by binding which of the following? a. bDystroglycan to laminin b. Actin to bdystroglycan c. Actin to the Z lines d. Z lines to M lines e. Z lines to the sarcolemma f. bdystroglycan to the Z lines

d. Presence of anti-pancreatic islet cell antibodies Type I diabetes is characterized by the auto-immune destruction of the beta cells of the pancreatic islet.

A 14-year-old boy complains of a 3-week history of weight loss, excessive thirst, and urination. Type 1 diabetes mellitus is suspected. Laboratory testing of blood samples will most likely show: a. Increased erythrocyte sedimentation rate b. Increased serum creatinine level c. Presence of anti-pancreatic amyloid antibodies d. Presence of anti-pancreatic islet cell antibodies e. Presence of anti-streptolysin O antibodies f. Elevated concentration of C-peptide

a. Bronchodilation via smooth muscle relaxation This patient's symptoms point to a diagnosis of exerciseinduced asthma. Because his asthma only occurs when he is active and only some of the time, his disease can be classified as mild. The treatment of choice for such cases is an inhaled βagonist, such as albuterol, which directly promotes the relaxation of airway smooth muscle, leading to bronchodilation and increased air flow.

A 14yearold male soccer player complains of occasional shortness of breath, particularly during practice or games. You decide to initially treat his asthma with a β2agonist inhaler to be used when symptoms appear. By which mechanism will this agent improve his ability to breathe? a. Bronchodilation via smooth muscle relaxation b. A decrease of proinflammatory cells c. Prevention of histamine release d. Inhibition of leukotriene synthesis e. LeukotrieneI receptor antagonism f. Vasodilation of pulmonary capillaries

d. Vitamin D Excessive intake of vitamin D is associated with disturbances in calcium metabolism, resulting in hypercalcemia and hypophosphatemia. Clinical findings include nausea, vomiting, muscle weakness, calcification of soft tissues, renal stones, renal failure, and eventual death.

A 15-year-old boy complains of vomiting, anorexia, and weight loss. Laboratory studies reveal: serum calcium 13 mg/dL (normal 8.7-10.6 mg/dL); blood urea nitrogen 36 mg/dL (normal 7-22 mg/dL); serum creatinine, 1.8 mg/dL (normal 0.3-0.7 mg/dL). He has been taking several high-potency vitamin supplements daily for the past year as part of a 'body-building' program. An excess of which of the following vitamins is the most likely cause of these findings? a. Vitamin A b. Vitamin B c. Vitamin C d. Vitamin D e. Vitamin E f. Vitamin K

f. Unbound In the circulation, both T4 and T3 are highly bound to plasma proteins. Thyroidbinding globulin (TBG), albumin, and transthyretin (TTR) account for most of this binding. The affinity of these binding proteins is sufficiently high that, for T4, >99.98% of the hormone circulates tightly bound to protein. T3 is bound only slightly less: ~99.5% is protein bound. However, the free or unbound hormone in the circulation is responsible for the actions of the thyroid hormones on their target tissues.

A 15yearold girl presents with loss of the outer onethird of her eyebrows. Physical examination demonstrates slight enlargement of the thyroid gland and delayed relaxation phase of deep tendon reflexes. Blood work shows an elevation in creatine phosphokinase and TSH. Thyroid hormone therapy is ordered. Physiologically active thyroxine exists in which of the following forms? a. As a glucuronide b. Bound to albumin c. Bound to globulin d. Bound to prealbumin e. Bound to transthyretin f. Unbound

b. Greater than normal sodium reabsorption by the cortical collecting ducts The patient is treated with amiloride, which is a K+-sparing diuretic. Amiloride blocks Na+ channels in the principal cells of the cortical collecting ducts, limiting sodium reabsorption. Sodium reabsorption in the cortical collecting ducts is typically under the control of aldosterone. Patients with Liddle syndrome exhibit excess sodium reabsorption despite low levels of aldosterone and renin in the plasma, because of a mutation in the genes for the renal ENaCs, which increases ENaC activity and sodium reabsorption. Metabolic alkalosis, hypokalemia, and hypertension are also present due to the increased sodium (with subsequent water) reabsorption. An inability of the distal nephron to secrete hydrogen would cause RTA type I. An inability to concentrate urine occurs when patients are treated with loop diuretics. The amount of sodium reabsorbed by the proximal tubule remains constant primarily as a results of Na+/H+ exchange. The inability of the distal nephron to secrete potassium would result in hyperkalemia.

A 16-year-old girl presents for her annual high school athletic physical. She states that she seems more tired than usual, she has been having muscle cramps in her calves, and her legs get very weak and sore after running and playing soccer. Her blood pressure is 160/100 mm Hg, and her ECG shows a prolonged QT interval with the presence of a U wave. Blood analysis shows hypokalemia, metabolic acidosis, and decreases in plasma renin activity and aldosterone concentration. Her clinical condition is reversed after she is placed on the diuretic amiloride. Based on this finding, which of the following renal transport processes is the major defect causing metabolic disorder? a. Greater than normal sodium reabsorption by the proximal tubules b. Greater than normal sodium reabsorption by the cortical collecting ducts c. Inability of the distal nephron to secrete hydrogen d. Inability of the distal nephron to secrete potassium ions e. Inability to concentrate urine f. Decreased renal blood flow

b. Greater than normal sodium reabsorption by the cortical collecting ducts The patient is treated with Amiloride, a potassium sparing diuretic, which blocks sodium channels in the principal cells of the cortical collecting ducts, limiting sodium reabsorption. Sodium reabsorption from this region is normally under the control of aldosterone. In patients with Liddle syndrome, the cortical collecting ducts reabsorb excess sodium despite low levels of aldosterone and renin in the plasma because of a mutation in the genes for the renal ENaCs which increases ENaC activity and sodium retention. Metabolic alkalosis, hypokalemia, and hypertension are also present secondary to the increased sodium and water reabsorption.

A 16-year-old girl presents for her annual high school athletic physical. She states that she seems more tired than usual. She has been having muscle cramps in her claves, and her legs get very weak and sore after running and playing soccer. Her blood pressure is 160/100 mmHg and her ECG shows a prolonged QT interval and the presence of a U wave. Blood analysis shows hypokalemia, metabolic alkalosis, and decreases in plasma renin activity and aldosterone concentration. Her clinical condition is reversed after she is placed on the diuretic amiloride. Based on this finding, which of the following renal transport processes is the major defect causing her metabolic disorder? a. Greater than normal sodium reabsorption by the proximal tubule b. Greater than normal sodium reabsorption by the cortical collecting ducts c. inability of the distal nephron to secrete hydrogen d. Inability of the distal nephron to secrete potassium ion e. Inability to concentrate urine f. Greater than normal potassium reabsorption by the proximal tubule g. Greater than normal bicarbonate secretion from the cortical collecting ducts

c. Androgen insensitivity Androgen insensitivity syndrome, also known as testicular feminization syndrome, is where patients are genetically and gonadally male, but phenotypically female. Testes are present and functional, producing testosterone, but due to the absence of androgen receptors, tissues do not respond to testosterone androgen effects.

A 17yearold girl comes to see an endocrinologist because of primary amenorrhea. Breast development started several years ago, and she has otherwise been well. On examination breast development and height is in the tenth percentile, but she has no pubic or axillary hair. A pelvic examination reveals a blind vagina. These findings are most consistent with which of the following: a. Turner's syndrome b. Hypopituitarism c. Androgen insensitivity d. Klinefelter's syndrome e. Adrenogenital syndrome f. Congenital adrenal hyperplasia

d. 72 mEq/L Net acid excretion is the amount of acid excreted each day. It is calculated using the formula: Net Acid Excretion (NAE) = (UTitratable Acids + UNH4+ - UHCO3-) × Volume NAE=(24mEq/L+38mEq/L- 2mEq/L)×1.2L/day=72mEq/L Almost all of the acid excreted is buffered by either phosphate, called titratable acid, and ammonia. The titratable acid is equal to the mM of NaOH that must be added to the urine to raise its pH back to that of plasma. Bicarbonate must be subtracted from the sum of acid excreted because each milliequivalent of excreted bicarbonate represents the addition of 1 mEq of acid to the plasma.

A 17yearold girl went on a starvation diet for 3 days before prom so that she would look thin in her new dress. Her mother found her lethargic and hyperventilating, and took her to the Emergency Department for evaluation. Based on the following laboratory values, which of the following is her net acid excretion? Plasma pH = 7.26 Urine flow = 1.2 L/day Urine bicarbonate = 2 mEq/L Urine titratable acids = 24 mEq/L Urine ammonium = 38 mEq/L Urine pH = 5.4 a. 60 mEq/L b. 64 mEq/L c. 68 mEq/L d. 72 mEq/L e. 76 mEq/L f. 84 mEq/L

e. Reduced secretion of GnRH This patient has hypogonadotropic hypogonadism due to reduced secretion of GnRH. This is the only option that could be responsible for the hypogonadism described in this case. Hypersecretion of gonadotropins would promote normal pubertal development, not hypogonadism.

A 18-year-old male complains about continuous growth, lack of facial hair, small genitalia and lack of muscle development. Laboratory values include total testosterone 125 ng/dL (normal 300-1100 ng/dL) and luteinizing hormone (LH) < 2 mIU/mL (normal 6-23 mIU/mL). A major causal factor in some cases of hypogonadism is: a. Increased number of follicle stimulating hormone (FSH) receptors in the testis b. Hypersecretion of pituitary LH and FSH as the result of increased gonadotropin-releasing hormone (GnRH) c. Excess secretion of testicular activin by Sertoli cells d. Failure of the hypothalamus to respond to testosterone e. Reduced secretion of GnRH

d. Decreased glucose; elevated protein; many polymorphonuclear leukocytes A reduced glucose, elevated protein, and serum with many polymorphonuclear leukocytes are characteristic findings of bacterial meningitis.

A 19-year-old male presents to the emergency department with headache, neck stiffness and fever. You are concerned about bacterial meningitis. What are the classical laboratory findings for CSF in bacterial meningitis? a. Decreased glucose; decreased protein; many polymorphonuclear leukocytes b. Normal glucose; elevated protein; many polymorphonuclear leukocytes c. Elevated glucose; decreased protein; many polymorphonuclear leukocytes d. Decreased glucose; elevated protein; many polymorphonuclear leukocytes e. Elevated glucose; elevated protein; many lymphocytes f. Decreased glucose, normal protein, many polymorphonuclear leukocytes, many lymphocytes

c. Hyperpolarizing the membrane potential Hyperpolarization of the membrane potential causes the closing of voltage-gated potassium channels, decreasing the amount of potassium efflux from the cell.

A 2-day-old infant starts having brief tonic-clonic seizures throughout the day. His neurological function in between seizures is normal, and he has no other medical or neurological problems. The history reveals no readily apparent causes for the seizures, though the mother recalled that her first baby also developed seizures shortly after birth that only lasted for 2 weeks, with no subsequent episodes or developmental problems. Genetic analysis revealed a mutation of voltage-gated K+ channels consistent with a diagnosis of benign familial neonatal seizures. Which of the following would cause an immediate reduction in the amount of potassium leaking out of a cell? a. Decreasing the extracellular potassium concentration b. Decreasing the extracellular sodium concentration c. Hyperpolarizing the membrane potential d. Increasing the permeability of the membrane to potassium e. Reducing the activity of the sodium-potassium pump f. Increasing the activity of voltage-gated sodium channels g. Increasing the number of voltage-gated potassium channels

c. Defective epithelial chloride transport This child has cystic fibrosis (CF), a disease of the secretory processes of exocrine and eccrine glands of the body. Cystic fibrosis affects 1 in 2000 live births and is most common among Caucasians. The common defect in these patients is a defective regulation of an epithelial chloride transporter. Defective chloride transport is responsible for the development of the clinical manifestations of the disease. In sweat gland ducts, the defect leads to decreased reabsorption of sodium, and an increased sweat chloride, which is diagnostic for CF. In the respiratory mucosa, this defect leads to an increased viscosity and thickness of secretions, leading to mucus plugging and obstruction of the airways. Pancreatic abnormalities are common, with loss of exocrine pancreatic function and steatorrhea, with resulting fat-soluble vitamin deficiencies. In males, obstruction of the epididymis and vas deferens lead to infertility. Phenylalanine hydroxylase is responsible for the metabolism of phenylalanine. High plasma levels of phenylalanine lead to mental retardation, seizures, and skin pigment problems. These patients are not known, however, for having pulmonary complications. A deficiency of galactokinase leads to galactosemia. This leads to hepatomegaly, opacification of the lens, CNS abnormalities, heart disease, and kidney problems, but not pulmonary problems. Amino acid substitutions on the β polypeptide chain result in hemoglobinopathies, such as HbS. High concentrations of sickle hemoglobin (HbS) results in hemolytic anemia and vaso-occlusive crisis. The history and symptoms described in this case do not indicate sickle cell anemia. Defects in hypoxanthine-guanine phosphoribosyltransferase lead to the accumulation of uric acid, leading to gouty arthritis, nephropathy, and neurologic symptoms - not the pulmonary symptoms described.

A 2-year-old Caucasian boy is brought to the pediatric clinic for failure to gain weight. The child's mother states that he has frequent foul-smelling stools, with a greasy appearance, and has had three hospitalizations for pneumonia. Which of the following defects is responsible for this child's symptoms? a. Lack of phenylalanine hydroxylase b. Deficiency of galactokinase c. Defective epithelial chloride transport d. Amino acid substitution on the hemoglobin beta chain e. Defect in hypoxanthine-guanine phosphoribosyltransferase

d. Normal platelet count, bleeding time normal, PT prolonged, aPTT prolonged Warfarin ingestion has no effect on number of platelets, can eliminate A, B, E, and F. The prothrombin time of patients receiving a vitamin K-competing warfarin drug such as warfarin (anticoagulation therapy used in deep venous thrombophlebitis) will also be prolonged, usually in the range of one and one half to two times the normal PT time.

A 2-year-old boy accidentally ingests rat poison, which contains warfarin (Coumadin). Physical examination shows bleeding from the mouth and gastrointestinal tract. Which of the following sets of laboratory test results is most likely to be reported? a. Decreased platelet count, bleeding time prolonged, PT normal, aPTT normal b. Decreased platelet count, bleeding time normal, PT normal, aPTT normal c. Normal platelet count, bleeding time normal, PT normal, aPTT prolonged d. Normal platelet count, bleeding time normal, PT prolonged, aPTT prolonged e. Increased platelet count, bleeding time prolonged, PT normal, aPTT normal f. Increased platelet count, bleeding time prolonged, PT normal, aPTT prolonged

f. Tropheryma whippelii AS is a chronic systemic inflammatory disorder affecting primarily the axial skeleton, but may have multiple organ involvement. The pathogenesis of AS is incompletely understood, but the dramatic response of all aspects of the disease to blockade of TNF-a indicates that this cytokine almost certainly plays a major role in the immunopathogenesis. Prior to the 2000 report of the effectiveness of anti-TNF-a in the treatment of AS and other spondyloarthrides, NSAID therapy with indomethacin, and more recently COX-2 inhibitors, combined with exercise programs constituted the primary management. Tropheryma whippelii is the organism causing the rare chronic bacterial infection, Whipple's disease, which initially presents with arthritic symptoms. Antibiotic therapy is curative, whereas the untreated disease is fatal.

A 20-year-old male presents with stiffness in the lower back that improves with exercise. A pelvic radiograph reveals sacroiliitis. Patients with ankylosing spondylitis (AS) show a dramatic response to all symptoms with therapeutic blockade of which of the following? a. Prostaglandin E b. Cyclooxygenase 2 (COX-2) c. Tumor necrosis factor a (TNF-a) d. Na+-K+, ATPase e. Interleukin-6 f. Tropheryma whippelii

c. Intracellular volume Adding water will promote influx of fluid into the cell. The increase in fluid causes swelling of the brain resulting in cerebral edema and the pt's symptoms.

A 23-year-old man is brought to the emergency department after collapsing during basketball practice. On admission, he is lethargic and appears confused. His coach reports that it was hot in the gym and he was drinking a lot of water during practice. An increase in the following is most likely the cause of his symptoms? a. Intracellular tonicity b. Extracellular tonicity c. Intracellular volume d. Extracellular volume e. Plasma volume f. Plasma tonicity

b. Both the left and right eyes deviate toward the left When the head rotates in one direction, the hair cells mounted on the cristae rotate along with the head. However, the flow of endolymph is delayed and as a result the cupula is moved in a direction opposite to the movement of the head. When the head moves to the right, the cupula moves toward the left; this bends the stereocilia on the hair cells in the right horizontal canal toward the kinocilium and bends the stereocilia on the hair cells in the left horizontal canal toward the kinocilium. As a result, the hair cells in the right horizontal canal depolarize and those in the left horizontal canal hyperpolarize. The depolarization of the hair cells in the right horizontal canal stimulates the right vestibular nerve, which in turn causes the eyes to deviate toward the left. The movement of the eyes toward the left as the head deviates toward the right keeps the image on the retina in focus.

A 20yearold boxer suffers from disequilibrium after several blows to the ears. Which of the following normally happens when a person slowly rotates toward the right? a. The stereocilia on the hair cells in the right horizontal semicircular canal bend away from the kinocilium b. Both the left and right eyes deviate toward the left c. The hair cells in the left horizontal semicircular canal become depolarized d. The visual image on the retina becomes unfocused e. The endolymph in the left and right horizontal semicircular canals moves in opposite directions

A. Blood loss stimulates the arterial baroroceptors which respond to a fall in arterial pressure. The result is an increase in sympathetic outflow from the vasomotor center and a decrease in parasympathetic nerve activity. The increase in sympathetic nerve activity leads to peripheral vasoconstriction, increased total peripheral resistance, and a return of blood pressure toward normal. The decrease in parasympathetic nerve activity and sympathetic outflow would result in an increase in heart rate.

A 22-year-old man enters the hospital emergency room after severing a major artery in a motorcycle accident. It is estimated that he has lost approximately 700 mL of blood. His blood pressure is 90/55 mm Hg. Which of the following sets of changes would be expected in response to hemorrhage in this man?

b. The ciliary body Contracting the ciliary body increases the refractive power of the eye for near vision. When the ciliary muscle contracts, it pulls the suspensory ligaments toward the cornea, which causes the lens surface to bulge, increasing its refractive power. The muscles of the iris control the size of the pupils, and the extraocular muscles control the position of the eye in the socket.

A 22-year-old man sees his ophthalmologist because it is becoming increasingly difficult for him to read the newspaper. His vision problem most likely results from an inability to contract which of the following? a. The iris b. The ciliary body c. The suspensory ligaments d. The extraocular muscles e. The pupil

g. Preload The right and left ventricles are in series with one another such that the right and left ventricular outputs are essentially equal. Because the two ventricles beat at the same rate, their stroke volumes are the same. Although the left and right preloads are not identical, they are very similar as reflected by an average right atrial pressure of 2 mmHg and a left atrial pressure of 5 mm Hg. In contrast, the resistance of the pulmonary vasculature is much lower than that of the systemic circulation, yielding much lower pressures in the pulmonary artery than the aorta (mean pulmonary artery pressure = 15 mm Hg mean aortic pressure = 90 mmHg). Thus, the afterload and stroke work are greater on the left side than on the right side. Because the same cardiac output is ejected into a higher resistance, peak systolic pressure is higher on the left side than the right side. Only about 10% of the blood volume is within the pulmonary circulation at any one time, whereas approximately two-thirds of the blood volume is stored within the systemic veins and venules.

A 22-year-old man with no history of congenital heart disease has a normal physical examination prior to entering the military. Which of the following characteristics is most similar in the systemic and pulmonary circulations of this patient? a. Afterload b. Blood volume c. Peak systolic pressure d. Mean diastolic pressure e. Stroke work f. Resistance g. Preload

a. The proximal tubule The mechanism of diuresis involves the proximal tubule of the kidney. The enzyme carbonic anhydrase is found here, allowing the reabsorption of bicarbonate, sodium, and chloride. By inhibiting this enzyme, these ions are excreted, along with excess water, lowering blood pressure, intracranial pressure, and intraocular pressure.

A 23-year-old man expresses concern about his upcoming skiing trip to Breckenridge, CO (elevation ~10,000ft). He states that every time he goes there, he gets high altitude sickness that is relieved when he is given oxygen. The family physician gives the patient a prescription for oxygen to use when he arrives in Colorado, as well as a prescription for acetazolamide, a carbonic anhydrase inhibitor, to take for 2 days prior and throughout his four-day trip. Carbonic anhydrase inhibitors exert their diuretic effect by inhibiting the reabsorption of Na+ in which of the following parts of the nephron? a. The proximal tubule b. The thick ascending limb of the loop of Henle c. The distal convoluted tubule d. The cortical collecting duct e. The outer medullary collecting duct f. The macula densa g. The inner medullary collecting duct

e. Terminal ileal disease Vitamin B12 (cobalamin) absorption occurs primarily in the ileum. Cobalamin's primary function is to serve as a coenzyme for homocysteine, converting homocysteine to methionine. If cobalamin is deficient and methionine levels fall, then the body converts its stores of intracellular folate into N5-methyl-THF in an effort to produce more methionine. As a result, levels of 5,10-methylene-THF (the form of folate needed for DNA synthesis) falls, an effect that explains why folate and cobalamin deficiencies cause identical hematological abnormalities (i.e., megaloblastic anemia). All of the answer choices can be eliminated except pernicious anemia and terminal ileal disease. Pernicious anemia can be ruled out as addition of intrinsic factor does not correct the anemia.

A 22-year-old woman has an intermittent history of colicky, right lower quadrant pain and diarrhea. Laboratory studies show an anemia with a mean corpuscular volume of 120 μm3, a WBC count of 2000 cells/mm3, and a platelet count of 50,000 cells/mm3. The peripheral smear shows macro-ovalocytes and hypersegmented neutrophils. Serum vitamin B12level is decreased, and serum folate is normal. Addition of intrinsic factor or administration of broad-spectrum antibiotics does not correct the anemia. Which of the following is the most likely cause of the anemia? a. Alcohol excess b. Bacterial overgrowth c. Pernicious anemia d. Pregnancy e. Terminal ileal disease f. Lactose intolerance g. Overproduction of gastric acid

e. Increased central venous pressure Net filtration from systemic capillaries is dependent on the Starling forces and capillary permeability. The equation is Net filtration = Kf [(Pcapillary - Ptissue) - (pcapillary - ptissue)], where Kf is the filtration coefficient of the membrane, and is directly proportional to capillary permeability, Pcapillary and Ptissue are the hydrostatic pressures in the capillary and tissue (interstitial space), respectively, and pcapillary and ptissue are the osmotic (colloid oncotic) pressures in the capillary and interstitial space, respectively. Increasing central venous pressure

A 22-year-old woman is hospitalized with a history of respiratory distress, fever, and fatigue. ST segment and T wave abnormalities suggest myocarditis, which is attributed to an acute viral origin. Over the next several days, significant peripheral edema develops. The edema is most likely caused by which of the following? a. Decreased capillary permeability b. Decreased arterial pressure c. Increased plasma protein concentration d. Increased lymphatic flow e. Increased central venous pressure f. Increased cardiac output

a. Arterioles Sympathetic nervous system activation results in stimulation of alpha and beta adrenergic receptors. These in turn activate a G protein cascade which results in the release of calcium from the sarcoplasmic reticulum and increased muscle contraction. Ephedrine acts primarily on post-synaptic alpha-adrenergic receptors, and very weakly on beta-adrenergic receptors. There are no alpha receptors in the bronchioles, pupils, or ciliary bodies, so there is no contraction. Venules have almost no smooth muscle cells as they are capacitance vessels. Ephedrine is also taken up into neurotransmitting cells and is released along with norepinephrine from the sympathetic nerve terminals. Smooth muscle cells of the intestines relax when the alpha2-adrenergic receptors are stimulated by norepinephrine and ephedrine.

A 22-year-old woman presents at the student center with tachycardia and palpitations. She reports that she has been taking a diet supplement containing ephedrine that she purchased from an internet site. Activation of the sympathetic nervous system by ephedrine causes smooth muscle contraction in which of the following? a. Arterioles b. Bronchioles c. Ciliary bodies d. Intestines e. Pupils f. Venules

b. First-degree AV heart block In first-degree AV heart block, the PR interval is prolonged >20 ms, but without any dropped QRS waves. In this EKG, the PR interval is approximately 1 second, but there are no dropped QRS waves. In second and third degree AV block, QRS waves are dropped. In atrial fibrillation, there is an absence of P waves. In the common form of type I atrial flutter, the ECG shows sawtooth flutter (F) waves. Flutter waves are often visualized best in leads II, III, aVF, or V1 (see the image below). The flutter waves for typical (type I) atrial flutter are inverted (negative) in these leads because of a counterclockwise reentrant pathway.

A 220 lb. 60-year-old man with a past medical history significant for myocarditis, with no other cardiac risk factors presented to the Emergency Department following an episode of sharp substernal chest pain and shortness of breath. The patient reports that the pain started while he had been exercising. The patient reports no previous episodes of pain, and endorses a history of high athletic performance. On examination, his heart rate was 65 beats per minute, BP was 135/80, and respiratory rate was 12 breaths per minute. His EKG was obtained, and the standard lead II is shown below. What is his diagnosis? a. AV nodal rhythm b. First-degree AV heart block c. Second degree AV heart block d. Third degree AV heart block e. Atrial flutter f. Atrial fibrillation

d. Arterioles The catecholamines, norepinephrine and epinephrine, will activate both a and badrenergic receptors. When the a1adrenergic receptors are stimulated, they activate a G protein, which in turn activates phospholipase C, which hydrolyzes PIP2 and produces IP3 and DAG. The IP3 causes the release of Ca2+ from the sarcoplasmic reticulum, which in turn increases muscle contraction. a1Adrenergic receptors predominate on arteriolar smooth muscle, so these muscles contract when stimulated with norepinephrine. The bronchiolar, pupillary, and ciliary smooth muscles all contain b receptors, which cause smooth muscle relaxation. The intestinal smooth muscle relaxation is initiated by an a2 adrenergic receptor.

A 22yearold female takes a diet supplement containing ephedrine. Activation of the sympathetic nervous system by ephedrine causes tachycardia, palpitations, and contraction of the smooth muscle in which of the following? a. Bronchioles b. Pupils c. Intestine d. Arterioles e. Ciliary body

a. Hyperaldosteronism Aldosterone promotes the loss of both H+ and K+, producing both the metabolic alkalosis and hypokalemia described in this case.

A 23-year-old girl complains of a 3-month history of malaise and generalized muscle cramps. Laboratory results reveal: serum sodium 144 mmol/L; serum potassium 2.0 mmol/L; serum bicarbonate 40 mmol/L; arterial pH 7.5. Which of the following is the most likely cause of this patient's hypokalemic alkalosis? a. Hyperaldosteronism b. Hyperventilation c. Persistent diarrhea d. Renal failure e. Diabetes mellitus f. Heart failure

b. Golgi tendon organ The harder a muscle is stretched, the stronger the contraction, but when the tension becomes too great, the contracting muscle suddenly relaxes. The reflex that causes this sudden muscle relaxation is known as the Golgi tendon organ reflex. The Golgi tendon organ is connected in series with the extrafusal skeletal muscle fibers, which respond to muscle tension rather than muscle length. Increased tension leads to stimulation of lb afferents, which inhibit the homonymous muscle via spinal interneurons. Muscle spindles are spindle-shaped bundles of muscle fibers (intrafusal fibers) arranged in parallel with extrafusal skeletal muscle fibers. Muscle spindles sense the length of the muscle and are innervated by group la and II sensory afferent neurons. Muscle spindles respond to muscle length, not tension.

A 23-year-old weightlifter lifts 325 lbs. over his head and holds it there for 5 seconds, after which he suddenly drops it to the floor. Which of the following receptors is responsible for this sudden muscle relaxation? a. Free nerve ending b. Golgi tendon organ c. Muscle spindle d. Merkel's disk e. Pacinian corpuscle f. Ruffini's ending

a. Anemia In anemia, the reduced hematocrit reduces blood viscosity, which increases the velocity and thsu the turbulence of blood flow, which makes systolic murmurs common. An increased velocity of blood flow in anemia is also due to higher than normal heart rate and cardiac output, which is a compensatory response to increase oxygen delivery to the tissues in the face of reduced arterial oxygen content. An increased velocity of blood flow and turbulence can also result from flow through a narrowed opening, such as a stenotic heart valve. It is helpful to separate murmurs into systolic and diastolic type. During systole, the AV valves (tricuspid and mitral) close and blood is ejected from the ventricles as the semilunar valves (aortic and pulmonary) open. If there is a murmur during systole, the AV valves have not closed as they should (tricuspid or mitral regurgitation), the semilunar valves are narrowed (aortic or pulmonic stenosis), or there is a higher than normal velocity of blood flow across normal valves (anemia). However, the systolic murmur of mitral regurgitation is either early systolic (acute mitral regurgitation) or holosystolic, and cardiac output would be reduced. Third-degree heart block may be associated with a mid-systolic murmur, but cardiac output would not be increased. During diastole, the semilunar valves should close and the AV valves should open to allow filling of the ventricles. If a diastolic murmur is present, then there is either semilunar (aortic or pulmonic) regurgitation, or AV valve stenosis. In cardiac tamponade, cardiac output is reduced due to decreased filling of the heart. Heart sounds are distant due to accumulation of fluid within the pericardium.

A 23-year-old woman presents with fatigue. She is found to have a mid-systolic murmur and higher-than-normal cardiac output. The differential diagnosis based on these findings includes which of the following? a. Anemia b. Cardiac tamponade c. Mitral regurgitation d. Mitral stenosis e. Third-degree heart block f. Aortic stenosis g. Pulmonic regurgitation

c. Forced expiratory volume in 1 second (FEV1)/forced vital capacity (FVC) ratio of less than 75% Chronic obstructive pulmonary disease (COPD), including asthma, is characterized by an increased resistance to air flow, which makes it much more difficult to expel gas from the lung during expiration. The increased airway resistance decreases maximum breathing capacity and FEV1. The FEV1/FVC ratio is the most sensitive test for airway obstruction, where FEV1 is the amount of air that can be exhaled forcefully within 1 second, and FVC is the total amount of air exhaled through a maximal forceful expiration. Airway obstruction is indicated when the FEV1/FVC ratio falls below 75%.

A 23-year-old woman, who complains of chronic cough, shortness of breath, and chest tightness is diagnosed with asthma. Which of the following is the best clinical indicator of significant pulmonary obstruction in patients with asthma? a. Arterial hypoxemia b. Dyspnea c. Forced expiratory volume in 1 second (FEV1)/forced vital capacity (FVC) ratio of less than 75% d. Large anatomic dead space volume e. Rapid respirations f. V/Q mismatch

d. LH Ovulation is caused by a surge of luteinizing hormone (LH) that at the midpoint of the menstrual cycle.

A 23yearold woman who has been training for a marathon presents with secondary amenorrhea. A pregnancy test is negative, so the woman is started on an opioid blocker, which restores ovulation and menses. Ovulation is caused by a sudden increase in the secretion of which of the following hormones? a. Estrogen b. FSH c. GnRH d. LH e. Progesterone f. Progestin

c. Blood transfusion This patient's blood pressure is low and his heart rate is high, which are signs that he is suffering from hypovolemic shock. The low pressure stimulates baroreceptors to cause a reflex-mediated increase in sympathetic activity, which causes arteriolar vasoconstriction to prevent the arterial blood pressure from falling lower. In the kidney, arteriolar vasoconstriction results in reduced renal perfusion and a lower glomerular filtration rate, which helps the body to conserve fluids. Increasing the blood volume helps to maintain an adequate blood pressure and urinary output. Physiologic saline solution would not be as effective for fluid replacement as blood or a plasma expander because it does not cause fluid to remain within the plasma space, but leaks to the interstitium.

A 24-year-old bicyclist severely injures his right leg after colliding with an automobile. He is hemorrhaging from the wound but is stabilized, with a blood pressure of 90/40 mmHg and a heart rate of 100/min. He was not wearing protective head gear at the time of the accident, but shows no evidence of head trauma. His breathing is rapid and shallow. Blood pressure and urine flow could best be improved in this patient by: a. Oral administration of diuretic b. Administration of physiologic saline solution c. Blood transfusion d. Indirect cardiac massage e. Intravenous injection of norepinephrine

a. GABA GABA is the major inhibitory mediator on the brain. GABAA receptors are Cl- ion channels that increase the Cl- conductance into the cell, decreasing their excitability. Several anti-anxiety medications potentiate the increase in Cl- conduction at GABAA receptors. Glutamate, Histamine, and neuropeptide Y have been found to have excitatory effects on the CNS. Neurokinin-1 receptor inhibition has antidepressant activity in humans. nAChRs are seen most commonly at the neuromuscular junction. Beta-1-adrenergic receptor stimulation increases the heart rate and can enhance feelings of anxiety.

A 24-year-old medical physiology student develops apprehension, restlessness, tachycardia, and tachypnea as she enters the testing center for her NBME examination. Activation of which of the following receptors would be expected to decrease her anxiety? a. GABA b. Glutamate c. Histamine d. Neurokinin-1 e. Neuropeptide Y f. Nicotinic Acetylcholine receptor g. Beta-1-adrenergic

a. Bicarbonate CO2 is transported in arterial blood in three forms: as physically dissolved CO2 (about 5%), in combination the amino groups of Hb as Carbaminohemoglobin (about 10%), and as bicarbonate ion (85%). The amount of CO2 carried as carbonic acid is negligible. Carboxyhemoglobin is the combination of carbon monoxide with Hb.

A 24-year-old woman presents to her family physician with intractable hiccups. The patient is instructed to breathe into and out of a bag in order to rebreathe exhaled CO2. In the blood, the majority of CO2 is transported as which of the following forms? a. Bicarbonate b. Carbaminohemoglobin c. Carbonic acid d. Carboxyhemoglobin e. Dissolved CO2 f. Carboxylic acid g. Carbamino compound

c. GABA g-Aminobutyric acid (GABA) is the major inhibitory mediator in the brain. GABAA receptors are pentameric Cl- ion channels that are widely distributed in the CNS. The increase in Cl- conductance produced by GABAA receptors is potentiated by the anxiolytic drug, diazepam, and other benzodiazepines. Glutamate is the major excitatory transmitter in the brain. Neuropeptide Y is an excitatory neurotransmitter that has a stimulatory effect on food intake. Central nervous system actions of histamine have been implicated in arousal, sexual behavior, drinking, pain thresholds, and the sensation of itch. Antagonism of central NK1 receptors has antidepressant activity in humans.

A 24yearold medical student develops apprehension, restlessness, tachycardia, and tachypnea as he enters the testing center for his initial licensure examination. Activation of which of the following receptors would be expected to decrease his anxiety? a. Neuropeptide Y b. Glutamate c. GABA d. Histamine e. Neurokinin 1 (NK1, Substance P) f. Betaadrenergic

c. Glucose "Nutrient-coupled" Na+ absorption occurs throughout the small intestine. Glucose- and amino acid-coupled Na+ absorption occurs only in villous epithelial cells and not in crypt epithelial cells. Glucose- and amino acid-coupled Na+ absorption is mediated by specific apical membrane transport proteins. The Na/glucose cotransporter SGLT1 is responsible for glucose uptake across the apical membrane. The glucose and amino acid-coupled uptake of Na+ entry across the apical membrane increases [Na+]i, which in turn increases Na+ extrusion across the basolateral membrane through the Na-K pump. Oral rehydration using a saline solution containing glucose utilizes Na+/glucose co-transport to promote the absorption of sodium.

A 25-year-old woman comes to the physician because of a 2-day history of muscle cramps and profuse, watery stools. She returned from a trip to Pakistan 3 days ago. Her temperature is 37°C (98.6°F), pulse is 120/min, and blood pressure is 80/50 mm Hg. Stool culture shows numerous curved, gram-negative bacteria; there are no erythrocytes or leukocytes. Oral rehydration is initiated. The blood pressure increases, and the pulse decreases. The oral hydration formula most likely promotes sodium absorption via the gut by allowing cotransport with which of the following? a. Albumin b. Fatty acid c. Glucose d. Magnesium e. Potassium f. Lactose g. Chloride

b. Low Serum thyroxine (T4); High Serum thyroid-stimulating hormone (TSH); Normal (radioactive iodine) uptake The patient exhibits classic symptoms of primary hypothyroidism, probably secondary to Hashimoto's thyroiditis. Hashimoto's thyroiditis is an autoimmune lymphocytic infiltration of the thyroid gland. Decreased, serum T4 levels due to destruction of the thyroid, and increased serum TSH due to lack of feedback control would be expected.

A 25-year-old woman complains of chronic constipation and progressive weight gain over the past 6 months despite maintaining a pure vegan diet. Physical examination reveals a fatigued woman with brittle hair, face and eyelid puffiness, a symmetrically enlarged, non-tender thyroid gland, prolonged relaxation phase of deep tendon reflexes, and proximal muscle weakness in her lower extremities. Which set of the following results of thyroid function studies shown below would you expect in this patient? a. Low Serum thyroxine (T4); Low Serum thyroid-stimulating hormone (TSH); Low (radioactive iodine) uptake b. Low Serum thyroxine (T4); High Serum thyroid-stimulating hormone (TSH); Normal (radioactive iodine) uptake c. High Serum thyroxine (T4); Low Serum thyroid-stimulating hormone (TSH); Low (radioactive iodine) uptake d. High Serum thyroxine (T4); Low Serum thyroid-stimulating hormone (TSH); High (radioactive iodine) uptake e. Normal Serum thyroxine (T4); Normal Serum thyroid-stimulating hormone (TSH); Normal (radioactive iodine) uptake

e. Weight loss syndrome When body fat in a female falls to1ess than 15% of normal for her size and weight, hypothalamic secretion of gonadotropin-releasing hormone (GnRH) decreases. This results in decreased release of gonadotropins FSH and LH. Loss of gonadotropins results in insufficient ovarian synthesis of estrogen and amenorrhea. A progesterone challenge is used to evaluate the cause of amenorrhea once pregnancy is ruled out. Absence of withdrawal bleeding indicates inadequate levels of estrogen or an end-organ problem. The patient's gonadotropin levels are decreased, indicating that the cause of this patient's amenorrhea lies in the hypothalamic-pituitary axis (i.e. excessive weight loss). Low gonadotropin levels do not indicate primary ovarian disease, which is associated with increased gonadotropin levels. Hypopituitarism would cause low levels of cortisol and growth hormone, not increased levels as in this case. Physical examination of this patient did not reveal any of the classic findings of Cushing's syndrome (such as truncal obesity or buffalo hump). Although growth hormone levels are increased, the patient in this case has none of the classic findings of acromegaly (e.g. prominent jaw, enlarged hands and feet).

A 25-year-old woman has had secondary amenorrhea for the past 8 months. She plans to get married soon, and states that she has been trying to lose weight for her upcoming wedding. A urine pregnancy test is negative. The patient is given an intramuscular injection of progesterone. Ten days later she reports that she has had no withdrawal bleeding. Laboratory studies show the following serum levels: a. Acromegaly b. Cushing's syndrome c. Hypopituitarism d. Primary ovarian disease e. Weight loss syndrome f. Addison's disease

c. Sports drink Sports drinks contain electrolytes and carbohydrate that set up an osmotic gradient in the intestine to speed absorption of water to replace fluids lost during sweating.

A 25yearold female medical student from out of state wants to drive home over the 4th of July holiday. She hates to use public facilities and would prefer to minimize her urinary flow rate. Unfortunately, the air conditioning in her car is broken so she knows that she will have to consume 23 L of fluids to replace that lost due to sweating. What is her best choice to drink during the 8hours to result in a minimum of rest room breaks? a. Tap water b. Coffee c. Sports drink d. Soda e. Sweetened tea f. Distilled water

b. Increased serum aldosterone concentration This patient is hypotensive secondary to fluid loss. To compensate for her fluid depletion, the RAAS system is activated. Her serum aldosterone levels will be elevated, promoting reabsorption of sodium and water from the filtrate. Aldosterone also promotes the excretion of potassium, and serum levels will decrease. ADH will also be released, and serum levels will be elevated. This will result in a reduction of urine volume. Atrial natriuretic peptide is released in response to increased atrial stretch due to increased blood pressure.

A 26-year old woman is brought to the emergency department because of a 4-day history of flu-like symptoms accompanied by vomiting following each attempt to eat or drink. Her temperature is 38.5°C (101.3°F), pulse is 93/min, respirations are 24/min, and blood pressure is 105/70 mm Hg. Physical examination shows no other abnormalities. Which of the following additional findings is most likely in this patient? a. Decreased serum ADH (vasopressin) concentration b. Increased serum aldosterone concentration c. Increased serum atrial natriuretic peptide d. Increased urine sodium and chloride concentrations e. Increased urine volume f. Increased serum potassium concentration

d. High ADH, high renin, high ANG II, high aldosterone This patient is severely dehydrated as a result of sweating and lack of adequate fluid intake. The dehydration markedly stimulates the release of ADH and renin secretion, which in turn stimulates the formation of ANG II and secretion of aldosterone.

A 26-year-old construction worker is brought to the emergency room with a change in mental status after working a 10-hour shift on a hot summer day (average outside temperature was 97 F). The man had been sweating profusely during the day but did not drink fluids. He has a fever of 102 F, heart rate of 140 beats/min, and blood pressure of 100/55 mm Hg in the supine position. On examination, he has no perspiration, appears to have dry mucous membranes, and is poorly oriented to person, place, and time. Assuming that his kidneys were normal yesterday, which of the following sets of hormone levels describes his condition, compared with normal? a. High antidiuretic hormone (ADH), high renin, low angiotensin II (ANG II), low aldosterone b. Low ADH, low renin, low ANG II, low aldosterone c. High ADH, low renin, high ANG II, low aldosterone d. High ADH, high renin, high ANG II, high aldosterone e. Low ADH, high renin, low ANG II, high aldosterone f. Low ADH, low renin, low ANG II, high aldosterone g. High ADH, high renin, high ANG II, low aldosterone

f. Carbon monoxide poisoning Carbon monoxide poisoning not only reduces the oxygen carrying capacity of the blood, but also shifts the oxyhemoglobin dissociation curve to the left, altering the affinity of hemoglobin for oxygen, resulting in less availability of oxygen to the tissues. At very high doses, carbon monoxide also diffuses into cells and binds to myoglobin, producing a functionally hypoxic state at the level of mitochondria despite oxygen delivery at the capillary level.

A 26-year-old man comes to the emergency room complaining of lethargy, headache, nausea and extreme, weakness. His skin has a cherry-red color. He has a normal arterial PO2, but a low venous PO2of 20 mmHg (normal is 40 mmHg). You believe this situation is caused by: a. Cyanide poisoning b. Absolute right-to-left cardiopulmonary shunt c. A right-shifted hemoglobin association d. Hypoventilation e. Tuberculosis f. Carbon monoxide poisoning

e. Nitrogen gas embolism The bends, also known as decompression sickness (DCS) or Caisson disease occurs in scuba divers or high altitude or aerospace events when dissolved gases (mainly nitrogen) come out of solution in bubbles. Nitrogen or any gas from a diver's air tank increases in pressure as a diver descends. For every 33 ft in ocean water, the pressure due to nitrogen goes up another 11.6 pounds per square inch. As the pressure due to nitrogen increases, more nitrogen dissolves into the tissues. The longer a diver remains at depth, the more nitrogen dissolves. Unlike the oxygen in the air tank a diver uses to swim underwater, the nitrogen gas is not utilized by the body and builds up over time in body tissues. The underlying cause of symptoms throughout the body is due mainly to nitrogen bubbles being released when the diver returns to sea level and blocking blood flow and disrupting blood vessels and nerves by stretching or tearing them. They may also cause emboliLinks to an external site., blood coagulation and the release of vasoactive compounds.

A 26-year-old man is scuba diving off the coast of Bermuda in water 30 to 60 feet deep when he develops problems with his air tank and must ascend quickly to the surface. One hour later, he has pain in the muscles and joints in his legs. Which of the following is most likely responsible for these symptoms? a. Deep venous thrombosis b. Disseminated intravascular coagulation (DIC) c. Fat embolization d. Hemorrhage into muscles and joints e. Nitrogen gas embolism f. Intracranial hemorrhage

e. It spreads inward to all parts of the muscle via the T-tubules Depolarization of the muscle fiber is essential for muscle contraction. The AP is transmitted to all of the fibrils along the t-tubules stimulating the release of Ca2+ from the SR. the AP of cardiac muscle is longer with a plateau phase.

A 26-year-old medical student is an avid bodybuilder who lifts weights approximately 2 to 3 hours per day. Which of the following best describes the action potential of skeletal muscle during his workout? a. It causes the immediate uptake of Ca2+ into the lateral sacs of the SR b. It has a prolonged plateau phase c. It is longer than the action potential of cardiac muscle d. It is not essential for muscle contraction e. It spreads inward to all parts of the muscle via the T-tubules f. It stimulates the binding of tropomyosin to the actin filament stimulating cross-bridge cycling g. It has a longer absolute refractory period than that of smooth muscle

A. a The fetal hemoglobin (HbF) in fetal erythrocytes has a higher O2 affinity than the Hb inside adult RBCs (HbA). This difference is crucial for the fetus, whose blood must extract O2 from maternal blood in the placenta. An increased O2 affinity is represented on the Hb-O2 dissociation curve as a left shift from normal. Of the given curves, only curve a represents a leftward shift from the normal curve.

A 26-year-old pregnant woman is diagnosed with placenta previa, which requires premature delivery of her fetus of 28-week gestation. A blood sample is taken from both the mother and the newborn infant for determination of the oxyhemoglobin saturation curve. If curve N in the figure below is the oxyhemoglobin saturation curve of the mother who has normal HbA, which of the curves is most likely obtained from the premature infant? A. a B. b C. c D. d E. e

c. The conduction velocity of its action potential As axon diameterincreases, the conduction velocity of action potentials increases because the internal resistance of the axoplasm is inversely related to the internal cross-sectional area of the axon.

A 26-year-old woman presents with unilateral facial weakness. She states that whenever she tries to close her eyes, the upper eyelid on the affected side rolls upward. Electromyography on the affected side shows evidence of axonal degeneration. Which of the following characteristics of an axon is most dependent on diameter? a. The magnitude of its resting potential b. The duration of its refractory period c. The conduction velocity of its action potential d. The overshoot of its action potential e. The activity of its sodium-potassium pump

a. Decreased bile acid pool size Steatorrhea is defined as excess loss of fat in the stool. Numerous pathophysiological situations can cause the loss of excess fat in the stool including a decrease in bile acid pool size, inactivation or decreased intraluminal concentration of pancreatic lipase in the small intestine, decreased intestinal absorptive surface area (such as occurs in celiac sprue), or inability to form chylomicrons (such as in abetalipoproteinemia). A decrease in bile acid pool size results in an increased delivery of fats into the colon, which in turn inhibits fat absorption and promotes water secretion.

A 26yearold man presents to the emergency room with a 48hour bout of diarrhea with steatorrhea. Which of the following best accounts for the appearance of excess fat in the stool? a. Decreased bile acid pool size b. Increased bile salt reabsorption c. Increased colonic microbiota d. Increased chylomicron formation e. Increased lipase secretion f. Decreased VLDL circulation

b. Maintenance of the bloodtestis barrier The Sertoli cells rest on a basal lamina and form a layer around the periphery of the seminiferous tubules. They are attached to each other by specialized junctional complexes that limit the movement of fluid and solute molecules from the interstitial space and blood to the tubular lumen, and thus form a bloodtestis barrier that provides an immunologically privileged environment for sperm maturation. Sertoli cells are intimately associated with developing spermatozoa and play a major role in germcell maturation. They secrete a variety of serum proteins and an androgenbinding protein into the tubular fluid in response to FSH and testosterone stimulation. Testosterone is synthesized and secreted by the interstitial Leydig cells. Estrogen is produced in small amounts by the Sertoli cells before puberty.

A 26yearold man with Klinefelter's syndrome has seminiferous tubule dysgenesis. Which of the following is a function of Sertoli cells in the seminiferous tubules? a. Expression of surface LH receptors b. Maintenance of the bloodtestis barrier c. Secretion of FSH into the tubular lumen d. Secretion of testosterone into the tubular lumen e. Synthesis of estrogen after puberty f. Synthesis of testosterone during fetal development

c. A decrease in plasma bicarbonate due to renal compensation for the respiratory alkalosis that existed prior to treatment A patient with asthmatic bronchitis with is likely tachypneic to compensate for the increased airway resistance. Tachypnea causes a decrease in PCO2 as more CO2 is breathed off. A decrease in PCO2 drives the blood pH up resulting in a respiratory alkalosis. To compensate for this, bicarbonate is excreted from the kidney to improve the acid-base balance.

A 27-year-old man presents to the emergency department with asthmatic bronchitis that started 3 days ago. He is given an aerosolized bronchodilator treatment which relieves his symptoms. Arterial blood gases following bronchodilator therapy demonstrate metabolic acidosis with a normal anion gap. Which of the following explains his acid-base status? a. A laboratory error b. A decrease in plasma chloride concentration, resulting from the chloride shift after the treatment restored alveolar ventilation c. A decrease in plasma bicarbonate due to renal compensation for the respiratory alkalosis that existed prior to treatment d. An increase in lactic acid secondary to the hypoxemia that existed prior to treatment e. An increase in citrate from the vehicle used in the bronchodilator preparation f. An increase in plasma bicarbonate synthesis to compensate for the metabolic acidosis that existed prior to treatment g. A decrease in plasma H+ concentration, resulting from the respiratory alkalosis that existed prior to treatment.

b. Hypocapnia While the patient is experiences all of the above listed conditions, the only one of these conditions that would directly alter the acid-base status of the patient is the Hypocapnia. Recall the bicarbonate equilibrium equation. This patient is having an exacerbation of his chronic asthma. During this attack, he is tachypneic and is blowing off CO2. The result is hypocapnia which initially drives the bicarbonate equilibrium to the left, creating a respiratory alkalosis. To compensate for this, there is an increase in excretion of bicarbonate by the kidneys producing the resultant metabolic acidosis.

A 27-year-old man presents to the emergency room with severe acute asthma. He reports that his symptoms began three days ago. Bronchodilator agents are administered and an arterial blood gas analysis reveals a non-anion gap metabolic acidosis. Which of the following conditions, all of which were present before he came to the emergency room, is responsible for the non-anion gap metabolic acidosis? a. Tachypnea b. Hypocapnia c. Wheezing d. Hypoxemia e. Hyperinflation f. Alveolar collapse

D. D Because ventilation ceases during the apneic periods, CO2 accumulates increasing its partial pressure, while there is less oxygen available lowering its partial pressure. The accumulation of CO2 drives the pH down.

A 27-year-old man with a sleep disorder enrolls in a research study. During sleep evaluation, he is noted to snore loudly and stop breathing for prolonged periods of 30 to 40 seconds. Toward the end of one of these apneic periods, arterial blood gas analysis is done. Which of the following sets of findings is most likely in this patient? A. A B. B C. C D. D E. E F. F G. G

a. A During increase in sympathetic output to maximal values, several changes occur. First the mean systemic filling pressure increase markedly, but at the same time the resistance to venous return increase. Venous return is determined by the formula: (mean systemic filling pressure-right atrial pressure)/resistance to venous return. During maximal sympathetic output, the increase in systemic filling pressure is greater than the increase in resistance to venous return. Therefore, in this formula the numerator has a much greater increase than the denominator. The result is an increase in the venous return.

A 30-year-old male is resting and his sympathetic output increases to maximal values. Which of the following sets of changes would be expected in response to this increase in sympathetic output? a. A b. B c. C d. D e. E f. F g. G h. H

c. It is the final event required for conversion of the transitional circulation in the newborn to the adult circulatory pattern Functional closure of the ductus arteriosus is the final event required for conversion of the transitional circulation to the adult circulatory pattern. Immediately after birth, flow through the ductus switches, with blood flowing from the aorta to the pulmonary artery, but this ceases when the ductus closes. The ductus arteriosus is a systemic blood vessel and thus it constricts in response to high oxygen tensions and dilates in response to hypoxemia. Ductal sensitivity to oxygen is age dependent, however closure of the ductus arteriosus due to progressive constriction may be delayed in premature infants.

A 28-year-old woman gave birth without complications 48 hours ago to a full-term 8-lb 12-oz boy. Which of the following best describes the functional closure of the ductus arteriosus? a. It causes blood to flow from the aorta into the pulmonary artery b. It is independent of gestational age c. It is the final event required for conversion of the transitional circulation in the newborn to the adult circulatory pattern d. It occurs due to hypoxic pulmonary vasoconstriction e. It precedes functional closure of the foramen ovale f. Its closure is dependent on the closure of the ductus venosus g. It occurs just prior to delivery

a. A Diarrhea causes loss of bicarbonate, which increases plasma hydrogen ion concentration leading to metabolic acidosis, as evidenced by a reduction in plasma bicarbonate and a reduction in pH. Respiratory compensation as a result of peripheral chemoreceptor stimulation by the increased arterial [H+] causes the arterial PaCO2 to be decreased. Diarrhea is a normal anion gap types of metabolic acidosis because the loss of bicarbonate is replaced by an increase in serum chloride concentration, with no increase in the concentration of unmeasured anions, which makes A the correct answer.

A 28yearold student goes to Cancun for spring break. After running on the beach the morning he was to return, he feels so thirsty that he takes a drink of water from a garden hose at the hotel. After his return, the man presents in the Student Medical Center reporting foulsmelling diarrhea and flatulence over the past 5 days. Stool cultures confirm and infection with the protozoan Giardia lambia. Which of the following arterial blood gases would be expected in this patient? a. A b. B c. C d. D e. E

d. Intragastric volume The initial rate of emptying varies directly with the volume of the meal ingested. Increasing the volume, fat content, acidity, or osmolarity of the lumen of the small intestine inhibits gastric emptying via neural, hormonal, and paracrine feedback.

A 29-year-old internal medicine resident has a breakfast buffet after a long night of call. The rate of gastric emptying increases with which of the following? a. Acidity of the duodenum b. Fat content of the duodenum c. Intraduodenal volume d. Intragastric volume e. Osmolality of duodenum f. Rate of gastric accommodation g. Intraduodenal pressure

a. Umbilical vein Oxygenated and nutrient-heavy blood from the mother is transferred to the fetus through the placenta via the umbilical vein. Unlike arteries in the mother, the umbilical arteries carry deoxygenated blood. The relatively large volume of well-oxygenated umbilical venous blood (~50% of CCO) returns to the lower body from the placenta but bypasses the largely nonfunctional liver by shunting through the ductus venosus—the fetal first shunt.

A 29-year-old woman presents at the OB/GYN office with breast tenderness. She's concerned she may have a lump because her mother had breast cancer. She reports that she cannot be pregnant because she just finished her menstrual period about 2 weeks ago. An assay for human chorionic gonadotropin (hCG) in her urine is positive, confirming pregnancy. In the developing fetus, which vessel has the greatest partial pressure of oxygen? a. Umbilical vein b. Umbilical arteries c. Aorta d. Ductus arteriosus e. Ductus venosus f. Foramen ovale g. Pulmonary capillaries

b. Left optic tract Cutting the optic nerve causes blindness in the ipsilateral eye. Cutting the optic chiasm causes heteronymous bitemporal hemianopia. Cutting the optic tract causes homonymous contralateral hemianopia. Cutting the geniculocalcarine tract causes homonymous hemianopia with macular sparing.

A 29-year-old woman presents at the ophthalmologist's office complaining of slowly progressive loss of vision. Based on the visual defect shown here, the ophthalmologist determines that the patient has a right-sided homonymous hemianopia (vision field loss on the right side of both eyes). She refers the patient to a neurologist who orders a CT scan of the head. The CT scan demonstrates a high-density, space-occupying lesion, which is compressing which area of the brain? a. Left optic nerve b. Left optic tract c. Left occipital cortex d. Optic chiasm e. Right lateral geniculate nucleus f. Right visual cortex g. Right optic nerve

d. AntiMullerian hormone deficiency AntiMullerian hormone (AMH) or Mullerianinhibiting substance (MIS) causes Mullerian degeneration in the male fetus. The uterus is derived from the Mullerian ducts and its development is independent of the ovaries.

A 29yearold man who is a construction worker is brought to the emergency department because of severe, penetrating wounds to the abdomen. An emergent exploratory laparotomy is performed and the wounds are sutured. Before the abdominal cavity is closed, a small but wellformed uterus is found. Which of the following abnormalities during fetal development best explains the presence of a uterus in this otherwise normal male? a. 5Alpha reductase deficiency b. Deficiency of testis determining factor c. Luteinizing hormone secreting tumor d. AntiMullerian hormone deficiency e. Testosterone deficiency f. Hypersecretion of follicle stimulating hormone

a. Acetylcholinesterase Neostigmine is a cholinesterase inhibitor. Antagonizing cholinesterase increases the concentration of ACh in the synaptic cleft. The patient has myasthenia gravis (MG) which is a disease characterized by a deficiency in Ach receptors. Treatment with acetylcholinesterase inhibitors to increase the concentration of ACh in the synapse helps patients with MG improve.

A 30-year-old man complains of progressive muscle weakness and double vision (diplopia) and some difficulty in swallowing solid foods. Neurologic studies show a neuromuscular conduction defect, and diagnostic studies with an intravenous injection of edrophonium are found to rapidly increase his muscle strength. When the patient is treated with neostigmine, his status improves. The most probable target molecule in this treatment is: a. Acetylcholinesterase b. Choline acetyltransferase c. Monoamine oxidase d. Muscarinic acetylcholine (ACh) receptor e. Nicotinic ACh receptor

c. Hyperpolarizing the membrane potential The amount of potassium leaking out of the cell depends on its membrane potential, its concentration gradient, and its membrane conductance. According to the Nernst equation, the electrical gradient for K+ is inward and the concentration gradient is outward. Hyperpolarizing the membrane makes the inside of the cell more negative and therefore makes it more difficult for potassium to flow out of the cell. Decreasing the extracellular potassium concentration would increase the flow of potassium out of the cell, as would increasing the permeability of the membrane to potassium. Altering the activity of the sodiumpotassium pump or the extracellular sodium concentration has no immediate effect on the flow of potassium across the membrane. Decreasing the activity of the sodium potassium pump will ultimately depolarize the membrane, however, resulting in an increased flow of potassium out of the cell.

A 2dayold infant starts having brief tonicclonic seizures throughout the day. His neurological function in between seizures is normal, and he has no other medical or neurological problems. The history reveals no readily apparent causes for the seizures, though the mother recalled that her first baby also developed seizures shortly after birth that only lasted for 2 weeks, with no subsequent episodes or developmental problems. Genetic analysis revealed a mutation of voltagegated K+ channels consistent with a diagnosis of benign familial neonatal seizures. Which of the following would cause an immediate reduction in the amount of potassium leaking out of a cell? a. Decreasing the extracellular potassium concentration b. Decreasing the extracellular sodium concentration c. Hyperpolarizing the membrane potential d. Increasing the permeability of the membrane to potassium e. Reducing the activity of the sodiumpotassium pump

c. Blood pressure increases and heart rate decreases If intracranial pressure is rapidly elevated, cerebral blood flow is reduced. The increase in intracranial pressure stimulates the vasomotor center and produces an increase of systemic blood pressure that may lead to a restoration of cerebral blood flow. The increased blood pressure induces bradycardia mediated by the baroreceptor reflex.

A 2yearold boy is mauled by a black bear while hiking with his family in the Appalachian Mountains. A claw puncture wound to the skull compressed the underlying brain tissue. Which of the following occurs in response to an increased intracranial pressure? a. Blood pressure and heart rate increase b. Blood pressure and heart rate decrease c. Blood pressure increases and heart rate decreases d. Blood pressure decreases and heart rate increases e. Blood pressure and heart rate remain constant f. Blood pressure remains constant and heart rate increases

c. Infusion of a balanced electrolyte solution Vomiting can cause a large loss of electrolytes which results in a substantial decrease in the plasma volume. Blood volume decreases and causes very low arterial pressure. Infusion of a balanced electrolyte solution is the appropriate therapy for fluid volume replacement.

A 30-year-old female enters the University Hospital Cleveland Medical Center Emergency Room with complaint of severe vomiting. She has pale skin, tachycardia, an arterial pressure of 80/50, and difficulty walking. What therapy do you recommend to prevent shock? a. Packed red cell infusion b. Administration of an antihistamine c. Infusion of a balanced electrolyte solution d. Infusion of a sympathomimetic (elicits sympathetic response) drug e. Administration of a glucocorticoid f. Rapid peritoneal hemodialysis

b. Respiratory acidosis The accumulation of CO2 or hypercapnia, such as by hypoventilation, produces a respiratory acidosis resulting in a pH decrease.

A 30-year-old female patient with hypercapnia that is not the result of compensation for an acid-base disturbance is seen in the clinic. Thus, this patient is suffering from: a. Normal acid-base balance b. Respiratory acidosis c. Respiratory alkalosis d. Metabolic acidosis e. Metabolic alkalosis

c. Folic acid Folate deficiency during the initial weeks of gestation is a risk factor for neural tube defects. A supplement of 400 μg of folate daily is recommended for women of gestational age. A mother with a history of having children with neural tube defects needs folic acid supplementation of 4000 μg/day. Neural tube defects result from a failure to close a portion of the neural tube or reopening of a region of the tube after closure. Myelomeningocele is an extension of CNS tissue through a vertebral column defect. The recurrence risk is about 5%.

A 30-year-old female with a past medical history of having a baby born with a myelomeningocele plans to become pregnant. She is concerned about the possibility of having another child with this condition and asks what she can do to help prevent the defect from occurring again. Which of the following dietary supplements would you recommend? a. Vitamin A b. Vitamin B12 c. Folic acid d. Iron e. Vitamin K f. Vitamin E

d. The work of breathing Surfactant is composed of a mixture of dipalmitoylphosphatidylcholine (DPPC), other lipids and proteins which reduces the surface tension (increases the compliance) of the lung, thereby reducing the work of breathing. Without surfactant, the respiratory muscles may not provide adequate ventilation, leading to hypercapnia and hypoxia.

A 30-year-old man develops adult respiratory distress syndrome (ARDS) after near-drowning due to a boating accident. Conventional mechanical ventilation and inhaled nitric oxide (NO) do not provide sufficient oxygenation. The partial pressure of arterial carbon dioxide (PaCO2) and fraction of inspired oxygen (FIO2) ratio as well as shunt fraction (Qs/Qt) improve greatly after bronchial instillation of surfactant. The improvements in respiratory function occurred because surfactant decreased which of the following? a. Bronchiolar smooth muscle tone b. Arterial bicarbonate concentration c. Lung compliance d. The work of breathing e. Functional residual capacity (FRC) f. Pulmonary vascular resistance

c. It has a lower glucose concentration than plasma. The CSF glucose concentration is normally 60% of the plasma glucose concentration, and under non-pathological conditions, this ratio changes proportionately in response to a rising or falling plasma glucose event with a 4‐hour lag time. The chart below summarizes concentrations of various solutes in the plasma and the CSF.

A 30-year-old man hits his head on the windshield in an automobile accident. MRI of the brain reveals enlarged ventricles, suggestive of normal pressure hydrocephalus. Lumbar puncture confirms normal intracranial pressure, and his symptoms are found to improve after removal of a volume of his CSF. Which of the following best describes the CSF? a. It is absorbed by the choroid plexus. b. It circulates in the epidural space. c. It has a lower glucose concentration than plasma. d. It has a higher protein concentration than plasma. e. Its absorption is independent of CSF pressure. f. It is produced in the subarachnoid space. g. It circulates outside of the blood brain barrier.

e. β-oxidation of fatty acids Conditions such as prolonged fasting, consumption of a low-carbohydrate diet, or untreated diabetes mellitus lead to the production of three water-soluble byproducts of incomplete FA oxidation, substances collectively known as ketone bodies: acetoacetate, β-hydroxybutyrate, and acetone. What the conditions have in common is the accelerated β-oxidation of fatty acids, which produces acetyl CoA faster than the citric acid cycle can consume it.

A 30-year-old man who has type 1 diabetes mellitus develops diabetic ketoacidosis (DKA). Which of the following biochemical reactions most likely provides the substrate that is necessary to synthesize ketoacids? a. Cholesterol synthesis b. Citric acid cycle c. Gluconeogenesis d. Glycolysis e. β-oxidation of fatty acids f. Protein catabolism g. Oxidative phosphorylation

e. Vitamin B12 (cobalamin) This question can be triaged without knowledge of the symptomology of deficiency in each of these nutrients. The bulk of folate, iron, protein, and calcium absorption takes place in the duodenum. Resection of the terminal ileum results in deficiency of cobalamin only.

A 30-year-old woman comes to the physician because of a 2- month history of unsteady gait and numbness of both legs. Eight years ago, she underwent resection of the terminal ileum because of severe Crohn disease. Physical examination shows mild spastic weakness. Sensation to pinprick, vibration, and fine touch is decreased in the upper and lower extremities. A deficiency of which of the following is the most likely underlying cause of these findings? a. Folic acid b. Iron c. Protein d. Calcium e. Vitamin B12 (cobalamin) f. Bile salts g. Vitamin D

a. Hypoxic hypoxia (hypoxemia) Alveolar hypoventilation (as evidenced by the higher than normal value of Paco2) is a type of hypoxic hypoxia or hypoxemia (as evidenced by the decreased PaO2). Anemic hypoxia is characterized by a decreased concentration of hemoglobin (anemia) or a reduction in the saturation of hemoglobin with oxygen (SaO2) expected for a given Pao2, as would occur in carbon monoxide poisoning or methemoglobinemia. Stagnant hypoxia is characterized by a decreased cardiac output; in this patient, cardiac output, calculated as (V ̇ O2/CaO2 - CvO2), is 5 L/min, which is normal. In histotoxic hypoxia, oxy gen extraction is impaired, and thus CaO2 - CvO2 would be less than normal and Svo2 would be greater than normal.

A 30yearold female patient is admitted to the Emergency Department with dyspnea, tachycardia, confusion, and other signs of hypoxia. The following laboratory data were obtained while the patient was breathing room air: PaO2 = 67 mmHg PaCO2 = 60 mmHg pH = 7.27 [HCO3-] = 26 mEq/L [Hb] = 15 g% SaO2 = 90% PvO2 = 30 mmHg SvO2 = 55% V ̇O2 = 350 mL/min CaO2CvO2 = 7 mL O2/100 mL Which of the following is the most appropriate classification of the patient's hypoxia? a. Hypoxic hypoxia (hypoxemia) b. Anemic hypoxia c. Stagnant (hypoperfusion) hypoxia d. Histotoxic hypoxia e. Carbon monoxide poisoning f. Emphysema

c. Conduction system defect The patient has hypertrophic cardiomyopathy (idiopathic hypertrophic subaortic stenosis), which is the most common cause of sudden cardiac death in young people. Because of asymmetric hypertrophy of the interventricular septum, blood flow is obstructed below the level of the aortic valve during systole. This produces a systolic ejection murmur. Conduction system abnormalities are responsible for fatal ventricular arrhythmia and sudden death. Sitting up reduces venous return to the heart (decreases preload) and intensifies the murmur (greater obstruction) while reclining increases venous return and decreases murmur intensity.

A 30yearold man with a family history of sudden cardiac death has a systolic murmur that increases in intensity when he is sitting up. An echocardiogram shows abnormal movement of the anterior mitral valve Y leaflet against an asymmetrically thickened interventricular septum. The mechanism for sudden cardiac death in this patient's family is most closely attributed to which of the following conditions? a. Acute myocardial infarction (MI) b. Congenital bicuspid aortic valve c. Conduction system defect d. Dissecting aortic aneurysm e. Mitral valve prolapse f. Aortic stenosis

d. Inadequate LH surge to luteinize the corpus luteum Although the patient produces enough LH to induce rupture of her follicles, there is not enough LH to luteinize the corpus luteum. If the patient produced high amounts of periovulatory LH, she would not suffer from inadequate luteinization.

A 31-year-old woman attempting to become pregnant after several months sees her gynecologist who diagnoses her with luteal insufficiency, even though ovulation does occur. Which of the following is most likely responsible? a. High serum levels of periovulatory luteinizing hormone (LH) b. Elevated levels of periovulatory gonadotropin releasing hormone (GnRH) c. Insufficient priming of the corpus luteum by follicle stimulating hormone (FSH) d. Inadequate LH surge to luteinize the corpus luteum e. High levels of follicular phase estrogen f. Continuous secretion of FSH

b. ST segment depression occurs ST segment and T wave changes can be indicative of cardiac ischemia. ST segment elevation and depression can indicate ischemia, while T wave inversion is another potential sign of cardiac ischemia. During the stress test, the patient will experience an increase in sympathetic stimulation due to the increase in physical activity. This will cause an increase in the patient's heart rate (tachycardia), which is non-pathological. The increase in cardiac output causes a corresponding increase in mean arterial blood pressure, which is also non-pathologic. A diastolic murmur is heard during diastole of the heart. Causes of diastolic murmurs include atrioventricular valve stenosis or aortic regurgitation. Causes of a widened QRS complex include right or left BBB, pacemaker, hyperkalemia, ventricular preexcitation as is seen in Wolff-Parkinson-White pattern, and a ventricular rhythm. Though not visualized in all EKGs, the U wave on its own is not pathological. It is best visualized in leads V2 and V3. Prominent U waves are indicative of hypokalemia, and are commonly seen in bradycardia. Prominence of the U wave increases as the heart rate decreases.

A 32-year-old female complains of intermittent chest discomfort that occurs most frequently when she drinks lots of coffee to stay up to meet deadlines at work. She is referred to cardiology for an exercise stress test to rule out cardiac ischemia as the cause for her angina. The test will be considered positive if: a. Mean arterial blood pressure increases b. ST segment depression occurs c. Tachycardia develops d. A diastolic murmur is heard e. The QRS complex widens f. The U wave appears

e. Increased plasma norepinephrine concentration After surgery, there are increased blood levels of norepinephrine and sympathetic activity to the gut, which decreases gut motility and bowel sounds. Increased sympathetic activity to the gut decreases motility and bowel sounds, not decreased sympathetic activity. Food in the stomach stimulates the gastrocolic reflex, which increases gut motility and bowel sounds. Increased parasympathetic activity to the gut increases motility and bowel sounds. Acetylcholine is a parasympathetic mediator and thus will increase motility and bowel sounds, not decrease them.

A 32-year-old female undergoes an appendectomy without complications, but has markedly reduced bowel sounds and an absence of flatus or defecation during the immediate postoperative period. Which of the following mechanisms best explains this decrease in bowel sounds? a. Decreased sympathetic activity to the gut b. Increased activity of the gastrocolic reflex c. Increased parasympathetic activity to the gut d. Increased plasma acetylcholine concentration e. Increased plasma norepinephrine concentration

c. Mast cells Mast cells are large granulocytes that contain granules filled with histamine and heparin. Histamine is the main chemical mediator responsible for these lesions. Eosinophils contain major basic protein which is toxic to parasites, some role in allergic reactions. Lymphocytes are agranular. Neutrophils have two types of granules (specific and azurophilic) that contain lysosomal enzymes, peroxidase, collagenase, and other enzymes capable of digesting foreign material. Platelets are cell fragments responsible for clotting.

A 32-year-old man reports that he was eating peanut butter cookies and began to feel itchy. He then developed a rash over most of his body. The lesions disappeared within 3 to 4 hours. He describes the rash as roughly circular, raised lesions with central clearing and red borders. The chemical mediator responsible for these lesions is derived from which of the following cells? a. Eosinophils b. Lymphocytes c. Mast cells d. Neutrophils e. Platelets f. Erythrocytes

d. Pancreatitis This patient is showing typical symptoms of cholelithiasis (gall stones): right upper quadrant pain after a high fat meal, nausea/vomiting. Significant risk factors for the development of cholelithiasis include pregnancy and obesity.

A 32-year-old pregnant woman in the third trimester is admitted to the emergency department because of severe right upper quadrant pain, nausea, and vomiting following a meal of chicken fried steak. Her blood pressure is 130/84 mm Hg, heart rate is 105 beats/min, and respirations are 30/min. Body mass index before pregnancy was 45 kg/m2. Physical examination shows abdominal guarding and diaphoresis. Serum bilirubin levels and white cell count are both normal. This patient is most likely suffering from which of the following conditions? a. Cholelithaisis b. Constipation c. Hepatitis d. Pancreatitis e. Peritonitis f. Peptic ulcer disease g. Gastroparesis

e. Postsynaptic nicotinic acetylcholine receptors on the motor end plate Pt has MG. Myasthenia gravis is now recognized to be an acquired autoimmune disorder in which the spontaneous production of anti-AChR antibodies results in progressive loss of muscle AChRs and degeneration of postjunctional folds.

A 32-year-old woman from the IT department presents to the employee health clinic late in the afternoon complaining of fatigue, muscular weakness, and double vision. She indicates that the symptoms have been getting worse over the past 2 months and that she gets worse the longer she works at the computer screen. Cranial nerve examination discloses impaired movement of the right eye and bilateral ptosis, which worsen with repetitive eye movements. An MRI of the chest shows enlargement of the thymus gland. The neuropathy of this clinical presentation is most likely caused by antibodies against which of the following? a. Acetylcholine b. Acetylcholinesterase c. Postsynaptic muscarinic acetylcholine receptors d. Presynaptic nicotinic acetylcholine receptors in autonomic ganglia e. Postsynaptic nicotinic acetylcholine receptors on the motor end plate

c. Excess calcium release from the SR during muscle contraction Malignant hyperthermia is a life-threatening increase in metabolic rate and body temperature attributed to a mutation of the ryanodine receptor in skeletal muscle. The ryanodine receptor or calcium release channel on the SR is normally opened when skeletal muscle is activated. The flow of calcium through the open ryanodine receptor binds to troponin and initiates muscle contraction. The metabolic activity accompanying muscle contraction can warm the body. If the mutated ryanodine receptor causes uncontrolled release of calcium from the SR, the body temperature can rise to levels that cause brain damage.

A 32-year-old woman undergoing an appendectomy develops malignant hyperthermia following halothane anesthesia. What changes in skeletal muscle cause body temperature to increase in this condition? a. An increase in the refractory period of the alpha-motoneurons b. inability of skeletal muscle cells to repolarize c. Excess calcium release from the SR during muscle contraction d. Production of endogenous muscle pyrogens e. Rapid repetitive firing of the presynaptic terminals of alpha-motoneurons f. A decrease in the absolute refractory period of the alpha-motoneurons g. Decreased potassium efflux from the potassium muscle cell

c. Glucagon-like polypeptide 1 (GLP-1) GLP-1 is a product of glucagon metabolism in the L cells of the lower intestine. GLP-1 has no definite biological activity by itself but is processed further by removal of its amino-terminal amino acid residues and the product, GLP-1 (7-36), is a potent stimulator of insulin secretion that also increases glucose utilization.

A 32-year-old woman who has been dieting for several weeks breaks down and eats half a pan of frosted brownies. Insulin secretion following a carbohydrate-rich meal is stimulated by which of the following? a. Cholecystokinin b. Gastrin c. Glucagon-like polypeptide 1 (GLP-1) d. Serotonin e. VIP f. Acetylcholine g. Secretin

d. Prolactin inhibits gonadotropin releasing hormone secretion by the hypothalamus Prolactin is a singlechain protein structurally homologous to growth hormone, which is secreted by the anterior pituitary and has the principal physiologic effects of lactation (i.e., milk production), decreased reproductive function, and suppressed sexual drive. Normal adult serum levels of prolactin are about the same or only slightly higher in females compared to males. Consistent with its role in lactogenesis, prolactin secretion increases during pregnancy. Suckling increases prolactin secretion, but milk ejection during suckling is due to oxytocin release. Prolactin inhibits reproductive function by inhibiting hypothalamic GnRH release and pituitary gonadotropin secretion. Hyperprolactinemia is the most common pituitary hormone hypersecretion syndrome in both males and females. Pregnancy and lactation are the most important physiological causes of hyperprolactinemia. Prolactinsecreting pituitary adenomas, hypothyroidism, and druginduced inhibition or disruption of dopaminergic receptor function are other common causes of hyperprolactinemia.

A 32yearold male taking chlorpromazine for his schizophrenia presents with diminished libido and decreased beard growth. His blood prolactin level of 75 μg/L confirms the presence of hyperprolactinemia. Which of the following statements about prolactin is correct? a. Normal adult serum levels of prolactin are much higher in women than men b. Prolactin causes milk ejection during suckling c. Prolactin inhibits the growth of breast tissue d. Prolactin inhibits gonadotropin releasing hormone secretion by the hypothalamus e. Prolactin inhibits gonadotropin secretion by the pituitary gland

e. Pancreatic enzyme secretion Inflammation or removal of the upper small intestine leads to a decrease in pancreatic and hepatobiliary function. The proximal small intestine contains a number of receptors that monitor the physical and chemical composition of the chyme emptied from the stomach. Stimulation of these receptors releases secretin which acts on pancreatic ductal cells to increase HCO3 secretion as well as cholecystokinin which acts on pancreatic acinar cells to increase pancreatic enzyme secretion. Stimulation of proximal small intestine receptors also activates neural reflexes which initiate pancreatic enzyme and bicarbonate secretion, stimulate gall bladder emptying, and provide feedback for inhibitory regulation of gastric function. Removal of these reflexes decreases pancreatic secretion and gallbladder emptying and increases gastric emptying and acid output.

A 32yearold man with AIDS presents with a fever, anorexia, weight loss, and GI bleeding. Physical examination reveals a palpable abdominal mass. Endoscopy and biopsy reveal a proximal smallbowel malignancy requiring surgical resection. Removal of proximal segments of the small intestine would most likely result in a decrease in which of the following? a. Basal acid output b. Maximal acid output c. Gastric emptying of liquids d. Gastric emptying of solids e. Pancreatic enzyme secretion f. Gastric accommodation

c. Increases beginning less than 2 weeks after fertilization Human chorionic gonadotropin (hCG) begins to appear in the maternal blood approximately 6 to 8 days following ovulation, upon implantation of the fertilized ovum in the endometrium. The secretion of hCG is essential to prevent involution of the corpus luteum and to stimulate secretion of progesterone and estrogens, which continues until the placenta becomes large enough to secrete sufficient quantities of those hormones. Following a peak at 7 to 9 weeks, hCG secretion gradually declines to a low level by 20weeks' gestation.

A 32yearold woman presents at her physician's office complaining of nausea and vomiting. The history reveals that her symptoms have been present for over a month and that they seem to be worse in the morning. A urine sample is taken and shows that the woman is pregnant. The plasma concentration of hCG: a. Increases steadily throughout pregnancy b. Increases beginning about 2 months after fertilization c. Increases beginning less than 2 weeks after fertilization d. Peaks 46 months after fertilization e. Decreases steadily throughout pregnancy f. Increases during the first 46 weeks of pregnancy, decreases during the next 612 weeks, then increases again

A. A The effects of a prolonged stay in space are similar to those of prolonged bed rest. Blood volume, red cell mass, muscle strength, and work capacity, and cardiac output all decrease.

A 35-year-old man travels to Mars. His exercise equipment malfunctions so that he is subjected to prolonged weightlessness without appropriate exercise. Which of the following sets of changes best describe the physiologic changes that occur in this man? A. A B. B C. C D. D E. E F. F

a. The patient's systemic vascular resistance was much greater than his pulmonary vascular resistance δP = Q x R The right and left sides of the heart have the same cardiac output (Q). Systemic resistance exceeds pulmonary resistance (R), as indicated by the aortic pressure (93 mm Hg) versus the pulmonary artery pressure (20 mm Hg).

A 33-year-old man complained about chest pain on exertion. He was referred to a cardiologist, who carried out a number of studies, including right- and left-sided catheterization. Among the data obtained during these studies were the findings that at the time of his initial examination the patient's mean aortic pressure was 93 mm Hg and his mean pulmonary artery pressure was 20 mm Hg. These findings can be explained as follows: a. The patient's systemic vascular resistance was much greater than his pulmonary vascular resistance b. The patient's aortic compliance was much greater than his pulmonary artery compliance c. The patient's left ventricular stroke volume was much greater than his right ventricular stroke volume d. The total cross-sectional area of the patient's pulmonary artery was much greater than the total cross-sectional area of the aorta e. The duration of rapid ejection phase of the patient's left ventricle exceeded the duration of the rapid ejection phase of the right ventricle

a. a. Pancreatic bicarbonate secretion Pancreatic bicarbonate secretion into the small intestine is essential for neutralization of gastric acid emptied into the small intestine. Unlike the gastric mucosal lining, the mucosal surface of the small intestine does not provide a significant endogenous defense mechanism against the insult of HCl. Upon delivery into the proximal small intestine, hydrogen ions stimulate the release of the hormone secretin from the intestinal wall, which in turn stimulates pancreatic bicarbonate secretion. In fact, the acid output of the stomach during a meal is matched equally by the pancreatic output of bicarbonate. Although the liver secretes bicarbonate and bile contains bicarbonate, the amounts are not sufficient for acid neutralization.

A 33yearold female, who has been taking large doses of NSAIDs for her menstrual cramps, presents with burning epigastric pain. The pain improves after eating a meal, suggesting a duodenal versus a gastric ulcer. Which of the following is the major factor that protects the duodenal mucosa from damage by gastric acid? a. a. Pancreatic bicarbonate secretion b. The endogenous mucosal barrier of the duodenum c. Duodenal bicarbonate secretion d. Hepatic bicarbonate secretion e. Bicarbonate contained in bile f. Absence of G cells in the duodenum

d. It stimulates production of somatomedins GH has several diabetogenic effects including increasing lipolysis and increasing gluconeogenesis. In the circulation, GH has a halflife of ~25 minutes. Somatostatin is a potent inhibitor of GH secretion while ghrelin appears to be involved in the postmeal stimulation of growth hormone secretion. GH provokes the secretion of another circulating factor that mediates the action of GH. Initially called sulfation factor because of how it was assayed, this intermediate was subsequently termed somatomedin because it mediates the somatic effects of GH. We now know that somatomedin is in fact two peptides resembling proinsulin and thus termed insulinlike growth factors 1 and 2.

A 33yearold major league baseball player takes human growth hormone to increase his performance. Which of the following best describes the action of human growth hormone? a. It decreases lipolysis b. It has a long halflife c. It inhibits protein synthesis d. It stimulates production of somatomedins e. Its secretion is stimulated by somatostatin and inhibited by ghrelin f. It decreases gluconeogenesis

e. Folic acid deficiency This patient is anemic as evidenced by the Hb<14 g/dL. White count is normal, suggesting a normal bone marrow. His red cells are larger than normal (normal MCV = 90). His lack of vegetable consumption suggests either a vitamin B12 or folic acid deficiency, however the body has sufficient stores of B12 to last 4-5 years, so B12 deficiency is not likely. The body only stores folic acid for 3-6 months, so 1 year without vegetables would result in folic acid deficiency.

A 34-year-old man with schizophrenia has had chronic fatigue for 6 months. He has a good appetite, but has refused to eat vegetables for 1 year because he hears voices saying that vegetables are poisonous. His physical and neurological examinations are normal. His hemoglobin level is 91. g/dL, leukocyte count is 10,000/mL3, and MCV is 122. Which of the following is the most likely diagnosis? a. Acute blood loss b. Sickle cell c. Aplastic anemia d. Hemolytic anemia e. Folic acid deficiency f. Von Willebrand's disease g. Hemophilia type A

C. C The oxygen carrying capacity of the blood is reduced in an anemic person, but the arterial PO2 and oxygen saturation of hemoglobin are both normal. The decrease in arterial oxygen content is compensated for by an increase in the extraction of oxygen from hemoglobin which reduces the PO2 of the venous blood. The unloading of oxygen at the tissue level is enhanced by increases in levels of 2,3DPG in an anemic patient because 2,3DPG causes a right shift of the oxygen hemoglobin dissociation curve.

A 34year woman is anemic with a blood hemoglobin concentration of 7.1 g/dL. Which of the following sets of changes has occurred in this woman, compared to normal? a. A b. B c. C d. D e. E f. F g. G

b. Insulinoma This patient is exhibiting signs and symptoms consistent with hypoglycemia. An insulinoma is an islet cell tumor (usually benign) that releases high and unregulated concentrations of insulin into the bloodstream. The elevated insulin promotes glucose uptake into the tissues and drives blood glucose levels to critically low levels.

A 34yearold woman is brought to the emergency department by her husband because of confusion for 2 hours. She is unconscious on arrival. Her husband says that she has a 1year history of episodes of nervousness, lightheadedness, and dizziness that resolve after she eats a meal. Physical examination shows no other abnormalities. Her serum glucose concentration is 25 mg/dL. After an intravenous infusion of 0.9% saline and glucose, she regains consciousness. Which of the following is the most likely diagnosis? a. Cushing syndrome b. Insulinoma c. Multiple endocrine neoplasia syndrome d. Pancreatic gastrinoma e. Pheochromocytoma f. Addison's disease

d. Inhibition of H+/K+ ATPase activity Omeprazole is a proton pump inhibitor that inhibits the activity of the of H+/K+ ATPase preventing acid secretion into the lumen of the stomach.

A 35-year-old woman is diagnosed with gastroesophageal reflux disease. Omeprazole is administered. Which of the following is the most likely mechanism of action of this drug? a. Blockade of gastrin receptors b. Blockade of H1 receptors c. Blockade of M3 receptors d. Inhibition of H+/K+ ATPase activity e. Inhibition of synthesis of gastrin f. Stimulation of somatostatin release g. Stimulation of acetylcholine release

b. Decreases the metabolic breakdown of acetylcholine This patient has myasthenia gravis with classic clinical features including fluctuating fatigue and weakness that improve after administration of acetylcholinesterase inhibitors. Inhibition of acetylcholinesterase prevents the breakdown of acetylcholine allowing for more of it to bind to its receptor on the post-synaptic membrane.

A 35-year-old woman presents at her ophthalmologist's office with a chief complaint of bilateral drooping eyelids. A complete history reveals that the woman has also been experiencing generalized fatigue and weakness that only improves if she takes frequent naps. The ophthalmologist suspects a synaptogenic basis for the ptosis and refers her to a neurologist. The patient is found to have circulating antibodies to nicotinic acetylcholine receptors on the motor endplate a drug is administered that increases the force of muscle contraction but causes bradycardia in the patient. What is the most likely mechanism of action of the drug? a. Decreases the concentration of calcium in the extracellular fluid b. Decreases the metabolic breakdown of acetylcholine c. Increases alpha motoneuron discharge rate d. increases the affinity of the skeletal muscle acetylcholine receptors to acetylcholine e. Increase the amount of acetylcholine released by alpha motoneurons f. Increases the concentration of acetylcholine released from pre-synaptic vesicles g. Decreases the number of acetylcholine receptors on the motor end plate

g. Folate This patient has a macrocytic anemia found with folate of vitamin B12 deficiency. Iron-deficiency anemia, the most common type of anemia, and glucose-6-phosphate deficiency, the most common metabolic disorder of red blood cells, are both associated with microcytosis (low MCV). Niacin (vitamin B3) and zinc deficiencies are causes of malabsorption. Niacin deficiency presents with pellagra.

A 35-year-old woman presents to her family physician's office with fatigue of at least 3 months' duration. Her explanation is that keeping up with her twin 4-year-oldss really tires her out, and she does not even have enough energy to make nutritious meals for her and her husband. Vital signs and ECG are normal, but a third heart sound is heard with auscultation and she is pale. Blood results are as follows: Hb 8 g/dL, hematocrit 30%, MCV 115 fL, WBC 8000/mL, platelets 200,000/mL. A deficiency of which of the following substances can most likely account for these findings? a. Calcium b. Vitamin D c. Iron d. Glucose-6-phosphatase e. Zinc f. Niacin g. Folate

c. Stimulation of osteoclast activity Sustained secretion of parathyroid hormone results in bone resorption through the indirect activation of osteoclasts. PTH binds to its receptor on osteoblasts and promotes the secretion of cytokines including RANK, MCSF, and IL6 which in turn increase the activity of the osteoclast. Bone resorption results in the release of calcium ion into the ECF leading to hypercalcemia.

A 35yearold man has an adenoma of the parathyroid gland, with increased serum concentrations of parathyroid hormone (PTH) and calcium. In this patient, PTH induces which of the following processes to cause hypercalcemia? a. Production of 25hydroxycholecalciferol b. Shift of Ca2+ from the intracellular to the extracellular fluid compartment c. Stimulation of osteoclast activity d. Suppression of renal production of 1,25dihydroxycholecalciferol e. Hypersecretion of renal inorganic phosphate f. Negative feedback on the PTHR1 receptor of chief cells

f. Pulses of GnRH, administered every 1 to 2 h by subcutaneous pump Pulsatile GnRH discharge elicits a corresponding pulsatile release of LH and FSH, continuous administration of GnRH—or intermittent administration of high doses of GnRH analogs—suppresses the release of gonadotropins. Clinical problems requiring upregulation of gonadotropin secretion, which leads to stimulation of the gonads, are therefore best treated by a pulsatile mode of GnRH administration. Females treated with exogenous gonadotropins or GnRH analogs— pulsatile administration with a programmed infusion pump—can have normal folliculogenesis, ovulation, and pregnancy.

A 35yearold woman desperately wants one child, but has isolated hypogonadotropic hypogonadism of hypothalamic origin (all pituitary hormones and her ovaries and reproductive tract are normal, but FSH and LH secretion are minimal). With which one of the following could you treat her to induce a normal ovulation and pregnancy? a. Estrogen alone, given so it peaks twice each month b. Progesterone alone, given so it peaks once each month c. Estrogen, and perhaps progesterone, taken on a regular monthly schedule (two peaks of estrogen, one of progesterone) d. Daily shots of equal amounts of both FSH and LH every day each month e. Continuous administration of estrogen and progesterone f. Pulses of GnRH, administered every 1 to 2 h by subcutaneous pump

d. Pituitary microadenoma This patient is exhibiting signs consistent with Cushing's syndrome. These signs include signs including adiposity of the trunk, neck, and facies; hypertension; loss of subcutaneous adipose and connective tissue in the extremities with associated easy bruising; loss of bone mineral; muscle weakness and wasting; and hyperglycemia. To determine the source of the excess cortisol secretion, the dexamethasone suppression test is performed. In a subject with a normal hypothalamic-pituitary-adrenal (HPA) axis, a supraphysiological dose of dexamethasone can inhibit ACTH secretion by the pituitary leading to drop in cortisol level in serum, urine and saliva; such inhibition does not occur in patients with Cushing syndrome. In subjects with Cushing disease, a higher dose of dexamethasone (usually 8 mg) is required to suppress ACTH secretion and drop cortisol level in serum, urine, and saliva.

A 35yearold woman has hypertension and truncal obesity. Serum studies show: Which of the following is the most likely cause of this woman's increased serum cortisol concentration? a. Adrenocortical adenoma b. Ectopic corticotropinreleasing hormone producing neoplasm c. Ectopic corticotropinsecreting neoplasm d. Pituitary microadenoma e. Selfadministration of synthetic glucocorticoids

d. Low urinary sodium concentration The student is dehydrated and will, therefore, be retaining sodium to maintain his extracellular fluid (ECF) volume.

A medical student complains of severe vomiting and diarrhea for 2 days since eating at the Roadkill Café. His blood pressure is 120/70 mmHg supine and 90/50 mmHg standing. His skin and mucous membranes are dry. Which of the following laboratory finding would you expect to find? a. Low serum aldosterone concentration b. High atrial natriuretic peptide levels c. Low plasma renin activity d. Low urinary sodium concentration e. High urine volume

a. Estrogens and progesterone The placenta produces all of the hormones listed in the five answers at various times during pregnancy but it is the production of progesterone and estrogens (estradiol and estriol) from maternal and fetal precursors, which take over the function of the corpus luteum after the sixth week of pregnancy.

A 35yearold woman presents at her obstetrician/gynecologist's office for her annual wellwoman examination. She reports that she may have "a touch of the flu" because she has been tired and nauseated the past week and also has had fleeting episodes of lower abdominal pain. She wasn't sure when her last menstrual period started but after looking at a calendar, realized that it had been 38 days. A right adnexal mass was palpated on routine pelvic examination and subsequently observed on ultrasound. Urinalysis confirmed that the woman was pregnant and serum levels of the tumor marker CA125 were elevated. The gynecologist informed the woman that she may have to have her ovary removed, but that they need to wait another week or two to do the laparotomy in order to protect her fetus. Ovariectomy before the sixth week of pregnancy leads to abortion, but thereafter has no effect on pregnancy because the placenta secretes adequate amounts of which of the following hormones? a. Estrogens and progesterone b. Estrogen and relaxin c. Progesterone and human chorionic gonadotropin (hCG) d. Human chorionic somatomammotropin (hCS) and hCG e. Growth hormone releasing hormone (GnRH) and corticotropin releasing hormone (CRH) f. Oxytocin and prolactin

c. Increased pressure within the afferent arterioles Juxtaglomerular cells are sensitive to changes in afferent arterial intraluminal pressure. Increased pressure in the afferent arteriole leads to a decrease in renin release, whereas decreased pressure leads to an increase.

A 36-year-old man presents with low renin essential hypertension. Renin release from the juxtaglomerular apparatus is normally inhibited by which of the following? a. Aldosterone b. Beta-adrenergic agonists c. Increased pressure within the afferent arterioles d. Prostaglandins e. Stimulation of the macula densa f. Sympathetic stimulation

d. Production of antibodies against nACHRs Pt has myasthenia gravis. Giving an acetylcholinesterase inhibitor leaves more Ach in the synaptic cleft to improve symptoms.

A 36-year-old woman is evaluated because of a two-week history of fatigue and occasional blurry vision. She works as a third-grade teacher and reports to generally feeling fine while at work. But when she returns home from work, she fatigues easily and may experience blurred vision. Neurologic examination shows normal strength and reflexes, but when asked to quickly look upward from a downward position after initial over-elevation, her eyelids droop. This finding disappears after the administration of edrophonium. Which of the following best explains the process by which this patient's disease produces symptoms of muscle weakness? a. Thyroid-stimulating hormone receptor of the thyroid gland is overstimulated by an autoantibody resulting in excess thyroid hormone production, thyroid cell growth, and ultimately fatigue and weakness b. Production of autoantibodies against P/Q-type calcium channels at the motor nerve terminals c. Ingestion of a toxin that binds to presynaptic nerve cells and prevents the release of neurotransmitters d. Production of antibodies against nACHRs

c. Increased pressure in the afferent arterioles Juxtaglomerular cells are sensitive to changes in afferent arterial intraluminal pressure. Increased pressure within the afferent arteriole leads to a decrease in renin release, whereas decreased pressure tends to increase renin release. Angiotensin appears to inhibit renin release by initiating the flow of calcium into the juxtaglomerular cells. Renin release is increased in response to increased activity in the sympathetic neurons innervating the kidney. Prostaglandins, particularly PGI2 and PGE2 stimulate renin release. Stimulation of the macula densa leads to an increase in renin release, and although the mechanism is not fully understood, it appears that increased delivery of NaCl to the distal nephron is responsible for stimulating the macula densa. Aldosterone does not appear to have any direct effect ton renin release.

A 36yearold African American man presents with low renin essential hypertension. Renin release from the juxtaglomerular apparatus is normally inhibited by which of the following? a. Aldosterone b. Betaadrenergic agonists c. Increased pressure in the afferent arterioles d. Prostaglandins e. Stimulation of the macula densa f. Increased sodium chloride concentration in the distal tubule

c. Restrict water intake and treat with arginine vasopressin (AVP) and see if urinary volume declines Central diabetes insipidus is confirmed when the patient does not concentrate his urine with water deprivation, but does respond to AVP treatment by decreasing urine output.

A 36yearold male patient with a recent head injury complains of polyuria. You suspect central diabetes insipidus. How could you confirm the diagnosis? a. Restrict water intake and treat with insulin and see if urinary volume is decreased b. Restrict water intake and treat with angiotensin II and see if urinary volume is decreased c. Restrict water intake and treat with arginine vasopressin (AVP) and see if urinary volume declines d. Restrict water intake and treat with a loop diuretic to determine if urinary flow rates decrease e. Compare urinary production rates following consumption of 3 L of water and 3 L of isotonic saline to determine whether they differ f. Restrict water intake and treat with aldosterone to see if urinary volume increases

c. Vitamin C Because we cannot synthesize vitamin C, we rely on dietary sources such as citrus fruits, leafy green vegetables, and tomatoes to obtain necessary stores of the vitamin. Although rare in the United States, scurvy may be seen in patients with poor dietary intake (homeless, poor, elderly). Scurvy leads to features of defective collagen synthesis and vascular wall weakness. Purpura and ecchymoses, especially in the oral mucosa and lower legs may be seen. Bleeding into the joint spaces (hemarthroses) and intracranial hemorrhages are severe consequences.

A 37-year-old homeless woman presents to the emergency room and is noted to have bleeding gums. She has several ecchymoses over his shins and forearms. A complete blood count shows hemoglobin of 11 g/dL and an elevated mean corpuscular volume (MCV). Which vitamin deficiency is responsible for his symptoms? a. Vitamin A b. Vitamin B12 c. Vitamin C d. Vitamin D e. Vitamin E f. Vitamin K

e. E Aldosterone increases Na+ reabsorption from the principal cells of the cortical and medullary collecting ducts.

A 42-year-old man presents with fatigue, loss of stamina, and frequent urination. He is not taking any medicaitons currently. Physical examination is normal except for a blood pressure of 165/95 mm Hg. Serum electrolytes show sodium, 152 mEq/L; potassium, 3.1 mEq/L, chloride, 112 mEq/L; and bicarbonate, 32 mEq/L. Aldosterone concentration is elevated and plasma renin activity is low, consistent with primary hyperaldosteronism. Alodsterone increases Na+ reabsorption at which point in this schematic diagram of the nephron? a. A b. B c. C d. D e. E

a. Hemoglobin's affinity for oxygen is increased P50 is the PO2 at which Hb is half saturated. A decreased P50 indicates Hb is becoming saturated at lower values of PO2. This is an indication of an increased Hb affinity for oxygen.

A 37-year-old man presents with low exercise tolerance. Blood work shows a normal hematocrit and hemoglobin concentration but a decreased P50. Which would be true of his oxyhemoglobin transport and dissociation? a. Hemoglobin's affinity for oxygen is increased b. O2 loading at the alveolar-capillary level is less than normal c. O2 saturation is lower than normal at any PaO2 d. O2 unloading is increased at the tissue level e. The differential diagnosis includes a point mutation resulting in increased binding of H+ to the Hb chains f. Carbon dioxide production at the tissues is increased g. Carbon dioxide offloading at the alveolar-capillary level is greater than normal

c. Sodium Increased Sodium conductance causes increased depolarization.

A 37-year-old woman presents with severe migraine headaches that are accompanied by hemiparalysis. Genetic analysis confirms the suspicion of an inherited channelopathy. The membrane potential will depolarize by the greatest amount if the membrane permeability increases for which of the following ions? a. Chloride b. Potassium c. Sodium d. Chloride and potassium e. Sodium and potassium

f. Increased by 16-fold

A 37-year-old woman undergoes a CT scan of the abdomen, which reveals a large peritoneal mass. A subsequent magnetic resonance angiography study showed that the abdominal aorta was constricted to one-half of its resting diameter. As a result, resistance to blood flow through the vessel would be which of the following? a. Decreased by 16-fold b. Decreased half c. Constant d. Increased by half e. Doubled f. Increased by 16-fold

f. Increase by 16-fold Poiseuille's law indicates that flow is directly proportional to the fourth power of vessel radius, but indirectly proportional to the fourth power of vessel radius. As radius decreases, resistance increases by the fourth power of the magnitude of vessel change.

A 37-year-old woman undergoes a CT scan of the abdomen, which reveals a large peritoneal mass. A subsequent magnetic resonance angiography study showed that the abdominal aorta was constricted to one-half of its resting diameter. As a result, resistance to blood flow through the vessel would be which of the following? a. Decreased by half b. Decreased 16-fold c. Increased by half d. Doubled e. Increased by 8-fold f. Increase by 16-fold

b. Isovolumetric relaxation The second heart sound is caused by closing of the aortic and pulmonary valves which occurs during isovolumetric relaxation.

A 38-year-old man has a murmur that ceases with the onset of the second heart sound. The second heart sound occurs at the onset of which phase of the cardiac cycle? a. Isovolumetric contraction b. Isovolumetric relaxation c. Rapid ejection d. Rapid ventricular filling e. Systole

f. Isovolumetric relaxation The second heart sound occurs at the onset of isovolumetric relaxation. On the pressure volume loop, the second heart sound is heard at point 7. This is immediately following ejection when the aortic valve closes, and isovolumetric relaxation begins.

A 38-year-old man has a murmur that ceases with the onset of the second heart sound. The second heart sound occurs at the onset of which phase of the cardiac cycle? a. Isovolumetric contraction b. Rapid ejection c. Systole d. Closing of the mitral valve e. Rapid ventricular filling f. Isovolumetric relaxation

b. Anterior pituitary Tumors of the somatotropes of the anterior pituitary gland secrete large amounts of growth hormone, leading to acromegaly in adults. When the epiphyses have not yet fused to the long bones, growth is stimulated by excess growth hormone leading to gigantism in children. Once the epiphyses have closed, linear growth is no longer possible, and growth hormone produces the pattern of bone and soft tissue abnormalities typical of acromegaly. Hypersecretion of growth hormone is accompanied by hyper secretion of prolactin in up to 40% of patients with acromegaly. Human growth hormone also has intrinsic lactogenic activity. Acromegaly can be caused by hypothalamic tumors that secrete growth hormone releasing hormone (GHRH), but these are rare.

A 39yearold man with an enlarged head, hands, and feet, osteoarthritic vertebral changes and hirsutism presents with a complaint of gynecomastia and lactation. The patient is most likely suffering from a tumor in which of the following locations? a. Hypothalamus b. Anterior pituitary c. Posterior pituitary d. Adrenal cortex e. Breast tissue f. Testicle

a. Chromaffin The hypertension in the absence of other physiological findings as well as the elevated excretion of epinephrine and metanephrine suggest over production of catecholamines which are produced in the chromaffin cells of the adrenal medulla.

A 39yearold woman comes to the physician for a followup examination because she recently was diagnosed with hypertension. Her blood pressure is 156/100 mm Hg. Physical examination shows no other abnormalities. Serum studies show normal findings. A 24hour urine collection shows three times the normal excretion of epinephrine and metanephrine. The excessive epinephrine production in this patient is most likely caused by which of the following cell types? a. Chromaffin b. Juxtaglomerular c. Zona fasciculata d. Zona glomerulosa e. Zona reticularis f. Corticotrophs

b. Increased maximal acid output Inflammation of the proximal small intestine results in a decrease in the feedback regulation of gastric function by reducing the input of the enterogastric reflex and enterogastrone to gastric emptying and gastric acid secretion. Absent inhibitory input, basal and maximal acid output are increased, and the gastric emptying of liquids and solids is increased.

A 42-year-old airline pilot presents to his family physician with a chief complaint of midepigastric pain that is relieved by antacids or eating. Endoscopic evaluation reveals the presence of a duodenal ulcer. Based on the diagnosis, which of the following also would be expected? a. Decreased basal acid output b. Increased maximal acid output c. Decreased gastric emptying of liquids d. Increased frequency of antral contractions e. Decreased orad (proximal) stomach compliance f. Enhanced gastric emptying of solids

d. 21aHydroxylase Ambiguous genitalia in genotypic females may result from disorders of adrenal function. Several forms of congenital adrenal hyperplasia have been described, including the deficiency of several enzymes involved in steroid synthesis: the sidechaincleavage enzyme, 17αhydroxylase, 21αhydroxylase, 11βhydroxylase, and 3βhydroxysteroid dehydrogenase. Deficiencies in 21αhydroxylase, 11βhydroxylase, and 3βhydroxysteroid dehydrogenase all lead to virilization in females—and thus ambiguous genitalia—as a result of the hypersecretion of adrenal androgens. 21α hydroxylase deficiency, by far the most common, accounts for ~95% of cases. 21αhydroxylase deficiency reduces the conversion of progesterone to 11deoxycorticosterone—which goes on to form aldosterone—and also reduces the conversion of 17α hydroxyprogesterone to 11deoxycortisol—which is the precursor of cortisol. As a result, adrenal steroid precursors are shunted into androgen pathways.

A 3monthold infant presents with ambiguous genitalia. Physical examination reveals hypotension and dryness of the mucous membranes. Laboratory studies show a decrease in serum cortisol and an increase in plasma adrenocorticotropic hormone (ACTH). A random urine sodium is markedly increased. A chromosome study shows an XX genotype. The patient most likely has a deficiency of which of the following enzymes involved in adrenal steroid synthesis? a. 11Hydroxylase b. 17bHydroxylase c. 18aHydroxylase d. 21aHydroxylase e. Oxidoreductase f. Cortisone

g. Pneumocytes Infants born prematurely (before 32 weeks' gestational age) experience severe respiratory distress because of structurally immature lungs that lack of surfactant. Type II pneumocytes are responsible for the production of surfactant.

A 4 hour old female newborn delivered at 30 weeks' gestation has respiratory distress. Her temperature is 36.5°C (97.7°F), pulse is 160/min, respirations are 85/min, and blood pressure is 68/40 mm Hg. Arterial blood gas analysis on room air shows: pH: 7.18, PCO2: 78 mm Hg, PO2: 55 mm Hg. Endotracheal intubation and mechanical ventilation are required. The primary cause of this patient's condition is a dysfunction of which of the following cell types? a. Alveolar macrophages b. Pulmonary erythrocytes c. Pulmonary chondrocytes d. Pulmonary vascular endothelial cells e. Pulmonary vascular smooth muscle cells f. Parietal pleural endothelial cells g. Pneumocytes

d. Cortical collecting tubule ADH increases the water permeability of the kidney tubule in the segments distal to the distal convoluted tubule. AVP dramatically increases the water permeabilities of the collecting tubules (ICT and CCT) and ducts (OMCD and IMCD) by causing AQP2 water channels to insert into the apical membrane. high levels of AVP cause substantial water reabsorption to occur in AVP-sensitive nephron segments. Because water is reabsorbed, they filtrate in the cortical collecting tubule will be hypertonic and isotonic to the plasma.

A 4-year-old boy on vacation with his family is rushed to the emergency room after accidentally ingesting approximately 2L of seawater. He is confused and complains of being "light-headed." He is also extremely thirsty. Lab results reveal a plasma osmolarity of 350 mOsm and significantly elevated levels of anti-diuretic hormone (ADH). In the presence of ADH, the filtrate in the kidney will be isotonic to plasma in which of the following? a. Afferent arteriole b. Descending limb of the loop of Henle c. Ascending limb of the loop of Henle d. Cortical collecting tubule e. Renal pelvis f. Efferent arteriole

c. Uncompensated Metabolic acidosis This patient has a reduced PCO2 (normal 33-45 mm Hg) and a reduced serum bicarbonate (normal 22-28 mEq/L). The reduced PCO2 would drive the blood pH up, creating a compensated respiratory alkalosis. To compensate, bicarbonate is excreted, lowering its plasma concentration.

A 40-year old woman is admitted to the intensive care unit with hypotension and shortness of breath. Arterial blood gasses reveal a PCO2 of 10mmHg and a bicarbonate concentration of 12 mEq/L. These findings are indicative of which of the following acid-base states? a. Normal b. Compensated Respiratory acidosis c. Uncompensated Metabolic acidosis d. Compensated Respiratory alkalosis e. Uncompensated Metabolic alkalosis f. Uncompensated Respiratory alkalosis g. Uncompensated Metabolic alkalosis

c. Lipid solubility A hydrophobic hormone must diffuse through the phospholipid bilayer of the cell to exert its effects. Its lipid solubility will affect the degree to which the hormone is able to diffuse.

A 40yearold male with acute promyelocytic leukemia is started on treatment with alltrans retinoic acid (ATRA). Which of the following characteristics of a hydrophobic hormone that binds to nuclear receptors is most important in governing its diffusibility through a cell membrane? a. Diameter b. Electrical charge c. Lipid solubility d. Molecular weight e. Threedimensional shape f. Receptor type

d. Respiratory alkalosis The arterial pH can be calculated using the HendersonHasselbalch equation. pH = 6.1 + log [HCO3-] / (PaCO2 × 0.03 mmol/L/mmHg) pH = 6.1 + log 12 mmol/L / (10 mmHg × 0.03 mmol/L/mmHg) pH = 6.1 + log 40 = 6.1 + 1.6 = 7.7 The patient has an alkalemia due to hyperventilation and therefore is suffering from a respiratory alkalosis. In an acute respiratory alkalosis, the bicarbonate typically decreases by 2 mM for each 10 mmHg decrease in PCO2; in a chronic respiratory alkalosis, the bicarbonate typically decreases 4 mM for each 10 mmHg decrease in PCO2. In this case the PCO2 has decreased by 30 mmHg. Because bicarbonate has decreased by 12 mM, the diagnosis is consistent with a chronic respiratory alkalosis.

A 40yearold woman is admitted to the intensive care unit with hypotension and shortness of breath. Arterial blood gases reveal: PaCO2, 10 mmHg, and bicarbonate, 12 mEq/L. Which of the following is her acidbase status? a. Normal b. Respiratory acidosis c. Metabolic acidosis d. Respiratory alkalosis e. Metabolic alkalosis

a. Cholecystokinin (CCK) This woman has a risk profile (female, fat, forties) and symptomatology consistent with gallstones (cholelithiasis). Contraction of the gallbladder following a fatty meal often exacerbates the pain caused by gallstones. The release of CCK is stimulated by dietary fat, and protein and is the hormone responsible for stimulation of gallbladder contraction. In addition to gallbladder contraction, CCK also stimulates pancreatic enzyme secretion, causes the pancreas to get larger (grow) in size and decreases the rate of gastric emptying. Gastrin produced by the G cells of the gastric antrum and duodenum, stimulates gastric acid and pepsinogen secretion and gastric mucosal growth. Gastrin secretion is stimulated by gastric distention, protein digestive products (e.g. amino acids), and vagal discharge, but not dietary fat. Pepsin is a protease produced by the chief cells of the stomach that begins gastric digestion of protein, but does not cause gallbladder contraction. Pepsinogen release is stimulated by vagal stimulation, gastrin, local acid production, secretin, CCK, and histamine. Pepsinogen is converted to its active form pepsin by gastric acid. Somatostatin is produced by the D cells of the pancreatic, gastric and intestinal mucosa and is not associated with pain in the right upper quadrant. Secretin is produced by the S cells of the duodenum in response to acidification of the duodenal mucosa. Secretin stimulates the secretion of bicarbonate-containing fluid from the pancreas and biliary ducts to provide the right environment for enzymes to function. Secretin also inhibits gastric acid production and gastric emptying, but does not cause gallbladder contraction. Somatostatin is a universal inhibitory hormone that inhibits the secretion of most gastrointestinal hormones, gallbladder contraction, gastric acid and pepsinogen secretion, pancreatic and small intestinal fluid secretion, and both glucagon and insulin release.

A 42-year-old obese woman experiences episodic upper right quadrant abdominal pain. She notes that the pain increases after the ingestion of a fatty meal. The action of which of the following hormones is responsible for the postprandial intensification of her symptoms? a. Cholecystokinin (CCK) b. Gastrin c. Pepsin d. Secretin e. Somatostatin f. Histamine

c. Gastrin Hypersecretion of gastric acid is caused by an increase in plasma levels of gastrin. This patient has Zollinger-Ellison (ZE) syndrome. On rare occasions, patients with one or more ulcers have very high rates of gastric acid secretion. The increased acid secretion in these patients is most often a result of elevated levels of serum gastrin, released from a pancreatic islet cell adenoma or gastrinoma.

A 42-year-old salesman presents with the chief complaint of intermittent mid-epigastric pain that is relieved by antacids or eating. Gastric analysis reveals that basal and maximal acid output exceed normal values. The hypersecretion can be explained by an increase in the plasma concentration of which of the following? a. Somatostatin b. Histamine c. Gastrin d. Secretin e. Enterogastrone f. Acetylcholine g. Serotonin

e. von Willebrand disease Pt has a normal platelet count which allows us to rule out aplastic anemia, vitamin B12 deficiency, and NSAID overdose. The most likely cause is an inherited platelet disease. von Willebran disease is the most common inherited bleeding disorder. Hemophilia is an X-linked recessive trait that results in a deficiency in Factor VIII. Females are generally carriers with 50% of normal levels of Factor VII but no bleeding problems.

A 42-year-old woman presents to her doctor's office with heavy menstrual bleeding for up to two weeks' duration for each of the past five cycles. She also reports that she has a tendency to bruise easily, and has had several episodes of epistaxis (nosebleed) over the past couple of months. Blood analysis shows: Hb 8g/dL; Hematocrit 24%, MCV 70, platelet count 230,000/microliter. Which of the following is a likely cause of her bleeding disorder? a. Aplastic anemia b. Hemophilia A c. Nonsteroidal anti-inflammatory drug overdose d. Vitamin B12 deficiency e. von Willebrand disease f. Vitamin K deficiency

c. Left ventricular end-diastolic volume LVEDV = preload

A 42-year-old woman with mitral prolapse is admitted to the hospital for evaluation of her cardiac function. Which of the following values is the best index of the preload on her heart? a. Blood volume b. Central venous pressure c. Left ventricular end-diastolic volume d. Pulmonary capillary wedge pressure e. Left ventricular end-systolic pressure f. Cardiac output

b. Fat induced release of cholecystokinin from the small intestine The delivery of food into the small intestine is characterized by prompt emptying of the gallbladder, resulting from fatinduced release of cholecystokinin. Secretin stimulates pancreatic bicarbonate secretion. Enterogastrone is inhibitory to gastric function; glucagon is involved in nutrient metabolism; motilin is an interdigestive hormone responsible for migrating motor complex activity.

A 42yearold obese female presents to the Emergency Department with right upper quadrant pain, nausea, and vomiting. The pain is not related to food intake and lasts for several hours before resolving slowly. Ultrasound images are suggestive of gallstones with cystic duct obstruction. Which of the following is the primary physiological stimulus of gall bladder contraction in the digestive period? a. Acidinduced release of secretin from the small intestine b. Fat induced release of cholecystokinin from the small intestine c. Calorie induced release of enterogastrone from the small intestine d. Distension induced release of glucagon from the small intestine e. Amino acid induced release of motilin from the small intestine f. Protein induced release of somatostatin from the small intestine

d. Chelating calcium The citrate ion has three anionic carboxylate groups that avidly chelate calcium and reduce the concentration of free calcium in blood. Because free calcium (Ca2+) is required for multiple steps in both coagulation pathways, citrate is a useful anticoagulant in vitro. The citrate ion is rapidly metabolized; thus, blood anticoagulated with citrate can be infused into the body without untoward effects. Oxalate, another calciumchelating anticoagulant, is toxic to cells.

A 42yearold patient with a rare blood type is scheduled for surgery that will likely require a transfusion. Because the patient has a rare blood type, an autologous blood transfusion is planned. Prior to surgery, 1500 mL of blood is collected. The collection tubes contain calcium citrate, which prevents coagulation by which of the following actions? a. Blocking thrombin b. Binding factor XII c. Binding vitamin K d. Chelating calcium e. Activating plasminogen f. Blocking fibrinogen

b. Antidiuretic hormone (ADH) The supraoptic nucleus (SON) is a nucleus of magnocellular neurosecretory cells in the hypothalamus of the mammalian brain. The nucleus is situated at the base of the brain, adjacent to the optic chiasm. The cell bodies produce the peptide hormone vasopressin, which is also known as anti-diuretic hormone (ADH). This chemical messenger travels via the bloodstream to its target cells in the papillary ducts in the kidneys, enhancing water reabsorption. In the cell bodies, the hormones are packaged in large, membrane-bound vesicles that are transported down the axons to the nerve endings in the posterior pituitary gland.

A 43-year-old man develops a brain tumor that impinges on the supraoptic nucleus in the hypothalamus. As a result, the secretion of which of the following hormones is affected? a. Adrenocorticotropic hormone (ACTH) b. Antidiuretic hormone (ADH) c. Follicle-stimulating hormone (FSH) d. Growth hormone e. Prolactin f. Luteinizing hormone (LH)

c. Glutamate In long-term potentiation, glutamate released from the presynaptic terminal activates NMDA receptor channels, which allow Ca2+ to enter the postsynaptic cell.

A 43-year-old pregnant woman develops preeclampsia at 32 weeks' gestation. Intravenous labetalol is given to reduce blood pressure, and magnesium sulfate, which blocks N-methyl-D-aspartate (NMDA) receptors in the central nervous system, is ordered for the prevention of eclamptic seizures until the fetus can be delivered. Which of the following activates the NMDA receptor? a. Acetylcholine b. Gamma-aminobutyric acid (GABA) c. Glutamate d. Glycine e. Kainate f. Aspartate g. D-serine

a. At low lung volumes compared with high lung volumes One of the most powerful determinants of airway resistance (RAW) is lung volume. RAW is extremely high at residual volume (RV) but decreases steeply as lung volume (VL) increases. One reason for this effect is obvious: all pulmonary airways—including the conducting airways, which account for virtually all of RAW—expand at high VL, and resistance falls steeply as radius increases. A second reason is the principle of interdependence; alveoli tend to hold open their neighbors by exerting radial traction or mechanical tethering. This principle is especially important for conducting airways, which have thicker walls than alveoli and thus a lower compliance. Greatest aggregate resistance is in the pharynx-larynx and large airways (diameter > 2 mm, or before about generation 8.

A 43-year-old woman with a history of asthma presents to the emergency department with an acute asthma attack after her bronchodilator inhaler ran out the day before. Airway resistance is greater at which of the following? a. At low lung volumes compared with high lung volumes b. At lower values for Reynolds number c. During inspiration compared with expiration d. In the total cross-section of the small airways compared with the total cross-section of the central airways e. With laminar flow compared to turbulent flow f. In the lower airways compared with upper airways g. With sympathetic stimulation of bronchial smooth muscle compared to parasympathetic stimulation

e. Glutamate The NMDA receptor channel is a large channel permeable to Ca2+ K+ and Na+ It is activated by glutamate, but unlike other glutamate receptor channels, the NMDA channel is blocked by Mg2+in its resting state. Depolarization of the cell membrane to approximately -40 mV removes the Mg2+blockade. Therefore, the NMDA channel is only opened when the cell is depolarized by other excitatory neurotransmitters. Preeclampsia is the development of high blood pressure during pregnancy. Other signs of preeclampsia include protein in the urine and severe edema (or swelling). The only treatment for preeclampsia is delivery of the baby, which may mean a premature birth. Magnesium sulfate is the treatment of choice for preventing and treating eclamptic seizures, demonstrating superiority over phenytoin and diazepam in two large clinical trials.

A 43yearold pregnant woman develops preeclampsia at 32 weeks gestation. Magnesium sulfate, which interacts with NMDA receptors in the central nervous system, is ordered for the prevention of eclamptic seizures until the fetus can be delivered. Which of the following activates the NMDA receptor? a. Glycine b. Acetylcholine c. Substance P d. Histamine e. Glutamate f. Serotonin

b. Glomerular filtration rate (GFR) in the affected kidney will decrease The presence of a partial urinary tract obstruction results in a marked decline in GFR on the affected side.

A 44 year-old male presents to the emergency department with severe back pain radiating to the left flank. A radiological exam reveals a kidney stone partially blocking the ureter of one kidney. Based on your knowledge of renal physiology, which of the following statements is correct? a. The patient will have problems with salt and water balance due to a loss of 50% of renal function b. Glomerular filtration rate (GFR) in the affected kidney will decrease c. Urinary flow will be reduced by approximately 50% d. Hydrostatic pressure in the tubules will be decreased e. Renal failure will occur f. Tubular secretion of calcium and phosphate from the affected kidney will increase

d. Increase the plasma levels of angiotensin II ANF released in response to increased atrial stretch (high BP). ANG II released by the kidneys as a sympathetic response to hemorrhage.

A 44-year-old man is attempting to shave for the first time with a straight razor. He loses focus for a few seconds and accidentally severs his carotid artery. His wife calls 911 and the man is rushed to the hospital after considerable hemorrhaging. A 25% loss of blood volume is likely to: a. Increase unstressed volume b. Decrease the plasma levels of vasopressin c. Increase tissue interstitial pressure d. Increase the plasma levels of angiotensin II e. Increase the plasma levels of ANF f. Decrease the levels of renin

e. Testosterone and dehydroepiandrosterone (DHEA) sulphate Excess hair growth in females is due to androgens. As this patient has no signs of Cushing's syndrome, androgen levels produced by the ovaries and adrenals should be obtained first. As the patient shows no signs of hypercortisolism, cortisol measurement would be of little benefit. T4 levels should be measured in conjunction with thyroid binding globulin or free thyroxine if thyroid involvement is suspected. Estrogen and progesterone levels are not indicated in cases of hirsutism. Hirsutism is due to excess androgen, not increased estrogen and progesterone. Gonadotropin measurements are helpful only after androgen (testosterone) levels are measured first to differentiate between ovarian and hypothalamic pituitary disease. Measuring 17-alpha-hydroxyprogesterone is used to diagnose congenital adrenal hyperplasia and would not useful in the diagnosis of the cause of hirsutism described in this case.

A 44-year-old woman complains of increased 'hairiness' that has developed over her face and body over the past 2 years. Physical exam reveals increased hair growth over the upper lip, chin, chest, and abdomen, but no truncal obesity or purple striae. Laboratory values of which hormones would provide the most pertinent diagnostic information? a. Cortisol and thyroxine (T4) b. Estrogen and progesterone c. Follicle stimulating hormone (FSH) and luteinizing hormone (LH) d. 17-alpha-hydroxyprogesterone e. Testosterone and dehydroepiandrosterone (DHEA) sulphate f. Oxytocin and testosterone

c. Inhibits H+ -K + ATPase Several isoforms of the H-K pump are present in the kidney and exhibit differential sensitivities to inhibition by drugs such as omeprazole.

A 45-year-old man is diagnosed with gastroesophageal reflux disease. Omeprazole treatment is initiated. Which of the following is the mechanism of action of this drug? a. Binds to H2 receptors b. Creates a cytoprotective layer in the stomach c. Inhibits H+ -K + ATPase d. Prevents absorption of dietary acids e. Stimulates serotonin receptors f. Increases secretin secretion from S cells of the duodenum g. Inhibits histamine release from ECL cells

d. Lipase A large amount of fat appearing in the stool indicates an impairment in fat digestion and absorption. Lipase is secreted by the pancreas to aid in fat digestion. Chronic pancreatitis results in decreased secretion of pancreatic enzymes, resulting in insufficient lipase secretion.

A 45-year-old man with chronic pancreatitis has a 9-kg (20- lb) weight loss and diarrhea. Analysis of a 24-hour stool sample shows 28 g of fat. A deficiency of which of the following enzymes is the most likely cause? a. Amylase b. Carboxypeptidase c. Lactase d. Lipase e. Lipoprotein lipase f. Sucrase g. Trypsin

b. Gastrin Severe peptic ulcer disease in the absence of predisposing factors such as NSAID use or H. pylori infection should raise a suspicion for Zollinger-Ellison syndrome (also known as gastrinoma). In this syndrome gastrin-secreting gut tumors lead to hypergastrinemia and gastric acid hypersecretion.

A 45-year-old woman complains of episodic epigastric pain. She denies fevers, chills, diaphoresis, flushing, chest pain or shortness of breath. An upper endoscopy, reveals severe and atypical peptic ulcer disease. Biopsy results are Helicobacter pylori negative and she denies taking NSAIDs. Continued workup should include serum levels of which of the following? a. Serotonin b. Gastrin c. Somatostatin d. Vasoactive intestinal peptide (VIP) e. Cholecystokinin (CCK) f. Histamine

e. Increased osmolarity This patient is exhibiting signs and symptoms consistent with diabetes. To best answer this question, you need to determine from the question stem whether this patient has type I or type II diabetes. His age coupled with the more recent onset of symptoms suggest that this is type II diabetes over type I and answer choice D can be eliminated. His urine is negative for ketones so it is unlikely that he would have an elevated serum concentration of betahydroxybutyrate, there is also not enough evidence to suggest that he would have a metabolic disturbance that would elevate the anion gap such as acidosis. Decreased concentration of Cpeptide suggests reduced insulin secretion, which is not the dysfunction seen in type II diabetes. Increased serum osmolality is the best answer choice, given the elevated fasting serum glucose level.

A 45yearold man comes to the physician because of increased frequency of urination. He voids approximately 12/day and 3/night. He describes the inability to maintain an erection but reports no decrease in libido. Physical examination shows that he is oriented to person and place, but not to time. He is 177 cm (5 ft 10 in) tall and weighs 109 kg (240 lb); BMI is 34 kg/m2. He has an elevated waisttohipratio. Cardiac auscultation reveals an audible S1 and S2, with a regular rate and rhythm. The lungs are clear. There are no palpable abdominal or scrotal masses. Urinalysis shows 3+ glucose and is negative for ketones. Fasting serum glucose is 195 mg/dL and the fasting insulin level of 78 mIU/L (normal: <25mIU/L). Which of the following blood laboratory findings in serum is most likely in this patient? a. Decreased concentration of Cpeptide b. Elevated anion gap c. Elevated concentration of betahydroxybutyrate d. Increased antibodies against islet cell proteins e. Increased osmolarity

d. Hypokalemia Aldosterone produces hypokalemia by increasing renal K+ excretion. Hypokalemia causes muscle weakness by hyperpolarizing the cell membrane and preventing the membrane potential from reaching threshold. Excess aldosterone induces metabolic alkalosis characterized by increased plasma HCO3−concentration and decreased H+ and Cl- in the extracellular fluid. Thus, hyperaldosteronism is associated with hypochloremia not hyperchloremia. Increased aldosterone levels produce hypokalemia, not hyperkalemia. Aldosterone does not affect plasma Ca2+ concentrations or cause hypocalcemia.

A 46-year-old man complains of progressive muscle weakness. Laboratory studies show a serum pH of 7.6, HCO3−of 32 mEq/L, and an elevated plasma aldosterone concentration. The most likely cause of muscle weakness in this patient is: a. Hyperchloremia b. Hyperkalemia c. Hypocalcemia d. Hypokalemia e. Hyponatremia f. Hyperphosphatemia

a. A higher than normal flow of hypotonic urine Pt has central diabetes insipidus. The mass in his pituitary is inhibiting him from secreting ADH and as a result he is producing more urine that is hypotonic in nature.

A 46-year-old man presents to his physician with a 12-week history of frontal headaches. A computed tomography (CT) of the brain shows a mass in the posterior pituitary. The patient also complains of increased thirst and waking up frequently during the night. Which of the following best describes his urine? a. A higher than normal flow of hypotonic urine b. A higher than normal flow of hypertonic urine c. A normal flow of hypertonic urine d. A lower than normal flow of hypotonic urine e. A lower than normal flow of hypertonic urine f. A normal flow of isotonic urine

c. ACTH, cortisol, and norepinephrine In response to emotional stress, ACTH is rapidly released from the anterior pituitary and quickly stimulates the release of cortisol. Sympathetic stimulation results in the rapid release of norepinephrine. Norepinephrine is found in many somatic tissues in amounts that roughly parallel the extent of sympathetic innervation of the tissue. In other words, the norepinephrine in these other tissues is not made there but is derived from the sympathetic nerve endings in them. Epinephrine however must be synthesized from norepinephrine in the adrenal medulla, and as a result its secretion is delayed.

A 46yearold man has lost innervation to both his adrenal glands. Otherwise, his nervous system is intact and functioning normally, and his adrenals have adequate circulation. Late one afternoon, he receives a telephone message that one of his children has been seriously injured. Which of the following hormones will increase quickly in his plasma? a. ACTH, cortisol, epinephrine, and norepinephrine b. ACTH, epinephrine, and norepinephrine c. ACTH, cortisol, and norepinephrine d. Only ACTH and norepinephrine e. Only ACTH f. None of the above

c. Aortic dissection The patient's description is classic for aortic dissection with pain to the left arm and radiating into the left area of the back. Marfan's syndrome is suggested by his extreme height.

A 47-year-old male arrives in the emergency department with acute anterior chest pain, which radiates to the left arm and back and has lasted for 1 h. He is 86 in tall and weighs 190 lbs. His blood pressure is 115/70 mmHg, pulse 116/min, and respiratory rate 20/min. The lungs are clear and an electrocardiogram (ECG) shows no abnormalities. Which of the following is the most likely diagnosis? a. Gastroesophageal reflux disease b. Myocardial infarction (MI) c. Aortic dissection d. Cervical disc disease e. Pericardial tamponade f. Angina pectoris

d. Increased sympathetic activation The renal sympathetic nerves are able to enhance renin release from granular cells during times when there are changes to volume homeostasis. Prolonged essential hypertension causes damage to the kidney tubule that alters effective circulating volume, which in turn increases the sympathetic activation of the kidney resulting in increased plasma renin.

A 47-year-old woman comes to the physician because of dizziness and a headache. Two years ago, she was diagnosed with essential hypertension. Laboratory studies show an increased plasma renin activity with normal aldosterone/renin ratio. Her increased plasma renin activity is most likely due to which of the following? a. Decreased sympathetic activation b. Increased Cl- delivery to the macula densa c. Increased Na+ delivery to the macula densa d. Increased sympathetic activation e. Primary hyperaldosteronism f. Increased parasympathetic activation g. Increased Na+ reabsorption from the kidney

a. A decreased pulse pressure Aortic stenosis is a disease that commonly shows up on the NBME. Aortic stenosis is the obstruction of blood flow through the aortic valve due to reduced opening (stiffening) of the valve. As a result, less blood leaves the heart and enters the systemic circulation. This will cause a reduced pulse pressure (a hallmark of the disease). Pulse pressure is the difference between systolic and diastolic pressure (SBP-DBP= PP). Less blood leaving the heart for the systemic circulation will cause a reduced arterial pressure. More blood in the heart after contraction results in an increased LV diastolic pressure. Ejection fraction is typically reduced, but may be preserved until late stages of the disease. Though there is less blood flow through the systemic circulation, the heart continues to extract its normal load of oxygen from the blood to maintain its normal function. As the primary disorder is a reduction in blood flow, there is a reduction in stroke volume. Remember the equations for EF and SV. SV=EDV-ESV, EF=SV/EDV *100%. Reduced SV combined with an increased EDV results in a reduced EF.

A 47-year-old woman is brought to the emergency department because she fainted at the gym during her daily aerobic workout. A prominent systolic murmur is heard and a presumptive diagnosis of aortic stenosis is made. Which of the following is consistent with that diagnosis? a. A decreased pulse pressure b. An increased arterial pressure c. A decreased left ventricular diastolic pressure d. An increased ejection fraction e. A decreased cardiac oxygen consumption f. An increased stroke volume

d. Na+ reabsorption by the distal nephron decreases Factors that decrease excretion of potassium: Decreased tubular flow, decreased aldosterone levels, decreased potassium intake, decreased sodium reabsorption, increased excretion of organic ions, decreased renin plasma levels.

A 47-year-old woman presents for her annual physical examination. One year ago the patient was started on a diet and exercise regimen when her BP was 130.85 mmHg. She has lost 10 pounds and reduced her BMI since then but her BP on this visit was found to be 145/98 mmHg. The patient is started on a combination of a low dose of hydrochlorothiazide accompanied by a K+ sparing diuretic. The amount of potassium excreted by the kidney will decrease if which of the following occurs? a. Distal tubular flow increases b. Circulating aldosterone levels increase c. Dietary intake of potassium increases d. Na+ reabsorption by the distal nephron decreases e. The excretion of organic ions decreases f. Circulating renin levels increase

d. Na+ reabsorption by the distal nephron decreases Factors that decrease excretion of potassium: Decreased tubular flow, decreased aldosterone levels, decreased potassium intake, decreased sodium reabsorption, increased excretion of organic ions, decreased renin plasma levels.

A 47-year-old woman presents for her annual physical examination. One year ago the patient was started on a diet and exercise regimen when her BP was 130.85 mmHg. She has lost 10 pounds and reduced her BMI since then but her BP on this visit was found to be 145/98 mmHg. The patient is started on a combination of a low dose of hydrochlorothiazide accompanied by a K+ sparing diuretic. The amount of potassium excreted by the kidney will decrease if which of the following occurs? a. Distal tubular flow increases b. Circulating aldosterone levels increase c. Dietary intake of potassium increases d. Na+ reabsorption by the distal nephron decreases e. The excretion of organic ions decreases f. Circulating levels of renin increase g. Circulating aldosterone levels remain constant

a. Replacement of the squamous epithelium of the esophagus with columnar epithelium Barrett's esophagus appears when the squamous epithelium of the esophagus is replaced by brownish metaplastic columnar epithelium extending up from the stomach in a tonguelike or circumferential fashion. Barrett's esophagus is present in up to 10% of patients with chronic reflux and is associated with an increased risk of neoplasia.

A 47yearold woman with a longstanding history of heartburn complains of severe heartburn. You make the diagnosis of gastroesophageal reflux disease (GERD). An upper endoscopy study is positive for Barrett's esophagus. Which of the following correctly describes Barrett's esophagus? a. Replacement of the squamous epithelium of the esophagus with columnar epithelium b. Replacement of the columnar epithelium of the esophagus with squamous epithelium c. The presence of moderatetohigh grade dysplasia in the esophagus d. The presence of lowtomoderate grade dysplasia in the esophagus e. The presence of any stricture within the esophagus f. Increased pressure at the lower esophageal sphincter

a. Abnormal sweat chloride test An abnormal sweat chloride test is an expected diagnostic feature of cystic fibrosis (CF), which is what this patient has. The chloride channel is thought to be regulated by the CF transmembrane regulator (CFTR) protein, which is defective in CF and causes elevated sodium and chloride in sweat.

A 5-year-old boy has a history of growth retardation, pulmonary infections, and bulky, oily, malodorous stools. Which of the following test results would be expected in this patient? a. Abnormal sweat chloride test b. Low C3 complement level c. Abnormal nitroblue tetrazolium (NBT) dye test d. Positive wheal and flare reaction with antigen scratch testing e. Sputum with Gram-positive diplococci f. Leukocytosis and thrombocytopenia

d. Iron is transported in the blood bound to transferrin Iron is transported in the blood bound to the betaglobulin, transferrin. Excess iron is stored in all cells, but especially in hepatocytes where it combines with apoferritin. The stored form is called ferritin. The rate of iron absorption is extremely slow, with a maximum of only a few milligrams per day. Iron is absorbed primarily in the ferrous form. Therefore, ferrous iron compounds, rather than ferric compounds, are effective in treating iron deficiency.

A 47yearold woman with hypermenorrhea develops an irondeficiency anemia requiring iron supplements. Which of the following best describes iron digestion and absorption? a. About 100 mg of iron is absorbed per day b. Iron is rapidly absorbed from the small intestine c. Iron is transported into enterocytes by a ferroportin transporter on the apical membrane d. Iron is transported in the blood bound to transferrin e. Iron is oxidized from the ferrous to the ferric state during absorption f. Iron absorption is dependent on normal levels of hemoglobin

f. Glucose into skeletal muscle and fat cells Glucose transport into skeletal muscle cells occurs passively, down the concentration gradient of glucose. Though glucose transport into the cell requires a GLUT transporter, the transporters facilitate the diffusion of glucose, which is a passive process. The transport of all of the other molecules given requires the use of a pump or active transporter of some kind, indicating an active, uphill process.

A 48-year-old executive was referred for a life insurance physical examination for his new corporation. His body mass index was 34, indicating clinical obesity, and his blood pressure was 145/92 mm Hg. Blood tests showed hyperlipidemia and hyperglycemia with normal insulin levels, consistent with type 2 diabetes mellitus (T2DM). T2DM adversely affects many cellular processes. Which of the following transport processes is a passive downhill process? a. Calcium into the sarcoplasmic reticulum (SR) b. Potassium into kidney tubule epithelial cells c. Hydrogen into the lumen of canaliculi of the parietal cells of the stomach d. Phosphate into epithelial cells lining the proximal tubule of the kidney e. Sodium out of neurons f. Glucose into skeletal muscle and fat cells g. Chloride out of erythrocytes

c. Anterior infarction Within hours after an acute myocardial infarction of the anterior ventricle, ST segment elevation appears in leads I, aVL, and the left precordial leads, V3-6. Reciprocal ST depression occurs in leads II, III, and aVF. After some days or weeks, the ST segment abnormalities subside, the dead muscle and scar tissue become electrically silent. The infarcted area is therefore negative relative to the normal myocardium during systole, and it fails to contribute its share of positivity to the ECG complexes. Manifestations of this negativity include the appearance of Q waves and failure of progression of the R wave in the precordial leads.

A 48-year-old man develops chest pain while running. His wife takes him to the Emergency Department, where the following ECG is obtained. The electrocardiographic changes are consistent with a diagnosis of which of the following? a. Hyperkalemia b. Hypokalemia c. Anterior infarction d. Posterior infarction e. Ventricular premature beat f. Atrial flutter

b. A decrease in the number of presynaptic neurotransmitter vesicles

A 48-year-old man has hepatic cancer that is unresponsive to standard therapy. He enrolls in a clinical study of a novel chemotherapeutic agent that, as a side effect, blocks kinesin, a component of the cellular microtubular transport system. One week later, he develops skeletal muscle weakness. An alteration in which of the following components of the neuromuscular junction is the most likely cause of the muscle weakness? a. A decrease in the number of postsynaptic neurotransmitter receptors b. A decrease in the number of presynaptic neurotransmitter vesicles c. A decrease in the presynaptic neuron calcium permeability d. Impaired α-motoneuron action potential conduction e. Impaired skeletal muscle action potential conduction

c. A decrease in core body temperature from 37 C to 32 C Decreasing the temperature causes the Hb-O2 dissociation curve to shift to the left, whereas increasing the temperature has the opposite effect. As the temperature increases, the amount of O2 bound to Hb decreases progressively. In other words, high temperature decreases the O2 affinity of Hb and leads to release of O2. Low temperature increases the O2 affinity of Hb and leads to retention of oxygen. Increasing the PCO2, lowering the pH, increasing 2,3-BPG, and increasing the body temperature all cause the curve to shift to the right. Transfusion with blood with an elevated P50 would also shift the curve to the right.

A 48-year-old man presents to the emergency department with chest pain and shortness of breath. His ECG shows ST-segment elevation and cardiac enzymes are elevated. After cardiac catheterization revealed 90% occlusion of his left anterior descending coronary artery, the patient is scheduled for coronary artery bypass graft surgery. During surgery, his oxygen saturation shifts from a to b, as shown in the figure below. Which of the following can best account for this shift? a. A change in PCO2 from 40 to 46 mm Hg b. A change in pH from 7.4 to 7.3 c. A decrease in core body temperature from 37 C to 32 C d. An increase in erythrocyte [2,3-bisphosphoglycerate] e. Transfusion of blood with a higher P50 than normal f. Vigorous exercise for 45 minutes g. An increase in core body temperature from 37 C to 40 C

b. Intestinal smooth muscle In intestinal smooth muscle, the upstroke of the action potential is caused by the flow of calcium into the cell. In cells of the cardiac ventricular muscle, the plateau phase of the AP, but not the upstroke is accompanied by the flow of calcium into the cells. Skeletal muscle fibers and nerve cells have the same activity. In both, the upstroke of the AP is caused by the flow of Na+ into the cell.

A 48-year-old woman with advanced breast cancer presents with severe nausea, vomiting, and dehydration. She is not undergoing chemotherapy currently, though was previously treated with 5-fluorouracil and doxorubicin. Laboratory findings reveal elevated serum-ionized calcium. Parathyroid hormone (PTH) levels are undetectable, but there is an increase in PTH-related peptide (PTHrP). The increased flow of calcium into the cell is an important component of the upstroke phase of the action potential in which of the following a. Cardiac ventricular muscle b. Intestinal smooth muscle c. Nerve cells d. Presynaptic nerve terminal e. Skeletal muscle f. Endothelial cells

c. The intact pleura causes the lungs and chest wall to recoil away from each other throughout a tidal breath When the pleura, and hence the lungchest wall system, are intact, the inward elastic recoil of the lung opposing the outward elastic recoil of the chest wall results in a subatmospheric (negative) pressure with the pleural space. When one reaches lung volumes in excess of ~70% of the total lung capacity, the chest wall recoil is also inward.

A 48yearold coal miner complains of shortness of breath and a productive cough. The history reveals that he has smoked 1 to 2 packs of cigarettes per day since he was 16 years old. Pulmonary function studies are ordered including an esophageal balloon study to measure intrapleural pressures. Normally, intrapleural pressure is negative throughout a tidal inspiration and expiration because of which of the following? a. The lungs have the tendency to recoil outward throughout a tidal breath b. The chest wall has the tendency to recoil inward throughout a tidal breath c. The intact pleura causes the lungs and chest wall to recoil away from each other throughout a tidal breath d. The intact pleura causes the lungs and chest wall to recoil in the same direction throughout a tidal breath e. There is always a small leak in the visceral pleura causing some air to escape into the pleural space during a tidal breath f. The diaphragm remains in a contracted state throughout the respiratory cycle

b. Conn syndrome Conn syndrome, also known as primary hyperaldosteronism is excess production of aldosterone by the adrenal glands resulting in very low renin levels. The excess of aldosterone causes highly elevated levels of serum sodium, low potassium levels, and resulting plasma volume expansion. The increase in blood pressure increases GFR and causes a decrease in renin. Addison's disease, also known as primary adrenal insufficiency and hypocortisolism, is a long term disorder in which the adrenal glands do not produce enough steroid hormones. Addison's disease arises from problems with the adrenal gland such that not enough of the steroid hormone cortisol and possibly aldosterone is produced. Addison's can be eliminated based on the hypertension and elevated aldosterone levels. Cushing's syndrome is a collection of signs and symptoms due to prolonged exposure to cortisol. Though cortisol can exhibit mineralocorticoid activity, the postprandial glucose of 110 mg/dL suggests that this patient is not insulin resistant and prolonged cortisol exposure is unlikely. Congenital adrenal hyperplasia due to 21 hydroxylase deficiency can cause reduction of aldosterone and cortisol levels depending on the severity of the deficiency. Pheochromocytoma is a tumor of the medulla of the adrenal gland that results in high amount of catecholamines. Adrenocortical carcinoma may present differently in children and adults. Most tumors in children are functional, and virilization is by far the most common presenting symptom, followed by Cushing's syndrome and precocious puberty. Among adults presenting with hormonal syndromes, Cushing's syndrome alone is most common, followed by mixed Cushing's and virilization (glucocorticoid and androgen overproduction). Feminization and Conn occur in less than 10% of cases. Rarely, hypersecretion of catecholamines has been reported in adrenocortical cancers.

A 48yearold man comes to the physician for a routine examination. He reports occasional headaches, feeling tired and having muscle weakness. His temperature is 37.2 C, pulse 76/min, respirations are 13/min, and blood pressure is 165/95 mm Hg. Physical examination is normal except for obesity. Laboratory studies show hematocrit is 33%, sodium is 1148 mEq/L, potassium is 2.9 mEq/dL, creatinine is 1.0 mg/dL, and 2hour postprandial glucose is 110 mg/dL. Plasma renin activity is nearly undetectable, and serum aldosterone is increased. Which of the following is the most likely underlying condition? a. Addison's disease b. Conn syndrome c. Cushing syndrome d. 21hydroxylase deficiency e. Pheochromocytoma f. Adrenocortical carcinoma

b. Hypoglycemia Cortisol is the prototypical naturally occurring glucocorticoid. The ability of cortisol to increase plasma [glucose] largely results from its ability to enhance mobilization of amino acids from proteins in many tissues and to enhance the ability of the liver to convert these amino acids into glucose and glycogen by activating gluconeogenesis. In liver, cortisol induces the synthesis of enzymes involved in gluconeogenesis and aminoacid metabolism in support of gluconeogenesis, thus enhancing hepatic glucose production. In muscle, cortisol stimulates the breakdown of muscle protein, which releases amino acids for uptake by the liver. Similarly, cortisol promotes lipolysis in adipose tissue. Without these actions, blood glucose level remains low and hypoglycemia results. To compensate for the reduced production of glucose, glucose uptake will actually increase.

A 48yearold man with a history of Addison disease cannot produce adequate amounts of cortisol, either basal or stressinduced. Under basal conditions, his metabolism is not greatly affected, but he experiences severe weakness when confronted with a stressor. Laboratory tests performed during periods of stress will most likely show which of the following? a. Decreased glucose uptake in muscles and adipose b. Hypoglycemia c. Increased circulating amino acids d. Increased gluconeogenesis e. Increased lipolysis f. Hypokalemia

b. An increase in the gastric emptying rate of liquids The fundus of the stomach acts as a holding area for liquids during the gastric phase of digestion. Without the fundus, liquids pass more quickly through the stomach.

A 49-year-old female is referred to surgery for removal of a tumor from the fundic region of the stomach. Resection of the fundus will likely result in which of the following outcomes? a. A decrease in the contractile frequency of the stomach b. An increase in the gastric emptying rate of liquids c. A decrease in the gastric emptying rate of solids d. An increase in maximal acid output e. A decreased gastrin release in response to ingestion of a meal f. An increase in gastric emptying rate of solids g. An increase in accommodation of the stomach

d. It is as diffusible through the membrane as water The osmolality of a substance is the number of osmoles per kg of solvent. One osmole (Osm) equals the gram molecular weight of a substance divided by the number of free moving particles that each molecule liberates in solution. Osmotically active substances in the body are dissolved in water, and the density of water is 1. Thus, osmolar concentrations can be expressed as osmoles (or milliosmoles) per liter of water. If the osmolality is zero, there are no free moving particles, and thus, the molecule is as diffusible as water through the membrane.

A 49-year-old male in end-stage renal failure is able to perform peritoneal dialysis at home. The osmolality of the solution chosen for peritoneal dialysis will determine the rate of ultrafiltration. Which of the following statements best characterizes a molecule whose osmolality is zero? a. It will not permeate the membrane b. It can only cross the membrane through the lipid bilayer c. It causes water to flow across the membrane d. It is as diffusible through the membrane as water e. It is transported across the membrane by a carrier f. It rapidly diffuses across the membrane down its concentration gradient

b. Decreased serum K+ Abnormally prominent U waves are characteristically seen in severe hypokalemia. Thumb adducting into palm is sign of latent tetany (Trousseau sign) due to hypocalcemia.

A 49-year-old woman complains of muscle weakness. Physical examination shows diastolic hypertension. When the blood pressure is taken, the patient's thumb adducts into the palm. A random urine potassium is increased. An ECG shows prominent U waves. Which of the following laboratory findings is most likely to be reported? a. Decreased serum HCO3 b. Decreased serum K+ c. Decreased serum Na+ d. Decreased serum PO42- e. Increased serum Ca2+ f. Increased serum Na+

c. Lower blood pressure Calcium channel blockers are antihypertensive drugs because they block the inward movement of calcium by binding to L-typed calcium channels on the heart and smooth muscle of the coronary and peripheral vasculature. Calcium channel blockers inhibit the inward movement of calcium in the heart by binding to L-type calcium channels and therefore, would decrease the force of the contraction. Calcium channel blockers do not lengthen the plateau phase of the ventricular action potential like β-blockers which directly depress SA and AV nodal activity.

A 49-year-old woman with coronary artery disease is prescribed a calcium channel blocker to facilitate coronary vasodilatation. Because of their myocardial side-effects, drugs that block L-type calcium channels would be expected to do which of the following? a. Increase heart rate by enhancing the firing rate of the sinoatrial (SA) node b. Trigger vasoconstriction c. Lower blood pressure d. Increase the force of contraction of the heart e. Lengthen the plateau phase of the ventricular action potential f. Increase venous return

f. Dystrophin This patient has Duchenne Muscular Dystrophy. He exhibits several hallmark signs of the disease including an abnormal gait, Gowers' sign (The child helps himself to get up with upper extremities: first by rising to stand on his arms and knees, and then "walking" his hands up his legs to stand upright), and overall lower extremity weakness for his age. DMD is caused by a mutation of the dystrophin gene at locus Xp21, located on the short arm of the X chromosome. Dystrophin is responsible for connecting the cytoskeleton of each muscle fiber to the underlying extracellular matrix. The absence of dystrophin permits excess calcium to penetrate the sarcolemma. Alterations in calcium and signalling pathways cause water to enter into the mitochondria, which then burst. DMD has an X-linked recessive inheritance pattern, and the patient's family history indicates that his mother is a carrier of the DMD gene mutation.

A 5-year-old boy is brought to the physician by his parents because of an 8-month history of difficulty walking. His parents say that he limps when he walks and has a waddling gait; he also has difficulty standing. When getting up from a sitting position, he uses his hands to walk up his thighs and push his body into a standing position. His parents have not noticed any weakness of his arms. His mother is an only child, but she has an uncle who became bedridden as a child and died of respiratory arrest. Physical examination shows prominent calf muscles. Muscle strength is 4/5 at both hips but normal elsewhere. This patient most likely has a mutation in the gene coding for which of the following proteins? a. Actin b. Troponin c. Frataxin d. Myelin e. Myosin f. Dystrophin

c. Histamine Histamine is released from basophils and mast cells in response to foreign pathogens as part of the immune response. Histamine increases the permeability of the capillaries to white blood cells and some proteins, to allow them to engage pathogens in the infected tissues. Histamine is involved in the inflammatory response and has a central role as a mediator of itching.

A 5-year-old boy with a family history of seasonal allergies develops erythema, itching, and swelling of the skin after a subcutaneous injection of ragweed pollen. Which of the following chemical mediators is most responsible for this skin reaction? a. Bradykinin b. Complement c. Histamine d. Nitric oxide (NO) e. Prostaglandins f. Epinephrine

e. Shivering As cold stress increases, cutaneous vasoconstriction is supplemented with heat production—thermogenesis—initially in brown adipose tissue and subsequently through shivering.

A 5-year-old girl falls through the ice while skating on an outdoor pond. She is removed from the water within 1 minute, but dry clothing is not available, and she is still cold and wet 20 minutes later. Which of the following mechanisms helps maintain the patient's core temperature during the period following her rescue? a. Cutaneous vasodilation b. Diving response c. Increased thermoregulatory set point d. Release of endogenous pyrogen e. Shivering

c. Niacin The patient's clinical manifestations are consistent with pellagra caused by niacin deficiency. This life-threatening disease is characterized by the three 'ds' (dermatitis, diarrhea, and dementia). Chronic alcoholism may inhibit niacin absorption from the gastrointestinal tract, resulting in niacin deficiency. Although rare, biotin deficiency has been observed in persons eating raw eggs or receiving chronic high doses of antibiotics or long-term total parenteral nutrition (TPN). Although homeless, poor dietary intake will probably not result in biotin deficiency because intestinal bacteria can synthesize large amounts of biotin. Experimental biotin deficiency is characterized by dermatitis, hair loss, muscle pain, paresthesia, and hypercholesterolemia, but not diarrhea and dementia as in this case. Folate deficiency in adults results in megaloblastic anemia, polyneuropathy, and depression, but not dementia as in this case.

A 50-year-old homeless man with a 30-year history of alcoholism complains of painful lesions on the mucous membranes of his mouth and tongue and has pigmented keratotic scaling lesions on his face and hands. In addition, he is weak, forgetful, and has burning sensations in various parts of the body and diarrhea. A deficiency of which of the following is most likely to account for these findings? a. Biotin b. Folic acid c. Niacin d. Riboflavin e. Vitamin C f. Vitamin D

b. Goblet cells Goblet cells are responsible for the production and secretion of mucus.

A 50-year-old man comes to the physician because of a cough productive of large quantities of mucus for 6 months. He has smoked 1 pack of cigarettes daily for 25 years. Which of the following cell types is the most likely cause of the increase in this patient's secretion of mucus? a. Columnar ciliated epithelial cells b. Goblet cells c. Interstitial cells d. Macrophages e. Pneumocyte epithelial cells f. Pulmonary vascular smooth muscle cells g. Basal cells

b. Plasma osmolarity (POsm) decreased, serum ADH increased Papilledema is swelling of the optic disc that is secondary to elevated intracranial pressure, this finding is likely due to the cerebral edema seen on MRI.

A 50-year-old man who has a 35-pack-year history for smoking cigarettes, develops a cough, a headache, and blurry vision. Physical examination shows papilledema and coarse inspiratory crackles in both lungs. A chest radiograph shows a centrally located mass in the left hilum. An MRI shows cerebral edema but no localizing masses. A sputum cytology shows small, hyperchromatic cells and necrotic debris. Which of the following laboratory results is most likely expected? a. Plasma osmolarity (POsm) decreased, serum ADH decreased b. Plasma osmolarity (POsm) decreased, serum ADH increased c. Plasma osmolarity (POsm) increased, serum ADH decreased d. Plasma osmolarity (POsm) increased, serum ADH increased e. Plasma osmolarity (POsm) normal, serum ADH normal f. Plasma osmolarity (POsm) decreased, serum ADH normal g. Plasma osmolarity (POsm) normal, serum ADH increased

d. Right ventricular infarction ST segment elevation is indicative of an infarction. ST segment elevation in the inferior leads suggests an infarction on the right side of the heart. Jugular vein distention suggests reduced blood flow from the right side of the heart which would occur if the cardiac muscle was impaired in its contraction due to infarction

A 52-year-old male with hypertension and high cholesterol presents to the hospital with mid-sternal chest pressure and on electrocardiogram is found to have ST segment elevation in the inferior leads, with reciprocal ST depressions in the anterior leads. On examination, the patient is pale and sweaty, and has a blood pressure of 80/50 mm Hg. The jugular veins are distended to an estimated pressure of 15 mm Hg, and the lung fields are clear. The most likely cause for the patient's low blood pressure is: a. Cardiac tamponade b. Acute ventricular septal defect c. Acute papillary muscle rupture d. Right ventricular infarction e. Acute pericarditis f. Bradycardia

e. Type 2 diabetes mellitus The results of the graph show a rapid spike in blood glucose level after administration of the glucose. After 30 minutes, insulin secretion increases, but blood glucose levels remain elevated even with increased plasma insulin levels. At time 120, plasma glucose levels still remain elevated well above normal, even with prolonged insulin exposure. These results suggest reduced sensitivity to insulin, which is the hallmark of type 2 diabetes.

A 52-year-old man comes to the physician for a health maintenance examination. Laboratory studies show a serum glucose of 400 mg/dL. He is scheduled for an oral glucose tolerance test after overnight fasting. The graph shows the results of this evaluation. Time 0 represents the time at which glucose was administered. Which of the following is the most likely explanation for these results? a. Beta cell hyperfunction b. Increased glycogenolysis c. Reduced somatostatin release d. Type 1 diabetes mellitus e. Type 2 diabetes mellitus f. Increased beta-oxidation of fatty acids g. Increased gluconeogenesi

C. C This patient with an extensive history of cigarette smoking has symptoms consistent with Chronic Obstructive Pulmonary Disease. A hallmark of this disorder is an increase in lung compliance due to emphysema and an increase in airway resistance due to chronic emphysema. The net result for the patient is air trapping in the lungs which inhibits expiration and drives the PCO2 up past normal levels (normal PCO2 is 33-45 mmHg). As PCO2 increases, the pH decreases. To compensate for this respiratory acidosis, the plasma bicarbonate level increases (normal 22-28 mEq/L) to neutralize the additional acid. Only answer choice C shows these changes.

A 52-year-old man has a 70-pack-year of smoking cigarettes. He states that he has coughed up a tablespoon or more of yellow-green sputum almost every morning for the past 20 years. Physical examination shows cyanosis of the skin and mucous membranes. On auscultation, there are scattered coarse inspiratory crackles that clear with coughing. Which of the following arterial blood gas values would most likely be reported? A. A B. B C. C D. D E. E F. F G. G

b. Plasminogen Plasminogen is the inactive precursor of plasmin, the proteolytic enzyme involved in clot dissolution. An infusion of tissue plasminogen activator (tPA) soon after a heart attack (and possibly a thrombolytic stroke) can lessen the chances of permanent damage. Thrombin, the enzyme ultimately responsible for the formation of fibrin monomers, is generated from prothrombin by activated factor X. Activation of factor X occurs via both extrinsic and intrinsic pathways. Kininogens are enzymes responsible for the production of peptides (kinins) associated with inflammation. Heparin is an anticlotting agent found on endothelial cell surfaces.

A 52-year-old man is brought to the Emergency Department with severe chest pain. Angiography demonstrates a severe coronary occlusion. A thrombolytic agent is administered to reestablish perfusion. Which of the following does the thrombolytic agent activate? a. Heparin b. Plasminogen c. Thrombin d. Kininogen e. Prothombin f. Factor VII

d. Vitamin K A, B, C, F, and G promote bleeding. Vitamin K promotes K dependent factors of coagulation cascade leading to clotting.

A 67-year-old woman with a history of venous thromboembolism is placed on warfarin prophylactically. Bleeding can occur if the blood concentration of warfarin becomes too high. Should bleeding occur, it may be prevented by the administration of: a. Aspirin b. Heparin c. tPA d. Vitamin K e. Fibrinogen f. Coumadin g. Tissue plasminogen activator

c. Plasminogen Plasminogen is the inactive precursor of plasmin, the proteolytic enzyme in involved in clot dissociation. An infusion of tissue plasminogen activator soon after a heart attack can lessen the chances of permanent damage. Thrombin is generated from prothrombin and is activated by factor X. activation of factor X occurs via both extrinsic and intrinsic pathways. Kininogens are enzymes responsible for the production of kinins associated with inflammation. Heparin is an anticlotting agent found on endothelial cell surface.

A 52-year-old man is brought to the emergency department with severe chest pain. Angiography demonstrates a severe coronary occlusion. A thrombolytic agent is administered to reestablish perfusion. Which of the following does the thrombolytic agent activate? a. Heparin b. Kininogen c. Plasminogen d. Prothrombin e. Thrombin f. Fibrinogen g. Vitamin K

a. Downstroke velocity of nerve cell action potentials The repolarization phase of the action potential is produced by decrease in Na+ conductance caused by the inactivation of Na+ channels, and the increase in K+ conductance due to the activation of K+ channels. Preventing the inactivation of Na+ channels will decrease the downstroke velocity of the action potential. This will slow down the normal repolarization phase of the action potential and thereby prolong the duration of the action potential. The relative refractory period is prolonged because of the prolonged duration of the action potential. The upstroke velocity and the magnitude depend on how rapidly and how long the sodium channels are opened. By preventing inactivation of the Na+ channel, the rate of the upstroke and the magnitude of the overshoot may be increased.

A 52-year-old man presents at the oral surgeon's office with an abscessed tooth. Prior to surgery on the tooth, the patient is given a shot of procaine. Preventing the inactivation of sodium channels by local anesthetics will decrease which of the following? a. Downstroke velocity of nerve cell action potentials b. Duration of nerve cell action potentials c. Magnitude of the overshoot in nerve cell action potentials d. Relative refractory period of nerve cells e. Upstroke velocity of nerve cell action potentials f. Duration of the absolute refractory period g. Magnitude of the membrane potential threshold

a. Downstroke velocity of nerve cell action potentials

A 52-year-old man presents at the oral surgeon's office with an abscessed tooth. Prior to surgery to extract the tooth, the patient is given a shot of procaine. Preventing the inactivation of sodium channels by local anesthetics will decrease which of the following? a. Downstroke velocity of nerve cell action potentials b. Duration of nerve cell action potentials c. Magnitude of the overshoot in nerve cell action potentials d. Relative refractory period of nerve cells e. Upstroke velocity of nerve cell action potentials

f. Sodium Aldosterone is secreted in response to hyponatremia to increase sodium reabsorption from the proximal tubule.

A 52-year-old man presents to his internist for a 6-month check-up following diuretic therapy and recommended diet changes for his essential hypertension. His blood pressure is 145/95 mmHg and serum aldosterone levels are increased. Aldosterone secretion is increased when there is a decrease in the plasma concentration of which of the following? a. Chloride b. Potassium c. Creatinine d. Bicarbonate e. Urea f. Sodium

a. Cortisol The pituitary secretes ACTH with a circadian rhythm. Input from hypothalamic nuclei to the corticotrophs—via both CRH and AVP— appears to modulate the circadian secretion of ACTH and thus the circadian secretion of cortisol as well. As a result, superimposed on the circadian rhythm of ACTH is the pulsatile secretion of ACTH. ACTH secretory activity is greatest in the early morning and diminishes late in the afternoon and early evening. The overall result is that cortisol secretion is greatest in the early morning and declines throughout the day.

A 52yearold woman with a chief complaint of snoring is referred for a sleep study. As shown in the graph below, the concentration of a hormone varied over the 24hour period of study. This diurnal variation in plasma level results from the secretion of which of the following hormones? a. Cortisol b. Estrogen c. Insulin d. PTH e. Thyroxine f. Testosterone

d. Atrial fibrillation Absence of P waves indicates not normal sinus rhythm or sinus tachycardia. Ventricular tachycardia will present with QRS abnormalities, particularly widened QRS complex. Not multifocal atrial tachycardia because there are no P waves present. MAT has P waves with inconsistent morphologies. In the common form of type I atrial flutter, the ECG shows sawtooth flutter (F) waves. Flutter waves are often visualized best in leads II, III, aVF, or V1 (see the image below). The flutter waves for typical (type I) atrial flutter are inverted (negative) in these leads because of a counterclockwise reentrant pathway

A 53-year-old man with a history of hyperthyroidism is evaluated because of acute onset of heart palpitations and shortness of breath. Results of electrocardiography (ECG) are shown. Which of the following is the most likely explanation for this patient's signs? a. Sinus tachycardia b. Normal sinus rhythm c. Ventricular tachycardia d. Atrial fibrillation e. Multifocal atrial tachycardia f. Atrial flutter.

b. B The 1a afferents, which innervate the muscle spindles, have a phasic and tonic component. B illustrates the response of 1a afferents to sudden movement of a limb. The highfrequency burst of action potentials encodes the velocity of the initial movement, whereas the steady firing encodes the position of the limb when movement is completed. A and E illustrate the behavior of a tonic receptor, which discharges at the same rate for as long as the stimulus is present. The patterns of sensory loss are often indicative of the level of nervous system involvement. In the spinal cord, segregation of fiber tracts and the somatotopic arrangement of fibers give rise to distinct patterns of sensory loss. Lesions that involve onehalf of the spinal cord lead to loss of proprioception on the ipsilateral side and loss of pain and temperature sensation on the contralateral side. This presentation is called BrownSequard syndrome, which may be accompanied by contralateral hemiparesis with lesions in the high cervical spinal cord.

A 53yearold man develops loss of pain and temperature sensation in his right leg and loss of proprioception in his left leg (BrownSequard syndrome). These symptoms appear 6 weeks following total prostatectomy for prostate cancer. A bone CT scan reveals metastases compressing the patient's left hemicord. His urologist refers him to a neurologist who wishes to confirm normal proprioception in the left leg. Which figure below illustrates the train of action potentials normally seen in a sensory nerve encoding the velocity of limb movement in response to sudden movement? a. A b. B c. C d. D e. E

B. B A cardiac and respiratory arrest generates a mixed acid-base disorder with the coexistence of two primary acid-base disorders, namely, respiratory and metabolic acidosis. The respiratory acidosis is due to alveolar hypoventilation, which increases arterial PCO2 and decreases pH. Cardiac arrest decreases cardiac output and thus oxygen delivery to the tissues resulting in stagnant hypoxia. The resultant increase in anaerobic glycolysis produces large amounts of lactic acid, which lowers plasma bicarbonate and decreases pH. The normal anion gap of about 12 mEq/L, calculated as the difference between the concentration of Na+ and major plasma anions, Cl- and HCO3-, increase due to the increase in the unmeasured organic anion, lactate. Because both independently existing disorder cause acidosis, mixed respiratory and metabolic acidosis may cause dangerously low pH levels and a poor outcome.

A 54-year-old man goes out to shovel the snow so that he can drive his wife to her doctor's appointment. After getting some chest pain and feeling shortness of breath, he thinks he better go and rest for a while before finishing the task. When his wife comes downstairs, she finds him sitting with his head down on the morning paper at the kitchen table. When the ambulance arrives, he is still responsive, but has a cardiac and respiratory arrest en route to the hospital. Which of the following arterial blood gases would be expected given these findings? A. A B. B C. C D. D E. E F. F G. G

b. Antidiuretic hormone This patient is hyponatremic and hyperkalemic, indicating that aldosterone and renin can be immediately eliminated. Of the remaining hormones, only ADH is able to reduce serum sodium concentration.

A 54yearold man is admitted to the hospital after being found unresponsive by his friend. There is no suspicion of drug use. The patient is a nonsmoker and nondrinker. The patient has a 5year history of uncontrolled hypertension and takes furosemide, hydrochlorothiazide, atenolol, and enalapril. He has had worsening lethargy and confusion for the last few days. Blood pressure is 89/48 mm Hg. He has dry mucous membranes and flat neck veins. Cardiac examination shows a regular rate and rhythm. Pulmonary auscultation reveals labored breathing and decreased breath sounds at the lung bases. Laboratory studies of serum show: Serum sodium = 112 mEq/L, Serum potassium = 5.1 mEq/L, Serum osmolality = 230 mOsmol/kg H2O, Urine sodium 40 mEq/L, Creatinine 1.4 mg/dL. An increase of which of the following hormones is most likely responsible for this man's hyponatremia? a. Aldosterone b. Antidiuretic hormone c. Cortisol d. Somatostatin e. Thyroid stimulating hormone f. Renin

b. Increased afferent arteriolar resistance that reduced GFR NSAIDs inhibit prostaglandin synthesis, which in turn causes the constriction of afferent arterioles that can reduce the GFR. The decrease in GFR, in turn, leads to an increase in serum creatinine. Increased efferent arteriole resistance, and increased glomerular capillary filtration coefficient would both tend to increase rather than reduce GFR. Increasing muscle mass due to exercise would cause very little change in serum creatinine. Increased renal blood flow and increased glomerular capillary hydrostatic pressure increase GFR.

A 55-year-old male patient with hypertension has had his blood pressure reasonably well controlled by administration of a thiazide diuretic. During his last visit (6 months ago) his blood pressure was 130/75 mmHg and his serum creatinine was 1 mg/100 mL. he has been exercising regularly for the past 2 years, but recently has complained of knee pain and began taking large amounts of a non-steroidal anti-inflammatory drug. When he arrives at your office, his blood pressure is 155/85 and his serum creatinine is now 2.5 mg/100 mL. Which of the following best explains his increased serum creatinine? a. Increased efferent arteriolar resistance that reduced GFR b. Increased afferent arteriolar resistance that reduced GFR c. Increased glomerular capillary filtration coefficient that reduced GFR d. Increased angiotensin II formation that decreased GFR e. Increased muscle mass due to exercise f. Increased renal blood flow that reduced GFR g. Increased glomerular capillary hydrostatic pressure that reduced GFR

e. Reabsorption of vitamin B12 after addition of intrinsic factor In pernicious anemia, atrophy of the gastric mucosa in the corpus and an absence of parietal cells result in a lack in the secretion of both gastric acid and intrinsic factor (IF). Many patients with pernicious anemia exhibit antibody-mediated immunity against their parietal cells, and many of these patients also produce anti-IF autoantibodies. Because IF is required for cobalamin (vitamin B12) absorption in the ileum, the result is impaired cobalamin absorption. Because of this, the vitamin B12 deficiency seen in pernicious anemia can be treated with exogenous intrinsic factor. Because parietal cells are absent, the elevated plasma gastrin levels are not associated with enhanced gastric acid secretion.

A 55-year-old woman is diagnosed with vitamin B12deficiency caused by pernicious anemia. Which of the following laboratory findings is used to distinguish pernicious anemia from other causes of deficiency of vitamin B12or folic acid? a. Hypersegmented neutrophils b. Increased plasma homocysteine c. Increased urine methylmalonic acid d. Pancytopenia with megaloblastic bone marrow e. Reabsorption of vitamin B12 after addition of intrinsic factor f. Enhanced gastric acid secretion g. Reduced plasma gastrin levels

a. Autoimmune response Myasthenia gravis is an autoimmune disease which results from antibodies that block or destroy nicotinic acetylcholine receptors at the junction between the nerve and muscle. This prevents nerve impulses from triggering muscle contractions. The Tensilon test uses the drug Tensilon (edrophonium) to diagnose myasthenia gravis. Tensilon prevents the breakdown of the chemical acetylcholine, a neurotransmitter that nerve cells release to stimulate your muscles. People with the chronic disease myasthenia gravis don't have normal reactions to acetylcholine. Antibodies attack their acetylcholine receptors. This prevents muscles from being stimulated and makes muscles easy to tire. A person tests positive for myasthenia gravis if their muscles get stronger after being injected with Tensilon.

A 55-year-old woman visits her physician because of double vision, eyelid droop, difficulty chewing and swallowing, and general weakness in her limbs. All of these symptoms are made worse with exercise and occur more frequently late in the day. The physician suspects myasthenia gravis and orders a Tensilon test. The test is positive. What is the most likely basis for the symptoms described in this patient? a. Autoimmune response b. Botulinum toxicity c. Depletion of voltage-gated Ca2+ channels in certain motor units d. Development of macro motor unit units during recovery from poliomyelitis e. Overexertion f. Atrophy of skeletal muscle

b. Increased afferent arteriolar resistance that reduced GFR NSAIDs inhibit the synthesis of prostaglandins, which, in turn causes constriction of afferent arterioles that can reduce the GFR. The decrease in GFR, in turn, leads to an increase in serum creatinine. Increased efferent arteriole resistance, and increased glomerular capillary filtration coefficient would both tend to increase rather than reduce GFR. Increasing muscle mass due to exercise would cause very little change in serum creatinine.

A 55yearold male patient with hypertension has had his blood pressure reasonably well controlled by administration of a thiazide diuretic. During his last visit (6 months ago) his blood pressure was 130/75 mm Hg and his serum creatinine was 1 mg/100 mL. He has been exercising regularly for the past 2 years, but recently has complained of knee pain and began taking large amounts of an NSAID. When he arrives at your office, his blood pressure is 155/85 and his serum creatinine is now 2.5 mg/100 mL. Which of the following best explains his increased serum creatinine? a. Increased efferent arteriolar resistance that reduced GFR b. Increased afferent arteriolar resistance that reduced GFR c. Increased glomerular capillary filtration coefficient that reduced GFR d. Increased angiotensin II formation that decreased GFR e. Increased muscle mass due to the exercise f. Increased glomerular capillary hydrostatic pressure that reduced GFR

d. Third degree heart block Syncope (fainting) is a transient loss of consciousness caused by an inadequate blood flow to the brain. Transient decreases in cerebral blood flow are usually due to one of three general mechanisms: disorders of vascular tone or blood volume, cardiovascular disorders, or cerebrovascular disease. Approximately onefourth of syncopal episodes are of cardiac origin and are due to either transient obstruction of blood flow through the heart or sudden decreases in cardiac output due to cardiac arrhythmias, such as brady cardia, heart block, or sinus arrest (neurocardiogenic syncope). Thirddegree (complete) heart block results when conduction of the action potential from the atria to the ventricles is completely interrupted. Under these conditions, pacemaker cells within the HisPurkinje system or the ventricular muscle cause the ventricles to beat at a low rate (idioventricular rhythm) independently of the atria. Although the heart rate may be high enough to adequately perfuse the brain under resting conditions, Thirddegree heart block is caused by conduction system disturbances, inferior wall MI, and digitalis toxicity. When the conduction disturbance is due to disease in the AV node, the idioventricular rhythm is normally about 45 beats/min. When the conduction disturbance is below the AV node (infranodal block) due to disease in the bundle of His, firing of more peripheral ventricular pacemakers can decrease heart rate to below 30 beats/min with periods of asystole that may last a minute or more. The resultant cerebral ischemia causes dizziness and fainting (StokesAdams syndrome). Sinus arrhythmia is a change of the heart rate produced by the normal variation in the rate of phase 4 depolarization of the SA nodal pacemaker cells between inspiration and expiration. First degree heart block is defined as a higher than normal PR interval (greater than 0.2 seconds). Second degree heart block occurs when the action potential fails to reach the ventricles some, but not all, of the time. Tachycardia is a heart rate above 100 beats per minute.

A 55yearold male reports several recent episodes of syncope. An electrocardiogram is performed. Which of the following arrhythmias is most commonly associated with syncope? a. Sinus arrhythmia b. Firstdegree heart block c. Second degree heart block d. Third degree heart block e. Tachycardia f. Normal sinus rhythm

f. Increased production of estrogen by adipose tissue Obesity is known to increase the risk for cancer of the reproductive tract in women. The mechanism underlying this association can be explained by increased estrogenic stimulus to estrogentarget tissues as the result of three factors. First, increased adrenal secretory activity makes more androgen precursors available for conversion to estrogen in peripheral tissues. Second, the efficiency of conversion of androstenedione (A) to estrone (E1), is elevated in obese subjects because adipose tissue is the major tissue site of conversion. Third, plasma levels of SHBG, which binds estradiol (E2), are depressed in obese subjects and greater than normal amounts of serum estradiol are therefore available to target tissues from the circulation.

A 55yearold woman who is obese has a greater risk for endometrial carcinoma than a 55yearold woman with the same health history and status who is not obese. Which of the following best explains this increased risk? a. Accelerated catabolism of antioxidants b. Association of obesity with heavier menstrual periods c. Carcinogenic effects of dietary fats d. Greater average number of pregnancies e. Impairment of immune surveillance by T lymphocytes f. Increased production of estrogen by adipose tissue

d. Iron deficiency The presence of microcytic hypochromic red blood cells indicates iron deficiency anemia, lead poisoning, thalassemia, or sideroblastic anemia. The presence of occult blood in the stool reveals a probable source of iron loss through the gastrointestinal tract.

A 56-year-old female complains of generalized weakness and a 20 lb weight loss over the past 6 months. Blood pressure is 136/80 mmHg and pulse is 88/min. Laboratory values reveal a hematocrit of 31% (normal 40-45%), hemoglobin level of 10 g/dL (normal 14 g/dL), and a blood smear shows hypochromic microcytic cells. A stool test for occult blood is positive. Which of the following would be the most likely cause of the findings? a. Acute blood loss b. Folic acid deficiency c. Hemochromatosis d. Iron deficiency e. β-thalassemia trait f. Hemophilia type A

c. Right optic tract The patient has a lesion in the right optic tract that causes left homonymous hemianopia. Left homonymous hemianopia can arise from lesions in the right optic tract, right lateral geniculate body, right optic radiations or right occipital cortex.

A 56-year-old man presents with a 4-day history of impaired vision. On physical exam, you note a loss of vision of his left visual field in both eyes. The patient most likely has a lesion involving the: a. Right optic nerve b. Optic chiasm c. Right optic tract d. Right temporal lobe e. Right parietal lobe

a. Coronary blood flow Abnormalities in coronary blood flow result in ischemia in the ventricular muscle that leads to a current of injury. The current of injury Is reflected as an upward or downward shift in the ST segment of the ECG. The ECG above shows marked ST segment elevation in leads I, aVL, and V1-V6, indicative of acute myocardial infarction. The electrical activity of the heart does not reflect changes in ejection fraction, blood pressure, total peripheral resistance, cardiac output, stroke volume, or ventricular contractility.

A 56-year-old man was admitted to the hospital with angina and diaphoresis. A myocardial infarction is suspected, and a 12-lead ECG is recorded and shown below. The ECG is most effective in detecting a decrease in which of the following? a. Coronary blood flow b. Ejection fraction c. Mean blood pressure d. Cardiac output e. Total peripheral resistance f. Ventricular contractility g. Stroke volume

d. Competing with creatinine for renal tubular secretion Both cimetidine and creatinine compete for the same renal tubular secretion mechanism and cimetidine treatment will decrease creatinine secretion, thereby leading to an increase in the serum concentration of creatinine. Although acute renal failure leads to an increase in serum creatinine concentration, it also elevates serum K+ and BUN (blood urea nitrogen).

A 56-year-old obese woman suffering from heartburn and regurgitation of acid saliva after large meals is treated with cimetidine. During cimetidine treatment, her serum creatinine concentration increases significantly. The increase in serum creatinine with cimetidine treatment is most likely due to: a. Being reabsorbed by the nephron in exchange for creatinine b. Blocking synthesis of creatinine by skeletal muscle c. Causing acute renal failure d. Competing with creatinine for renal tubular secretion e. Inhibiting uptake of creatinine ingested with food f. Increasing creatinine production from the muscles

g. Ventricular end-diastolic volume Preload is the degree to which the myocardium contracts, an indication of the length of the sarcomere at the end of diastole. The Frank-Starling law states that the energy of contraction is proportional to the initial length of the cardiac muscle fiber. The variable most directly related to sarcomere length is the end-diastolic volume. Though pulmonary wedge pressure and end-diastolic pressure indirectly indicate the preload, the end-diastolic volume is the most direct measure.

A 56-year-old woman presents for her annual physical examination. Her physician auscultates a late systolic crescendo murmur with a midsystolic click. The murmur is best ehard over the apex, is loudest at S2, is shortened with squatting and is longer and more intense when venous return is decreased by standing or a Valsalva maneuver. Which of the following values is the best index of the preload on her heart? a. Blood volume b. Central venous pressure c. Pulmonary capillary wedge pressure d. Ventricular end-diastolic pressure e. Cardiac output f. Systemic vascular resistance g. Ventricular end-diastolic volume

a. Diffusing capacity of the lungs increases The lungs and heart are in series, so the entire cardiac output flows through the lungs. The increased pulmonary blood flow during exercise increases the surface area for diffusion, and therefore increases the diffusing capacity in accordance with Fick's law of diffusion. The increased perfusion of the lungs is accompanied by an even greater increase in ventilation, so the V/Q ratio of the whole lung, as well as most areas of the lung, increases during exercise. The increase in blood flow through the pulmonary circulation during exercise increases the diameter of the pulmonary vessels and therefore decreases their resistance. Systolic, diastolic, and mean PAPs increase slightly during exercise because of the increased pulmonary blood flow and blood volume.

A 56yearold man presents for his annual physical examination. His BMI has increased from 28 to 33 over the past year and the fat deposition is mainly around the abdomen. His blood pressure has increased from 125/85 to 140/95 mm Hg since the last visit. Other physical findings are unremarkable and he and his spouse state that he does not snore. Past medical history and social history are insignificant except for his sedentary lifestyle. Exercise stress testing is ordered prior to placing the patient on a regular exercise regimen. Aerobic exercise causes which of the following changes in pulmonary physiology? a. Diffusing capacity of the lungs increases b. Mean pulmonary arteriolar pressure decreases c. Overall V/Q ratio of the lungs decreases d. Pulmonary blood flow decreases e. Pulmonary vascular resistance increases f. Pulmonary capillary diameter increases

a. Left atrial pressure This ECG shows the irregularly irregular rhythm of atrial fibrillation. A fib occurs as a result of disorganization of the electrical activity of the atrium. The result is an inability to produce a coordinated atrial contraction. The absence of a normal atrial pulse results in decrased emptying of the left atrium and increased left atrial pressure. The venous a wave decreases as a result of the absent atrial pulse. Arterial blood pressure, stroke volume, and cardiac output all decrease. The oculocardic reflex is a decrease in HR that occurs when pressure is applied to the eyeball via connections between the ophthalmic branch of the trigeminal nerve (afferent) and the vagus nerve (efferent) to the SA node. Decreased HR results from increased parasympathetic stimulation of atrial musculature, not decreased sympathetic tone. The oculocardic3 reflex is similar to carotid sinus massage.

A 57-year old man complains of palpitations that are relieved by pressing on his eyeball. His ECG is shown below. An increase in which of the following is most likely to accompany this condition? a. Left atrial pressure b. Mean arterial pressure c. Stroke volume d. Sympathetic tone e. Venous a wave f. Cardiac output

d. Bile salts The bulk of bile salt reabsorption occurs in the ileum. The bulk of iron, folate, lactose, protein, carbohydrate, and fat reabsorption occurs in the duodenum. The bulk of Vitamin D reabsorption occurs in the jejunum.

A 57-year-old man undergoes resection of the distal 100cm of the terminal ileum as part of treatment for Crohn's disease. The patient likely will develop malabsorption of which of the following? a. Iron b. Folate c. Lactose d. Bile salts e. Protein f. Vitamin D g. Fatty acids

e. Secretion of renin Renal artery stenosis reduces the flow of filtrate through the kidney tubule and the effective circulating volume as detected by the macula densa. As a result, renin is released to increase the ECV. Afferent arteriolar resistance, GFR, glomerular hydrostatic pressure, interlobar artery pressure, and renal blood flow are all reduced. ANP is released in response to increased blood volume as indicated by increased atrial stretch.

A 57-year-old woman is admitted to the hospital because of a 2-day history of chest pain on exertion. This occurred while doing housework, which consisted of vacuuming and waxing her hardwood floors. She has a history of hypertension, angina, and diabetes. Cardiac examination shows S1 and S2 heart sounds that are somewhat faint. There are no murmurs or gallops. ECG and cardiac enzymes show no abnormalities. Cardiac catheterization shows areas of coronary artery narrowing (between 50 and 70%). CT angiography shows an 90% stenosis of the right renal artery. Which of the following is most likely increased in this patient? a. Afferent arteriolar resistance b. Glomerular filtration rate c. Glomerular hydrostatic pressure d. Interlobar artery pressure e. Secretion of renin f. Secretion of atrial natriuretic peptide g. Renal blood flow

g. Antidiuretic hormone ADH is a powerful vasoconstrictor in addition to is effect of decreasing plasma osmolality via renal water retention. Adenosine, ANP, b2-adrenergic agonist, NO, and lactate cause vasodilation. ACh can effect vasodilation by several mechnisms including activation of endothelial NO synthase and prostaglandin production. In skin, exogenous ACh increases both skin blood flow and bioavailable NO levels, but the relative increase is much greater in skin blood flow than NO.

A 57-year-old woman is undergoing a femoral popliteal bypass for her peripheral vascular disease. The vascular wishes to induce a localized arteriolar constriction to help control hemostasis. An increase in the local concentration of which of the following agents will cause systemic vasoconstriction? a. Adenosine b. Acetylcholine c. Atrial natriuretic peptide d. Beta-2-adrenergic agonist e. Nitric oxide f. Lactate g. Antidiuretic hormone

b. An increased left atrial pressure Atrial fibrillation is an arrhythmia in which the electrical activity of the atrium becomes disorganized and therefore unable to produce a coordinated atrial contraction. The absence of an atrial pulse reduces the emptying of the atria during diastole and results in an enlarged left atrium and increased left atrial pressure. The venous A wave represents atrial contraction and disappears due to the absence of an atrial beat. Decreased filling of the heart results in a decrease in stroke volume. Heart rate increases because the continuous electrical activity of the atria initiates a high rate of ventricular activity. Systemic blood pressure typically falls because of inadequate filling of the ventricles and the resulting decrease in stroke volume.

A 57yearold male complains of an irregular heart beat that he notices is relieved by pressing on his eyeball. An electrocardiogram reveals atrial fibrillation. Which of the following is most likely to accompany this condition? a. An increased venous A wave b. An increased left atrial pressure c. A decreased heart rate d. An increased stroke volume e. An increased mean arterial blood pressure f. An increased cardiac output

b. Angiotensin II Angiotensin II is a powerful vasoconstrictor that is formed when renin is released from the kidney in response to a fall in blood pressure or vascular volume. Renin converts angiotensinogen to angiotensin I. Angiotensin II is formed from angiotensin I by angiotensinconverting enzyme localized within the vasculature of the lung. All the other listed substances cause vasodilation.

A 57yearold woman is undergoing a femoral popliteal bypass for her peripheral vascular disease. The vascular surgeon wishes to induce a localized arteriolar constriction to help control hemostasis. An increase in the local concentration of which of the following agents will cause systemic vasoconstriction? a Nitric oxide b. Angiotensin II c. Atrial natriuretic peptide d. A beta2adrenergic agonist e. Adenosine f. Antidiuretic hormone

b. Excretion of nitrogen increases In untreated diabetes mellitus, the excretion of nitrogen is increased (negative nitrogen balance) because of reduced protein synthesis and decreased uptake of amino acids by cells.

A 58-year-old female patient with type II diabetes mellitus decides to discontinue insulin treatment because she believes she can control the disease with diet alone. What would you expect to happen to this patient shortly after insulin is discontinued? a. Blood glucose concentration decreases b. Excretion of nitrogen increases c. Glycogen content in the liver increases d. Extracellular concentration of potassium decreases e. Rate of conversion of alanine to glucose decreases f. Rate of lipolysis increases

c. Chronic obstructive pulmonary disease (COPD) The interpretation of the arterial blood gas is compensated respiratory acidosis. The primary disturbance is an elevation in arterial PCO2 dure to alveolar hypoventilation from the impaired mechanics of breathing in COPD. The hypercapnia lowers the ratio of bicarbonate ion to dissolved CO2 in the plasma, and thus lowers the pH according to the Henderson-Hasselbalch equation. To compensate for the acidosis, the increase in the next excretion of H+, which increases the plasma [HCO3-] returns the pH back to normal. Narcotic overdose would be associated with an acute, uncompensated respiratory acidosis. Anxiety-induced hyperventilation would lower arterial PCO2, and increase arterial pH, characteristic of respiratory alkalosis. Ketoacidosis secondary to excessive alcohol ingestion, starvation, or diabetes would cause a metabolic acidosis with a compensatory decrease in arterial PO2. Salicylate toxicity results in a combined respiratory alkalosis (due to direct stimulation of the medullary respiratory center) and metabolic acidosis (due to accumulation of organic acid).

A 58-year-old man is transported to the emergency department due to impaired breathing and shortness of breath. Arterial blood gases show the following: pH=7,35, PaO2=60 mmHg, PaCO2=60 mmHg, [HCO3-]=31 mEq/L. Which of the following is the most likely diagnosis of this patient? a. Alcoholic ketoacidosis b. Anxiety-induced hyperventilation c. Chronic obstructive pulmonary disease (COPD) d. Narcotic overdose e. Salicylate overdose f. Asbestosis g. Hyperemesis-induced dehydration

e. It is used as a source of energy for the transport of other ions

A 58-year-old man with a history of HTN and renal disease presents at his physician's office with a complaint of headaches. His blood pressure is 190/115 mmHg and laboratory results show an elevated plasma renin activity with hypernatremia. Which of the following best describes the sodium gradient across the nerve cell membrane? a. It is a result of the Donnan equilibrium b. It is maintained by a Na+/Ca2+ exchanger c. It is significantly changed during and action potential d. It is the primary determinant of the resting membrane potential e. It is used as a source of energy for the transport of other ions

e. Vitamin K In addition to treatment with packed red blood cells, and fresh frozen plasma (FFP), IV or oral vitamin K is the first line therapy for Coumadin (warfarin) toxicity/overdose. The administration of vitamin K is important to promote the endogenous synthesis of vitamin K-dependent clotting factors which are essential to promote the coagulation cascade and reverse bleeding.

A 67-year-old woman with a history of venous thromboembolism is placed on warfarin prophylactically. The blood concentration of Coumadin becomes too high and bleeding occurs. The bleeding can best be treated by administration of which of the following? a. Fibrinogen b. Platelets c. Protein C d. Thrombin e. Vitamin K f. Protein S g. Factor Xa

c. Decrease intake of H2O, maintain intake of Na+ This patient has SIADH, secondary to an ADH-secreting tumor in his lungs. The tumor is causing an increase in water reabsorption in the distal tubule. As a result, the plasma osmolarity is falling, and the patient is producing a hyperconcentrated urine. To remedy this, a reduction of water intake along with a constant intake of sodium, will improve plasma osmolarity back to normal levels.

A 58-year-old man with small cell carcinoma of the lung complains of a headache and blurry vision. Examination shows swelling of the optic nerve. An MRI of the head shows cerebral edema, but no evidence of metastatic disease. The serum Na+ level is 115 mEq/L and the random urine Na+ > 40 mEq/L (normal is 20 mEq/L). Which of the following is the most appropriate non-pharmacologic treatment for this patient? a. Decrease intake of Na+ and H2O b. Decrease intake of Na+, maintain intake of H2O c. Decrease intake of H2O, maintain intake of Na+ d. Increase intake of Na+ and H2O e. Maintain intake of Na+ and H2O f. Increase intake of H2O, increase intake of Na+

a. Atelectasis Atelectasis (lung collapse) is the most common cause of fever that occurs within the first 24-36 h after surgery. Lung consolidation and the loss of lung mass results in ipsilateral elevation of the diaphragm and inspiratory lag because the lung is not expanding properly on inspiration. Postoperative pneumonia usually develops later (3-10 days) after surgery not within 24-36 h as in this case. The most common cause of a lung abscess, aspiration, occurs much later than 24-36 h after surgery. The patient usually has a cough productive of foul-smelling sputum due to the presence of aerobes and anaerobes in the abscess. Signs of pulmonary embolization, such as dyspnea, tachycardia and hemoptysis, are not present, and pulmonary embolism usually occurs 5-7 days after surgery. Most pulmonary emboli are from leg deep venous emboli. In spontaneous pneumothorax, a portion of the lung collapses, producing hyper-resonance to percussion and physical findings similar to lung consolidation.

A 58-year-old woman develops a fever and dyspnea approximately 24 h after undergoing a laparoscopic cholecystectomy. Physical examination shows decreased percussion, increased tactile fremitus, and decreased breath sounds in the right lower lobe. Which of the following is the most likely diagnosis? a. Atelectasis b. Lobar pneumonia c. Lung abscess d. Pulmonary infarction e. Spontaneous pneumothorax f. Asbestosis

e. The concentration of calcium within the SR In cardiac muscle, the sarcoplasmic reticulum Ca2+-ATPase is responsible for pumping calcium back into the sarcoplasmic reticulum and lowering the concentration of calcium in the sarcoplasm. The activity of the Ca2+-ATPase activity is inhibited by the regulatory protein phospholamban (PLN). When PLN is inhibited, such as when it is phosphorylated by cAMP-dependent PKA, its ability to inhibit the SR Ca2+ pump is lost and Ca2+ is sequestered back into the sarcoplasmic reticulum.

A 59-year-old man with an ejection fraction of 15%, who is being treated with medications for his heart failure, is asked whether he would like to participate in a trial for an experimental drug. The drug being tested is designed to decrease the expression of phospholamban on ventricular muscle cells. Which of the following would be increased by decreasing phospholamban? a. The activity of the sodium-potassium pump b. The diastolic stiffness of the ventricular cells c. The activity of the L-type calcium channels d. The duration of the ventricular muscle action potential e. The concentration of calcium within the SR f. The concentration of calcium in the sarcolemma

e. The concentration of calcium within the SR Phospholamban is a protein contained within the sarcoplasmic reticulum (SR) that inhibits the activity of the SR calcium pump. Inactivation of phospholamban results in an increase in calcium sequestration by the SR. Increasing the concentration of calcium within the SR increases the force of the ventricular contraction.

A 59yearold male with an ejection fraction of 15%, who is being treated with medications for his heart failure, is asked whether he would like to participate in a trial for an experimental drug. The drug being tested is designed to decrease the expression of phospholamban on ventricular muscle cells. Which of the following would be increased by decreasing phospholamban? a. The activity of the sodiumpotassium pump b. The diastolic stiffness of the ventricular muscle cells c. The activity of the Ltype calcium channels d. The duration of the ventricular muscle action potential e. The concentration of calcium within the SR

d. Sodium (mEq/L) 130 ↓, Potassium (mEq/L) 2.8 ↓, Chloride (mEq/L) 80 ↓, and Bicarbonate (mEq/L) 35↑ With prolonged vomiting, deficits in sodium, potassium, and especially chloride occur, leading to profound hypochloremia. Reduced intravascular fluid volume stimulates renal compensation, which increases serum bicarbonate and pH. Thus, the laboratory values for D are consistent with hypochloremic metabolic alkalosis.

A 6-week-old boy is brought to his pediatrician by his mother because of progressively worsening projectile vomiting over the past 2 weeks. Physical examination reveals a wasted, dehydrated infant who is avidly sucking on a nipple. Which of the following laboratory values is most consistent with this child's problem? a. Sodium (mEq/L) 160, Potassium (mEq/L) 3.0 ↓, Chloride (mEq/L) 125 ↑, and Bicarbonate (mEq/L) 12 ↓ b. Sodium (mEq/L) 140 Normal, Potassium (mEq/L) 3.8 Normal, Chloride (mEq/L) 110 ↑, and Bicarbonate (mEq/L) 23 Normal c. Sodium (mEq/L) 138 Normal, Potassium (mEq/L) 2.5 ↓, Chloride (mEq/L) 110 ↑, and Bicarbonate (mEq/L) 23 Normal d. Sodium (mEq/L) 130 ↓, Potassium (mEq/L) 2.8 ↓, Chloride (mEq/L) 80 ↓, and Bicarbonate (mEq/L) 35↑ e. Sodium (mEq/L) 123 ↓, Potassium (mEq/L) 3.2 ↓, Chloride (mEq/L) 93 ∼, and Bicarbonate (mEq/L) 10↓

c. Thiamine deficiency The patient exhibits clinical features of Wernicke's encephalopathy, which is a complication of alcoholism and nutrition resulting in thiamine deficiency. Administering a glucose-containing solution to a patient with subclinical thiamine deficiency results in the consumption of the remaining thiamine in the pyruvate dehydrogenase complex reaction, which precipitates acute Wernicke's encephalopathy.

A 60-year-old man develops an acute onset of confusion, ataxia, nystagmus, and hypothermia shortly after the administration of an intravenous solution containing 5% glucose and normal saline. The pathogenesis of this patient's neurologic disorder is most closely related to which of the following conditions? a. Central pontine myelinolysis b. Purkinje cell atrophy c. Thiamine deficiency d. Viral encephalitis e. Vitamin B12 deficiency f. Scurvy

d. Sinoatrial heart block When a patient has no P waves and a low heart rate, it is likely that the impulse leaving the sinus node is totally blocked before entering the atrial muscle. This is called sinoatrial block. The ventricles pick up the new rhythm usually initiated in the AV node at this point, which reults in a heart rate of 40 to 60 beats/min. In contrast, during sinus bradycardia, you still have P waves associated with each QRS complex. In first-, second-, and third-degree heart block, you have P waves in each of these instances although some are not associate with the QRS complex. In atrial fibrillation, P waves are absent, but the QRS complexes occur irregularly.

A 60-year-old man has a blood pressure of 140/85 and weighs 200 lb. he reports that he is not feeling well, his EKG has no P-waves, he has a heart rate of 46, and the QRS complexes occur regularly. What is his likely condition? First-degree heart block b. Second-degree heart block c. Third-degree heart block d. Sinoatrial heart block e. Sinus bradycardia f. Atrial fibrillation g. Normal sinus rhythm

c. Calcitonin from the thyroid gland Calcitonin is released in response to elevated serum calcium levels and is responsible for depositing calcium ion on the bone to lower serum calcium.

A 60-year-old man with advanced lung cancer has increased serum concentrations of ionized calcium. Which of the following hormones is most likely to be secreted in response to his calcium concentration and from which site? a. Parathyroid hormone (PTH)-related peptide from the thyroid gland b. 1,25-Dihydroxyvitamin D from the kidneys c. Calcitonin from the thyroid gland d. PTH from the parathyroid gland e. Thyroid hormone from the thyroid gland f. 1,25-Dihydroxyvitamin D from the small intestine

g. 7000 Cardiac output is HR x Stroke Volume. The patient has a heart rate of 70 BMP and stroke volume can be determined from the P-V loop as SV = EDV (point B 150 mL) - ESV (point A -50 mL) = 100 mL. CO = 70 bpm * 100 mL= 7000 mL/min

A 60-year-old woman has a resting heart rate of 70 beats/minute, arterial pressure is 130/85 mm Hg, and body temperature is normal. Her pressure-volume diagram of the left ventricle is shown below. What is the patients cardiac output? a. 1000 b. 2000 c. 3000 d. 4000 e. 5000 f. 6000 g. 7000

b. At point D

A 60-year-old woman has a resting heart rate of 70 beats/minute, arterial pressure is 130/85 mm Hg, and body temperature is normal. Her pressure-volume diagram of the left ventricle is shown below. When does the second heart sound occur in the pressure-volume relationship? a. At point A b. At point D c. Between points A and B d. Between points B and C e. Between points C and D f. Between points D and A

g. G Moving from a supine to a standing position causes an acute fall in arterial pressure that is sensed by arterial baroreceptors located in the carotid sinuses and aortic arch. Activation of the baroreceptors results in a decrease in parasympathetic activity (or vagal tone) and an increase in sympathetic activity, which leads to an increase in plasma renin activity.

A 60-year-old woman has experienced dizziness for the past 6 months when getting out of bed in the morning and when standing up. Her mean arterial pressure is 130/90 mm Hg lying down and 95/60 when sitting. Which of the following sets of physiological changes would be expected in response to moving from a supine to an upright position? a. A b. B c. C d. D e. E f. F g. G h. H

e. It promotes sodium retention and potassium excretion in the distal tubules of nephrons Decreased blood perfusion of the kidney increases secretion of renin from the juxtaglomerular cells an enzyme that converts angiotensinogen to angiotensin I, which eventually results in aldosterone secretion from the adrenal gland. Compound X represents aldosterone, which acts primarily on the distal tubules of nephrons to promote sodium retention and potassium excretion.

A 60yearold hypertensive man is diagnosed with stenosis of a major renal artery. To compensate for decreased blood perfusion of the affected kidney, he produces an excess of the compound labeled X in the figure below. Which of the following best characterizes compound X? a. It causes arteriolar vasoconstriction b. It is produced by juxtaglomerular cells of the kidney c. It is secreted in response to infusions rich in Na+ d. It is under tight control of adrenocorticotropic hormone (ACTH) e. It promotes sodium retention and potassium excretion in the distal tubules of nephrons f. It is produced in the lungs and secreted in response to increased effective circulating volume

b. Left bundle branch block In the EKG, there is a QRS width greater than 0.12 seconds, this indicates a bundle branch block. There is also a left axis deviation which is consistent with a left bundle branch block.

A 60yearold woman had the following EKG recorded at a local emergency room following an automobile accident. Her weight was 70 kg and her aortic blood pressure was 140/80 mm Hg. What is the most likely diagnosis? a. Mitral valve stenosis b. Left bundle branch block c. Pulmonary valve stenosis d. Right bundle branch block e. Left ventricular hypertrophy f. ST segment elevation myocardial infarction

d. D Dilated cardiomyopathy is the most common form of cardiomyopathy. It is characterized by an increased end-diastolic volume and decreased ejection fraction due to dilated ventricles and decreased contractility. Only D demonstrates these changes with a rightward shift of the pressure-volume loop to a higher end-diastolic volume and a narrower loop representing a decreased stroke volume. A represents the changes that would be seen in aortic stenosis as there is no change in end-diastolic volume, but peak systolic volume is significantly increased due to increased afterload from the stenotic valve. B represents the changes that would be seen in impaired relaxation of the myocardium such as in left ventricular hypertrophy or diastolic heart failure. As shown, there is impaired filling of the heart demonstrated by a steepening of the bottom of the pressure-volume loop and a decreased end-diastolic volume. C demonstrates an increase in ejection fraction with no change in EDV, seen in instances of increased contractility such as during the response to exercise.

A 61-year-old gentleman experiences progressive dyspnea on exertion over the last few weeks. He also complains of swelling of his lower extremities and increased fatigue. After an echocardiogram is performed, he is diagnosed with dilated cardiomyopathy. Which of the following options represents this patient's pressure volume loop where the solid line represents the healthy baseline and the dotted line represents dilated cardiomyopathy? a. A b. B c. C d. D

e. Vitamin K Vitamin K is required for hepatic synthesis of seven proteins involved in blood coagulation.

A 61-year-old woman presents to her family physician with the chief complaint of frequent diarrhea accompanied by weight loss. He reports a tendency to bruise easily and laboratory data reveal a PT of 19 seconds (normal, 11-14 sec). the bruising and prolonged PT can be explained by a decrease in which of the following vitamins? a. Vitamin A b. Vitamin D c. Vitamin C d. Vitamin E e. Vitamin K f. Vitamin B12

b. Cyclic GMP Sildenafil protects cyclic guanosine monophosphate(cGMP) from degradation by cGMPspecific phosphodiesterase type 5 (PDE5) in the corpus cavernosum. Nitric oxide (NO) in the corpus cavernosum of the penis binds to guanylate cyclase receptors, which results in increased levels of cGMP, leading to smooth muscle relaxation (vasodilation) of the intimal cushions of the arteries of the penis.

A 61yearold man with erectile dysfunction asks his physician to prescribe Viagra (sildenafil citrate). Sildenafil produces its physiological effects by blocking the enzyme that hydrolyzes the second messenger by which nitric oxide produces its physiological effects. Which of the following is the second messenger? a. Gprotein b. Cyclic GMP c. Guanylyl cyclase d. cGMP phosphodiesterase e. Diacylglycerol f. Acetylcholine

d. It has a lower glucose concentration than plasma Concentration of glucose and protein in the CSF are lower than those of the plasma. Changes in CSF concentrations of these substances are helpful in detecting pathologic processes, such as tumor or infection, in which the blood-brain barrier is disrupted. CSF, which is in osmotic equilibrium with the ECF of the brain and spinal cord is made in the choroid plexus, circulates in the subarachnoid space, and is absorbed back into the circulation by the arachnoid villi.

A 62-year-old female with a history of multiple head injuries in the past becomes progressively more confused, and develops urinary incontinence and a gait disorder. An MRI of the brain (shown below) and lumbar puncture are performed. Intracranial pressure is found to be within normal limits. The patient's symptoms improved after ventriculoperitoneal shunting. Under normal conditions, which of the following statements correctly descries the cerebrospinal fluid (CSF)? a. It is absorbed into the choroid plexus b. It circulates in the epidural space c. It has a higher protein concentration than plasma d. It has a lower glucose concentration than plasma e. Its absorption is independent of CSF pressure f. It is produced by the arachnoid villi

c. Weak muscle fibers are recruited before strong muscle fibers In general, small, weak, slow, fatigue-resistant muscle fibers are innervated by small spinal motoneurons, whereas, large spinal motoneurons innervate large, fast, strong, easily fatigable muscle fibers. During most normal reflex or voluntary movements, small spinal motoneurons are recruited before large motoneurons. The slow fatigue-resistant muscle fibers have a dense capillary network for perfusion and use mitochondrial oxidative metabolism to produce adenosine triphosphate (ATP).

A 62-year-old male presents with a hypokinetic movement disorder characterized by bradykinesia (paucity and slowness of movement). During a normal voluntary movement, which of the following occurs? a. Large muscle fibers are recruited before small muscle fibers b. Fast muscle fibers are recruited before slow muscle fibers c. Weak muscle fibers are recruited before strong muscle fibers d. Poorly perfused muscle fibers are recruited before richly perfused muscle fibers e. Anaerobic fibers are recruited before aerobic fibers f. Only large muscle fibers are recruited

e. Increased total peripheral arteriolar resistance Given the tenderness, marked bruising, and drop in the blood pressure, this patient likely has significant internal bleeding. In response to hemorrhage, there is constriction of the peripheral blood vessels to return blood back to the internal organs. Constriction of these vessels cause an increase in total peripheral arteriolar resistance.

A 62-year-old man is involved in a head-on automobile collision and is examined by paramedics at the scene of the accident. The patient is conscious and responds to questions. There is marked tenderness along the left and right lower rib cage with significant bruising along the chest and abdomen. No open fractures are noted. His heart rate is 160 beats/min and his blood pressure is 70/40 mm Hg. The skin is cold and clammy. Which of the following hemodynamic changes is most likely to be present? a. Decreased arterial PO2 b. Decreased hemoglobin concentration c. Increased left ventricular end-diastolic pressure d. Increased mixed venous O2 (MVO2) content e. Increased total peripheral arteriolar resistance f. Decreased arterial PCO2 g. Increased effective circulating volume.

f. 350 mmol mannitol In order to reduced ICP, the excess fluid will need to be removed using an agent ant increases the osmotic pressure difference between the brain and the cerebral vessels. The solution must be hyperosmolar (>300 mOsm) and contain a solute that is impermeable to the blood-brain barrier to prevent further swelling. Only mannitol and urea are hyperosmolar and only mannitol is impermeable to the blood-brain barrier.

A 62-year-old man presents to the emergency department with an acute onset of aphasia and hemiparesis. A CT scan reveals an increase in intracranial fluid. Which of the following solutions will be most effective in reducing intracranial pressure (ICP) following a large hemispheric stroke? a. 150 mmol sodium chloride b. 150 mmol bicarbonate c. 250 mmol glycerol d. 250 mmol glucose e. 350 mmol urea f. 350 mmol mannitol

d. Increased concentration of cytosolic Ca2+ Normally, a Ca2+ ATPase pump pumps Ca2+ out of the cytosol into the interstitial fluid. Calcium within the cytosol activates phospholipase in the cell membrane, proteases in the cytosol, and endonucleases in the nucleus, which produces irreversible injury and death of the cell.

A 62-year-old man was admitted to the hospital with severe retrosternal chest pain that radiated down the left arm. Three days later, he developed a ventricular arrhythmia and died. In the hours shortly after the patient first experienced chest pain, which of the following biochemical changes would have marked the beginning of irreversible damage to the cardiac muscle? a. Decreased intracellular pH b. Decreased Na+, K+ - ATPase activity c. Increased activity of phosphofructokinase d. Increased concentration of cytosolic Ca2+ e. Increased conversion of pyruvate to lactate f. Increased pyruvate dehydrogenase activity

e. increased pulmonary capillary hydrostatic pressure In CHF, left ventricular dysfunction increases LV end-diastolic pressure, which raises left atrial pressure, pulmonary venous pressure, and pulmonary capillary hydrostatic pressure. This tends to drive fluid movement out of the pulmonary capillaries, according to Starling law. Thus pulmonary edema generally limited to the interstitium of the lungs is a hallmark of CHF.

A 62-year-old man with congestive heart failure develops increasing shortness of breath in the recumbent position. A chest x-ray reveals cardiomegaly, horizontal lines perpendicular to the lateral lung surface indicative of increased opacity in the pulmonary septa, and lung consolidation. Pulmonary edema in CHF is promoted by which of the following? a. Decreased pulmonary capillary permeability b. Decreased pulmonary interstitial oncotic pressure c. Decreased pulmonary interstitial hydrostatic pressure d. Decrease pulmonary capillary oncotic pressure e. increased pulmonary capillary hydrostatic pressure f. Increased pulmonary capillary oncotic pressure g. increased pulmonary interstitial hydrostatic pressure

C. C Medical treatment of gastric ulcers is aimed at restoring the balance between acid secretion and mucosal protective factors. Proton pump inhibitors are drugs that covalently bind and irreversibly inhibit the H+/K+ adenosine triphosphate (ATPase) pump, effectively inhibiting acid release. Therapy can also be directed toward histamine release, that is, H2 blockers, such as cimetidine (Tagamet), ranitidine (Zantac), famotidine (Pepcid), and nizatidine (Axid). These agents selectively block the H2 receptors int eh parietal cells. Antibiotic therapy is used to eradicate the H. pylori infection. NSAIDs can cause damage to the gastric mucosal barrier, which is a forerunner of gastric ulcer.

A 62-year-old man with dyspepsia and a history of chronic gastric ulcer has abdominal pain. Endoscopy shows a large ulcer in the proximal gastric body. Biopsies were positive for H. pylori. Which of the following are used clinically for treatment of gastric ulcers of various etiologies? a. A b. B c. C d. D e. E

D. D An enlarged pituitary gland causes compression of the optic chiasm which results in the visual field defect in D.

A 62-year-old woman is referred to a neurologist by her family physician because of a recent loss of initiative, lethargy, memory problems, and a loss of vision. She is diagnosed with primary hypothyroidism and an enlarged pituitary gland. She is referred to an endocrinologist for treatment of her thyroid problem and to a neuro-ophthalmologist for visual field evaluation. Which of the following visual field defects is most likely to be found? A. A B. B C. C D. D E. E

d. It has a lower glucose concentration than plasma CSF circulates throughout the subarachnoid space of the brain and spinal cord. The choroid plexuses secrete CSF into the ventricles, and the arachnoid granulations absorb it. For example, CSF has lower concentrations of K+ and amino acids than plasma does, and it contains almost no protein.

A 62-year-old woman with a history of multiple head injuries in the past becomes progressively more confused, and develops urinary incontinence and a gait disorder. Intracranial pressure is found to be within normal limits. The patient's symptoms improved after ventriculoperitoneal shunting. Under normal conditions, which of the following statements correctly describes the cerebrospinal fluid (CSF)? a. It is absorbed into the choroid plexus b. It circulates in the epidural space c. It has a higher protein concentration than plasma d. It has a lower glucose concentration than plasma e. Its absorption is independent of CSF pressure

d. Platelet function disorder The big clue for this question is the bleeding time. A bleeding time of 9-15 minutes is sign of platelet dysfunction. A bleeding time evaluation is used to measure the primary phase of hemostasis, which involves platelet adherence to injured capillaries and then platelet activation and aggregation. The bleeding time can be abnormal when the platelet count is low or the platelets are dysfunctional. Pt has a normal platelet count, but platelets are not functioning correctly. Antiphospholipid antibody syndrome causes increased coagulation, would have low bleeding time. Circulating anticoagulants should be suspected in patients with excessive bleeding combined with either a prolonged PTT or PT that does not correct when the test is repeated with a 1:1 mixture of normal plasma and the patient's plasma. Primary fibrinoloytic disorder will likely cause over coagulation. Isolated primary deficiencies of fibrinolytic factors (plasminogen, tissue plasminogen activator [tPA], and urokinase [uPA]) rarely cause intravascular thrombosis. On the other hand, overall functional fibrinolytic deficiency, as measured by appropriate clot lysis assays, has recently been shown to be a weak risk factor for thrombosis, and likely reflects alterations in inhibitor concentration or function. In both haemophilia A and B, there is spontaneous bleeding but a normal bleeding time, normal prothrombin time, normal thrombin time, but prolonged partial thromboplastin time.

A 63-year-old man with a 10-year history of osteoarthritis of the hips and knees develops urinary retention due to benign prostatic hyperplasia. He takes a nonsteroidal anti-inflammatory drug to relieve pain in his hips and knees. A transurethral resection of the prostate is performed to relieve the obstruction. Shortly after the procedure, there is profuse bleeding from the penis. Coagulation studies show a partial thromboplastin time (activated, aPTT) of 38 seconds (normal 30-50 seconds), a prothrombin time (PT) of 14 seconds (normal 9.5-13.5 seconds) a bleeding time is 14 minutes (normal 1-9 minutes), and a platelet count of 350,000/mm3. The d-dimer assay is negative. Which of the following is most likely responsible for the bleeding disorder? a. Antiphospholipid antibodies b. Circulating anticoagulant c. Coagulation factor deficiency d. Platelet function disorder e. Primary fibrinolytic disorder f. Hemophilia type A

d. Increased inhibition of the Na/K pump Digoxin and other cardiac glycosides cause direct vasoconstriction in the arterial and venous system in vascular smooth muscle. The positive inotropic effect of digitalis has the following 2 components: direct inhibition of the Na +/K +-ATPase, which leads to an increase in the intracellular concentration of calcium and associated increase in a slow inward calcium current during the action potential. Digitalis glycosides bind specifically to Na+/K+ -ATPase, inhibit its enzymatic activity, and impair active transport of extruding sodium and transport of potassium into the fibers (3:2 ratio). As a result, intracellular sodium gradually increases, and a gradual, small decrease in intracellular potassium occurs. Hypokalemia reduces the activity of the Na +/K +-ATPase. The Na +/K +-ATPase typically pumps potassium into the cell and sodium out of the cell. A reduction in the concentration of potassium outside of the cell inhibits the activity of the Na +/K +-ATPase to prevent driving extracellular plasma levels to dangerous lows. An excess of digitalis (digitalis toxicity) reduces the Na +/K +-ATPase activity, a reduction which is exacerbated by hypokalemia.

A 65-year-old man being treated with a beta-blocker and an ACE inhibitor for his heart failure presents to his cardiologist's office complaining of fatigue, weakness, shortness of breath, and an irregular heartbeat. An ECG reveals atrial fibrillation, so his cardiologist adds digoxin to his treatment regimen, but tells the patient that he will need to get his blood drawn to check for low K+ on a regular basis. Hypokalemia will increase the risk and severity of digitalis toxicity because of which of the following? a. Hypopolarization of cardiac muscle membranes b. Increased amplitude of cardiac muscle action potentials c. Increased excitability of cardiac muscle cells d. Increased inhibition of the Na/K pump e. Increased removal of cardiac cytosolic Ca2+ via the Na+/Ca2+ exchanger f. Decreased duration of cardiac muscle relative refractory period g. Hyperpolarization of the SA node pacemaker cells

b. A reduction in conduction velocity of the affected nerve Damage to the axon results in a loss of myelin as well as damage to the axon itself. As a result, conduction velocity in the affected nerve will decrease. The symptoms of peripheral nerve disease, or neuropathy, are numbness (i.e., a sensory deficit) and weakness (i.e., a motor deficit). Motor axons directly innervate and have "trophic" effects on skeletal muscle. If the axon is cut or dies, this trophic influence is lost and the muscle undergoes denervation atrophy. Peripheral neuropathy can affect both the upper and lower motor neurons.

A 65-year-old man is diagnosed with a form of peripheral neuropathy. Which of the following effects will this disorder have upon the patient? a. A loss in motor function, but sensory functions will remain largely intact b. A reduction in conduction velocity of the affected nerve c. An increase in the number of nodes of Ranvier d. Degeneration of myelin but the axon will typically remain intact e. Signs of an upper motor neuron paralysis

c. Reabsorption of bicarbonate ion in the proximal tubule, a large amount of H+ ion is secreted into the tubule lumen via a Na/K exchanger which secretes H+ into the lumen in exchange for Na+. Most of the secreted H+ combines with bicarbonate ion in the tubular fluid to form CO2 and water. The CO2 diffuses into the proximal tubular cells where the opposite reaction takes place to form H+ and HCO3-. The bicarbonate ion exits the cells on the basolateral side and enters the blood as reabsorbed bicarbonate. Carbonic anhydrase is located on the luminal surface of the cells as well as inside the cells to facilitate the above reactions. In type II RTA, a defect in proximal tubular bicarbonate reabsorption causes normal anion gap metabolic acidosis and bicarbonate wasting gin the urine (increased pH). Multiple myeloma, heavy metals, and carbonic anhydrase inhibitors are causes of type II RTA. The most common symptom of multiple myeloma is bone pain and compression fractures. Bence Jones proteins in the urine are also a common finding of multiple myeloma.

A 65-year-old man presents in the emergency department with a fracture of his right arm after slipping and falling on the ice. He reports that he has had back pain for the past 6 months. Blood results show Hb = 9 g/dL; hematocrit = 30%, BUN = 35 mg/dL; creatinine = 3 mg/dL. Urinalysis shows pH >5.3 and is positive for Bence Jones proteins. The patient is diagnosed with type II renal tubular acidosis (RTA) secondary to multiple myeloma. The transport of H+ into the proximal tubule is primarily associated with which of the following? a. Excretion of ammonium ion b. Excretion of potassium ion c. Reabsorption of bicarbonate ion d. Reabsorption of calcium ion e. Reabsorption of phosphate ion f. Excretion of sodium ion g. Reabsorption chloride ion

d. Oxygen extraction is higher than normal The lower than normal levels of venous oxygen tension and oxygen saturation indicate that the tissues have extracted more oxygen than normal. Oxygen extraction is the a-v oxygen content difference, and can be calculated from the data given, though that is not necessary to answer the question. Total oxygen content is the sum of the dissolved oxygen and the oxyhemoglobin content. Arterial oxygen content in this patient is therefore 21.6 mL O2/100 mL blood and venous oxygen content is 13.5 mL O2/100 mL blood with the a-v O2=8.1 mL O2/100 mL blood, compared with a normal value of approximately 5 mL O2/100 mL blood.

A 65-year-old man which chronic bronchitis is admitted to the emergency department with cyanosis and shortness of breath. Arterial and venous blood samples show the following: What do these data reveal about the patient's gas exchange and transport? a. Arteriovenous oxygen content difference is lower than normal b. Dissolved CO2 content in the arterial blood is lower than normal c. Dissolved O2 content the eh venous blood is higher than normal d. Oxygen extraction is higher than normal e. Oxyhemoglobin content in the arterial blood is higher than normal f. Ventilation is lower than normal while gas exchange at the capillary-alveolus interface is higher than normal g. Gas exchange at the capillary-alveolus interface is lower than normal while ventilation is higher than normal.

a. Cyclic guanosine monophosphate (cGMP) Nitric oxide is a potent vasodilator that acts by activating guanylyl cyclase, resulting in increased production of the second messenger cGMP.

A 65-year-old man with erectile dysfunction asks his physician to prescribe Viagra (sildenafil citrate). Sildenafil produces its effects by block the enzyme that hydrolyzes the second messenger by which nitric oxide produces its physiological effects. Which of the following is the second messenger? a. Cyclic guanosine monophosphate (cGMP) b. cGMP phosphodiesterase c. Diacylglycerol d. G protein e. Guanylate cyclase f. Adenylyl cyclase g. Calcium

d. Aspirin This patient is probably taking aspirin for her osteoarthritis. The risk of developing gastric ulcers is increased in all chronic nonsteroidal anti inflammatory drug (NSAID) users. NSAIDs function as cyclo oxygenase (COX) inhibitors, blocking prostaglandin production in the gastric mucosa. Prostaglandins are cytoprotective in the gastric mucosa preventing damage to the gastric mucosal barrier. Newer NSAIDs have been designed to reduce the risk of gastric complications of NSAID therapy.

A 65yearold woman with a history of congestive heart failure, osteoarthritis, and hypertension presents with pain and tenderness, which is relieved by food or antacids. Upper endoscopy reveals a gastric ulcer. Her current medications include digoxin, metoprolol, lisinopril, aspirin, hydrochlorothiazide, and vitamin D. Which of her medications is most likely to have contributed to her peptic ulcer disease? a. Digoxin b. Metoprolol c. Lisinopril d. Aspirin e. Hydrochlorothiazide f. Vitamin D

e. Parietal Parietal cells are responsible for secreting intrinsic factor which is necessary for the absorption of B12. Without parietal cells, exogenous B12 must be injected in the muscles to compensate for the inability to absorb B12 through the GI tract.

A 66-year-old man with Zollinger-Ellison syndrome undergoes a gastrectomy. He is informed that he will require treatment with intramuscular vitamin B12 (cyanocobalamin) for the rest of his life. This therapy is necessary because this patient lacks which of the following types of cells? a. Chief b. G (gastrin) c. Goblet d. Mucous neck e. Parietal f. D cells g. ECL

d. Lipase Lipase secreted from the pancreas is responsible for hydrolyzing triacylglycerides (TAGs) into monoacylglycerides (MAGs) which can then be absorbed by the small intestine. Without lipase, these fats remain undigested and appear in the feces resulting in steatorrhea.

A 67-year-old male with a history of alcohol abuse presents to the emergency room with severe epigastric pain, hypotension, abdominal distention, and diarrhea with steatorrhea. Serum amylase and lipase are found to be greater than normal, leading to a diagnosis of pancreatitis. The steatorrhea can be accounted for by a decrease in the intraluminal concentration of which pancreatic enzyme? a. Amylase b. Trypsin c. Chymotrypsin d. Lipase e. Colipase f. Pepsin g. Cholecystokinin

c. Increased preload ANP is released in response to increased stretch of the atria. If there is increased stretch on the atria it is due to increased fluid returning to the heart. Increased preload is the best answer.

A 67-year-old man is evaluated because of a one-week history of worsening fatigue, limb edema, shortness of breath, and a new requirement for using pillows to sleep. Laboratory studies show increased atrial natriuretic factor. Which of the following is the most likely cause of these findings? a. Improving cardiovascular function b. Increased afterload c. Increased preload d. Increased natriuresis

b. Zenker's diverticulum Zenker's diverticulum is a protrusion of the pharyngeal mucosa resulting from a loss of elasticity of the upper esophageal sphincter. Dysphagia and regurgitation slowly progress over years in middle-aged or elderly patients. Other symptoms may include throat discomfort, coughing, halitosis, and a protrusion in the neck. Treatment consists of surgical upper esophageal myotomy, and often diverticulectomy.

A 67-year-old man presents with mild dysphagia and throat discomfort that has slowly progressed over the past 5 years. He reports waking up at night choking and regurgitating undigested food. He has been troubled by halitosis and has noticed a small protrusion in his neck over the past year. He denies weight loss or heartburn. Which of the following is most consistent with his symptoms? a. Achalasia b. Zenker's diverticulum c. Gastroesophageal reflux disease d. Schatzki's ring e. Esophageal cancer

b. Pulmonary embolism Pulmonary embolism is characterized by the sudden onset of dyspnea (difficulty breathing) and chest pain along with rapid respiration rate (tachypnea), pleuritic chest pain (increased pain with deep breathing), and the presence of a new right axis deviation on the ECG.

A 67-year-old man suffering from metastatic prostate cancer arrives at your emergency department 2 h after the sudden onset of anterior chest pain and shortness of breath. The pain is enhanced by deep breathing movements. He is hypotensive and exhibits a rapid pulse and respiration rate. His blood pressure is 100/56 mmHg, pulse 136/min and respiration rate 28/min. Although the lungs are clear, an electrocardiogram (ECG) shows a right axis deviation not observed before. Which of the following is the most likely diagnosis? a. Pneumonia b. Pulmonary embolism c. Pneumothorax d. Lung cancer e. Esophageal spasm f. Atrial flutter

c. Compensated respiratory acidosis The patient has an elevated PCO2 as well as an elevated bicarbonate with a near-normal pH. This suggests that the initial injury was the increase in PCO2 that drove down the pH, a respiratory acidosis. The elevated bicarbonate serves to compensate for the decrease in pH. This patient's acid-base status is compensated respiratory acidosis.

A 67-year-old man with COPD comes to the physician for a routine follow-up examination. His temperature is 36.5 C, pulse is 85/min, respirations are 22/min, blood pressure is 132/78 mm Hg. Physical examination shows a barrel chested man in no acute distress. Auscultation reveals no wheezes but a prolonged expiratory phase is present. His arterial blood gas shows a plasma bicarbonate concentration of 32 mEq/L, an arterial PCO2 of 58 mm Hg, and a pH of 7.39. Which of the following best describes this patient's acid-base status? a. Compensated metabolic acidosis b. Compensated metabolic alkalosis c. Compensated respiratory acidosis d. Compensated respiratory alkalosis e. Uncompensated metabolic alkalosis f. Uncompensated metabolic acidosis g. Uncompensated respiratory alkalosis

e. Increased V wave With a history of rheumatic heart disease and a holosystolic murmur heard over the mitral area, this patient likely has mitral regurgitation. Regurgitation of the mitral valve results in backward flow of blood from the left ventricle to the left atrium during systole, resulting in an increased left atrial pressure, which is manifest as an increased v wave in a jugular pressure recording. Blood pressure is typically normal in patients with chronic mitral regurgitation, not decreased. Left ventricular preload is increased, and this elicits a greaterthan normal stroke volume. However, the forward stroke volume, the volume entering the aorta, does not increase, so there is no increase in pulse pressure or cardiac output.

A 67yearold man with a history of rheumatic heart disease presents with difficulty breathing when he exercises, which has worsened over the last year. Auscultation reveals a holosystolic murmur at the left 5th intercostal space along the midclavicular line. The murmur is loudest at the apex, radiates to the axilla, and is enhanced during expiration and when the patient is instructed to make a fist. Which of the following findings is most likely to be present? a. Decreased arterial pressure b. Decreased left ventricular preload c. Increased cardiac output d. Increased pulse pressure e. Increased V wave

e. Bone marrow All white blood cells originate from the bone marrow from myelocytes or lymphocytes.

A 68-year-old man with a history of chronic alcoholism developed chest pain and cough with an expectoration of sputum. A blood sample revealed that this white blood cell count was 42,000/mL. What is the origin of these white blood cells? a. Pulmonary alveoli b. Bronchioles c. Bronchi d. Trachea e. Bone marrow f. Thymus g. Adrenal gland

c. Afterload If afterload (aortic pressure) is reduced, stroke volume increases due to the decrease in end-systolic volume, accompanied by a smaller reduction in end-diastolic volume. This is the basis for giving an arterial dilator to enhance cardiac output in heart failure patients. A decrease in end-diastolic volume (preload), stroke volume, and heart rate all directly reduce the cardiac output equation. Decreased contractility indicates that the heart is likely putting out less blood.

A 68-year-old man with hypertension is evaluated for a six-month history of shortness of breath. He describes becoming short of breath with exertion, such as when walking up a flight of stairs. He also reports having difficulty lying flat at night and needs to use three pillows to breathe well. Blood pressure is 170/90 mm Hg. Auscultation reveals bibasilar crackles. There is no dilation of his neck veins, no ascites, and no lower extremity edema. Chest radiography shows cardiomegaly and signs of pulmonary edema. A decrease in which of the following is most likely to result in increased cardiac output in this patient? a. End-diastolic volume b. Stroke volume c. Afterload d. Contractility e. Heart rate f. Preload g. Aortic compliance

a. Decreased diffusing capacity of the lung Pulmonary fibrosis causes an increase in the thickness of the diffusional barrier, which results in a decrease in the diffusing capacity of the lung per Fick's Law of Diffusion. Fibrosis is characterized by a decrease in lung compliance and an increase in lung elastance which results in findings typical of a restrictive lung disease. Pulmonary function test values characteristic of restrictive disease includes a decrease in lung volumes and capacities, and a normal or increased FEV1/FVC ratio. Airway radius is decreased which results in an increase in airway resistance at low lung volumes, but in restrictive disease the airway resistance is normal when corrected for lung volume versus obstructive diseases where there is increased airway resistance.

A 68-year-old woman with pulmonary fibrosis presents with a complaint of increasing dyspnea while performing activities of daily living. She is referred for pulmonary function testing to assess the progression of her disease. Which of the following laboratory values is consistent with her diagnosis? a. Decreased diffusing capacity of the lung b. Increased residual volume c. Decreased forced expiratory volume exhaled in 1 second (FEV1)/forced vital capacity (FVC) d. Increased lung compliance e. Increased airway resistance corrected for lung volume f. Decreased pulmonary vascular resistance g. Increased turbulent flow in large-diameter vessels

a. It inhibits both gastrin and acetylcholine mediated secretion of acid Histamine stimulates gastric acid secretion by direct and indirect mechanisms. Histamine binds directly to its receptor on the parietal-cell membrane and synergistically stimulate acid secretion. The histamine receptor on the parietal cell is an H2 receptor that is coupled to the Gαs GTP-binding protein. Histamine activation of the receptor complex stimulates the enzyme adenylyl cyclase, which, in turn, generates cAMP. The resulting activation of protein kinase A leads to the phosphorylation of certain parietal-cell proteins, including the H-K pump. In the indirect method, ACh and gastrin indirectly induce acid secretion as a result of their stimulation of histamine release from ECL cells. H2 receptor antagonists (i.e., H2 blockers), such as cimetidine and ranitidine, not only block the direct action of histamine on parietal cells but also substantially inhibit the acid secretion stimulated by ACh and gastrin (which activate ECL cells). The effectiveness of H2 blockers in controlling acid secretion after stimulation by most agonists is well established in humans.

A 68-year-old woman with rheumatoid arthritis, who has been taking NSAIDs for the past 10 years, complains of burning epigastric pain that is relieved by antacids, but worsened with food. Her doctor discontinues the NSAIDs and recommends cimetidine because it is inexpensive and over the counter. Which of the following best describes the pharmacological blockade of histamine H2 receptors in the gastric mucosa? a. It inhibits both gastrin and acetylcholine mediated secretion of acid b. It inhibits gastrin-induced but not meal-stimulated secretion of acid c. It has no effect on either gastrin-induced or meal-stimulated secretion of acid d. It prevents activation of adenylyl cyclase by gastrin e. It causes an increase in potassium transport by gastric parietal cells. f. It stimulates the release of somatostatin from antral D cells inhibiting acid secretion from g. It stimulates gastrin-mediated secretion of acid and inhibits acetylcholine-mediated secretion of acid

d. D Alcoholic ketoacidosis is associated with a decrease in pH (high anion gap metabolic acidosis) and a compensatory rise in alveolar ventilation, which lowers the arterial PCO2. Hyperventilation also causes a slight increase in PO2.

A 69-year-old alcoholic came into the Emergency Department after he blacked out and fell, hitting his head on the kitchen floor. Measurement of serum Na+, glucose, BUN, and osmolality indicate an osmolar gap between calculated and measured osmolarity. Which of the following changes in arterial blood gas values are consistent with a presentation of ethanol-induced coma? a. A b. B c. C d. D e. E

d. Causing smooth muscle relaxation by stimulating the formation of nitric oxide, thereby causing an increase in the level of cyclic guanosine monophosphate (cGMP) Acting via the formation of a nitrosothiol intermediate, nitroglycerin and other nitrates increase the formation of nitric oxide. The nitric oxide activates guanylyl cyclase and thereby causes an increase in the level of cGMP. The cGMP activates its own kinase, which phosphorylates phosphatases. These phosphatases then dephosphorylate (inactivate) myosin light chains and cause smooth muscle relaxation. The relaxation of large veins and arteries reduces preload and afterload, respectively, and decreases the myocardial oxygen demand. Nitrates also redistribute blood flow along collateral vessels, and this could contribute to their beneficial effects. Nitroglycerin and other nitrates are useful in the management of Prinzmetal's angina (also called variant angina or vasospastic vasospasm) and chronic stable angina. Tolerance rapidly develops to the effects of nitrates because the drugs cause cellular depletion of sulfhydryl compounds.

A 69-year-old man complains of episodes of "squeezing" substernal chest pain, that occur when he physically exerts himself. He sometimes feels the pain in his jaw, and it often radiates down his left arm. After thorough evaluation, he is diagnosed with chronic stable angina (exertional angina) and instructed to take nitroglycerin. The mechanism by which nitroglycerin exerts its beneficial effects in angina is by: a. Blocking α1-adrenergic receptors and reducing myocardial oxygen consumption by decreasing cardiac pre-load and afterload b. Blocking β1-adrenergic receptors and reducing myocardial oxygen consumption by preventing an increase in heart rate and contractility c. Blocking slow Ca2+ channels and reducing myocardial oxygen consumption by decreasing cardiac contractility, heart rate, and arterial pressure d. Causing smooth muscle relaxation by stimulating the formation of nitric oxide, thereby causing an increase in the level of cyclic guanosine monophosphate (cGMP) e. Preventing platelet aggregation by inhibiting platelet cyclooxygenase activity f. Inhibiting muscarinic G-protein activity reducing potassium influx resulting in reduced membrane depolarization

c. Dihydrotestosterone (DHT) The prostate gland is one of the specific target tissues for DHT. DHT is primarily responsible for prostatic enlargement associated with aging in the male.

A 69-year-old man is diagnosed with benign prostatic hypertrophy (BPH). Which of the following hormones is primarily responsible for prostatic enlargement in BPH? a. Testosterone b. Androstenedione c. Dihydrotestosterone (DHT) d. Estrone e. Estradiol f. Luteinizing hormone

e. Sympathetic nerve activity Renin secretion is stimulated by the sympathetic nerves stimulating the juxtaglomerular apparatus. Sympathetic nerve activity increases when baroreceptors sense low blood pressure. Increasing mean arterial blood pressure decreases sympathetic nerve activity. ANG II decreases renin release by negative feedback. ANP reduces renin release. Increase in GFR is sensed by the macula densa and leads to the secretion of adenosine or ATP which contracts the afferent arteriole and decreases renin release. Decreased GFR results in increased renin release. An increase in both total peripheral resistance and cardiac output result in an increase in mean arterial pressure which causes a decrease in sympathetic nerve activity.

A 69-year-old man presents with symptoms of thirst and dizziness, and physical evidence of orthostatic hypotension and tachycardia, decreased skin turgor, dry mucous membranes, reduced axillary seating, and reduced jugular venous pressure. He was recently placed on an ACE inhibitor for his hypertension. Urinalysis reveals a reduction in the fractional excretion of sodium and the presence of acellular hyaline casts. The internist suspects acute renal failure of prerenal origin associated with increased renin secretion by the kidney. A stimulus for increasing renal renin secretion is an increase in which of the following? a. Angiotensin II b. Atrial natriuretic peptide (ANP) c. Glomerular filtration rate d. Mean arterial blood pressure e. Sympathetic nerve activity f. Total peripheral resistance g. Cardiac output

e. Sympathetic nerve activity A low effective circulating volume, sensed by baroreceptors located in the central arterial circulation, signals medullary control centers to increase sympathetic outflow to the JGA, which in turn increases renin release. An increase in ANG II results in vasoconstriction, and eventually increased mean arterial pressure, resulting in decreased renin secretion. ANP is released in response to increased atrial stretch, an indicator of increased mean arterial pressure, renin secretion in this case would be inhibited. An increase in mean blood pressure inhibits renin release from the juxtaglomerular apparatus.

A 69-year-old man presents with symptoms of thirst and dizziness, and physical evidence of orthostatic hypotension and tachycardia, decreased skin turgor, dry mucous membranes, reduced axillary sweating, and reduced jugular venous pressure. He was recently placed on an angiotensin-converting enzyme inhibitor for his hypertension. Urinalysis reveals a reduction in the fractional excretion of sodium and the presence of acellular hyaline casts. The internist suspects acute renal failure of pre-renal origin, which has increased renin secretion by the kidney. A stimulus for increasing renal renin secretion is an increase in which of the following? a. Angiotensin II b. Atrial natriuretic peptide c. Glomerular filtration rate d. Mean blood pressure e. Sympathetic nerve activity f. End-diastolic volume

d. Titin Titin is a large protein located between the Z line at the end of the sarcomere and the M line in the middle of the sarcomere. The resistance of the muscle to stretch is determined by the elasticity of the titin molecule. The titin in cardiac muscle is much stiffer than in the skeletal muscle, so it is more difficult to stretch cardiac muscle cells than it is to stretch skeletal muscle fibers.

A 70-year-old male with edema and congestive heart failure has diastolic dysfunction characterized by decreased ventricular filling due to reduced ventricular muscle compliance. Which one of the following proteins determines the normal stiffness of ventricular muscle? a. Calmodulin b. Troponin c. Tropomyosin d. Titin e. Myosin light chain kinase f. Actin

a. Intrinsic factor Intrinsic factor is secreted from parietal cells and is necessary for the absorption of vitamin B12. The patient has diminished capacity to secrete acid because of chronic gastritis. Because acid and IF are both secreted by parietal cells, a diminished capacity to secrete acid is usually associated with a diminished capacity to secrete intrinsic factor. Ptyalin, also known as salivary amylase, begins carbohydrate digestion in the mouth. Ptyalin secretion is not affected by gastritis. Chymosin is a proteolytic enzyme synthesized by chief cells in the stomach. Its role is to curdle or coagulate milk in the stomach.

A 71-year-old man with upper abdominal pain and blood in the stool takes NSAIDs for the pain and washes it down with whiskey. Pentagastrin administration produced lower than predicted levels of gastric acid secretion. Secretion of which of the following substances is most likely to be diminished in this patient with gastritis? a. Intrinsic factor b. Ptyalin c. Chymosin d. Lipase e. Trypsin f. Vitamin B12 g. Lactase

b. The substantia nigra Parkinson's disease is characterized by resting tremor rigidity and akinesia. It is caused by destruction of the dopamine secreting neurons within the substantia nigra of the basal ganglia. Levo (L)-dopa is a precursor for dopamine. L-dopa, rather than dopamine, is administered because it can cross the blood- brain barrier, but dopamine cannot. In contrast to the resting tremor of Parkinson's disease, cerebellar disease is characterized by an intention tremor. In contrast to damage to the nigrostriatal dopaminergic system in Parkinson's disease, Huntington's disease results in a loss of the intrastriatal GABAergic and cholinergic neurons in the caudate nucleus and putamen of the basal ganglion.

A 72-year-old man visits his physician because he finds it difficult to hold his hand steady when painting. Examination reveals a resting tremor and rigidity. The symptoms are relieved by single dose of levodopa, suggestive of Parkinson's disease. This patient's neurological signs are most likely related to a lesion within which of the following? a. The cerebellum b. The substantia nigra c. The premotor area d. The caudate nucleus and putamen e. The hippocampus

d. Hyperkalemia This question can be triaged without interpreting the EKG. Of the given options only hyperkalemia is alluded to in the question lead-in. The patient has a history of hypertension that is being managed with a potassium-sparing diuretic, in the absence of metabolic potassium deficiency, this patient could become hyperkalemic. The EKG shown also exhibits changes typical of hyperkalemia. Early EKG changes of hyperkalemia include the following: Tall, peaked T waves with a narrow base, best seen in precordial leads, shortened QT interval, ST-segment depression. EKG changes seen with increasing levels of serum potassium include peaked T waves, prolonged PR interval, decreased or disappearing P wave, widening of the QRS, and amplified R wave. The EKG above shows a widened QRS complex, diminished P waves, peaked T waves as well as a prolonged PR interval.

A 72-year-old woman with a history of HTN presents to emergency department with generalized weakness. Her medications include spironolactone, potassium sparing diuretic and an ACE inhibitor. Serum creatinine and BUN are elevated. The ECG is shown below. Which of the following electrolyte disturbances should be corrected to have the ECG return to normal? a. Hypocalcemia b. Hypokalemia c. Hypercalcemia d. Hyperkalemia e. Hypermagnesemia f. Hypercapnia g. Hypophosphatemia

b. The substantia nigra Parkinson's disease is characterized by resting tremor rigidity and akinesia. It is caused by destruction of the dopamine secreting neurons within the substantia nigra of the basal ganglia. Levo (L) dopa is a precursor for dopamine. Ldopa, rather than dopamine, is administered because it can cross the blood brain barrier, but dopamine cannot. In contrast to the resting tremor of Parkinson's disease, cerebellar disease is characterized by an intention tremor. In contrast to damage to the nigrostriatal dopaminergic system in Parkinson's disease, Huntington's disease results in a loss of the intrastriatal GABAergic and cholinergic neurons in the caudate nucleus and putamen of the basal ganglion.

A 72yearold man visits his physician because he finds it difficult to hold his hand steady when painting. Examination reveals a resting tremor and rigidity. The symptoms are relieved by single dose of levodopa, suggestive of Parkinson's disease. This patient's neurological signs are most likely related to a lesion within which of the following? a. The cerebellum b. The substantia nigra c. The premotor area d. The caudate nucleus and putamen e.The hippocampus f. The anterior hypophysis

a. Insufficient time for ventricular filling The conduction pathway must slow the impulse down in the AV node to allow time for atrial systole, which always follows atrial excitation. Electrical excitation precedes mechanical contraction.

A 73-year-old patient in the cardiology clinic suffers from an arrhythmia, which prevents the electrical impulse from slowing down at the atrioventricular (AV) node. Which of the following would be a direct result of this type of arrhythmia? a. Insufficient time for ventricular filling b. Augmentation of ventricular contraction c. Increase in venous return d. Insufficient time for atrial filling e. Increased aortic pressure f. Decreased vascular tone

c. Aortic stenosis Systolic ejection murmur with radiation to carotid is hallmark sign of aortic stenosis. SOB due to pulmonary congestion, syncope due to reduce CO. Both

A 75-year-old woman makes an appointment to see her physician because of exertional dyspnea and an episode of syncope while dancing with her husband at their granddaughter's wedding. A systolic ejection murmur is auscultated that radiates to the carotid arteries. Her signs and symptoms are most likely due to which of the following? a. Aortic regurgitation b. Pulmonic regurgitation c. Aortic stenosis d. Mitral stenosis e. Tricuspid stenosis f. Myocardial infarction

e. Stabilization of cardiac remodeling Ventricular remodeling refers to the changes in left ventricular mass, volume, and shape and the composition of the heart that occur after cardiac injury or abnormal hemodynamic loading conditions. Left ventricular remodeling contributes to the progression of heart failure. Angiotensin-converting enzyme (ACE) inhibitors can relieve the symptoms of heart failure with a depressed ejection fraction by stabilizing or reversing cardiac remodeling. This effect is due not only to angiotensin suppression by inhibition of ACE but also to bradykinin upregulation by inhibition of kininase II. An increase in ventricular end-diastolic pressure would not result from administration of Lisinopril and would result in further decompensation of the patient's CHF, not an improvement. Similarly, increased wall thickness, decreased cardiac output, and reduced effective circulating volume are signs of progression of heart failure, not an improvement. An increase in cardiac contractility would improve heart failure, but Lisinopril is not a positive inotropic agent or a negative chronotropic agent.

A 75-year-old woman presents with fatigue and orthopnea. Rales are heard widely over both lung fields. After several days of furosemide to treat the patient's fluid retention, Lisinopril therapy is initiated. Which of the following is primarily responsible for the improvement in her condition with Lisinopril? a. An increase in cardiac contractility b. An increase in ventricular end-diastolic pressure c. An increase in ventricular wall thickness d. A reduction in heart rate e. Stabilization of cardiac remodeling f. A decrease in cardiac output g. An decrease in effective circulating volume.

g. Increased renal bicarbonate ion production This patient has a respiratory acidosis as indicated by the significantly increased PCO2 and the corresponding decrease in plasma pH. The elevated bicarbonate concentration is due to increased production of bicarbonate ion in the kidney tubule

A 75-year-old woman with chronic bronchitis comes to the physician for a follow-up examination. The following data are obtained from an arterial blood sample: pH = 7.39, PCO2 = 55 mm Hg, Plasma bicarbonate concentration 32 mEq/L. Which of the following is the major compensatory response to this patient's acid-base disorder? a. Decreased renal ammonium ion excretion b. Hyperventilation c. Hypoventilation d. Increased excretion of free H+ e. Increased renal bicarbonate ion excretion f. Increased production of free H+ g. Increased renal bicarbonate ion production

b. A reduction in heart size In heart failure, the inability to pump enough blood to satisfy the energy requirements of the tissues leads to an increase in end diastolic volume and enddiastolic pressure, which can lead to pulmonary edema, hence the term congestive heart failure. In addition, the large enddiastolic volume increases the wall stress that must be developed by the heart with each beat, and this increases the myocardial requirement for oxygen. Administration of a positive inotropic drug such as digoxin increases the stroke volume, which increases the cardiac output to the tissues. The greater ejection fraction lowers the end diastolic volume, which decreases heart size and thus wall stress, according to LaPlace's Law. As a result, myocardial oxygen demand is lowered.

A 75yearold woman presents with fatigue, edema, and shortness of breath. Her physician prescribes a diuretic and a positive inotropic agent. Which of the following changes is primarily responsible for the improvement in her condition? a. A reduction in heart rate b. A reduction in heart size c. in ventricular enddiastolic pressure d. An increase in wall thickness e. An increase in cardiac excitability f. A decrease in stroke volume

g. Arteriosclerosis This patient has isolated systolic hypertension as evidenced by his elevated systolic pressure of 160 mmHg, but a normal diastolic pressure of 80 mmHg. Isolated systolic hypertension can be caused by underlying conditions such as overactive thyroid (hyperthyroidism), diabetes, and heart valve problems, but the most common cause is arteriosclerosis. It is the most common form of high blood pressure in people older than age 60.

A 77-year-old man comes to the physician because of occasional episodes of weakness and dizziness. Physical examination shows brisk, symmetrical pulses, a pulse of 90/min, and respirations are 12/min. His lungs are clear to auscultation. A chest x-ray reveals an enlarged left heart. The graph shows an aortic pressure pulse contour from a normal individual and one obtained from the patient. The patient most likely has which of the following conditions? a. Aortic valve stenosis b. Renal artery stenosis c. Mitral regurgitation d. Mitral valve stenosis e. Patent ductus arteriosus f. Aortic aneurysm g. Arteriosclerosis

e. Vital capacity Spirometry is used only to measure lung volumes and capacities that can be exchanged with the environment. Of the listed lung volumes and capacities listed, only vital capacity

A 78-year-old woman presents to her family physician's office with a chief complaint of fatigue and shortness of breath. The doctor indicates that he wants her to go to the hospital to get some pulmonary function tests, but there is one who is able to do in the office. A spirometer can be used to directly measure which of the following? a. Functional residual capacity b. Peak flow rate c. Residual volume d. Total lung capacity e. Vital capacity

d. Vitamin K Patients with CF have difficulties in absorption of fatsoluble vitamins - A, D, E, and K. Vitamin K deficiency leads to hypoprothrombinemia and a bleeding diathesis. Vitamin K is a cofactor required for carboxylation of four clotting factors: factors II, VII, IX, and X. Vitamin K is found in green vegetables, dairy products, and liver.

A 7yearold patient with cystic fibrosis (CF) has a 4week history of greasy, foul smelling diarrhea. In which of the following vitamins or minerals would he be most likely to be deficient? a. Iron b. Vitamin B6 c. Vitamin C d. Vitamin K e. Folic acid f. Vitamin B12

c. Osteoclasts Osteoprogenitor cells are "stem cells" for bone. They have the capacity to divide and proliferate to form osteoblasts, bone forming cells which actively produce bone tissue. Osteoblasts are the "bone forming cells" which secrete the collagen and ground substance that constitutes unmineralized bone (osteoid), and subsequently are responsible for calcification of the matrix. These cells also communicate with one another and with osteocytes by gap junctions. Osteocytes are mature bone cells, differentiated from osteoblasts, which are responsible for maintaining the bone matrix. They can synthesize and resorb (break down) the matrix to maintain homeostasis. Each osteocyte occupies a space, the lacuna, which conforms to the shape of the cell surrounded by matrix secreted when the cell was an osteoblast. Osteoclasts are large multinucleated cells whose function is to resorb bone. Osteoclasts dissolve the matrix and osteoid with acids and hydrolytic enzymes. Osteoclasts are phagocytic and are derived from monocytes and not from the same line as the other bone cells.

A 90yearold woman is brought to the emergency department 30 minutes after she fell while climbing the steps into her house. Physical examination shows tenderness over the right shin area. An xray of the right lower extremity shows a fracture of the tibia. A DEXA scan shows decreased bone density. Increased activity of which of the following cell types is the most likely cause of the decrease in bone mass in this patient? a. Chondrocytes b. Osteoblasts c. Osteoclasts d. Osteocytes e. Osteoprogenitor cells f. Osteons

c. Ascending limb of the loop of Henle Dehydration causes a decrease in the ECF which is sensed by the stretch receptors in the low pressure receptors in the great veins, right and left atria, and pulmonary vessels leading to an increase in ADH secretion from the posterior pituitary. The ascending limb of the loop of Henle is not affected by ADH and remains impermeable to water, thus as sodium and other electrolytes are reabsorbed from the ascending limb, its filtrate becomes hypotonic. The glomerular filtrate and proximal tubular fluid remain isotonic to plasma, which in the case of dehydration is higher than normal. In the presence of ADH, the cortical and medullary collecting tubules become permeable to water due to the insertion of AQP2 channels in the luminal membrane, and the filtrate within these portions of the nephron reaches osmotic equilibrium with the surrounding interstitial fluid.

A 92-year-old man presents with dehydration following 4 days of persistent diarrhea. Under this circumstance, hypotonic fluid would be expected in which of the following? a. Glomerular filtrate b. Proximal tubule c. Ascending limb of the loop of Henle d. Cortical collecting tubule e. Distal collecting duct f. Glomerular capsule g. Distal tubule

d. Polymerization of HbS as it is deoxygenated Persons with sickle cell anemia have a homozygous substitution of valine for glutamine in the sixth position of the beta hemoglobin chain. During periods of high oxygen consumption, the abnormal HbS polymerizes and distorts the normal shape of red blood cells. The sickled cells cause vasoocclusion in multiple organs, leading to renal papillary necrosis and hematuria, acute chest syndrome, ischemic retinopathy, and functional asplenism and splenomegaly.

A 9yearold African American boy is brought to the emergency department by his mother who states that he was complaining of muscle aches and pain while playing basketball, which became worse whenever he was running up and down the court. She reports that he was sick with a fever last week, but she thought he was feeling better, so she let him go to summer basketball camp. Blood tests show anemia, increased reticulocyte count, and crescent shaped cells. Hgb electrophoresis confirms the presence of HbS. The primary mechanism for the change in RBC shape during a sickle cell crisis is which of the following? a. A decrease in erythrocyte volume during dehydration b. A rightward shift in the oxyhemoglobin dissociation curve of HbS compared with normal c. Low levels of erythropoietin d. Polymerization of HbS as it is deoxygenated e. The presence of antibodies against the red blood cell membrane f. Breakdown of the extracellular matrix of the erythrocyte

e. Increased net fluid movement out of the pulmonary capillaries Pulmonary artery wedge pressure (PAWP) measured with a Swan Ganz catheter is an index of the pulmonary capillary hydrostatic pressure. Normal PAWP is 212 mmHg. Thus, an elevated PAWP of 25 mmHg will drive fluid movement out of the pulmonary capillaries, according to Starling's law. Because of the paucity of smooth muscle in the pulmonary vasculature, pulmonary vascular resistance is distributed evenly across the arterioles, capillaries, and venules and there is a steady decline in pressures across the pulmonary circulation, with no large pressure drop across any element, such as seen in the systemic arterioles. As a result, PAWP is also an index of the pressures downstream in the left atrium and in the left ventricle at the end of diastole; that is, PAWP is an index of left ventricular preload.

A SwanGanz catheter is placed in a postmyocardial infarction (MI) patient in the Medical Intensive Care Unit. The patient is found to have a pulmonary artery wedge pressure of 25 mmHg, which is indicative of which of the following? a. Decreased pulmonary capillary hydrostatic pressure b. Decreased left atrial pressure c. Decreased left ventricular enddiastolic pressure d. Decreased left ventricular preload e. Increased net fluid movement out of the pulmonary capillaries f. Decreased radius of pulmonary capillaries

d. Number of motor units recruited The amount of tension generated in the muscle is dependent on the number of motor units recruited. Increased number of motor units results in increased tension in the muscle.

A demonstration is performed during a lecture on muscle physiology in which a student is asked to fully extend his right arm with the palm up. Two large textbooks are placed on his palm, one at a time. Which of the following facilitates the maximum amount of tension that allows the student to keep his arm extended in place under the increasing weight of the books? a. Amount of Ca2+ released from the sarcoplasmic reticulum b. Amount of muscle phosphocreatine c. Amplitude of the action potential d. Number of motor units recruited e. Rate of cross-bridge recycling f. Size of the motor unit recruited

c. Urine flow The increase in blood glucose concentration will result in a filtered load of glucose in excess of what the proximal tubule is able to absorb. As a result, glucose will remain in the filtrate, where it will act as an osmotic diuretic increasing urinary flow. The excess blood glucose will cause water to shift from the intracellular compartment to the extracellular compartment, causing a decrease in intracellular volume and, by dilution, a decrease in plasma sodium concentration. The accompanying increase in ketoacid production will result in a metabolic acidosis (low pH) and a compensatory increase in alveolar ventilation, lowering alveolar PCO2.

A diabetic patient presents with a blood glucose concentration of 600 mEq/L. Which of the following values would likely be above normal in hyperglycemia? a. Arterial pH b. Alveolar PCO2 c. Urine flow d. Intracellular volume e. Plasma sodium concentration f. Venous return

b. Compliance Surfactant is used to break the surface tension between water molecules at the alveoli-air interface. Without surfactant, the downward pull of the force between water molecules causes the lung to collapse. Without surfactant, the compliance of the lungs is dramatically reduced, and the alveoli cannot overcome the water tension.

A female newborn delivered at 32 weeks' gestation develops severe respiratory distress within hours of birth. Despite resuscitative efforts, the patient dies. Examination of the lungs at autopsy shows lung alveoli with radii of less than 50 μm (N=100). Which of the following is most likely decreased in the lungs of this newborn? a. Airway resistance b. Compliance c. Elastic recoil d. Surface tension e. Vascular resistance f. Transpulmonary pressure g. Bronchiole diameter

c. Impaired endothelin B receptor function Hirschsprung disease or aganglionic megacolon is an inherited disorder of abnormal colonic motility due to a congenital absence of the ganglion cells in both the myenteric and submucousal plexuses of a segment of the distal colon, as a result of failure of the normal cranial-to-caudal migration of neural crest cells during development. The action of endothelin on the endothelin B receptor is necessary for normal migration of certain neural crest cells. Megacolon develops in knockout mice lacking endothelin B receptors. In addition, one cause of congenial aganglionic megacolon in humans appears to be a mutation in the endothelin B receptor gene. The disease is typically diagnosed in infancy, and affects as many as 1 in 5000 live births. The absence of peristalsis in patients with this disorder causes feces to pass the aganglionic region with difficulty, and children with the disease my defecate as infrequently as once every 3 weeks. Findings also include abdominal distention, anorexia, and lassitude. The symptoms of Hirschsprung disease can be relieved completely if the aganglionic portion of the colon is resected and the portion of the colon above it anastomosed to the return. If an extensive segment is involved, patients may require a colectomy.

A full-term newborn infant with abdominal distention has not had a bowel movement for 5 days. An x-ray of the abdomen shows a narrowed colon, bowel obstruction, and dilated intestine above the obstruction. A suction rectal biopsy is done at the bedside. The pathology report indicates the absence of ganglion cells and the presence of non-myelinated nerves in the biopsy segment. What is the underlying cause of the bowel obstruction in the patient? a. Activation of intestinal opioid receptors b. Carcinoid tumor c. Impaired endothelin B receptor function d. Impaired gastrocolic reflex e. Increased activity of splanchnic sympathetic nerves f. Increased absorption of fluids from the small intestine g. Delayed activity of the interstitial cells of Cajal

E. E-D ERV is the maximal volume of gas that can be exhaled in excess of a passive tidal expiration. ERV is not labeled but can be calculated from the difference between the functional reserve capacity (FRC) and the residual volume (RV), labeled as E and D respectively. The FRC is the volume of gas remaining in the lungs following a passive, tidal exhalation. The residual volume is the volume of gas is the volume of gas remaining in the lungs after a maximal expiration. A is the inspiratory reserve volume, B is the inspiratory capacity, C is the vital capacity.

A healthy 30-year-old woman is referred for a life insurance physical exam. History reveals that she has never smoked and vesicular breath sounds are heard at the periphery of the lung with auscultation. In the patient's spirometry tracing below, the expiratory reserve volume (ERV) equals which of the following? A. point C B. Point D C. Point E D. C+D E. E-D F. A+B+C G. C-A

d. Intercostals and abdominal recti Though quiet expiration is passive and does not utilize the contraction of muscles to occur, forced expiration utilizes the accessory muscles of expiration to make intrapulmonary pressure more positive. These muscles include the abdominal muscles, the intercostals, and the neck and back muscles.

A healthy, 25-year-old medical student participates in a 10-km run for the American Heart Association. Which of the following muscles does the student use (contract) during forced expiration? a. Diaphragm and external intercostals b. Diaphragm and internal intercostals c. Diaphragm only d. Intercostals and abdominal recti e. Scaleni f. Sternocleidomastoid muscles g. Diaphragm and external obliques

f. Inositol 1,4,5-trisphosphate Remember the Gq pathway. Gq induces PLC to cleave PIP2 into IP3 and DAG. IP3 diffuses through the membrane of the ER or SR, binds to its receptor and causes release of calcium.

A hormone is known to activate phospholipase C with subsequent release of calcium from internal stores. The release of calcium most likely occurs as a result of an increase in the concentration of which of the following intracellular second messengers? a. Calcium b. ATP c. cAMP d. cGMP e. Diacylglycerol f. Inositol 1,4,5-trisphosphate

b. Hypokalemia, hypochloremia, and metabolic alkalosis Analysis of serum electrolytes reveals low potassium (hypokalemia), low chloride (hypochloremia), and metabolic alkalosis. These abnormalities arise from two sources. First, gastric juice contains potassium and chloride in concentrations higher than found in the plasma. Loss of gastric juice through vomiting or drainage leads to depletion of these electrolytes from the plasma. Second, the metabolic abnormalities are exacerbated by the student's dehydration. Contraction of the vascular volume leads to orthostatic hypotension and the activation of renal mechanisms important for conserving volume. As a result, water, sodium, and bicarbonate are reabsorbed at the expense of increased potassium and hydrogen excretion.

A medical student presents to the emergency room with a 2-day his- tory of severe vomiting and orthostatic hypotension. Which of the following metabolic abnormalities are most likely present in this patient? a. Hypokalemia, hypochloremia, and metabolic acidosis b. Hypokalemia, hypochloremia, and metabolic alkalosis c. Hyperkalemia, hyperchloremia, and metabolic alkalosis d. Normal serum electrolytes and metabolic acidosis e. Normal serum electrolytes and metabolic alkalosis f. Normal serum electrolytes and normal acid-base balance

f. Protein excretion is increased by sympathetic stimulation of the kidney Approximately two thirds of the 40 to 150 mg of protein excreted per day by the kidney is derived from plasma proteins. The remainder is derived from the tubular secretion of a mucoprotein, the Tamm Horsfall protein, that is present in tubular casts appearing in urinary sediment. Not all plasma proteins are filtered equally because glomerular permeability is related to molecular size and charge. The larger and negatively charged proteins are poorly filtered. Most of the filtered protein is reabsorbed in the proximal tubule unless the filtered load exceeds the tubular capacity. Such overload would occur following damage to the glomerular basement membrane and breakdown of normal barriers, or following an increase in the plasma concentration of a small protein, such as myoglobin. Protein excretion is also increased by sympathetic stimulation, such as that occurring during exercise. In this situation, renal vasoconstriction reduces the glomerular filtration rate, which, by increasing the transit time of glomerular filtrate, favors diffusion of proteins across the basement membrane. The presence of protein in the urine indicates glomerular dysfunction. RBC casts are indicative of glomerulonephritis. A red color indicates the presence of hemoglobin, myoglobin, or red food.

A hospitalized patient becomes oliguric with increasing BUN and creatinine. Urine specific gravity is low and the urine sediment has red cell casts with hematuria and proteinuria. Which of the following statements concerning the normal renal handling of proteins is correct? a. Proteins are more likely to be filtered if they are negatively charged than if they are uncharged b. Proteins can be filtered and secreted but not reabsorbed by the kidney c. Most of the protein excreted each day is derived from tubular secretion d. Protein excretion is directly related to plasma protein concentration e. Protein excretion depends on the rate of tubular flow f. Protein excretion is increased by sympathetic stimulation of the kidney

e. Pulmonary edema Lung compliance is defined as the ease with which the lungs are expanded, and is calculated as the change in lung volume per change in transpulmonary pressure. Curve Z has a lower slpe than normal and is the curve of an individual with decreased lung compliance. In pulmonary edema, the abnormal accumulation of fluid in the lungs causes a pulmonary impairment that prevents the lungs from expanding and lowers compliance. Asthma causes an increase in airway resistance, and is not associated with a change in lung compliance. Emphysema and aging-associated changes in the lung cause an increase in lung compliance. An L/S ratio greater than 2 indicates normal biochemical maturation of the lung in utero, indicating normal compliance.

A hospitalized patient has tachypnea and significantly labored respirations requiring mechanical ventilation. Based on the pressure-volume curve of the lungs shown as curve Z in the figure below, which of the following is the most likely diagnosis for the patient? a. Asthma b. Emphysema c. Dyspnea with aging d. Newborn with lecithin to sphingomyelin (L/S) ratio greater than 2 e. Pulmonary edema f. Type II pneumocytes hyperplasia g. Hypersecretion of surfactant

d. A decrease in the excretion of creatinine The excretion of creatinine, which is neither reabsorbed nor secreted in any significant amount, is dependent on filtration, which is in turn dependent on renal plasma flow. The decrease in renal plasma flow that accompanies chronic renal failure results in a decrease in creatinine excretion and an increase in plasma creatinine concentration. The increase in plasma creatinine concentration is used to assess the percentage of nonfunctioning nephrons in renal failure. Interestingly, the remaining nephrons adapt to renal failure. To maintain Na+ balance, less Na+ is reabsorbed, so the fractional excretion (the fraction of filtered Na+ that is excreted) goes up. Although the remaining nephrons are able to excrete a larger than normal amount of H+, secretion cannot fully compensate for the reduced number of nephrons because there is a limit to the amount of NH4+ that can be synthesized by the proximal tubules. Therefore, despite the overall increase in net acid excretion, H+ accumulation leads to a metabolic acidosis. Even creatinine secretion can be increased so the plasma creatinine concentration does not increase proportionally to the amount of renal damage. The anion gap increases because of the reduced excretion of phosphate and other anions that are included in the anion gap. Free water clearance decreases because there is decreased filtration. Therefore, to prevent overhydration in patients with renal failure, water intake must be limited.

A hypertensive patient develops chronic renal failure from progressive nephrosclerosis. Which of the following should you expect to occur as a result? a. A decrease in the fractional excretion of sodium b. An increase in the free water clearance c. A decrease in net acid excretion d. A decrease in the excretion of creatinine e. No change in the anion gap f. An increase in bicarbonate production

d. graded contraction occurs only in the skeletal muscles Though a single cardiac fiber is able to execute graded contractions, the contraction of cardiac muscle as a whole does not contract with varying degrees of strength. This is an ability reserved for the skeletal muscle in which there are graded changed in the membrane potential. These allow for the skeletal muscle to elicit varying degrees of strength.

A major difference between the cardiac muscle and skeletal muscle is that a. only cardiac cells are made up of sarcomeres b. the sliding filament hypothesis explains only skeletal muscle contraction c. length-tension relationship in the heart does not depict an optimum length d. graded contraction occurs only in the skeletal muscles e. only skeletal muscle has both thick and thin filaments f. Cardiac muscle lacks sarcomeres

e. 150 beats/min The rhythm of this EKG appears regular, so to determine the rate divide 300 by the number of large boxes between each QRS complex. 300/2=150 beats/min.

A male patient has a myocardial infarction at age 55. He is now 63 years old. Standard limb lead II is shown below. What is his heart rate? a. 40 beats/min b. 50 beats/min c. 75 beats/min d. 100 beats/min e. 150 beats/min f. 180 beats/min

b. Hypokalemia, hypochloremia, and metabolic alkalosis Analysis of serum electrolytes reveals low potassium (hypokalemia), low chloride (hypochloremia), and metabolic alkalosis. These abnormalities arise from two sources. First, gastric juice contains potassium and chloride in concentrations higher than found in the plasma. Loss of gastric juice through vomiting or drainage leads to depletion of these electrolytes from the plasma. Second, the metabolic abnormalities are exacerbated by the student's dehydration. Contraction of the vascular volume leads to orthostatic hypotension and the activation of renal mechanisms important for conserving volume. As a result, water, sodium, and bicarbonate are reabsorbed at the expense of increased potassium and hydrogen excretion.

A medical student presents to the emergency room with a 2day his tory of severe vomiting and orthostatic hypotension. Which of the follow ing metabolic abnormalities are most likely present in this patient? a. Hypokalemia, hypochloremia, and metabolic acidosis b. Hypokalemia, hypochloremia, and metabolic alkalosis c. Hyperkalemia, hyperchloremia, and metabolic alkalosis d. Normal serum electrolytes and metabolic acidosis e. Normal serum electrolytes and metabolic alkalosis f. Normal serum electrolytes without metabolic disturbance

e. Suppression of ADH secretion due to polydipsia Excess psychogenic polydipsia causes complete suppression of ADH secretion and extreme polyuria, as evidenced by a urine volume of 7.0 L in 24 hours. Kidneys work normally to remove the excessive ingested water by creating maximally dilute urine.

A middle-aged man is evaluated for polydipsia and polyuria. His 24-hour urine volume is 7.0 L with a low osmolality, and is negative for red blood cells, leukocytes, protein, and glucose. His blood pressure and heart rate are both normal. Which of the following is a likely cause of polyuria in this patient? a. Acute attack of diabetes mellitus without glucosuria b. Acute renal failure with a severe decrease in glomerular filtration rate (GFR) c. Cerebrovascular accident damaging the anterior pituitary gland d. Renal supersensitivity to the effects of antidiuretic hormone (ADH) e. Suppression of ADH secretion due to polydipsia f. Hyperproduction of mineralocorticoid from the adrenal cortex

d. Chronic respiratory acidosis with considerable renal compensation Chronic respiratory acidosis with considerable renal compensation is indicated by the arterial pH being only slightly acidic despite elevated CO2 levels. The patient's history indicates severe chronic obstructive pulmonary disease (COPD) due to chronic asthma. The laboratory data indicate hypercapnia that is associated with HCO3− generation by the kidney. Due to obstructive disease, the patient has increased CO2 production, and the patient's lung problem of poor alveolar ventilation enhances CO2 retention. The increased CO2 retention is associated with the observed hypoxemia.

A middle-aged woman has had asthma since childhood and has been a heavy smoker since her early teens. During the past few years, she has experienced progressive dyspnea (breathing difficulty) and somnolence (sleepiness). Physical examination reveals a cachectic (general ill health and malnutrition) female with shortness of breath, prolonged expirations, and frequent coughing. Laboratory data are as follows: arterial pH 7.35; arterial HCO3−32 mEq/L; arterial PCO260 mmHg; and arterial PO2, 60 mmHg. Which of the following is a likely diagnosis? a. Acute metabolic acidosis with renal compensation b. Acute respiratory acidosis without renal compensation c. Chronic metabolic acidosis with considerable renal compensation d. Chronic respiratory acidosis with considerable renal compensation e. Respiratory acidosis with metabolic acidosis

f. Congenital adrenal hyperplasia As the infant has normal female internal structures and ovaries, the formation of ambiguous external genitalia is probably due to excess adrenal androgens produced by congenital adrenal hyperplasia. Individuals with Turner's syndrome have normal female external genitalia, but an absence of internal structures with streak ovaries. Thus, this syndrome does not match the patient description in this case. In Klinefelter's syndrome the individual has a male phenotype and male internal sex organs, not ovaries, uterine tubes, uterus, and vagina and female phenotype, as described here.

A neonate is brought to the endocrinologist with ambiguous external genitalia. It is determined that the infant has ovaries and internal female genitalia, but external virilization (ambiguous genitalia). Which of the following diagnoses is the most likely? a. Turner's syndrome (45, XO) b. Klinefelter's syndrome (47, XXY) c. Hypersecretion of anti-mullerian hormone d. Deficient estrogens during fetal life e. 5α-reductase mutation f. Congenital adrenal hyperplasia

a. Increased chloride conductance

A new drug is developed that proposes to decrease the rate of action potentials by increasing the amount of inhibitory postsynaptic potentials a cell experiences. Which of the following is the most likely mechanism of action for such a drug? a. Increased chloride conductance b. Decreased potassium conductance c. Spontaneous release of a quantal package of Ach d. Increased sodium conductance e. Inhibition of the sodium potassium pump

b. The gastrocolic reflex Distention of the stomach by food initiates contraction of the rectum and often, a desire to defecate. This response is called the gastro colic reflex, but it may be mediated by the action of gastrin on the colon rather than being neurally mediated. This response leads to defecation after meals in infants and children. In adults, defecation may be inhibited by behavioral modification. The gastroileal reflex refers to the relaxation of the cecum and passage of chyme through the ileocecal valve when food leaves the stomach. Peristaltic rushes are very intense peristaltic waves that may occur with intestinal obstruction. The intestinointestinal reflex refers to a complete cessation of intestinal motility that may be caused by large dis tensions of the intestine, injury to the intestinal wall, or various intestinal bacterial infections. The defecation reflex refers to the sudden distention of the walls of the rectum produced by mass movement of fecal material into the rectum.

A new mother calls the pediatrician because she is concerned that her infant defecates after every meal. Which of the following is the cause of these normal bowel movements in newborns? a. The gastroileal reflex b. The gastrocolic reflex c. The intestinointestinal reflex d. The defecation reflex e. Peristaltic rushes f. Spinal cord reflex

d. 21α-hydroxylase The patient has a congenital adrenal hyperplasia, which is most commonly attributable to 21α-hydroxylase deficiency. Aldosterone and cortisol are not produced, and their precursors enter androgen synthesis pathways, thereby causing female genital abnormalities. Although a 11β-hydroxylase deficiency will produce congenital adrenal hyperplasia, CAH is most commonly produced by 21α-hydroxylase deficiency. Aldosterone is not produced, but its precursor, 11β-deoxycorticosterone acts as a mineralocorticoid while androgens are overproduced to cause masculinization similar to that found in 21α-hydroxylase deficiency.

A newborn presents with somewhat ambiguous external genitalia. The genitalia appear as an enlarged clitoris and a scrotum-like structure that appears to result from labial fusion. An ultrasound reveals normal ovarian development. The karyotype is 46 XX. The findings are most likely due to a deficiency in which one of the following enzymes a. 11β-hydroxylase b. 16α-hydroxylase c. 17α-hydroxylase d. 21α-hydroxylase e. 3β-hydroxysteroid dehydrogenase f. 5 α-reductase

b. The cardiac action potential passes through the AV node PR interval indicates the time between atrial and ventricular depolarization. The AV node is the intermediate point between these two occurrences.

A pacemaker is inserted in a patient in order to shorten the PR interval detected on his ECG. Which of the following events normally occurs during the PR interval? a. The ventricle is contracting b. The cardiac action potential passes through the AV node c. There is no change in the voltage tracing on the ECG d. The mitral and aortic valves are both closed e. The second heart sound is heard f. Ventricular contraction

a. Basolateral Na+K+ATPase, basolateral K+ channels, and apical Na+ channels A basolateral Na+/K+ pump will keep intracellular Na+ levels low to promote NaCl movement into the cell. Basolateral K+ channels are necessary to keep intracellular K+ levels from becoming too high. Apical Na+ channels are needed to facilitate NaCl movement into the apical portion of the cell.

A particular epithelium has a paracellular (between the cells) pathway permeable to water and chloride. In order to enhance the net movement of NaCl and water from apical to basolateral portions of the cell, the epithelium needs: a. Basolateral Na+K+ATPase, basolateral K+ channels, and apical Na+ channels b. Apical Na+K+ATPase, apical K+ channels, and basolateral Na+ channels c. Basolateral Na+K+ATPase, apical K+ channels, and basolateral Na+ channels d. Apical Na+K+ATPase, basolateral K+ channels, and apical Na+ channels e. Basolateral Na+K+ATPase, basolateral K+ channels, and basolateral Na+ channels f. Apical Na+K+ATPase, apical K+ channels, and apical Na+ channels

b. Epinephrine The movement of K+ into cells is facilitated by the presence of insulin and epinephrine. During exercise, epinephrine hastens the movement of K+ into muscle cells, preventing the accumulation of K+ in the extracellular space around active muscle cells. In cases of life-threatening hyperkalemia, insulin is often injected (along with glucose) to reduce the plasma K+ concentration.

A patient comes into the Emergency Department exhibiting signs of hyperkalemia. The extracellular potassium of a hyperkalemic patient can be decreased by administering which of the following drugs? a. Atropine b. Epinephrine c. Glucagon d. Lactic acid e. Isotonic saline f. Ringer's solution

d. Purkinje fibers The most rapid conduction of the action potential occurs through the Purkinje fibers. The slowest conduction occurs in the AV node. Pacemaker cells located within the SA node initiate the cardiac action potential normally. The action potential propagates from the SA node into the atrial muscle fibers. It then passes through the AV node and the His-Purkinje network to the ventricular muscle fibers. The rapid conduction of the action potential through the His-Purkinje net- work ensures rapid and synchronous activation of the entire ventricular muscle. The slow conduction through the AV node produces a delay between atrial and ventricular systole, allowing the ventricle to receive the blood ejected by the atria before it contracts.

A patient complaining of an irregular heart beat is referred for a cardiac electrophysiological (EP) study. Propagation of the action potential through the heart is fastest in which of the following? a. SA node b. Atrial muscle c. AV node d. Purkinje fibers e. Ventricular muscle f. Bundle of His

g. Resistive work of breathing Methacholine is a cholinergic agonist, which causes constriction of bronchial smooth muscle. Bronchoconstriction reduces airway radius, which increases airway resistance and thus the resistive work of breathing. Methacholine decreases the anatomic dead space but has no significant effect on lung compliance and thus does not affect elastic work of breathing.

A patient complains of paroxysmal episodes of not being able to catch her breath. When no abnormalities are detected with conventional pulmonary function testing, the pulmonologist orders a methacholine challenge test. Which of the following will increase as a result of stimulating cholinergic receptors on the bronchial smooth muscle? a. Airway diameter b. Anatomic dead space c. Compliance of the lungs d. Elastic work of breathing e. Tidal volume f. Diameter of the alveoli g. Resistive work of breathing

c. Tissue plasminogen activator tPA will dissolve the clot. Pt is within the window for tPA therapy, stroke is non-hemorrhagic.

A patient had a non-hemorrhagic ischemic stroke visualized by non-contrast computed tomography of the head at two hours post symptom onset. Which of the following is the most appropriate therapy? a. Warfarin b. Calcium channel blockers c. Tissue plasminogen activator d. Prevention of vasospasm e. Infusion of mannitol f. Hyperventilation

d. Plasma atrial natriuretic factor concentration Sustained hypersecretion of aldosterone results in sustained reabsorption of sodium from the tubule and secretion of potassium. The reabsorption of sodium promotes reabsorption of water and raises the blood volume and pressure. The sustained increase in blood volume causes the atria of the heart to stretch promoting release of atrial natriuretic factor. Aldosterone has a negative feedback effect on renin and ANG II.

A patient has a tumor of the adrenal gland, causing a sustained hypersecretion of aldosterone (primary aldosteronism). Which of the following parameters is elevated above that found in a normal individual ingesting an identical diet? a. Plasma renin concentration b. Plasma angiotensin II concentration c. Plasma potassium concentration d. Plasma atrial natriuretic factor concentration e. Plasma chloride concentration f. Plasma angiotensin I concentration g. Plasma phosphate concentration

a. A large increase in antidiuretic hormone secretion To answer this question, you will need to keep in mind normal lab values. Normal urine volume/24 hr is 800-2000 mL. Normal urine 24-hour osmolality is 500-800 mOsm/kg while normal random urine osmolality is 300-900 mOsm/kg of water. Normal plasma osmolality is approximately 300 mOsm (275-295 mOsm). Normal serum potassium is 3.5-5.0 mM. These results are most consistent with a large increase in ADH secretion. The reduced urine output, increased urine osmolality, reduced plasma osmolality, and slightly elevated from normal blood pressure indicate that water is being reabsorbed from the kidney back into the plasma. The normal plasma potassium indicates that there is not an increase in aldosterone secretion because aldosterone promotes secretion of potassium, and a large increase in its secretion would significantly decrease plasma potassium.

A patient has the following clinical data: Urine volume/24h = 600 mL, Urine osmolality = 1100 mOsm, Plasma osmolality = 260 mOsm, Blood pressure = 132/80 mmHg, Plasma potassium = 4.1 mM. Which of the following could produce these data? a. A large increase in antidiuretic hormone secretion b. A large increase in aldosterone secretion c. A large increase in angiotensin II d. The total absence of aldosterone e. The total absence of antidiuretic hormone f. A significant decrease in atrial natriuretic peptide secretion g. A significant decrease in renin secretion

a. Volume depletion Hyperkalemia can result from an increased K+load, decreased K+excretion, or shift of K+from the intra- to the extracellular fluid, such as occurs in acidosis. Volume depletion decreases distal tubular flow and NaCl delivery to the distal tubule, which decreases K+excretion, leading to hyperkalemia. The most common cause of elevated potassium in lab results is a laboratory error called pseudohyperkalemia, a falsely elevated measurement due to hemolysis of the blood specimen with leakage of potassium from the lysed cells. The test should be repeated if you suspect pseudohyperkalemia.

A patient is brought to the Emergency Department with weakness, confusion, and shortness of breath. The EKG reveals QRS widening and flattened P waves typical of serum potassium concentrations exceeding 7.5 mEq/L. Hyperkalemia may be observed in patients with which of the following? a. Volume depletion b. Diuretic therapy c. Administration of insulin d. Metabolic alkalosis e. Stimulation of adrenal medulla

a. Gastrin On rare occasions, patients with one or more ulcers have very high rates of gastric acid secretion. The increased acid secretion in these patients is most often a result of elevated levels of serum gastrin, released from a pancreatic islet cell adenoma or gastrinoma. Because gastrin released from these islet cell adenomas is not under physiological control, but rather is continuously released, acid secretion is substantially increased under basal conditions.

A patient is evaluated because of a history of multiple peptic ulcers. Abdominal magnetic resonance imaging shows a pancreatic mass. Biopsy of the mass reveals a tumor of the pancreatic islet cells. Which of the following hormones does this tumor most likely secrete? a. Gastrin b. Insulin c. Somatostatin d. Glucagon e. Vasoactive intestinal peptide f. Histamine g. Secretin

c. Reduced conduction velocity of atrioventricular (AV) nodal fibers The P-R interval represents the time for electrical activity to pass through the AV node. As the drug in this case prolonged the P-R interval, it caused a longer delay in electrical activity than normal in the AV nodal fibers.

A patient is given a drug that prolongs the P-R interval of the electrocardiogram (ECG). What is the most likely action of the drug? a. Reduced conduction velocity of the atrial fibers b. Delayed repolarization of the ventricular fibers c. Reduced conduction velocity of atrioventricular (AV) nodal fibers d. Increased conduction velocity of the atrial fibers e. Increased force of contraction of the right ventricle f. Increased conduction velocity of the of the Purkinje fibers

c. Blocks the conversion of angiotensin I to angiotensin II. The reduced activation of AT1 receptors causes increased vasorelaxation. ACE stands for Angiotensin Converting Enzyme, and is responsible for the conversion of angiotensin I to angiotensin II. ANG II binds to AT1 receptors and cause vasoconstriction in the afferent and efferent arterioles, but preferentially constricts the efferent arteriole. ACE inhibitors prevent this conversion causing relaxation.

A patient is hypertensive and has a low ejection fraction. A cardiologist prescribes an ACE inhibitor for him. What is the mechanism of action and how is this drug helping the patient? a. Inhibits action of the NKCC cotransporter in the ascending loop of henle. Reduces patient's total blood volume. b. Blocks the production of renin. Renal filtration increases allowing for a reduced total blood volume. c. Blocks the conversion of angiotensin I to angiotensin II. The reduced activation of AT1 receptors causes increased vasorelaxation. d. Inhibits the Na/K pump in the distal part of the nephron. Increases sodium retention leading to increased blood volume and increased preload. e. Blocks the conversion of angiotensinogen to angiotensin I. This causes vasorelaxation and increases GFR. f. Stimulates the activity of AQP2 in the distal nephron allowing for more water to be secreted. This reduces total blood volume. g. Stimulates the production of aldosterone. Increases sodium excretion from the kidney tubule which lowers the blood volume.

c. C This diagram depicts cardiac (ventricular) function and venous function curves. With the onset of exercise, there is an increase in contractility, which shifts the cardiac function curve up. Also accompanying the onset of exercise are decreases in total peripheral resistance and venous compliance, both of which shift the vascular function curve to the right and increase its slope. The point at which the cardiac function and venous function curves intersect (C) represents the central venous pressure and cardiac output of the cardiovascular system under these conditions.

A patient is referred to the Heart Station for exercise stress testing. Baseline and exercise levels of cardiac and venous function are measured and plotted on the graphs below. The point marked "Control" represents baseline cardiovascular function in the resting state in the supine position. During treadmill exercise, there will be a shift from the resting state to which of the following points? a. A b. B c. C d. D e. E

b. Left ventricular mean systolic pressure If afterload has decreased, should have a lower mean systolic pressure indicating a reduced force against which the LV has to contract.

A patient presents to the emergency room with intermittent chest pain. The EKG and blood tests are negative for a myocardial infarction. However, an echocardiogram shows some thickening of the left ventricular muscle and narrowing of the aortic valve. Medications to lower afterload and preload are prescribed. Which of the following values should be measured to determine if the medications have been effective in lowering left ventricular afterload? a. Left ventricular end-diastolic pressure b. Left ventricular mean systolic pressure c. Pulmonary capillary wedge pressure d. Total peripheral resistance e. Mean arterial blood pressure f. Cardiac output

g. Left ventricular mean systolic pressure If left ventricular afterload has been reduced, left ventricular mean systolic pressure will also be reduced. Afterload is a measure of the force against which the left ventricle must contract to expel blood from the heart. A reduction in afterload allows the heart to pump out more blood while generating less force to do so. The pressure in the left ventricle is reduced as a result.

A patient presents to the emergency room with intermittent chest pain. The EKG and blood tests are negative for a myocardial infarction. However, an echocardiogram shows some thickening of the left ventricular muscle and narrowing of the aortic valve. Medications to lower afterload and preload are prescribed. Which of the following values should be measured to determine if the medications have been effective in lowering left ventricular afterload? a. Left ventricular end-diastolic pressure b. Cardiac output c. Pulmonary capillary wedge pressure d. Total peripheral resistance e. Mean arterial blood pressure f. Right atrial pressure g. Left ventricular mean systolic pressure

a. 0.54 The ejection fraction (EF) is equal to the stroke volume (SV) divided by the enddiastolic volume (EDV) and the stroke volume (SV) is equal to the enddiastolic volume (EDV) minus the endsystolic volume (ESV). EF = SV/EDV = EDV - ESV/EDV. In this case, the enddiastolic volume is 130 mL and the endsystolic volume is 60 mL. Therefore, the ejection fraction is 70 mL/130 mL or 0.54.

A patient undergoes cardiac catheterization to assess his left ventricular function prior to thoracic surgery. What is his ejection fraction, as determined from the left ventricular pressurevolume curve illustrated below? a. 0.54 b. 0.60 c. 1.20 d. 70 mL e. 120 mmHg

b. A vagotomy of the distal stomach Patients often experience a sense of fullness (early satiety), abdominal bloating, and nausea following a vagotomy of the antrum. The stomach has three important motor functions: storage, trituration, and emptying. The ability of the stomach to serve as a reservoir is characteristic of the orad stomach (gastric fundus and corpus). Upon ingestion of a mixed meal (liquids and solids) a vago-vagal-mediated reflex, receptive relaxation, limits the rise in intragastric pressure in response to an increase in intragastric volume. Sectioning of the vagus nerve or gastric resection of the orad stomach abolishes the reflex and leads to an increased intragastric pressure with a resultant increase in the gastric emptying of liquids. Trituration, the mechanical breakdown of large, solid food particles into smaller particles, is essential for the gastric emptying of solids. Trituration is the result of rhythmical contractions characteristic of the gastric antrum. Normally, solids do not empty until they are smaller than 1 to 2 mm in diameter. Distension of the antrum initiates both local, enteric neural reflexes and long, vago-vagal reflexes that increase the strength of antral contractions and promote the gastric emptying of solids. The loss of vagal input to the antrum markedly impairs the emptying of solids and often leads to an early sense of fullness.

A patient undergoes surgery for their GI symptoms. Which of the following procedures may result in nausea and a sense of early satiety? a. Surgical resection of the proximal small bowel b. A vagotomy of the distal stomach c. Surgical resection of the proximal stomach d. Surgical removal of the gastric antrum e. A vagotomy of the orad (proximal) stomach f. A vagotomy of the colon

d. The central chemoreceptors would be stimulated Increasing arterial PaCO2 stimulates both the central and peripheral chemoreceptors. In respiratory acidosis, the increase in CO2 drives the CO2 hydrolysis equation to the right, with dissociation of carbonic acid into H+ and HCO3-. The ratio of HCO3- to dissolved CO2 decreases, causing a fall in pH but the absolute concentration of plasma HCO3- increases slightly because some of the hydrogen ions are buffered by non-bicarbonate buffers. In renal compensation for the respiratory acidosis, the increased arterial hydrogen ion concentration increases H+ secretion by the distal tubule, lowering the pH of the urine and increasing NH4+ excretion. The excretion of H+ is accompanies by the generation of new bicarbonate, causing the plasma [bicarbonate] to increase to a greater extent, which helps the pH return to normal.

A patient with Guillain-Barre syndrome develops paralysis of the respiratory muscles that increases PaCO2 from 40 to 60 mm Hg and increases the concentration of hydrogen ion in arterial blood from 40 mEq/L (pH=7.4) to 50 mEq/L (pH 7.3). as a result, which of the following would happen? a. The plasma bicarbonate ion concentration would decrease b. the pH of the urine would increase c. The amount of ammonium excreted in the urine would decrease d. The central chemoreceptors would be stimulated e. The peripheral chemoreceptors would be inhibited f. The amount of calcium phosphate buffer in the blood would increase g. The plasma hydrogen ion concentration would increase

e. Its synthesis is increased in respiratory acidosis Ammonia (NH3) is produced from amino acids in the cells of the renal tubules (mainly the proximal tubules), and its rate of production increases during acidosis. This is important in acidosis because it increases the total amount of H+ ion that can be excreted in a given volume of urine. The NH3 freely diffuses into the tubular lumen, and because of the high pKa (9.2) of the reaction, essentially all of it com bines with H+ to form NH4+. This maintains the driving force for more NH3 to passively diffuse into the lumen. The NH4+ that is formed gets "trapped" in the tubules and excreted because the tubules are impermeable to this cation.

A patient with Type IV renal tubular acidosis (RTA) has a defect in aldosterone secretion and ammonium excretion. Which of the following statements about ammonia (NH3) is correct? a. It is impermeable to the epithelial cells of the proximal tubule b. It is classified as a titratable acid c. It is produced by epithelial cells in the distal nephron d. It reduces the concentration of bicarbonate in the plasma e. Its synthesis is increased in respiratory acidosis

e. Somatostatin Somatostatin is the central inhibitor of gastric acid secretion. Somatostatin inhibits gastric acid secretion by both direct and indirect mechanisms. In the direct pathway, somatostatin coming from two different sources binds to a Gαi-coupled receptor (SST) on the basolateral membrane of the parietal cell and inhibits adenylyl cyclase. The net effect is to antagonize the stimulatory effect of histamine and thus inhibit gastric acid secretion by parietal cells. Somatostatin also acts via two indirect pathways, both of which are paracrine. In the corpus of the stomach, D cells release somatostatin, which inhibits the release of histamine from ECL cells. Because histamine is an acid secretagogue, somatostatin thus reduces gastric acid secretion. In the antrum of the stomach, D cells release somatostatin, which inhibits the release of gastrin from G cells. Because gastrin is another acid secretagogue, somatostatin also reduces gastric acid secretion by this route. The gastrin released by the G cell feeds back on itself by stimulating D cells to release the inhibitory somatostatin.

A patient with alcoholic cirrhosis presents to the emergency room with hematemesis. After stabilizing him with IV fluids, the gastroenterologist administers an analogue of which of the following agents to inhibit gastric acid secretion and visceral blood flow? a. Acetylcholine b. Gastrin c. Histamine d. Pepsin e. Somatostatin f. Secretin g. Cholecystokinin

b. An increase in blood ammonia One of the actions of colonic bacteria is to convert NH3 to NH4+. Thus, a total colectomy would increase blood ammonia levels, which would be exacerbated in a person with cirrhosis. NH3 is in equilibrium with NH4+. Most of the NH4+ formed by oxidative deamination of amino acids in the liver is converted to urea, and the urea is excreted in the urine. The NH4+ forms carbamoyl phosphate, and in the mitochondria it is transferred to ornithine by ornithine carbamoyl- transferase forming citrulline. Citrulline is converted to arginine, after which urea is split off and ornithine is regenerated (urea cycle). Most of the urea is formed in the liver, and in severe liver disease the blood urea nitrogen falls and blood NH3 rises. Normally, about 5 to 10% of bile salts enter the colon. In the colon, bacteria convert the two primary bile acids, cholic acid and chenodeoxycholic acid to the secondary bile acids, deoxycholic acid and lithocholic acid, respectively. Lithocholate is relatively insoluble and is mostly excreted in the stool, but deoxycholate is reabsorbed from the colon, where it is transported back to the liver in the portal vein and reexcreted in the bile (enterohepatic circulation). Humans can survive after total removal of the colon if fluid and electrolyte balance is maintained. When total colec- tomy is performed, the ileum is brought out through the abdominal wall (ileostomy). Advances in surgical techniques make ileostomies relatively trouble-free and patients with them can lead essentially normal lives.

A patient with cirrhosis undergoes a total colectomy. Removal of the entire colon would be expected to cause which of the following? a. Decreased urinary urobilinogen b. An increase in blood ammonia c. Megaloblastic anemia d. Severe malnutrition e. Death f. Dehydration

d. Na+ reabsorption by the distal nephron decreases The amount of potassium excreted is controlled by the amount of potassium secreted by the distal tubule. Potassium secretion is a passive process that depends on the electrochemical gradient between the distal tubular cells and the tubular lumen and the permeability of the luminal cells to potassium. By inhibiting Na+ reabsorption, the intraluminal potential becomes less negative and K+ secretion is reduced. K+sparing diuretics such as amiloride act in this fashion. Aldosterone increases the intracellular potassium concentration by augmenting the activity of the NaK pump and increasing the potassium permeability of the luminal membrane. Increasing dietary intake increases the plasma potassium concentration, which in turn stimulates aldosterone production. Increasing the rate of distal tubular flow increases the rate of K+ secretion. The high flow maintains a low tubular K+ concentration and therefore increases the electrochemical gradient for K+ secretion.

A patient with hypertension is given a K+sparing diuretic. The amount of potassium excreted by the kidney will decrease if which of the following occurs? a. Distal tubular flow increases b. Circulating aldosterone levels increase c. Dietary intake of potassium increases d. Na+ reabsorption by the distal nephron decreases e. The excretion of organic ions decreases f. Circulating renin levels increase

c. Anemia The kidney produces a number of important hormones, including erythropoietin. Erythropoietin is necessary for the normal production of red blood cells. The anemia associated with renal failure results from the decrease in the synthesis of erythropoietin. Often, the first clinical sign of renal failure is the fatigue produced by anemia.

A patient with renal failure develops symptoms caused by the loss of a hormone produced by the kidney. Which of the following is the most likely diagnosis? a. Edema b. Hypertension c. Anemia d. Uremia e. Acidosis f. Alkalosis

b. Anemia The kidney produces several important hormones including erythropoietin (EPO), 1,25-(OH)2-vitamin D, renin, and prostaglandins. EPO is necessary for the normal production of red blood cells and is released from the endothelial cells of the peritubular capillaries in response to hypoxia. The anemia associated with renal failure results from the decrease in the synthesis of EPO. Often the first sign of renal failure is fatigue produced by anemia. Decreased conversion of 1-OH-vitamin D to 1,25-(OH)2-vitamin D in patients with chronic renal failure leads to renal osteodystrophy or secondary hyperparathyroidism. Renin production in patients with chronic renal failure is actually increased due to decrease in GFR leading to decreased salt delivery to the macula densa cells of the distal convoluted tubule. This activates the RAAS system which increases blood pressure as well as sodium and water retention leading to fluid overload/edema. Uremia is a clinical syndrome marked by large increases in BUN and creatinine which develops as a later complication of chronic renal failure when kidney function is markedly reduced. Metabolic acidosis is also a complication of chronic renal failure due to failure of the kidneys to excrete the daily acid load.

A patient with renal failure presents to the clinic with increasing fatigue for the past month. Based on a thorough history and physical, as well as diagnostic testing, it is determined that the symptoms are caused by the loss of a hormone produced by the kidney. Which of the following is the most likely diagnosis? a. Acidosis b. Anemia c. Edema d. Hypertension e. Uremia f. Alkalosis

d. The tension developed by the muscle being moved The GTO is located in the tendon of skeletal muscles and therefore is in series with the muscle. Each time the muscle contracts, the GTO is stretched in proportion to the tension developed by the muscle. The Ib afferent fibers (which innervate the GTO) produce a train of action potentials with a frequency that is in proportion to the deformation of the GTO. The muscle length and speed of shortening are sent to the CNS by Ia afferents that innervate the intrafusal fibers within muscle spindles.

A patient with severe kyphoscoliosis presents with increasing dyspnea consistent with a decrease in chest wall compliance. With contraction of the external intercostal muscles during inspiration, the Golgi tendon organ (GTO) provides the central nervous system with information about which of the following? a. The length of the muscle being moved b. The velocity of muscle movement c. The blood flow to the muscle being moved d. The tension developed by the muscle being moved e. The change in joint angle produced by the movement f. The number of muscle fibers recruited

c. Na+-K+-pump The intracellular Na+ concentration of renal epithelial cells is pumped out of renal epithelial cells by Na+-K+ pumps located on the basolateral surface of the epithelial cells. The Na+/H+ exchanger and the Na+-glucose cotransporter are located on the apical surface of the epithelial cells. Na+ is transported from the peritubular spaces to the capillaries by solvent drag.

A patient with uncontrolled hypertension is placed on a new diuretic targeted to the Na+ reabsorption site from the basolateral surface of the renal epithelial cells. Which of the following transport processes is the new drug affecting? a. Na+/H+-exchange b. Na+-glucose co-transport c. Na+-K+-pump d. Facilitated diffusion e. Solvent drag f. Water reabsorption

c. Foreign body in the bronchus The presence of bright red frothy blood indicates a respiratory tract origin caused by trauma to the tracheobronchial tree, probably due to an inhaled foreign body.

A previously healthy 2-year-old girl who developed a sudden onset of persistent coughing is brought to the pediatric clinic. She has no fever and now is coughing up bright red, frothy blood. What is the most likely cause of this child's problem? a. Arteriovenous malformation of the lung b. Cystic fibrosis c. Foreign body in the bronchus d. Pneumonia e. Tuberculosis f. Asthma

e. Norepinephrine ANG II preferentially constricts the efferent arteriole though it constricts the afferent and efferent. ACE inhibitors prevent the formation of ANG II. Prostaglandins dilate the afferent arteriole. Nitric oxide acts as a vasodilator and also serves as a signaling molecule of the macula densa to stimulate renin release. Adenosine constricts the afferent arteriole preferentially. Norepinephrine equally constricts the afferent and efferent arterioles.

A researcher in a laboratory wants to find an agent that will cause equal constriction of the afferent and efferent arterioles in the kidney and therefore decrease the renal blood flow and glomerular filtration rate. Which of the following agents is most appropriate? a. Angiotensin II b. ACE inhibitor c. Prostaglandins d. Nitric oxide e. Norepinephrine f. Adenosine

e. Pulmonary vascular resistance increases Pulmonary vascular resistance (PVR) varies with lung volume. PVR is lowest at functional residual capacity, and increases at both high and low lung volumes. The increased pulmonary vascular resistance seen at total lung capacity (TLC) is due to an increase in the resistance of the alveolar vessels (pulmonary capillaries) caused by compression by the high alveolar volume. The increased pulmonary vascular resistance seen at residual volume is due to an increase in the resistance of the extraalveolar vessels caused by compression by the positive intrapleural pressure generated during a forced expiration. As lung volume increases toward TLC, the compliance of the lungs (slope of pressurevolume curve) decreases and elastic recoil of the lung increases, that is, the lungs resist further expansion. Intrapleural pressure becomes more subatmospheric during inspiration, approaching -30 cm H2O at TLC; as a result, the transmural pressure across the intrathoracic airways (Pin - Pout) increases, which increases airway radius, and decreases airway resistance according to Poiseuille's law.

A sedentary individual goes to her physician's office for a cardiopulmonary exercise stress test evaluation prior to starting a weight loss exercise regimen. Physiological changes that occur during deep inspirations as lung volume approaches total lung capacity include which of the following a. Elastic recoil of the lungs decreases b. Lung compliance increases c. Intrapleural pressure becomes less subatmospheric d. Airway resistance increases e. Pulmonary vascular resistance increases f. Diffusing capacity of the lung increases

a. All transmitters released by a single neuron have the same effect on postsynaptic cells (e. g., excitatory or inhibitory). A single neuron can release excitatory or inhibitory neurotransmitters. This allows for the volume of the nervous system to remain relatively small, instead of requiring a neuron for each neurotransmitter.

A single neuron can release more than one neurotransmitter. All of the following statements about this are true EXCEPT: a. All transmitters released by a single neuron have the same effect on postsynaptic cells (e. g., excitatory or inhibitory). b. A neuron can differentially release transmitters depending on the frequency and pattern of action potentials. c. A neuron can release both a fast-acting transmitter such as acetylcholine, and a slow-acting transmitter such as a peptide. d. A neuron can store different transmitters in distinct populations of vesicles. e. Release of transmitters from synaptic vesicles is Ca2+-dependent, regardless of the chemical nature of the transmitter.

d. Extrinsic pathway The only abnormal lab finding in this patient is the partial thromboplastin time (PTT) which is significantly elevated from normal. The PTT is used to measure the activity of the extrinsic pathway.

A teenaged boy with numerous nosebleeds was referred to a physician for evaluation prior to a minor surgery. His prothrombin time (PT) was 11 sec (11-15 sec normal), partial thromboplastin time (PTT) was 58 sec (normal 25-40 sec) and bleeding time was 6.5 minutes (normal 2-7 min). which of the following is most likely abnormal in this young man. a. Number of platelets b. Function of platelets c. Intrinsic pathway d. Extrinsic pathway e. Clotting factor production by the liver f. Vitamin K level g. Fibrinogen production

e. Increased permeability of the distal nephron to water Vasopressin (Anti-diuretic hormone) promotes the insertion of aquaporin 2 (AQP2) channels into the distal nephron. This allows water to be reabsorbed from the renal tubule back into the circulation.

A trauma patient with multiple rib fractures requires intubation and mechanical ventilation. Mechanical ventilation causes an increase in the patient's vasopressin secretion and plasma levels. Which of the following is the effect of vasopressin on the kidney? a. Increased diameter of the renal artery b. Increased glomerular filtration rate c. Increased excretion of Na+ d. Increased excretion of water e. Increased permeability of the distal nephron to water f. Increased reabsorption of potassium g. Increased diameter of the afferent arteriole

c. Oxygen concentration gradient The transport of oxygen across a capillary wall is proportional to the capillary surface area, capillary wall permeability to oxygen, and oxygen gradient across the capillary wall. Thus, a twofold increase in the oxygen concentration gradient would result in the greatest increase in the transport of oxygen across the capillary wall. A twofold increase in the intercellular clefts in the capillary wall and the radius of the capillary would not significantly impact oxygen transport. A twofold increase in the thickness of the capillary wall would decrease oxygen transport.

A twofold increase in which of the following would result in the greatest increase in the transport of oxygen across the capillary wall? a. Capillary hydrostatic pressure b. Intercellular clefts in the capillary wall c. Oxygen concentration gradient d. Plasma colloid osmotic pressure e. Capillary wall hydraulic permeability f. Thickness of the capillary wall g. Radius of the capillary

a. The corpus luteum must secrete progesterone to sustain the endometrium. The corpus luteum in the ovary at the time of fertilization fails to regress and instead enlarges in response to stimulation by human chorionic gonadotropin (hCG) secreted by the placenta. During the first trimester, placental production of hCG sustains the corpus luteum and ensures continued progesterone secretion by the corpus luteum, which is essential for development of the fetus.

A young couple has been trying to conceive a baby. The Medical Director of the Fertility Center has advised the woman to take her basal temperature readings on a daily basis and for them to have intercourse at the time the woman appears to be ovulating. Once conception takes place, which of the following must occur in order for the pregnancy to proceed uneventfully? a. The corpus luteum must secrete progesterone to sustain the endometrium. b. The pituitary must secrete hCG to maintain the corpus luteum c. The pituitary must secrete prolactin to sustain the placenta d. The placenta must secrete FSH to maintain ovarian function e. The placenta must secrete LH to maintain ovarian function f. The pituitary must secrete estrogen to maintain ovarian function

d. Dihydrotestosterone The testosterone metabolite dihydrotestosterone (DHT) induces the formation of the male external genitalia and male secondary sex characteristics. The fetus develops with bipotential internal and external genitalia that can develop (at about 40 days gestation) into either a testis or ovary, depending upon which genes are expressed. When the embryo has functional testes, male internal and external genitalia develop. The Leydig cells of the fetal testis secrete testosterone and the Sertoli cells secrete Müllerian inhibiting substance (MIS), also known as antiMüllerian hormone (AMH). The development of male internal genitalia depends upon testosterone, which stimulates growth and development of the Wolffian ducts and MIS, which stimulates Müllerian duct regression.

A young couple presents with concerns that they have not been able to conceive a child. Physical examination of the 22yearold husband reveals mild obesity, gynecomastia, and decreased facial and axillary hair. He has male genitalia, but penile length is decreased and the testes are small. Chromosomal analysis reveals the XXY pattern of Klinefelter's syndrome. Which of the following is the principal androgen responsible for transforming undifferentiated external genitalia in the fetus into male external genitalia? a. Testosterone b. Androstenedione c. Androsterone d. Dihydrotestosterone e. Müllerianinhibiting substance f. Luteinizing hormone

d. Secretin An injection of acid into the duodenum will lower the pH of the lumen. In response to the decrease in pH, secretin is released from the pancreatic duct cell. Secretin is the most important humoral regulator of ductal bicarbonate secretion.

After an overnight fast, a 52-year-old man undergoes infusion of acid through a catheter into the upper duodenum. This most likely will increase pancreatic secretion mainly through the action of which of the following substances? a. Cholecystokinin b. Gastrin c. Glucagon d. Secretin e. Vasoactive intestinal polypeptide f. Histamine g. Somatostatin

a. Gonadotropins To address the absence of pituitary hormones, a glucocorticoid, oral thyroid hormone, and progesterone will all allow for normal physiological function of the important target tissues. Gonadotropins need not be administrated because normal reproductive function does not need to occur in this patient.

After delivery of her fourth child, a 40yearold woman had severe hemorrhage and hypotension. She then developed Sheehan's syndrome (from postpartum necrosis of the anterior pituitary gland). She lost all normal anterior pituitary function. She doesn't want any more children, but does desire to avoid bone loss, reduce her risk of atherosclerosis, and have a satisfactory sex life (e.g. adequate lubrication by vaginal mucus glands). She does not want to nurse her latest baby. To induce relatively normal physiological function of important target tissues, you would likely treat her with all of the following EXCEPT: a. Gonadotropins b. A glucocorticoid, given orally c. An estrogen d. Oral thyroid hormone, e.g., T4 and/or T3 e. Progesterone

a. Venous compliance Venous compliance decreases during hemorrhage to reduce the amount of blood in the peripheral circulation, and return blood to the central circulation. Veins and venules are compliance vessels with the capacity to store the majority of the circulating blood. A reduction in venous compliance helps to maintain a constant cardiac output in the face of blood volume loss.

After mild hemorrhage, compensatory responses initiated by the baroreceptor reflex keeps blood pressure at or close to its normal value. Which of the following values is less after compensation for the hemorrhage than it was before hemorrhage? a. Venous compliance b. Heart rate c. Ventricular contractility d. Total peripheral resistance e. Coronary blood flow f. Cerebral blood flow g. Cardiac output

d. Increase in serum creatinine to about four times normal A severe renal artery stenosis that reduces GFR to 25% of normal would also decrease renal blood flow but would cause only a transient decrease in urinary creatinine excretion. The transient decrease in creatinine excretion would increase serum creatinine (to about 4X normal), which would restore the filtered creatinine load to normal and therefore return urinary creatinine excretion to normal levels under steady-state conditions. Urinary sodium secretion would also decrease transiently but would be restored to normal so that intake and excretion of sodium are balanced. Plasma sodium concentration would not change significantly because it is carefully regulated by the antidiuretic hormone-thirst mechanism.

After receiving a renal transplant, a patient develops severe hypertension (170/110 mm Hg). A renal arteriogram indicates severe renal artery stenosis in his single remaining kidney, with a reduction in GFR to 25% of normal. Which of the following changes, compared with normal, would be expected in this patient, assuming steady-state conditions. a. Large increase in plasma sodium concentration b. Reduction in urinary sodium excretion to 25% of normal c. Reduction in urinary creatinine excretion to 25% of normal d. Increase in serum creatinine to about four times normal e. Normal renal blood flow in the stenotic kidney due to autoregulation f. Increased filtration across the glomerular capillary g. Reduction in urinary bicarbonate production to 25% of normal

b. At total lung capacity Airway resistance decreases as lung volume increases because the increasingly subatmospheric intrapleural pressure increases the radial traction on the airways, thereby increasing air- way diameter and decreasing airway resistance. During a forced expiration or at the residual volume, the intrathoracic pressure is positive, compressing the airways and increasing their resistance. The vagus nerve constricts air- way smooth muscle. Resistances in parallel add as reciprocals. Thus, the large number of small, peripheral airways increases the number of airways arranged in parallel, and lowers the total resistance of the peripheral airways compared to the total cross-section of the central airways.

Airway resistance is lowest in which of the following? a. During a forced expiration b. At total lung capacity c. At residual volume d. During vagal stimulation e. In the total cross-section of the central airways compared to the peripheral airways. f. At base tidal volume

c. CO2 is rapidly converted entirely to bicarbonate ions in plasma Approximately 11% of the CO2 produced by the systemic tissues is transported back to the lungs dissolved in the plasma. Of this 11%, 6% remains dissolved, while only 5% is converted to bicarbonate. The remaining 89% of incremental CO2 is transported back to the lungs by the red blood cells. CO2 enters the RBCs through AQP1 and Rh complex "gas channels". The majority of the CO2 transported into the RBCs is converted to bicarbonate by using water, catalyzed by carbonic anhydrase. During bicarbonate formation, there is a shift of Chloride into the erythrocytes as some of the newly formed bicarbonate moves out of the cell. Chloride shift occurs to balance the negative charge lost by the loss of bicarbonate anion. During bicarbonate formation, water also enters the cell causing it to swell. In the RBCs, about 21% of the incremental CO2 forms carbamino compounds with hemoglobin. The combined effects of the described intracellular and extracellular events is that ~10% of incremental CO2 formed in systemic tissues moves to the lungs as dissolved CO2, 6% in plasma and 4% inside erythrocytes. About 21% moves as carbamino compounds, almost exclusively inside erythrocytes as carbamino Hb. Finally, ~69% of incremental CO2 moves as , 5% that forms in plasma and 64% that forms inside the RBC.

All of the following statements concerning CO2 transport are true EXCEPT that: a. Compared to O2, dissolved CO2 plays a significant role in its transport b. The bulk of CO2 transport involves reversible combination of CO2 with water in red blood cells (RBCs) c. CO2 is rapidly converted entirely to bicarbonate ions in plasma d. Chloride ions are transported into erythrocytes to complement bicarbonate ion transported out of the cell e. CO2 combines with terminal amine groups, and these carbamino compounds are important in CO2 unloading f. Incremental CO2 enters the red blood cells using (aquaporin 1) AQP1 and Rh complex g. Erythrocytes swell in response to bicarbonate formation as they pass through systemic capillaries

c. Oligodendrocyte cells give rise to only one process that produces a myelin sheath for a single axon In the PNS, a single Schwann cell provides a single myelin segment to a single axon of a myelinated nerve. This situation stands in contrast to that in the CNS, where one oligodendrocyte myelinates many axons. Such an oligodendrocyte has 15 to 30 processes, each of which connects a myelin sheath to the oligodendrocyte's cell body. Each myelin sheath, which is up to 250 µm wide, wraps many times around the long axis of one axon.

All the statements about the glial cells of the nervous system are true EXCEPT: a. Microglia cells migrate to the injured site, proliferate and function chiefly as scavengers. b. The Schwann cells of the PNS are similar to the oligodendrocytes of the CNS. c. Oligodendrocyte cells give rise to only one process that produces a myelin sheath for a single axon d. Astrocytes have processes that terminate as end feet on neurons, blood vessels and pia mater. e. Astrocyte control extracellular potassium concentration ([K+]o) by a mechanism known as spatial buffering. f. Glial cells constitute half the volume of the brain and outnumber the neurons g. Glial cells are able to self-proliferate and replace themselves when lost

a. Heating of a patch of skin causes a relatively restricted vasodilation in the heated region An example of a relatively restricted or local sympathetic action is the vasodilation or vasoconstriction of blood vessels that occurs upon warming or cooling a patch of skin.

Although the sympathetic nervous system is often activated in such a way that it leads to mass activation of sympathetic responses throughout the body, it can also be activated to produce relatively discrete responses. Which of the following is an example of a local or discrete sympathetic action? a. Heating of a patch of skin causes a relatively restricted vasodilation in the heated region b. Food in the mouth causes salivation c. Emptying of the bladder may cause reflexive emptying of the bowel d. Dust particle in the eye causes increased tear fluid release e. Bright light introduced into one eye

d. Ileum Most vitamins are absorbed in the upper small intestine, but vitamin B12 (cobalamin) is absorbed primarily in the terminal ileum. In the stomach, vitamin B12 binds with intrinsic factor, a glycoprotein secreted by the parietal cells of the gastric mucosa. The vitamin B12-intrinsic factor complex is emptied from the stomach and propelled along the small intestine to the terminal ileum, where specific active transporters located on the enterocyte microvilli bind the vitamin B12-intrinsic factor complex and the complex is absorbed across the ileal mucosa. Pernicious anemia is a disease in which there is autoimmune destruction of the parietal cells. Vitamin B12 can also be produced by gastrectomy with removal of the intrinsic factor-secreting tissue or by dis- eases of the terminal ileum. Binding of the Vitamin B12-intrinsic factor complex requires Ca2+. Whereas vitamin B12 and folate absorption are Na+- independent, all seven of the other water-soluble vitamins are absorbed by carriers that are Na+ cotransporters.

An 18-year-old male with pernicious anemia lacks intrinsic factor, which is necessary for the absorption of cyanocobalamin. Vitamin B12 is absorbed primarily in which portion of the GI tract? a. Stomach b. Duodenum c. Jejunum d. Ileum e. Colon f. Cecum

b. Antithrombin III (ATIII) deficiency Antithrombin III deficiency (abbreviated ATIII deficiency) is a deficiency of antithrombin III. It is a rare hereditary disorder that generally comes to light when a patient suffers recurrent venous thrombosis and pulmonary embolism, and repetitive intrauterine fetal death (IUFD). Inheritance is usually autosomal dominant, though a few recessive cases have been noted. The patients are treated with anticoagulants or, more rarely, with antithrombin concentrate. Heparin enhances ATIII activity and neutralizes "activated serine protease coagulation factors. Patients with ATIII deficiency requiring anticoagulant therapy with heparin will need higher doses of heparin. ATIII binds to thrombin and then forms the thrombin-anti thrombin complex or TAT complex. This is a major natural pathway of anticoagulation. This binding of thrombin to AT is greatly enhanced in the presence of heparin. Remember, partial thromboplastin time (PTT) measures the overall speed at which blood clots by means of two consecutive series of biochemical reactions known as the "intrinsic" (now referred to as the contact activation pathway) and common coagulation pathways. Normal PTT times require the presence of the following coagulation factors: I, II, V, VIII, IX, X, XI and XII. Prolonged PTT may indicate heparin use, antiphospholipid antibody, coagulation factor deficiency, sepsis, or the presence of antibodies against coagulation. To answer this question, you will need to triage based on the patient history as well as the heparin reaction.

An 18-year-old man presents with pain and swelling in the right calf. He states that his mother had numerous deep vein thromboses and pulmonary emboli and recently died from complications related to pulmonary embolization. He is concerned that he might have a similar problem. A Doppler ultrasound reveals a deep thrombosis in the right anterior tibial vein. He is placed on a standard dose of heparin and the partial thromboplastin time (PTT) returns to normal. A repeat test on a different sample also returns normal. The dose of heparin is then markedly increased and the PTT becomes prolonged. What is the most likely diagnosis? a. Antiphospholipid syndrome (APL) b. Antithrombin III (ATIII) deficiency c. Factor V Leiden d. Protein C or S deficiency e. Von Willebrand's disease f. Hemophilia A g. Aplastic anemia

c. Hematocrit A complete AV canal defect results in significant left-to-right shunting and pulmonary hypertension. The pulmonary vascular obstructive disease results in hypoxemia, causing the kidneys to produce erythropoietin. When the hematocrit reaches 75%, the increased blood viscosity causes headache, joint pain, chest pain, dyspnea, and dizziness. Serious complications, such as a thromboembolic event, can occur.

An 18-year-old patient with trisomy 21 has an unrepaired complete atrioventricular (AV) canal defect and pulmonary hypertension. She is complaining of headache and dizziness. Which of the following tests would be most likely to reveal the etiology of these complaints? a. Electrocardiography (ECG) b. Electroencephalography (EEG) c. Hematocrit d. Sinus radiography e. Urinalysis f. Computed tomography (CT)

d. Calcification of the bone matrix Vitamin D deficiency causes defective calcification of the bone matrix as a result of inadequate delivery of calcium and phosphate to the sites of mineralization. The disease in children is called rickets and is characterized by growth retardation, weakness, and bowing of the weightbaring bones, dental defects, and hypocalcemia, which increases parathyroid hormone and urinary phosphate losses. Although breast feeding is the preferred nutritional source for infants, breastfed infants require daily vitamin D supplementation. In addition to dietary vitamin D deficiency, several different types of inheritance lead to the vitamin D deficiency disorders, including Xlinked dominant and autosomal dominant hypophosphatemic rickets, vitamin D dependent rickets Type I, an autosomal recessive disorder caused by inactivating mutations in the gene encoding 1ahydroxylase enzyme, and vitamin Ddependent rickets Type II, in which there is endorgan resistance to 1,25(OH)2D3, which is also usually inherited as an autosomal recessive disorder.

An 18monthold boy presents with delayed dentation, short stature, difficulty, painful walking, and bowing of the legs. The history reveals that he has been breastfed, but that he has not been receiving daily vitamin D supplementation. A defect in which of the following can explain his findings? a. Blood supply to the Haversian canals Y b. Bone formation by osteoblasts c. Bone resorptions by osteoclasts d. Calcification of the bone matrix e. Composition of bone collagen f. Bone phosphate levels

a. A deficiency in the brush border enzyme lactase Lactase is a brush border enzyme that hydrolyzes milk sugar (lactose) into glucose and galactose. Patients with a lactase deficiency may experience diarrhea, cramps, and intestinal gas. The diarrhea and cramping reflect the osmotic effect of the sugar on water flux across the intestine. Colonic bacteria metabolize lactose to fatty acids, CO2, and H2.

An 18yearold college student reports that she experiences severe abdominal bloating and diarrhea within 1 hour of consuming dairy products. A subsequent H2breath test is abnormal. The diarrhea and bloating can best be explained by which of the following? a. A deficiency in the brush border enzyme lactase b. Carbohydrateinduced secretory diarrhea c. Decreased intestinal surface area d. Decreased carbohydrate absorption e. A decrease in exocrine pancreatic secretion f. Decreased pool of bile acids

a. An increase in plasma alkaline phosphatase Infectious hepatitis is a systemic infection predominantly affecting the liver. When jaundice appears, serum bilirubin rises, and, in most instances, total bilirubin is equally divided between the conjugated (direct) and unconjugated (indirect) fractions. The bilirubin in serum represents a balance between input from production of bilirubin and hepatic/biliary removal of the pigment. Hyperbilirubinemia may result from (1) overproduction of bilirubin; (2) impaired uptake, conjugation, or removal of bilirubin; or (3) regurgitation of unconjugated or conjugated bilirubin from damaged hepatocytes or bile ducts. Alkaline phosphatase, which is excreted in bile, increases in patients with jaundice due to bile duct obstruction, but generally not when the jaundice is due to hepatocellular disease. Bile acids are synthesized in the liver by a series of enzymatic steps that also involve cholesterol catabolism. Liver disease decreases bile acid synthesis.

An 18yearold female decides to get a tattoo for her birthday. Two months later she presents with a fever, right upper quadrant pain, nausea, vomiting, and jaundice. Which of the following lab values would most likely be found in a patient with infectious hepatitis? a. An increase in plasma alkaline phosphatase b. An increase in plasma bile acids c. An increase in both direct and indirect plasma bilirubin d. An increase in direct bilirubin, and a decrease in indirect bilirubin in the plasma e. A decrease in both direct and indirect plasma bilirubin f. An increase in serum albumin

a. A drug that inhibits Ca2+ release from the sarcoplasmic reticulum Ca2+ signals muscle contraction, coupling the electrical signal to contact with the mechanical event. Because Ca2+ is initially released from the sarcoplasmic reticulum, it would be best to target this release to block muscle spasms.

An 70yearold man complains of lower back pain that is accompanied with strong muscle spasms. He is otherwise healthy. Which of the following drugs might be effective in reducing the magnitude of the spasms? a. A drug that inhibits Ca2+ release from the sarcoplasmic reticulum b. A drug that inhibits myosin light chain kinase c. A drug that blocks Ca2+ channels in the cell membrane d. An acetylcholinesterase inhibitor e. An inhibitor of skeletal muscle glucose uptake f. A drug that stimulates voltagegated Na+ channel opening in the cell membrane

a. Paralysis of vascular smooth muscle The 'rosy cheeks' produced by cold-induced vasodilation are attributed to cold-induced paralysis of vascular smooth muscle.

An 8-year-old girl developed 'rosy cheeks' while playing outside on a chilly day. What is thought to be the cause of this vasodilation? a. Paralysis of vascular smooth muscle b. A decreased thermoregulatory 'set point' c. Increased prostaglandin E2 (PGE2) production d. Increased parasympathetic activity e. Secretion of progesterone

a. Antidiuretic hormone (ADH) The findings are consistent with inappropriate secretion of ADH (SIADH). The findings of vomiting, headache, and hypertension suggest a central nervous system lesion. Excessive secretion of ADH leads to water retention, reduced serum sodium concentrations, and a mild increase in the extracellular fluid (ECF) volume. Management of SIADH patients includes fluid restriction and treatment of the underlying cause.

An 8-year-old girl has headache, vomiting, and hypertension. She appears adequately hydrated. Laboratory values are: serum sodium concentration 110 mEq/L (normal 135-145 mEq/L); blood urea nitrogen 10 mg/dL (normal 7-22 mg/dL); serum creatinine, 0.4 mg/dL (normal 0.3-0.7 mg/dL). The sodium concentration of a random sample of urine is 100 mEq/L (normal 130-260 mEq/24 h). An increase in which of the following hormones is the most likely cause of these findings? a. Antidiuretic hormone (ADH) b. Atrial natriuretic peptide (ANP) c. Estrogen d. Glucocorticoid e. Mineralocorticoid f. Thyroid hormone

d. Incomplete relaxation of the lower esophageal sphincter with swallowing coupled with increased resting tone of the lower esophageal sphincter Studies of esophageal motility in which investigators measured intraesophageal pressure demonstrated the presence of two defects in patients with achalasia: (1) failure of the LES to relax, and (2) impaired peristalsis in the distal two thirds of the body of the esophagus (i.e., the portion that consists of smooth muscle).

An 8-year-old girl is brought in for evaluation because of progressive difficulty swallowing food. Her mother reports that the girl frequently vomits at night. Medical history shows the girl has been hospitalized previously for aspiration pneumonia. Achalasia is suspected. Which of the following findings most likely would confirm the diagnosis? a. Complete relaxation of the lower esophageal sphincter with swallowing b. Decreased resting tone of the lower esophageal sphincter c. Increased resting tone of the upper esophageal sphincter d. Incomplete relaxation of the lower esophageal sphincter with swallowing coupled with increased resting tone of the lower esophageal sphincter e. Incomplete relaxation of the lower esophageal sphincter with swallowing coupled with increased resting tone of the upper esophageal sphincter f. Incomplete relaxation of the upper esophageal sphincter with swallowing coupled with increased resting tone of the lower esophageal sphincter g. Incomplete relaxation of the upper esophageal sphincter with swallowing coupled with decreased resting tone of the upper esophageal sphincter

c. Insulin-like growth factor I (IGF-1) The effects of growth hormones depend on an interaction between growth hormone and somatomedins, which are peptide growth factors secreted by the liver. IGF-I is a principal circulating somatomedin along with IGF-II. Dwarfed children with normal or elevated circulating growth hormone due to a congenital defect of the growth hormone receptor have Laron dwarfism. The laboratory findings in the patient described in this case are consistent with the diagnosis of Laron-type dwarfism. Children with Laron dwarfism usually have normal to high levels of growth hormone and very low levels of IGF-I.

An 8-year-old healthy boy is concerned about his very short stature. His growth hormone level is within normal limits despite the fact that his growth rate and bone age are below the norm for children of the same chronologic age. The level of which of the following hormones is most likely to be abnormal? a. Fibroblast growth b. Insulin c. Insulin-like growth factor I (IGF-1) d. Somatostatin e. Somatotropin f. Thyroid hormone

f. Atrial fibrillation The patient has an irregularly irregular and rapid pulse, suggesting atrial fibrillation.

An 82 year-old male presents to the hospital with the acute onset of shortness of breath. On examination he has an irregularly irregular pulse of 130, a blood pressure of 110/90, and a respiratory rate of 24. His jugular veins are distended to an estimated RA pressure of 14 and his carotid upstrokes are delayed. His chest exam reveals dullness to percussion 1/3 the way up bilaterally. His cardiac exam reveals a grade III/VI late peaking systolic ejection murmur with a normal first heart sound and a single second heart sound. His extremities reveal 1+ edema. This patient's heart rhythm is most likely to be: a. Normal sinus rhythm b. Sinus rhythm with Mobitz I (Wenckebach) A-V block c. Re-entry induced paroxysmal atrial tachycardia d. Third-degree heart block e. Sinus tachycardia f. Atrial fibrillation

d. Potassium conductance will increase

An 82-year-old woman is brought to the emergency department complaining of nausea, vomiting, muscle cramps, and generalized weakness. Laboratory analysis reveals significant hyperkalemia. Elevations of extracellular potassium ion concentration will have which of the following effects on cell membranes? a. Over time, the membrane will become more excitable b. The membrane potential will become more negative c. The activity of the Na/K pump will decrease d. Potassium conductance will increase e. Sodium conductance will increase

d. Its clearance is equal to renal plasma flow (RPF) If a substance disappears from the circulation during its passage through the kidney, it usually indicates that it has been totally secreted into the nephron in which case the clearance of the substance equals RPF. The clearance would not equal the GFR because normal filtration fraction is 20% which would not totally clear the plasma concentration of the substance. None of the substance is reabsorbed because none of the substance is detected in the renal vein. If the substance is bound to plasma proteins it can be secreted without being filtered. Even if it is entirely secreted by the kidney, its urinary concentration may be less than its plasma concentration if urinary flow rate is very high.

An 83-year old woman with a history of hypertension presents to her family physician's office with oliguria. Serum creatinine and BUN are elevated and a CT reveals that the patient's left kidney is hypoplastic (undersized). Renal function studies are performed to assess the renal handling of various substances. Substance X is injected into an arterial line. All of substance X appears in the urine and none is detected in the renal vein. What do these findings indicate about the renal handling of substance X? a. It must be filtered by the kidney b. It must be reabsorbed by the kidney c. Its clearance is equal to GFR d. Its clearance is equal to renal plasma flow (RPF) e. Its urinary concentration must be higher than its plasma concentration f. It bypasses filtration by the kidney in favor of filtration by the liver

d. A decrease in aortic compliance Pulse pressure is the difference between the systolic and diastolic pressure readings. It is measured in millimeters of mercury (mmHg). It represents the force that the heart generates each time it contracts. Theoretically, the systemic pulse pressure can be conceptualized as being directly proportional to stroke volume, or the amount of blood ejected from the left ventricle during systole and inversely proportional to the compliance of the aorta. A reduction in aortic compliance results in an increase in the pulse pressure.

An 83-year-old woman with long standing hypertension presents after a near syncopal episode upon standing. Her blood pressure is taken sitting and then standing. Systolic pressure decreased slightly and pulse pressure increased in the standing position. Which of the following can lead to an increase in pulse pressure? a. An increased heart rate b. A decreased stroke volume c. An increase in total peripheral resistance d. A decrease in aortic compliance e. A decrease in venous capacitance f. A decreased cardiac output g. An increase in glomerular filtration rate

a. Increased surface area of the membrane Diffusive clearance across the membrane depends on several factors: thickness of the membrane, surface area of the membrane, molecular weight of the solutes, the concentration gradient across the membrane for the solutes. Diffusive clearance is directly proportional to the surface area of the membrane, the concentration gradient of the solutes across the membrane, and the lipid solubility of the solutes. Diffusive clearance is inversely proportional to the thickness of the membrane and the molecular weight of the solutes.

An 83-year-old woman with sepsis develops multi-organ failure. Based on her blood urea nitrogen (BUN) > 100 mg/dL, she is placed on continuous venous hemodialysis. Which of the following factors will increase the diffusive clearance of solutes across the semi-permeable dialysis membrane? a. Increased surface area of the membrane b. Decreased concentration gradient for the solutes c. Decreased lipid solubility of the solutes d. Increased molecular weight of the solutes e. Increased thickness of the membrane f. Increased water tension across the surface of the membrane

a. Area of the membrane increases Fick's law indicates that diffusive clearance will increase if the surface area of the membrane increases, the thickness of the membrane decreases, the concentration gradient of the solute increases, the size of the solute molecule is small, and the molecule is lipid-soluble.

An 83-year-old woman with sepsis develops multi-organ failure. Based on her blood urea nitrogen of >100 mg/dL, she is placed on continuous venovenous hemodialysis. Which of the following factors will increase the diffusive clearance of solutes across the semipermeable dialysis membrane? a. Area of the membrane increases b. Concentration gradient for the solute decreases c. Lipid solubility of the solute decreases d. Size of the solute molecule increases e. Thickness of the membrane increases f. Composition of the solute changes g. Rate of fluid flow past the membrane

f. Ruffini ending The Ruffini ending is a tonic receptor that produces a train of action potentials proportional to the intensity of pressure applied to the skin. The Pacinian corpuscle is a very rapidly adapting receptor that fires once or twice in response to skin deformation, but can produce a continuous train of action potentials if the stimulus is repetitively applied and withdrawn. Therefore, the Pacinian corpuscle is used to encode vibration.

An 86yearold woman develops unilateral vesicular eruption on the trunk in a T8 dermatomal pattern. Fluorescent staining of the skin scrapings confirms a diagnosis of herpes zoster. The woman complains of significant pain, as well as increased sensitivity to touch. Which of the following receptors is responsible for measuring the intensity of a steady pressure on the skin surface? a. Pacinian corpuscle b. Tactile nerve ending c. Merkel's disk d. Meissner's corpuscle e. Krause ending f. Ruffini ending

b. Increase, decrease or show no change PTH reduces the reabsorption of phosphate from the proximal tubule of the kidney, which means more phosphate is excreted through the urine. However, PTH enhances the uptake of phosphate from the intestine and bones into the blood. In the bone, slightly more calcium than phosphate is released from the breakdown of bone. In the intestines, absorption of both calcium and phosphate is mediated by an increase in activated vitamin D. The absorption of phosphate is not as dependent on vitamin D as is that of calcium. The end result of PTH release is a small net drop in the serum concentration of phosphate.

An adult with a parathyroid gland tumor has a very elevated plasma PTH level. This person's urinary phosphate excretion is expected to X, whereas the plasma phosphate level is likely to Y. a. Increase, increase b. Increase, decrease or show no change c. Decrease, increase or show no change d. Decrease, decrease e. Show no change, show no change f. Show no change, increase

d. Ovulation Ovulation occurs at the midpoint of every normal menstrual cycle, triggered by the LH surge, which in turn is stimulated by rapidly rising levels of estradiol. Estradiol secretion by the dominant follicle increases rapidly near the end of the late follicular phase. This dramatic rise in circulating estradiol switches the negativefeedback response of estradiol on the hypothalamus and anterior pituitary to a positive feedback response and also sensitizes the anterior pituitary to GnRH. The result is the LH surge, which generally begins 24 to 36 hours after peak estradiol secretion. Ovulation usually occurs ~36 hours after onset of the LH surge, and ~12 hours after its peak.

An assay of serum samples from a normal woman with a history of regular 28day cycles indicates that, during the last 12 h, there has been a peak in the serum concentration of estradiol17β in the absence of any detectable progesterone. Which of the following can be expected to occur within 3 days? a. Cessation of menstruation b. Decreased basal body temperature c. Onset of menstruation d. Ovulation e. Regression of the corpus luteum f. Parturition

f. Tetralogy of Fallot In tetralogy of Fallot there is an interventricular septal defect and increased resistance in the pulmonary valve or pulmonary artery. This causes partial blood shunting toward the left side of the heart without going through the lungs. This results in severely decreased arterial oxygen content. The interventricular septal defect causes equal systolic pressures in both ventricles. The end result is right ventricular hypertrophy and a wall thickness similar to the left ventricle.

An echocardiogram was performed on a 50-year-old femal patient. The results indicated a thickened right ventricle. Other data indicated that the patient had severely decreased arterial oxygen content and equal systolic pressure in both cardiac ventricles. What condition is present? a. Cardiac tamponade b. Decompensated heart failure c. Pulmonary valve stenosis d. Pulmonary valve regurgitation e. Patent ductus arteriosus f. Tetralogy of Fallot

e. Stimulation of adenylate cyclase Stimulation of either adenylate or guanylate cyclase induces smooth muscle relaxation. The cyclic nucleotides produced by these enzymes stimulate cAMP and cGMPdependent kinases, respectively. These kinases phosphorylate, among other things, enzymes that remove Ca2+ from the cytosol, and in doing so they inhibit contraction. In contrast, either a decrease in K+ permeability or an increase in Na+ permeability results in membrane depolarization and contraction. Likewise, inhibition of the sarcoplasmic reticulum Ca2+ATPase, one of the enzymes activated by cyclic nucleotidedependent kinases, would also favor muscle contraction. Smooth muscle does not express troponin.

An experimental drug is being tested as a potential therapeutic treatment for asthma. Preclinical studies have shown that this drug induces relaxation of cultured porcine tracheal smooth muscle cells pre contracted with acetylcholine. Which of the following mechanisms of action is most likely to induce this effect? a. Decreased affinity of troponin C for Ca2+ b. Decreased plasma membrane K+ permeability c. Increased plasma membrane Na+ permeability d. Inhibition of the sarcoplasmic reticulum Ca2+ATPase e. Stimulation of adenylate cyclase f. Upregulation of the Na+/K+ATPase

e. Stimulation of adenylate cyclase Stimulation of AC or GC induces smooth muscle relaxation. AC and C stimulate cAMP and cGMP-dependent kinases that phosphorylate enzymes that remove Ca2+ from the cytosol leading to decreased contraction. Decreasing K+ permeability or increased Na+ permeability leads to depolarization resulting in contraction. Inhibition of the Ca2+-ATPase favors contraction. No Tn in smooth muscle.

An experimental drug is being tested as a potential therapeutic treatment for asthma. Studies have shown that this drug induces the relaxation of cultured porcine tracheal smooth cells pre-contracted with acetylcholine. Which of the following mechanisms of action is most likely to induce this effect? a. Decreased affinity of troponin C for Ca2+ b. Decreased plasma membrane K+ permeability c. Increased plasma membrane Na+ permeability d. Inhibition of the sarcoplasmic reticulum Ca2+-ATPase e. Stimulation of adenylate cyclase

d. A similar increase in total body water Because both infusions are isotonic, there will be no shift in fluid between compartments. However, as the patient is anephric (without kidneys), the infusions will remain in the patient's circulation, leading to an increase in the total body water of the patient.

Approximately 30 minutes following the infusion of 1 liter of isotonic saline (0.9% NaCl) or 1 liter of isotonic glucose (5% dextrose) in an anephric adult, you would expect these infusions to cause: a. Similar decreases in plasma osmolality b. Similar decreases in extracellular volume c. Increase in cell volume in either group following either infusion d. A similar increase in total body water e. Increase in cellular osmolality following either infusion f. Increase in plasma osmolality following either infusion

b. Lactate Role of astrocytes in providing lactate as fuel for neurons. Neurons have two fuel sources. They can obtain glucose directly from the blood plasma or they can obtain lactate from astrocytes. In the direct path, the oxidation of one glucose molecule provides 30 ATP molecules to the neuron. In the transastrocyte path, conversion of two lactates to two pyruvates, and then the subsequent oxidation of the pyruvate, provides 28 molecules of ATP to the neuron.

Astrocytes provide fuel for their neighboring neurons through releasing which of the following molecules: a. Glucose b. Lactate c. Glutamate d. Pyruvate e. Glycogen f. Acetylcholine g. Glutamine

a. Point A The Na+ equilibrium potential is approximately +60 mV and is based on the ratio of the intracellular and extracellular Na+ concentrations. During an action potential, the peak of the action potential (point A) is close, but not equal, to the Na+ equilibrium potential. The membrane potential doesn't reach the Na+ equilibrium potential because the Na+ channels start to inactivate and the K+ channels begin to activate during the upstroke of the action potential.

At which point on the action potential shown in the diagram below is the membrane closest to the Na+ equilibrium potential? a. Point A b. Point B c. Point C d. Point D e. Point E

G. G The basic mechanism of Cheyne-Stokes breathing can be attributed to a buildup of carbon dioxide which stimulates hyperventilation, followed by a depression of the respiratory center due to low PCO2 of the respiratory neurons. It should be clear that the greatest depth of breathing occurs when the neurons of the respiratory center are exposed to the highest levels of carbon dioxide (W). This increase in breathing causes CO2 to be blown off and thus PCO2 of the lung is at its lowest value at about point Y on the diagram. PCO2 of the pulmonary blood gradually increase from point Y to point Z, reaching its maximum value at point V. Thus, it is the phase lag between the PCO2 at the respiratory center and the PCO2 of the pulmonary blood that leads to this type of breathing. The phase-lag often occurs with left heart failure, due to enlargement of the left ventricle which increases the time required for blood to reach the respiratory center. Another cause of Cheyne-Stokes breathing is increased negative feedback gain in the respiratory control areas, which can be caused by head trauma, stroke, and other types of brain damage.

Cheyne-Stokes breathing is an abnormal breathing pattern characterized by a gradual increase in the depth of breathing, followed by a progressive decrease in the depth of breathing that occurs again and again about every minute, as shown in the following diagram. Which of the following time points (V-Z) are associated with the highest PCO2 of lung blood and higest PCO2 of neurons in the respiratory center? a .A b. B c. C d. D e. E f. F g. G

c. 40 mmHg and the patient is ischemic Cerebral Perfusion Pressure (CPP) is defined as the difference between the Mean Arterial Pressure (MAP) and the Intracranial Pressure (ICP). This represents the pressure gradient driving cerebral blood flow (CBF) and hence oxygen and metabolite delivery. Remember MAP is calculated at (1/3)SBP+(2/3)DBP or ((SBP)+2(DBP))/3. The MAP is 110 mm Hg, and so CPP is 40 mm Hg. To determine whether the patient is ischemic, stable, or hyperemic, remember the ranges illustrated in the following graph: A CPP of 40 mm Hg is in the ischemic range.

Colton, a 45yearold professor, suffered a major concussion from a car accident. In the ED, he presents with a systolic BP of 150 mmHg and diastolic BP of 90 mmHg, as well as his intracranial pressure is 70mmHg. What is his central perfusion pressure and what does this mean for Colton? a. 130 mmHg and the patient is ischemic b. 130 mmHg and the patient is hyperemic c. 40 mmHg and the patient is ischemic d. 40 mmHg and the patient is hyperemic e. 130 mmHg and the patient is stable f. 90 mmHg and the patient is ischemic

a. VLDLs

Dietary fat, after being processed, is extruded from the mucosal cells of the gastrointestinal tract into the lymphatic ducts in the form of: a. VLDLs b. HDLs c. Triglycerides d. Monoacylglycerols e. Free fatty acids f. Cholesterol g. Mixed micelles

e. Depolarization of the membrane potential Explanation: The inhibition of Na+/K+-ATPase induces a rise in sodium concentration inside cells. This sodium increase induces in its turn an increase in the intracellular calcium concentration, via the sodium-calcium exchanger. The sodium-calcium exchanger is particularly active in myocardium and in smooth vascular muscles. The rise in intracellular calcium increases the force of contraction of the heart and the contracture of smooth vascular muscles. The inhibition of Na+/K+-ATPase reduces cellular polarization (depolarizing effect).

Digitalis, a drug used in the treatment of heart failure, produces its therapeutic effect by inhibition of the Na+/K+-ATPase pump. Which of the following cellular effects is this drug most likely to have? a. Increased intracellular potassium concentration b. Increased excitability of nerve cells c. Decreased intracellular volume d. Decreased intracellular sodium concentration e. Depolarization of the membrane potential

c. An alpha adrenergic blocker Pheochromocytoma is a tumor of the adrenal medulla characterized by an excessive release of catecholamines. Severe hypertension can result from the increase in heart rate and contractility (b1adrenergic activation by catecholamines) and vascular resistance (aadrenergic activation). Blocking the aadrenergic receptors will cause a decrease in TPR and a decrease in blood pressure.

During a routine physical examination, a 35yearold male is found to have a blood pressure of 170/105 mmHg. History reveals episodes of headache accompanied by palpitations, diaphoresis, and anxiety. A tentative diagnosis of pheochromocytoma is confirmed when blood pressure falls in response to the administration of which of the following? a. An ACE inhibitor b. A beta adrenergic agonist c. An alpha adrenergic blocker d. A loop diuretic e. An AII antagonist f. A thiazide diuretic

B. B The action of secretagogues on gastric acid secretion occurs via at least two parallel and perhaps redundant mechanisms. In the first, acetylcholine (ACh), gastrin, and histamine bind directly to their respective receptors on the parietal-cell membrane and synergistically stimulate acid secretion. ACh is released from endings of the vagus nerve (cranial nerve X), and as we will see below, gastrin is released from G cells. Histamine is synthesized from histidine in ECL cells of the lamina propria. In the second mechanism, ACh and gastrin indirectly induce acid secretion as a result of their stimulation of histamine release from ECL cells. Secretin appears to reduce acid secretion by at least three mechanisms: (1) inhibition of antral gastrin release, (2) stimulation of somatostatin release, and (3) direct downregulation of the parietal-cell H+ secretory process.

During a study of gastric parietal cells, an investigator attempts to elicit maximum hydrochloric acid secretion from the stomach of an experimental animal. Which of the following combinations of substances is most likely to lead to this desired effect? A. A B. B C. C D. D E. E F. F G. G

a. Increased right ventricular pressure An occlusion in the lungs will restrict blood flow from the R side of the heart to the lungs. Increased R ventricular pressure is an indicator of decreased blood flow out of the heart to the lungs.

During a transatlantic flight, an elderly woman complains of chest pain and difficulty breathing, and she suddenly collapses. A nurse onboard the plane elevates the woman's legs, which helps to restore consciousness but does not alleviate her chest pain or shortness of breath. After landing, the woman is rushed to a hospital for evaluation of a suspected pulmonary embolism. Which of the following is most likely to confirm the diagnosis? a. Increased right ventricular pressure b. Hypoxic vasoconstriction c. Decreased jugular venous pulse d. Atrioventricular conduction block e. Acute pulmonary edema f. Increased mean arterial pressure

b. Irritant receptors Irritant receptors respond to the presence of foreign bodies, irritants, or immunological challenges. When stimulated, these receptors can trigger a cough which minimizes the chances that the offending foreign body will be pulled deeper into the lungs. Peripheral chemoreceptors respond to changes in partial pressure of oxygen, while central chemoreceptors respond to change in partial pressure of carbon dioxide. J receptors have unknown function but respond to events such as pulmonary edema, pulmonary emboli, pneumonia, congestive heart failure and barotrauma, which cause a decrease in oxygenation and thus lead to an increase in ventilation/respiration. In the limbs, proprioceptors are sensors that provide information about joint angle, muscle length, and muscle tension, which is integrated to give information about the position of the limb in space. Stretch receptors are mechanoreceptors responsive to distention of various organs and muscles, and are neurologically linked to the medulla in the brain stem via afferent nerve fibers. When stimulated, beta-2-adrenergic receptors result bronchiole dilation. Alpha-1-adrenergic receptors result in bronchiole constriction.

During an experiment on the cough reflex in humans, a volunteer inhales air containing different amounts of particles that will impact and adhere to mucus primarily in the trachea. Blockade of which of the following receptors would most likely prevent this volunteer's reflex to initiate a cough? a. Chemoreceptors b. Irritant receptors c. J receptors d. Proprioceptors e. Stretch receptors f. Beta-2-adrenergic receptors g. Alpha-1-adrenergic receptors

c. Cytochrome c release One of the pathways that leads to apoptosis is damage to the mitochondria, triggered by such agents as ROS, increases in [Ca2+] in the mitochondrial matrix, and caspases. The result is the opening of a large pore in the mitochondrial inner membrane—the mitochondrial permeability transition pore (MPTP)—followed by mitochondrial swelling, rupture of the outer mitochondrial membrane, and release of cytochrome c into the cytosol. There, apoptotic protease-activating factor (Apaf-1) complexes with the cytochrome c and ATP, forming a wheel-like structure that contains seven of each molecule—an apoptosome.

During an experimental study, an investigator finds that the regulation of cell cycle and programmed cell death may be initiated by the mitochondrion. The interaction of the mitochondrion with the activation of the caspase family of proteases and subsequent apoptosis is most likely mediated by which of the following? a. Calcium release b. cAMP production c. Cytochrome c release d. GTP binding e. Nitric oxide release f. Telomerase reproduction

e. At the beginning of diastole Blood flow through the coronary vessels of the left ventricle is determined by the ratio of perfusion pressure to vascular resistance. The perfusion pressure is directly related to the aortic pressure at the ostia of the coronaries. Myocardial vascular resistance is significantly influenced by the contractile activity of the ventricle. During systole, when the ventricle is contracting, vascular resistance increases substantially. Flow is highest just at the beginning of diastole because, during this phase of the cardiac cycle, aortic pressure is still relatively high and vascular resistance is low due to the fact that the coronary vessels are no longer being squeezed by the contracting myocardium.

During the cardiac cycle, when does the highest coronary blood flow per gram of left ventricular myocardium occur. a. When aortic pressure is highest b. When left ventricular pressure is highest c. At the beginning of isovolumic contraction d. When aortic blood flow is highest e. At the beginning of diastole f. During systole

d. Both A and C An excess of growth hormone secretion in prepubertal individuals can cause gigantism while an excess in GH after puberty causes acromegaly. Chronic oversecretion of GH, such as occurs in patients with GH producing tumors in acromegaly, is accompanied by insulin resistance and often by glucose intolerance or frank diabetes.

Excessive secretion of growth hormone (GH) in prepubertal individuals can cause which of the following abnormal states? a. Gigantism b. Acromegaly c. Hyperglycemia d. Both A and C e. Both A and B

c. A muscarinic antagonist Organophostphate poisoning causes overstimulation of nAChRs and mAChRs leading to excess ACh release. Treatment is with anti-cholinergic drugs which counteract the excess ACh and upregulate AChE. A muscarinic antagonist such as Atropine block the action of ACh peripherally and prevent lethality of poisoning.

Farmer Brown arrives at the emergency room after inhaling insecticide that she was spraying on her fields. She has constricted pupils, wheezy breathing (constricted airway), and a slowed heart rate. An appropriate treatment would be: a. Acetylcholine b. Acetylcholinesterase c. A muscarinic antagonist c. A muscarinic agonist d. Anti-acetylcholinesterase e. A β-receptor antagonist f. Sodium channel blocker

A. A Fluid deprivation results in an increase in plasma osmolarity which is sensed by the osmoreceptors of the hypothalamus. These receptors stimulate the release of ADH from the posterior pituitary which increases water reabsorption from the distal kidney tubule and also stimulates thirst to promote water intake.

Following 24 hours without any fluid intake, characterize the changes in plasma osmolarity, ADH secretion, and thirst. A. A B. B C. C D. D E. E F. F G. G

a. It acts directly on bone cells to increase Ca2+ resorption and mobilize Ca2+ In the kidney, PTH promotes Ca2+reabsorption, phosphate loss, and 1-hydroxylation of 25-hydroxyvitamin D. The net effect of persistent increases of PTH on bone is to stimulate bone resorption, thus increasing plasma [Ca2+].

Following neck surgery, a patient develops circumoral paresthesia and a long QT interval on the EKG consistent with hypocalcemia resulting from injury to the parathyroid glands. Which of the following best describes parathyroid hormone (PTH)? a. It acts directly on bone cells to increase Ca2+ resorption and mobilize Ca2+ b. It acts directly on intestinal cells to increase Ca2+ absorption c. It is synthesized and secreted from the oxyphil cells in the parathyroid glands d. It increases phosphate reabsorption in the renal proximal tubular cells e. Its secretion is increased in response to an increase in plasma-free Ca2+ concentration f. It acts directly on the kidney cells to increase Ca2+ secretion from the tubule

c. a rapid (minutes) return of plasma pH and glucose to normal will cause potassium to shift into the cells Insulin is a potent stimulus for hypokalemia, sparing body potassium from urinary excretion by transporting it into cells. The administration of a potassium salt prevents hypokalemia from occurring.

For a diabetic patient (Type 1) who is being successfully treated for ketoacidosis by the administration of insulin, a potassium salt might also be administered because: a. insulin directly stimulates the excretion of potassium by the kidney Y b. a rapid (minutes) return of plasma pH and glucose to normal will lower the total body potassium content c. a rapid (minutes) return of plasma pH and glucose to normal will cause potassium to shift into the cells d. a rapid (minutes) return of plasma glucose to normal will cause a decrease in plasma volume. e. Insulin stimulates the uptake of hydrogen ion by the cell

e. Diabetes insipidus Free water clearance is the amount of water excreted in excess of that required to make the urine isotonic to plasma. It is calculated using the formula: CH2O = V − Cosm. Free water clearance is positive when the urine is dilute (more than a sufficient amount of water is excreted), and free water clearance is negative when the urine is concentrated (not enough water is excreted to make the urine isotonic to plasma). An increase in free water clearance can lead to hypernatremia; a decrease in free water clearance can lead to hyponatremia. In diabetes insipidus, very little water is reabsorbed in the distal nephron, and, therefore, the free water clearance is very high. In heart failure or renal failure, very little free water can be generated even if the urine is dilute because the glomerular filtration rate is decreased. With diuretic therapy, Na+ excretion is increased. Therefore, the increased water excretion is accompanied by an increased Na+ excretion and the amount of free water generated is limited. Although the water loss is proportionally greater than the solute loss in diabetes mellitus, the amount of water excreted is much less and the solute concentration significantly higher than in diabetes insipidus, so the free water clearance is much less in diabetes mellitus than in diabetes insipidus.

Free water clearance by the kidney is increased by which of the a. Heart failure b. Renal failure c. Diuretic therapy d. Diabetes mellitus e. Diabetes insipidus f. Liver failure

e. Diabetes insipidus Free water clearance is the amount of water excreted in excess of that required to make the urine isotonic to plasma. It is calculated using the formula: CH2O = VCosm. Free water clearance is positive when the urine is dilute (more than a sufficient amount of water is excreted), and free water clearance is negative when the urine is concentrated (not enough water is excreted to make the urine isotonic to plasma). An increase in free water clearance can lead to hypernatremia; a decrease in free water clearance can lead to hyponatremia. In diabetes insipidus, very little water is reabsorbed in the distal nephron, and, therefore, the free water clearance is very high. In heart failure or renal failure, very little free water can be generated even if the urine is dilute because the glomerular filtration rate is decreased. With diuretic therapy, Na+ excretion is increased. Therefore, the increased water excretion is accompanied by an increased Na+ excretion and the amount of free water generated is limited. Although the water loss is proportionally greater than the solute loss in diabetes mellitus, the amount of water excreted is much less and the solute concentration significantly higher than in diabetes insipidus, so the free water clearance is much less in diabetes mellitus than in diabetes insipidus.

Free water clearance by the kidney is increased by which of the following? a. Heart failure b. Renal failure c. Diuretic therapy d. Diabetes mellitus e. Diabetes insipidus f. Liver failure

d. Decreased ability to concentrate urine Furosemide inhibits the Na/Cl/K co-transport in the ascending limb of the loop of Henle. This causes marked natriuresis and diuresis and also reduces the urine concentrating ability. Furosemide is a potent diuretic and is used to treat severe edema and heart failure. It also increases the renal excretion of potassium and calcium resulting in hypokalemia and hypocalcemia.

Furosemide (Lasix) is a diuretic that also produces natriuresis. Which of the following is an undesirable side effect of furosemide due to its site of action on the renal tubule? a. Edema b. Hyperkalemia c. Hypercalcemia d. Decreased ability to concentrate urine e. Heart failure f. Uremia

a. Cause a reflex decreasing sympathetic output to the heart, while increasing parasympathetic output. An increase in blood pressure is sensed by the baroreceptors as increased stretch in the carotid and aortic vessels. The increase arterial pressure raises the firing rate of the afferent baroreceptor nerves. The increased firing rate is detected by the medulla which coordinates afferent baroreceptor signals. After the medulla has processed the information from the baroreceptors, it signals back to the periphery by efferent pathways using the sympathetic and parasympathetic nervous systems. Increased baroreceptor activity instructs the medulla to decrease sympathetic output and increase parasympathetic output. The net result is vasodilation and bradycardia to return the blood pressure to normal

Homeostasis is often maintained through feedback loops. Blood pressure control is an example of a feedback loop in which the CNS monitors blood pressure through afferents from baroreceptors. Deviation from your body's internal blood pressure "set point" triggers an autonomic response (vasoconstriction or vasodilation) to return blood pressure to its normal set point. An increase in blood pressure, above the body's set point, would: a. Cause a reflex decreasing sympathetic output to the heart, while increasing parasympathetic output. b. Cause a reflex decreasing sympathetic output to the heart, while decreasing parasympathetic output. c. Only decrease parasympathetic output. d. Increasing sympathetic output while decreasing parasympathetic output e. Only decrease sympathetic output f. Only increase parasympathetic output while maintaining constant sympathetic output g. None of the above

c. Tissue CO2 production to tissue blood flow Of the answer choices given, only the amount of CO2 made by the tissue and the blood flow will alter differences in CO2 content between the arterial and venous blood.

In a steady state, the difference in CO2 content between the arterial blood entering a tissue and the venous blood leaving the tissue is determined by which of the following ratios? a. Alveolar ventilation to tissue blood flow b. Alveolar ventilation to tissue O2 consumption c. Tissue CO2 production to tissue blood flow d. Tissue CO2 production to tissue O2 consumption e. Tissue CO2 production to venous PCO2 f. Tissue O2 production to tissue O2 consumption

c. Cerebral Cortex The choroid plexus and several restricted areas of the brain lack a BBB; that is, they are supplied by leaky capillaries. Intra-arterially injected dyes can pass into the BECS at these sites through gaps between endothelial cells. The BECF in the vicinity of these leaky capillaries is more similar to blood plasma than to normal BECF. The small brain areas that lack a BBB are called the circumventricular organs because they surround the ventricular system; these areas include the area postrema, posterior pituitary, median eminence, organum vasculosum laminae terminalis, subfornical organ, subcommissural organ, and pineal gland. As part of the systemic circulation, the liver would also be stained by the dye. The cerebral cortex has a blood-brain barrier and would not be stained by the dye.

If a blue dye is injected into the femoral artery (systemic circulation), which area would remain its normal color? a. Choroid plexus b. Pineal gland c. Cerebral Cortex d. Area postrema e. Liver f. Posterior pituitary g. Subcommissural organ

d. An increase in intracellular fluid (ICF) volume 2 L of sweat containing NaCl has been lost. ECF has been lost. Because Na+ is primarily an extracellular cation, ingested pure water is hypotonic to the ICF and will be drawn into the more hypertonic intracellular medium, increasing ICF volume. Drinking pure water after sweating will cause most of the water to move into the ICF volume. Pure water moves into the intracellular space to make it more hypotonic and increase the ICF volume.

If a healthy 70 kg man loses 2 L of sweat while doing yard work and simultaneously drinks 2 L of pure water, which of the following body fluid changes would be expected? a. An increase in extracellular osmolarity b. An increase in extracellular fluid (ECF) volume c. An increase in intracellular osmolarity d. An increase in intracellular fluid (ICF) volume e. An increase in plasma Na+ concentration

c. lung to collapse inward and the chest wall to collapse inward If a person experiences a puncture wound to the chest cavity, so that air enters the thorax from the atmosphere, PIP is increased to the same level as barometric pressure. This condition is called a pneumothorax. With no vacuum to counter their elastic recoil, alveoli will collapse inward. The chest wall movement counters the movement of the lungs, and as a result the chest wall springs outward.

If a person suffered a stab injury and air entered the intrapleural space (pneumothorax), the most likely response would be for the: a. lung to expand outward and the chest wall to spring inward b. lung to expand outward and the chest wall to spring outward c. lung to collapse inward and the chest wall to collapse inward d. lung to collapse inward and the chest wall to spring outward e. lung volume to be unaffected and chest wall to spring outward f. lung volume to be unaffected and the chest wall to collapse inward g. lung to collapse inward and the chest wall to be unaffected

a. Ca2+ Driving force on an ion is the difference in millivolts between the membrane potential and the equilibrium potential for that ion. In this cell EK=-61 mV, ECl=-61 mV, ENa=+61 mV, and ECa=525 mV. Ca2+ is the ion with the equilibrium potential farthest from membrane potential, meaning that Ca2+ would have the greatest tendency to cross the membrane through an open channel (greatest driving force).

If the membrane potential of this cell is -80 millivolts, the driving force is greatest for which ion? a. Ca2+ b. Cl- c. K+ d. Na+ e. HCO3- f. Glucose

c. An increase in atrial and end-diastolic ventricular pressure with increased stroke volume If the venous return is doubled there would be increased atrial and end-diastolic pressures along with an increased stroke volume by the Starling mechanism.

If the venous return to the right atrium should increase from a normal value of about 5 L/min (in the adult) to about 10 L/min, what would a normal heart most likely develop? a. An increase in atrial and end-diastolic ventricular pressure with decreased stroke volume b. A decrease in atrial and end-diastolic ventricular pressure with decreased contractility c. An increase in atrial and end-diastolic ventricular pressure with increased stroke volume d. Bradycardia and cardiac shock e. Bradycardia and very low end-diastolic pressure of the right ventricle f. Tachycardia and ventricular hypertrophy

c. of the combination of pore size and negative charges lining the pore Substances of low molecular weight (<5500 Da) and small effective molec¬ular radius (e.g., water, urea, glucose, and inulin) appear in the filtrate in the same concentration as in plasma (UFX/PX ≈ 1). In these instances, no sieving of the contents of the fluid moving through the glomerular "pores" occurs, so that the water moving through the filtration slits by convection carries the solutes with it. Electrical charge also makes a major contribution to the permselectivity of the glomerular barrier. the glycocalyx overlying the endothelial cells, the glomerular basement membrane, and the epithelial podocytes are covered with negative charges from anionic proteoglycans. Proteins are large and negatively charged, they are repelled by the negative charge of the glomerular barrier and are prevented from filtration by the epithelial podocytes.

In a healthy person, there is very little protein in the glomerular filtrate. This is because a. all serum proteins are too large to fit through the glomerular pores b. of the positive charges lining the pores, which repel serum proteins c. of the combination of pore size and negative charges lining the pore d. Filtered proteins are actively reabsorbed by glomerular epithelial cells e. none of the above f. All of the above

b. An increase in sympathetic tone to the ventricles In exercise, an increased sympathetic tone to the ventricles occurs, along with a decrease in parasympathetic tone. In addition, catecholamines are released increasing contractility.

In a healthy subject, running is usually associated with a decrease in the end-systolic volume of the right and left ventricles and with an increase in their stroke volume. What is the probable mechanism for this response? a. A decrease in catecholamine release b. An increase in sympathetic tone to the ventricles c. An increase in parasympathetic tone to the ventricles d. A decrease in venous return of blood to the heart e. An increase in pulmonary and systemic resistance f. A reduction in cardiac output

c. Return of plasma pH and glucose to normal is accompanied by a shift of potassium into cells Potassium uptake increases in cells treated with insulin and the concomitant administration of K+ with insulin prevents hypokalemia.

In addition to treating a diabetic patient (type 1) for ketoacidosis with insulin, why is a potassium salt often administered? a. Insulin directly increases the excretion of potassium by the kidney b. Return of plasma pH and glucose to normal will lower the total body potassium content c. Return of plasma pH and glucose to normal is accompanied by a shift of potassium into cells d. Plasma potassium is usually low in diabetic ketoacidotic patients e. Return of plasma glucose to normal will cause a decrease in the plasma volume f. Insulin indirectly increases the excretion of potassium by the kidney

f. Osteoporosis Osteoporosis is a reduction in bone mass, and develops as a result of decreased synthesis (osteoblastic activity) and increased resorptive activity (osteoclastic activity). Osteoporosis is associated with the loss of estrogen after menopause, but can also result from physical inactivity, increased cortisol, hyperthyroidism, and calcium deficiency. Without treatment, bone loss will continue, resulting in significant bone loss over time.

In aging, if osteoclast activity is constant, but osteoblast activity progressively decreases, which disease will likely develop? a. Osteoclastoma b. Osteomalacia c. Paget's disease d. Osteogenesis imperfecta e. Scurvy f. Osteoporosis

d. High muscle oxidative capacity Endurance training increases muscle oxidative capacity, which reduces the ventilatory demands of exercise, thus increasing exercise capacity for a high level of performance.

In an elite marathon runner, performance in a marathon race is limited by a number of factors. Among world class marathon runners, which of the following factors is considered to be critical for a high level of performance? a. Muscle ATP content b. Adequate fat stores c. High muscle glycogen content d. High muscle oxidative capacity e. High percentage of type II muscle fibers f. Low capillary density

a. increased, increased, decreased Chronic respiratory acidosis is caused by insufficient pulmonary ventilation result in in an increased in PCO2. Acidosis, in turn, stimulates the secretion of hydrogen ions into the tubular fluid and increased renal tubular production of NH4+, which further contributes to the excretion of hydrogen ion sand the renal production of HCO3, thereby increasing plasma bicarbonate concentration. The increased tubular secretion of hydrogen ions also reduces urine pH

In chronic respiratory acidosis with partial renal compensation, you would expect to find the following changes, compared to normal: ________ urinary excretion of NH4+; ______ plasma HCO3 concentration; and ________ urine pH. a. increased, increased, decreased b. increased, decreased, decreased c. no change in, increased, decreased d. no change in, no change in, decreased e. increased, no change in, increased f. decreased, no change in, increased

b. Thelarche Thelarche is breast development which occurs in women at puberty in response to ovarian steroid hormones.

In males, testosterone (or DHT) stimulates all the following EXCEPT: a. Pubertal growth of the penis, which will be permanent b. Thelarche c. Lowering of pitch of the voice, which will be permanent d. Hair growth on chest and face e. Hair growth up the midline of the abdomen

e. Ovulation Ovulation occurs at point A on the graph. In response to estrogen secretion by the ovary, the endometrial lining of the uterus undergoes proliferation of both glandular epithelium and supporting stroma during the first 10 to 14 days of the menstrual cycle. Following ovulation, the glands begin to secrete mucus and the stroma undergoes pseudodecidual reaction in preparation for potential pregnancy. When ovulation is not followed by implantation of a fertilized ovum, progesterone secretion declines as the corpus luteum involutes, and the endometrial lining is almost completely shed during menses.

In the following graph of changes in endometrial thickness during a normal 28day menstrual cycle, the event designated A corresponds most closely to which of the following phases? a. The menstrual phase b. The maturation of the corpus luteum c. The early proliferative phase d. The secretory phase e. Ovulation f. Folliculogenesis

d. D The graph illustrates the development of pressure in the aorta, the left atrium, and the left ventricle during a single cardiac cycle. At point time D, the pressure in the LV is less than the pressure of the LA and therefore the mitral valve opens and ventricular filling begins. Although the volume in the LV is increasing, the pressure is falling. During this time period, the recoil of the ventricle causes its pressure to decrease as it is filling. Later in diastole, the pressure of the blood returning from the lung causes both volume and pressure in the ventricle to increase (time A). At time B, the atrial and ventricular pressure curves separate as the mitral valve closes and isovolumetric contraction occurs. At time E, ventricular filling begins to slow.

In the hemodynamic pressure tracings below, rapid ventricular filling begins at which point? a. A b. B c. C d. D e. E

a. A The astroglial endfoot is in close apposition to the cerebral capillary. A. Astroglial Endfood B. Endothelial Cell C. Tight Junction D. Pericyte E. Capillary Lumen

In the image below, which of the following represents the astroglial endfoot? a. A b. B c. C d. D e. E

a. A 75yearold man who has smoked two packs of cigarettes per day for 60 years. His breath sounds are decreased bilaterally and his chest xray shows flattening of the diaphragm Cigarette smoking is the major cause of COPD. In obstructive lung diseases, the increase in airway resistance causes a decrease in expiratory flow rates and "airtrapping" which results in an increased residual volume, and thus total lung capacity. This hyperinflation pushes the diaphragm into a flattened position. Asbestosis and pulmonary fibrosis are restrictive lung diseases, in which curve C would be the typical MEFV curve. Decreased effort would decrease flow rates during the effort dependent portion of a MEFV curve, but not during the effort independent portion.

In the maximal expiratory flowvolume curves below, curve A would be typical of which of the following clinical presentations? a. A 75yearold man who has smoked two packs of cigarettes per day for 60 years. His breath sounds are decreased bilaterally and his chest xray shows flattening of the diaphragm b. A 68yearold man who presents with a dry cough that has persisted for 3 months. His chest xray shows opacities in the lower and middle lung fields. The man states that he was exposed to asbestos for approximately 10 years when he worked in a factory in his 30s c. A 57yearold woman with pulmonary fibrosis who presents to the emergency room with shortness of breath d. An 84yearold woman with a history of myocardial infarction who reports shortness of breath that worsens in the recumbent position e. A healthy, 22yearold man getting his army enlistment physical exam. He has never smoked, but is tired that morning and does not use much effort while exhaling f. A 16yearold boy who is out of breath after completing the 100meterdash at a track meet

c. papilla, taste bud, taste pore, taste receptors Taste receptors are located mainly on the dorsal surface of the tongue, concentrated within small but visible projections called papillae. Papillae are shaped like ridges, pimples, or mushrooms, and each is a few millimeters in diameter. Each papilla in turn has numerous taste buds. One taste bud contains 50 to 150 taste receptor cells, numerous basal and supporting cells that surround the taste cells, plus a set of sensory afferent axons. The chemically sensitive part of a taste receptor cell is a small apical membrane region near the surface of the tongue. The apical ends have thin extensions called microvilli that project into the taste pore, a small opening on the surface of the tongue where the taste cells are exposed to the contents of the mouth.

In the morphological organization from large to small, order the components of the sense of taste: a. taste bud, papilla, taste pore, taste receptors b. taste receptor, taste bud, papilla, taste pore c. papilla, taste bud, taste pore, taste receptors d. taste bud, taste pore, papilla, taste receptors e. taste papilla, taste pore, taste bud, taste receptor f. papilla, taste bud, taste receptors, taste pore g. taste pore, taste bud, papilla, taste receptor

a. Vision The visual receptor cells, the rods and cones, are depolarized when the eyes are in the dark. When exposed to light, they hyperpolarize. Light causes the rods and cones to hyperpolarize by activating a G protein called transducin, which leads to the closing of Na+ channels. Auditory receptors are depolarized by the flow of K+ into the hair cells. Touch receptors are activated by opening channels through which both Na+ and K+ can flow. Depolarization is caused by the inward flow of Na+. Smell and taste receptors are activated by G protein mediated mechanisms, some of which cause the receptor cell to depolarize; other G proteins cause the release of synaptic transmitter without any change in membrane potential.

In which one of the following sensory systems does stimulation cause the receptor cell to hyperpolarize? a. Vision b. Hearing c. Taste d. Touch e. Smell

c. Mullerian ducts to disappear, leaving Wolffian ducts intact Sertoli cells of the testis produce a nonsteroid macromolecule—anti Mullerian hormone (AMH) or Mullerianinhibiting substance (MIS)— that causes Mullerian degeneration in the male fetus. During embryogenesis in males, AMH—which is secreted by the Sertoli cells in the testis—causes involution of the Mullerian ducts. In females, the Mullerian ducts differentiate spontaneously in the absence of AMH, and the Wolffian ducts involute spontaneously in the absence of testosterone.

MIS (antiMullerian factor) causes the: a. Wolffian ducts to develop into male internal structures b. Mullerian ducts to develop into female internal structures c. Mullerian ducts to disappear, leaving Wolffian ducts intact d. Gonadal ridges to develop into testes instead of ovaries e. Testes to secrete testosterone in early fetal life, inducing Wolffian ducts to develop into male external structures f. Mullerian and Woffian ducts to develop into female internal structures

d. Step 4 Latch bridges are unphosphorylated myosin crossbridges that are bound to actin. These crossbridges cycle very slowly or not at all and are responsible for the ability of smooth muscle to maintain its tone for a long time without expending energy for crossbridge cycling. The enzyme myosin light chain phosphatase is responsible for dephosphorylating cycling crossbridges (Step 4). The smooth muscle relaxes when Ca2+ is removed from the myoplasm and the latch bridges detach from actin. Crossbridge cycling in smooth muscle cannot begin until the myosin light chains are phosphorylated (Step 1). Phosphorylation is enzymatically stimulated by myosin light chain kinase (MLCK).

Shown below is a diagram of the chemical reactions that occur during crossbridge cycling in smooth muscle. Which of the following steps is responsible for the formation of latch bridges? a. Step 1 b. Step 2 c. Step 3 d. Step 4 e. Step 5

c. Shrink initially and then swell until lysis When placed in the hypertonic urea solution, the cell will initially lose water and shrink, but because it is also permeable to urea the cell will then swell when exposed to extracellular hypertonicity and eventually rupture due to the high concentration of urea that has entered the cell.

Suppose the plasma membrane of a particular mammalian cell is much more permeable to water and urea than to any other solute. The water permeability is higher than the urea permeability. The cell is initially in a normal physiological saline solution and then is suddenly moved into a solution consisting of 600 mM urea as the only solute. The cell will: a. Swell initially and continue to swell until lysis b. Swell initially until it reaches osmotic equilibrium with the medium c. Shrink initially and then swell until lysis d. Shrink initially and stay shrunken e. Swell initially and then shrink back to normal volume f. Remain the same volume

e. Carbon monoxide poisoning Cyanosis is the blue color of the skin produced by desaturated hemoglobin. Cyanosis appears when 5 g of hemoglobin per 100 mL of blood are desaturated. For a person with a normal hemoglobin concentration of 15 g/100 mL, cyanosis appears when one-third of the blood is desaturated. For a person with polycythemia (a higher-than- normal concentration of hemoglobin), cyanosis may appear when only one-fourth of the hemoglobin is desaturated (e.g., if hemoglobin concentration is 20 g/100 mL). This individual may not be hypoxic because of the high concentration of saturated hemoglobin. On the other hand, a person with anemia (a lower than normal concentration of hemoglobin) may have a significant portion of the hemoglobin desaturated without displaying cyanosis. This individual will not appear cyanotic but may be hypoxic.

The affinity of hemoglobin for oxygen is increased by which of the following? a. Metabolic acidosis b. Exercise c. Hypoxemia d. Anemia e. Carbon monoxide poisoning f. Increased core body temperature

b. Autoregulation Hyperemia is the increase in blood flow to the area which explains the abrupt increase in flow, but not the return to normal levels. Autoregulation is the ability of a vascular bed to adjust blood flow in response to external stimuli. Ischemia is the absence of blood flow.

The blood flow through an organ is measured while the perfusion pressure is varied experimentally. An abrupt, sustained increase in perfusion pressure increases flow initially, but over the course of 1 minute, the flow returns nearly to the baseline level despite continued elevation of the perfusion pressure. The organ under study is exhibiting which of the following? a. Active hyperemia b. Autoregulation c. Ischemia d. Reactive hyperemia e. Static equilibrium

d. Autoregulation Autoregulation is the ability of an organ to regulate the flow of blood through its vascular bed in response to changes in the perfusion pressure through the organ.

The blood flow through the kidney is measured while the perfusion pressure is varied experimentally. An abrupt, sustained increase in perfusion pressure increases flow initially, but over the course of 1 minute, the flow returns nearly to the baseline level despite continued elevation of the perfusion pressure. The kidney is exhibiting which of the following? a. Active hyperemia b. Ischemia c. Reactive hyperemia d. Autoregulation e. Sympathetic stimulation f. Dynamic steady state

c. They are suspended in a medium whose osmotic strength is 200 mOsm Conditions that cause the erythrocyte to swell: suspension in a hypotonic medium (medium with osmotic strength <300 mOsm), suspension in a medium with a sodium concentration <140 mmol/L, any combination of one of these factors with a normal level of the other. Ouabain inhibits the Na/K pump causing [Na+]i to rise which in turn stimulates an influx of water into the cell.

The cellular volume of human erythrocytes will decrease when: a. They are suspended in a medium whose osmotic strength is 300 mOsm and that has a sodium concentration of 140 mmol/L b. The cells' sodium pump is inhibited with ouabain c. They are suspended in a medium whose osmotic strength is 200 mOsm d. They are suspended in a medium whose osmotic strength is 450 mOsm e. They are suspended in a medium whose osmotic strength is 300 mOsm and that has a sodium concentration of 100 mmol/L f. They are suspended in a medium whose osmotic strength is 100 mOsm and that has a sodium concentration of 110 mmol/L

c. Red blood cell hemoglobin Aerobic metabolism produces 13,000 to 24,000 mmol CO2 per day. This yields close to that amount of H+ions produced per day via the reaction CO2 +H2O<-->H2CO3<-->H+ + HCO3-. At the tissues, CO2 diffuses into the red blood cells, where the enzyme carbonic anhydrase accelerates the above reaction. The H+ produced is buffered mainly by the large amount of hemoglobin in the red blood cells. Bicarbonate is not an effective buffer of volatile acid (from CO2).

The daily production of hydrogen ions from CO2 is primarily buffered by which of the following? a. Extracellular bicarbonate b. Red blood cell bicarbonate c. Red blood cell hemoglobin d. Plasma proteins e. Plasma phosphate f. Plasma glucose

D. Slower cycling rate of the smooth muscle myosin cross-bridges Slower cycling rate of the cross-bridges in smooth muscle means that a higher percentage of cross-bridges is active at any point in time. The more active cross-bridges there are, the greater the force generated. Although the relatively slow cycling rate means that it takes longer for the myosin head to attach to the actin filament, it also means that the myosin head remains attached longer, prolonging contraction. Because of the slow cross-bridge cycling rate, smooth muscle actually requires less energy to maintain a contraction compared with skeletal muscle.

The delayed onset and prolonged duration of smooth muscle contraction, as well as the greater force generated by smooth muscle compared with skeletal muscle, are all consequences of which of the following? a. Greater amount of myosin filaments present in smooth muscle B. Higher energy requirement of smooth muscle C. Physical arrangement of actin and myosin filaments D. Slower cycling rate of the smooth muscle myosin cross-bridges E. Slower uptake of Ca2+ ions following contraction

e. S > T > U Compliance is the change in lung volume that occurs for a given change in the transpulmonary pressure. (The transpulmonary pressure is the difference between the alveolar pressure and pleural pressure). Because compliance is equal to the slope of the volume-pressure relationship, it should be clear that curve S represents the highest compliance, and that curve U represents the lowest compliance.

The diagram above shows three different compliance curves (S, T, and U) for isolated lungs subjected to various transpulmonary pressures. Which of the following best describe the relative compliances for the three curves? a. S < T < U b. S < T > U c. S = T = U d. S > T < U e. S > T > U f. S < T = U g. S = T < U

b. B During inspiration (curve ABC), the respiratory muscles pull the chest wall out and diaphragm down and intrapleural pressure (PIP) becomes more negative (subatmospheric). The muscles must overcome the elastic recoil forces of the lungs and the resistance of the airways to airflow. The PIP necessary to overcome the elastic forces of the lung is depicted by dashed line AC. The PIP necessary to overcome the airway resistance is the difference between dashed line AC and curve ABC. The maximum airflow occurs at point B, where the difference between the two is the greatest.

The diagram below illustrates the change in intrapleural pressure during a single tidal breath. At which point on the diagram is inspiratory airflow the greatest? a. A b. B c. C d. D e. E

f. Inspired lung volume The equal pressure point is the point at which the pressure inside the airways equals the intrapleural pressure. The intra-airway pressure closest to the alveoli equals the sum of the recoil pressure (exerted by the alveoli) and the intrapleural pressure (produced by the muscles of expiration). The equal pressure point moves further away from the lungs if the recoil force is increased and moves closer to the lungs when the intrapleural pressure is increased. Increasing the lung volume expands the alveoli, making their recoil force greater and the intrapleural pressure less (more negative). This moves the equal pressure point toward the mouth. If airway resistance increases by increasing airway smooth muscle tone or increasing lung compliance, then a greater expiratory effort and consequently a greater intrapleural pressure will be necessary to expel the gas from the lungs. The higher intrapleural pressure when airway resistance is increased will cause the

The diagram below illustrates the intrapleural pressure generated by a patient during a forced expiration. The equal pressure point will move closer to the mouth and the forced expiratory volume will increase if there is an increase in which of the following? a. Pulmonary vascular resistance b. Lung compliance c. Airway resistance d. Expiratory effort e. Airway smooth muscle tone f. Inspired lung volume

d. Movement of Na+ into the cell After a stimulus is delivered to the cell and the membrane potential reaches threshold, the cell rapidly depolarizes due to the opening of many voltage-gated sodium channels which permit the influx of sodium into the cell. The increase in sodium conductance is responsible for the upstroke of the action potential.

The diagram shows the change in membrane potential during an action potential in a giant squid axon. Which of the following is primarily responsible for the change in the membrane potential between points B and D? a. Inhibition of Na+/K+-ATPase b. Movement of K+ into the cell c. Movement of K+ out of the cell d. Movement of Na+ into the cell e. Movement of Na+ out of the cell f. Hyper-excitation of the Na+/K+-ATPase

c. Movement of K+ out of the cell Repolarization (the downstroke of the AP) is due to the delayed opening of voltage-gated potassium channels which allow the efflux of potassium out of the cell. These channels are stimulated by depolarization but open more slowly than voltage-gated sodium channels. This allows time for the cell to rapidly depolarize resulting in the upstroke of the action potential. Potassium channels slowly open and potassium conductance out of the cell leads to the return of the membrane potential to baseline, then hyperpolarized levels.

The diagram shows the change in membrane potential during an action potential in a giant squid axon. Which of the following is primarily responsible for the change in the membrane potential between points D and E? a. Inhibition of Na+/K+-ATPase b. Movement of K+ into the cell c. Movement of K+ out of the cell d. Movement of Na+ into the cell e. Movement of Na+ out of the cell f. Hyper-excitation of the Na+/K+ ATPase

c. As an antihypertensive, since ACE converts the inactive angiotensin I to the active angiotensin II ACE converts inactive ANG I to active ANG II. ANG II stimulates the release of aldosterone which promotes the reabsorption of sodium and water from the kidney tubule. The overall effect of ACE is to raise the blood pressure. As captopril inhibits the activity of ACE, its clinical usefulness is as an anti-hypertensive medication.

The drug captopril prevents the action of angiotensin-converting enzyme (ACE). Its clinical usefulness, therefore, is: a. As an antihypertensive because it prevents renin secretion b. As an antihypertensive because it prevents the action of renin on angiotensinogen c. As an antihypertensive, since ACE converts the inactive angiotensin I to the active angiotensin II d. As a pressor, since ACE converts the active angiotensin I to the inactive angiotensin II e. As a pressor, since the action of ACE prevents angiotensins stimulation of aldosterone secretion f. As a pressor, since ACE promotes the reabsorption of sodium and water from the kidney tubule

c. Point C Point C indicates the beginning of ejection. At this point, blood is leaving the heart so the aortic valve must open here.

The figure below represents a pressure-volume loop of the left ventricle for a single cardiac cycle. The point in the figure at which the aortic valve opens is: a. Point A b. Point B c. Point C d. Point D e. Point E

d. +40 degrees The mean electrical axis (MEA) represents the average direction traveled by the ventricular muscle action potentials as they propagate through the heart. The propagation path and the mass of tissue through which the action potentials travel influence the MEA. Normally the MEA ranges from +30° to +100°. The MEA is approximately perpendicular to the axis of the limb lead with the smallest QRS wave magnitude. In this case, the smallest deflection is in lead III. Therefore, the MEA lies along lead aVR. Because the QRS complex is negative in aVR, the MEA is approximately +30 degrees. Because the QRS complex is greater in lead II than in lead I, the MEA is between +60 degrees and +30 degrees.

The following 6-lead frontal ECG was performed as part of an annual physical exam. What is the mean electrical axis of the patient? a. +10 degrees b. -10 degrees c. +20 degrees d. +40 degrees e. +70 degrees f. -120 degrees

d. 3:1 The ratio of the blood flow through vessels Y and Z is inversely proportional to their resistance. Because vessel Y has half the resistance of vessel Z, it has twice the blood flow. The blood flowing through vessel X is the sum of the blood flowing through vessels Y and Z (2 + 1 = 3). Therefore, the ratio of the blood flowing through vessels X and Y is 3:2. QVessel Y ∞ 1/1 ; QVessel Z ∞ 1/2 QVessel Y/QVessel Z = 1/1 × 2/1 1/2 × 2/1 = 2/1 QVessel X = QVessel Y + QVessel Z = 2 + 1 = 3 QVessel X /QVessel Y = 3/2

The following diagram illustrates the relative resistance of three vessels. Which of the following is the ratio of the flow in vessel X to the flow in vessel Y? a. 1:1 b. 3:2 c. 2:1 d. 3:1 e. 4:3 f. 3:4

b. Emphysema The forced vital capacity (FVC) is the vital capacity measured with a forced expiration (FVC for patient Z= 4.0 L). The forced expiratory volume in one second (FEV1) is the amount of air that can be expelled from the lungs during the first second of a forced expiration (FEV1 for patient Z = 2.0 L). FEV1/FVC is a function of airway resistance. Airway resistance is often increased in emphysema which causes FEV1/FVC to decrease. FEV1/FVC is 50% in patient Z and 80% in the normal curve. FEV1/FVC ratio is not usually affected in pleural effusion and pneumothorax because airway resistance is normal. FVC is often decreased in asbestosis, fibrotic pleurisy, silicosis, and tuberculosis and the FEV1/FVC ratio is either normal or slightly increased.

The following diagram shows forced expirations from a person with healthy lungs (curve X) and from a patient (curve Z). Which of the following can best explain the results from the patient? a. Asbestosis b. Emphysema c. Fibrotic pleurisy d. Pleural effusion e. Pneumothorax f. Silicosis g. Tuberculosis

c. Cheyne-Stokes breathing Cheyne-Stokes breathing is the most common type of period respiration. It is characterized by a cycle of progressively deeper, and sometimes faster, breathing followed by a gradual decrease with periods of minimal breathing or apnea in between. Biot breathing is an abnormal pattern of breathing characterized by groups of quick, shallow inspirations followed by regular or irregular periods of apnea. Kussmaul breathing is a deep and labored breathing pattern often associated with severe metabolic acidosis, particularly diabetic ketoacidosis (DKA) but also kidney failure. Hyperpnea is increased depth and rate of breathing. It may be physiologic—as when required to meet metabolic demand of body tissues (for example, during or after exercise, or when the body lacks oxygen at high altitude or as a result of anemia)—or it may be pathologic, as when sepsis is severe. Tachypnea is abnormally rapid breathing. Eupnea is normal, good, unlabored breathing, sometimes known as quiet breathing or resting respiratory rate. Apnea is suspension of breathing. Below is a chart of various common breathing patterns.

The following diagram shows the depth of respiration of a 45-year-old man who suffered a head on automobile accident. This "crescendo-decrescendo" pattern of breathing is called which of the following? a. Apnea b. Biot breathing c. Cheyne-Stokes breathing d. Hyperpnea e. Eupnea f. Tachypnea g. Kussmaul breathing

C. multipolar The image is a multipolar neuron. There are four basic types of neurons: unipolar, bipolar, multipolar, and pyramidal. Unipolar neurons have only one structure that extends away from the soma. These neurons are not found in vertebrates but are found in insects where they stimulate muscles or glands. A bipolar neuron has one axon and one dendrite extending from the soma. An example of a bipolar neuron is a retinal bipolar cell, which receives signals from photoreceptor cells that are sensitive to light and transmits these signals to ganglion cells that carry the signal to the brain. Multipolar neurons are the most common type of neuron. Each multipolar neuron contains one axon and multiple dendrites. Multipolar neurons can be found in the central nervous system (brain and spinal cord). Pseudounipolar cells share characteristics with both unipolar and bipolar cells. A pseudounipolar cell has a single process that extends from the soma, like a unipolar cell, but this process later branches into two distinct structures, like a bipolar cell. Pyramidal neurons have a pyramidal shaped cell body as well as pyramidal dendritic projections from the cell body.

The following image represents what kind of neuron? A. Bipolar B. Unipolar C. Multipolar D. Prymidal E. Apolar F. Pseudounipolar G. Pseudobipolar

c. Increasing the frequency of stimulation of the fiber Increasing the frequency of stimulation of the fiber increases the sarcoplasmic concentration of Ca2+ resulting in an increase of force generation in the fiber. Increasing the amplitude of depolarization and increasing the number of Na+ channels will not affect Ca2+ release. Decrease in extracellular K+ and increase in muscle permeability to K+ would reduce the excitability of the muscle cell.

The force produced by a single skeletal muscle fiber can be increased by which of the following: a. Decreasing extracellular K+ concentration b. Increasing the amplitude of the depolarizing stimulus c. Increasing the frequency of stimulation of the fiber d. Increasing the number of voltage-gated Na+ channels to the sarcolemma e. Increasing the permeability of the sarcolemma f. Shortening the length of the sarcomere in the muscle fiber g. Decreasing the amount of calcium released into the sarcoplasm

a. Hypokalemia, hypochloremia, and metabolic alkalosis In this question, most of the lead-in is not necessary to answer the question. The patient's symptoms of vomiting and orthostatic hypotension are all that is required to answer the question. Vomiting causes a loss of the acid load of the stomach and causes an upward shift in pH resulting in a metabolic alkalosis. Gastric juices contain hydrogen, potassium, and chloride, and persistent vomiting results in a loss of these electrolytes leading to hypochloremia and hypokalemia. Vomiting also results in dehydration, stimulating water, sodium, and bicarbonate reabsorption at the expense of potassium and hydrogen excretion

The friends of a 26-year-old man plan a bachelor party for him in Las Vegas. After a round of golf, the group heads to the pool. They order several rounds of drinks for him over the next four hours and also order lunch poolside. Most of the group orders hamburgers and French fries, but the groom-to-be is watching his weight and opts for a club sandwich with a side of coleslaw. Later that night, they go for dinner and to the casinos, where they drink more. Early the next morning, the groom-to-be becomes ill. He believes it is just a hangover, but presents to the emergency department 36 hours with persistent vomiting and orthostatic hypotension. Which of the following metabolic abnormalities are most likely present in this patient? a. Hypokalemia, hypochloremia, and metabolic alkalosis b. Hypokalemia, hypochloremia, and metabolic acidosis c. Hyperkalemia, hyperchloremia, and metabolic alkalosis d. Hyperkalemia, hyperchloremia, and metabolic acidosis e. Hypokalemia, hypochloremia, and respiratory alkalosis f. Hyperkalemia, hypochloremia, and respiratory acidosis g. Normal serum electrolytes and acid-base balance

a. Increases the area available for gas exchange by opening closed alveoli As aging occurs, more and more of the basal alveoli collapse, making it impossible for gas exchange to take place, particularly at the base of the lung. To overcome this situation, positive pressure (PEEP) is applied during expiration. Thus, PEEP helps open closed basal alveoli and prevents other alveoli from collapsing. This procedure increases the functional residual capacity (FRC), increases the area available for gas exchange, and greatly improves oxygenation of the blood.

The fundamental benefit of applying positive end-expiratory pressure (PEEP) to an 80-year-old patient with a large closing volume would be that PEEP: a. Increases the area available for gas exchange by opening closed alveoli b. Increases compliance, making it easier to breathe c. Promotes an increase in pulmonary blood flow d. Significantly increases the PaO2 when the patient is breathing air e. Stimulates breathing by tonically activating pulmonary stretch receptors f. Increases the concentration of surfactant surrounding the alveoli

e. 1:20 Compliance is defined as the change in volume divided by the change in pressure (dV/dP). The lower the compliance, the stiffer the vessel becomes. The venous system is much more compliant than the arterial system. Arterial compliance = dV/dP = 250 mL/160 mmHg = 1.56 mL/mmHg Venous compliance = dV/dP = 1500 mL/50 mmHg = 30 mL/mmHg Arterial compliance/Venous compliance = 1.56 mL/mmHg/30 mL/mmHg= 1:19.2

The graph below illustrates the pressure-volume curves for the arterial and venous systems. Which of the following is the approximate ratio of the arterial compliance to the venous compliance? a. 15:1 b. 10:1 c. 1:1 d. 1:10 e. 1:20 f. 1:15

b. Levels of ACh in the neuromuscular junction Tensilon inhibits the activity of acetylcholine esterase allowing more ACh to remain in the NMJ and bind to nAChRs.

The increased muscle strength observed during the Tensilon (edrophonium) test is due to an increase in which of the following? a. Amount of acetylcholine released from the motor nerves. b. Levels of ACh in the neuromuscular junction c. Number of ACh receptors on the motor end-plate d. Synthesis of norepinephrine from the adrenal chromaffin cells e. SERCA2a activity from the sarcoplasmic reticulum f. Phospholamban activity on the sarcoplasmic

a. Foramina of Monro The two lateral ventricles each communicate with the third ventricle, which is located in the midline between the thalami, through the two interventricular foramina of Monro.

The lateral ventricles are joined to the third ventricle via the a. Foramina of Monro b. Cerebral aqueduct of Sylvius c. Dura mater d. Foramina of Luschka e. Arachnoid villus f. Pia mater g. Subarachnoid space

e. Sodium When the permeability of a particular ion is increased, the membrane potential moves toward the equilibrium potential for that ion. The equilibrium potentials for chloride (-80 mV) and potassium (-92 mV) are close to the resting membrane potential, so increases in their permeability have little effect on the resting membrane potential. The equilibrium potential for sodium (+60 mV) is very far from the resting membrane potential. Thus, increasing the permeability for sodium causes a large depolarization.

The membrane potential will depolarize by the greatest amount if the membrane permeability increases for which of the following? a. Potassium b. Sodium and potassium c. Chloride d. Potassium and chloride e. Sodium

b. inhibition of action potential generation by near-threshold EPSPs. GABA is the most prevalent inhibitory neurotransmitter in the Central Nervous System, and its binding to an ion-channel will cause an inhibitory effect on the cell.

The most likely effect of opening GABA-activated ion channels in an adult neuron is a. depolarization. b. inhibition of action potential generation by near-threshold EPSPs. c. opening of channels with a reversal potential positive to threshold. d. opening of voltage-dependent K+ channels. e. opening of voltage-dependent Na+ channels.

c. Glutamate Glutamate is the most prevalent excitatory neurotransmitter in the CNS and has a role in the vast majority of excitatory activity in the human brain.

The most prevalent excitatory neurotransmitter in the Central Nervous System is: a. Norepinephrine/Noradrenaline b. Dopamine c. Glutamate d. Glutamine e. GABA f. Serotonin g. Acetylcholine

d. D There is a marked increase in progesterone secretion following ovulation. Almost all the progesterone secreted in nonpregnant women is secreted by the corpus luteum. Secretion of both progesterone and estrogen is controlled by luteinizing hormone (LH) released by the adenohypophysis, and LH release itself is under the direction of a hypothalamic releasing factor.

The normal pattern of progesterone secretion during the menstrual cycle is exhibited by which of the following curves? a. A b. B c. C d. D e. E

b. 510nm The spectral sensitivity of rods (obtained with a spectrophotometer) peaks at ~500 nm;

The peak sensitivity of rods is closest to which wavelength? a. 420nm b. 510nm c. 605nm d. 700nm

D. D The plateau phase of the ventricular muscle action potential is due to an influx of calcium into the cell. Although calcium channels begin to open during the upstroke, the greatest numbers of calcium channels open during the plateau. The upstroke (B) is due primarily to the opening of Na+ channels. The initial repolarization (C) is dependent on the inactivation of Na+ channels and the opening of transient K+ channels. Repolarization (E) is produced by the inactivation of Ca2+ channels and the activation of the delayed rectifier K+ channels. The resting membrane potential (A) is determined by potassium conductance.

The phases of the ventricular muscle action potential are represented by the lettered points on the diagram below. At which point on the ventricular action potential is membrane potential most dependent on calcium permeability? A. A B. B C. C D. D E. E

e. salt, sour

The taste qualities that signal through an initial direct ionotropic receptor/channel are: a. umamii, salt b. sweet, umami c. bitter, sour d. salt, bitter e. salt, sour

c. Fetal adrenal gland and placenta The fetoplacental unit (the placenta and fetal adrenal cortex) is involved in the synthesis of estrogen during pregnancy in humans. Fetal adrenocorticotropic hormone (ACTH) stimulates the secretion of dehydroepiandrosterone (DHEA) by the fetal adrenal, which is converted to 16-OH-DHEA by the fetal liver. The 16-OH-DHEA or 16-OH DHEAs (sulfated form) is taken up by the placenta and converted to estriol. Because the fetal placental unit requires a functioning fetal pituitary, adrenal, and liver and placenta, the plasma concentrations of estriol are used to determine the viability of the fetus.

The primary source of estriol (E3) during pregnancy is the: a. Maternal ovary b. Fetal ovary c. Fetal adrenal gland and placenta d. Endometrium e. Corpus luteum f. Fetal pituitary gland

a. Acetylcholine Acetylcholine released by preganglionic sympathetic fibers of these nerves acts on nicotinic acetylcholine receptors, causing cell depolarization and an influx of calcium through voltage-gated calcium channels. Calcium triggers the exocytosis of chromaffin granules and, thus, the release of adrenaline (and noradrenaline) into the bloodstream.

The release of epinephrine from the chromaffin granules of the adrenal medulla into the bloodstream in response to neural stimulation is mediated by which of the following? a. Acetylcholine b. γ-Aminobutyric acid (GABA) c. Aldosterone d. Dopamine e. Serotonin

c. Left ventricle and aorta The pressure gradient between regions of the cardiovascular system is directly proportional to the resistance of the intervening structures. During ventricular ejection, the aortic valves are open and do not offer any significant resistance to blood flow. Therefore, there is very little, if any, pressure difference between the left ventricle and the aorta. Because the tricuspid valve is closed during ventricular ejection, there is an appreciable pressure difference between the right ventricle and the left atrium, although this pressure difference is opposite in direction to the flow of blood through the circulatory system. Although pulmonary vascular resistance is relatively small compared with systemic vascular resistance, it nonetheless produces a pressure drop between the right ventricle and the left atrium. Because most of the resistance in the systemic vasculature occurs at the level of the arterioles, there is a large pressure gradient between the aorta and the capillaries.

The spouse of a 58-year-old-man calls 9-1-1 because her husband complains of chest pain radiating down his left arm. He is transported to the Emergency Department, where an electrocardiogram and cardiac enzymes indicate a recent myocardial infarction. The man is sent for a cardiac catheterization, including coronary angiography and hemodynamic recordings throughout the cardiac cycle. No valvular defects were present. During ventricular ejection, the pressure difference smallest in magnitude is between which of the following? a. Pulmonary artery and left atrium b. Right ventricle and right atrium c. Left ventricle and aorta d. Left ventricle and left atrium e. Aorta and capillaries f. Venules and vena cava

b. Emphysema This patient has emphysema. The loss of alveolar walls with destruction of associated capillary beds in emphysematous lungs reduces the elastic recoil and increases the compliance. Compliance is equal to the change in lung volume for a given change in transpulmonary pressure. As indicated in the graph, an increase in transpulmonary pressure results in a much greater increase in lung volume in the patient than in the normal lung, showing increased lung compliance. Asbestosis, silicosis, and tuberculosis are all associated with deposition of fibrous tissue in the lungs which decreases compliance. Mitral obstruction, rheumatic heart, and aortic stenosis can cause pulmonary edema which also decreases pulmonary compliance.

The volume-pressure curves shown in the next diagram were obtained from a young, healthy subject and a patient. Which of the following best describes the condition of the patient? a. Asbestosis b. Emphysema c. Mitral obstruction d. Rheumatic heart disease e. Silicosis f. Tuberculosis g. Aortic stenosis

a. Bipolar Bipolar neurons have two processes that exit the cell body

This is an example of what type of nerve cell? a. Bipolar b. Unipolar c. Multipolar d. Pyramidal

d. Vasodilation of cutaneous vasculature Vasodilation of cutaneous vasculature with exercise promotes heat loss from the skin to lower the increased body temperature.

Walking on a treadmill increased the body temperature of a 67-year-old male patient from 98.1°F to 103.2°F. What will the increase in body temperature cause? a. Inhibition of postganglionic sympathetic cholinergic activation b. Increased thyroid stimulating hormone (TSH) secretion c. A shift to the left in the O2 dissociation curve d. Vasodilation of cutaneous vasculature e. A decrease in the basal metabolic rate (BMR) f. An increase in oxygen affinity of the hemoglobin molecule

b. Sinus tachycardia There is a P wave before every QRS complex, P waves have consistent morphology. PR interval is < 20 ms. QRS complex is narrow. ST segment shows no elevation or depression. Rate is elevated as determine by the previous question so, the patient must have sinus tachycardia.

What is his current diagnosis? a. Normal sinus rhythm b. Sinus tachycardia c. ST segment elevation myocardial infarction d. First degree heart block e. Second degree heart block f. Third degree heart block

e. 38 mOsm/L To answer this question, you will need to keep in mind that all of these compounds will dissociate into their individual ions in solution. NaCl dissociates into Na+ and Cl- for a total osmolarity of 24 mmol. KCl dissociates into K+ and Cl- for a total osmolarity of 8 mmol. CaCl2 dissociates into 1 molecule of Ca2+ and 2 molecules of Cl- for a total osmolarity of 6 mmol. Adding all of these together yields a total calculated osmolarity of 38 mOsm/L.

What is the calculated osmolarity of a solution containing 12 millimolar NaCl, 4 millimolar KCl and 2 millimolar CaCl2? a. 16 mOsm/L b. 26 mOsm/L c. 29 mOsm/L d. 34 mOsm/L e. 38 mOsm/L f. 40 mOsm/L

c. C Placing a cell in a hypertonic environment will promote the movement of water out of the cell leading to cell shrinkage. The water moves in order to equilibrate the difference in osmolarity between the cell and its surroundings. When water flows out of the cell its osmolarity increases and the movement of water into the surrounding causes a decrease in the osmolarity of the surrounding.

When a red blood cell is placed in a hypertonic environment, initially its volume will (X) because of the movement of (Y). a. A b. B c. C d. D e. E

c. Be subnormal and not increase after a single TRH injection High circulating levels of T3/T4 (thyroid hormone) will exhibit a strong negative feedback effect on TSH. Prolonged elevation of thyroid hormone is not easily overcome and will instead require more than a single TRH injection to simulate production of TSH.

When concentrations of thyroid hormones have been abnormally high for 2 to 3 months due to an overdose with T4, then the circulating TSH concentration will: a. Increase b. Be subnormal but increase after a single TRH injection c. Be subnormal and not increase after a single TRH injection d. Remain normal and not increase after a single TRH injection e. Remain normal and increase after a single TRH injection f. Decrease

d. Increases extracellular [Na+] The Na+/K+-ATPase is responsible for pumping Na+ out of the cell and K+ into the cell. If the pump is inhibited, sodium will accumulate in the cell, while potassium accumulates outside of the cell. The concentration of K+ in the ventricle will increase, while the concentration of Na+ in the ventricle will decrease.

When ouabain, a drug that blocks Na+/K+ ATPase, is given in the ventricle, all of the following cellular responses are expected EXCEPT: a. Halts CSF formation b. Inward Na+ ion gradient across the choroidal basolateral membrane is blocked. c. Increases extracellular [K+] d. Increases extracellular [Na+] e. Decreases transepithelial water movement f. Reduced activity of the sodium-calcium exchanger (NCX) g. Increase in intracellular calcium

e. Competition for a carrier protein The finding that the flux of leucine is altered by changes in the flux of alanine suggests that the two amino acids are utilizing a similar carrier protein. As the concentration of alanine increases, more alanine is bound to the carrier protein allowing for more alanine to enter the cell. Alanine occupies a position on the carrier protein that would otherwise be occupied by leucine and as a result, leucine flux decreases.

When the concentration of extracellular alanine increases and the flux of alanine into a cell increases, the flux of leucine into the cell is found to decrease. How might this be explained? a. Increased activity of the Na/K ATPase b. Diffusion c. Restricted diffusion d. Decreased activity of the Na/Ca exchanger e. Competition for a carrier protein f. Increased activity of the alanine protein pump g. Decreased activity of the H+/amino acid exchanger

c. They remain open for only a short period of time after the cell is sufficiently depolarized Na+ channels normally reside in a closed conformation at the resting membrane potential and their opening is a transient process determined by the kinetics of channel activation and inactivation.

Which if the following is true about voltage-gated Na+channels in the axon? a. They remain open as long as the membrane is sufficiently hyperpolarized b. They remain open as long as the membrane is sufficiently depolarized c. They remain open for only a short period of time after the cell is sufficiently depolarized d. They belong to a family of channels that are made of five subunits e. They have about the same relative permeability ratio (PNa/PK) as ACh channels

d. The amount of sodium entering the nerve with each action potential increases In order for propagation of an action potential to occur, the depolarization produced by one action potential must depolarize the adjacent patch of excitable membrane to the threshold level. In demyelinating diseases, such as multiple sclerosis, too much charge leaks from the membrane and as a result, not enough charge is available to bring the next patch of membrane to threshold. Increasing the duration of the action potential increases the amount of charge entering the cell and therefore increases the probability that the next patch of excitable membrane will be depolarized to threshold. Increasing the duration of the refractory period will not affect the amount of charge entering the cell. Depolarizing the membrane and increasing potassium conductance will make it more difficult to produce an action potential. If membrane capacitance is increased, the amount of charge required to excite the next patch of membrane will be increased.

Which of the following best explains why increasing the duration of the action potential can restore nerve conduction in patients with Multiple Sclerosis? a. The capacitance of the nerve fiber membrane is increased b. The duration of the refractory period is increased c. The potassium conductance of the membrane is increased d. The amount of sodium entering the nerve with each action potential increases e. The membrane potential becomes more positive

d. A marked decrease in pulmonary diffusing capacity Cranial blood flow is maintained within normal limits by arterial CO2 levels. A significant decrease in pulmonary diffusing capacity would lead to increased arterial blood CO2 levels, thereby producing hypoxic-ischemic injury.

Which of the following causes of brain hypoxia would most strongly stimulate the aortic and carotid chemoreceptors? a. Carbon monoxide poisoning b. Severe anemia c. Formation of methemoglobin d. A marked decrease in pulmonary diffusing capacity e. Acute respiratory alkalosis f. Surfactant hypersecretion

b. Increased plasma parathyroid hormone concentration Increased levels of parathyroid hormone stimulate calcium reabsorption from the TAL and distal tubules. Extracellular fluid volume expansion, increased blood pressure, decreased plasma phosphate concentrations, and metabolic alkalosis are all associated with decreased calcium reabsorption by the tubule.

Which of the following changes would tend to increase Ca2+ reabsorption in the renal tubule? a. Extracellular fluid expansion b. Increased plasma parathyroid hormone concentration c. Increased blood pressure d. Decreased plasma phosphate concentration e. Metabolic alkalosis f. Intracellular fluid loss

E. E A 50% reduction in renal efferent arteriolar resistance would reduce glomerular capillary hydrostatic pressure (upstream from the efferent arterioles), and therefore reduced GFR, while increasing hydrostatic pressure in the peritubular capillaries (downstream from the efferent arterioles) and increasing renal blood flow.

Which of the following changes would you expect to find after acute administration of a vasodilator drug that caused a 50% decrease in renal efferent arteriolar resistance and no change in afferent arteriolar resistance or arterial pressure? A. A B. B C. C D. D E. E F. F G. G

c. The conduction velocity of its action potential The conduction velocity of an action potential along an axon is proportional to the axon's diameter for both unmyelinated and myelinated axons. The resting membrane potential, the duration of the relative refractory period, and the magnitude of the action potential are dependent on the type and density of electrically excitable gates and the ability of the Na+,K+-ATPase to establish and maintain the concentration gradients. These characteristics are not related in any systematic way to the axon diameter.

Which of the following characteristics of an axon is most dependent on its diameter? a. The magnitude of its resting potential b. The duration of its refractory period c. The conduction velocity of its action potential d. The overshoot of its action potential e. The activity of its sodium-potassium pump

f. Increased oxygenation of the ductus arteriosus Immediately after birth, the ductus arteriosus remains open but blood flow, which follows the path of least resistance, now begins to shunt (left to right) from the descending aorta to the pulmonary circulation because of the two events that change the relationship between pulmonary and systemic vascular resistance: (1) increased systemic resistance because of the removal of the placenta, and (2) decreased pulmonary resistance because of the expansion of the lungs. Within a few hours after term birth, the ductus arteriosus closes functionally because its muscular wall constricts (see Fig. 57-5ALinks to an external site.). Usually, all blood flow through the ductus arteriosus ceases within 1 week after birth. The relatively rapid functional closure of the ductus arteriosus after birth is primarily the result of an increase of the arterial blood (caused by breathing) that perfuses this vessel, a decrease in circulating PGE2 (caused by placenta removal and increased pulmonary PGE2 uptake), and a decrease in PGE2 receptors in the ductus wall. As the of blood flowing through the ductus arteriosus rises from 18 to 22 mm Hg in utero to ~60 mm Hg a few hours after birth, the smooth muscle in the wall of the ductus arteriosus contracts, which reduces or eliminates flow through the ductus. This contraction also constricts the vasa vasorum of the ductus, contributing to smooth-muscle apoptosis in the vessel wall. The foramen ovale closes over several months of neonate development.

Which of the following circulatory effects occurs shortly after a newborn takes his or her first breath? a. Increased pulmonary vascular resistance b. Secretion of surfactant c. Decreased aortic vascular resistance d. Prostaglandin secretion in the ductus arteriosus e. Anatomic closure of the foramen ovale f. Increased oxygenation of the ductus arteriosus g. Increased patency of the ductus venosus

c. C In the absence of ADH secretion, there is a marked increase in urine volume because the late distal and collecting tubules are relatively impermeable to water. As a result of increased urine volume, there is dehydration and increased plasma osmolarity and high plasma sodium concentration. The resulting decrease in extracellular fluid volume stimulates renin secretion, resulting in an increase in plasma renin concentration.

Which of the following would be expected in a patient with diabetes insipidus due to a lack of antidiuretic hormone secretion? a. A b. B c. C d. D e. E f. F

e. Increased potassium permeability An increase in potassium permeability causes a decrease in the membrane potential of the AV node leading to extreme hyperpolarization making it more difficult for the membrane potential to reach threshold. The result is a decrease in heart rate. An increase in sodium and calcium permeability and norepinephrine increase the membrane potential causing a tendency to increase the heart rate.

Which of the following conditions at the A-V node will cause a decrease in heart rate? a. Increased sodium permeability b. Decreased acetylcholine levels c. Increased norepinephrine levels d. Increased norepinephrine levels e. Increased potassium permeability f. Increased calcium permeability

d. Hyponatremia The upstroke of the action potential is caused by an inward flow of sodium ions, and therefore its magnitude depends on the extracellular sodium concentration. Decreasing the external Na+ concentration decreases the size of the action potential, but has little effect on the resting membrane potential because the permeability of the membrane to Na+ at rest is low. Conversely, increasing the external K+ concentration decreases the resting membrane potential. Changes in external Ca2+ concentration affect the excitability of nerve and muscle cells, but not the magnitude of the resting potential or the action potential.

Which of the following conditions will produce a decrease in the magnitude of a nerve membrane action potential? a. Hyperkalemia b. Hypokalemia c. Hypernatremia d. Hyponatremia e. Hypocalcemia

c. Tricuspid valve stenosis Mitral regurgitation and aortic stenosis are murmurs heard during the systolic period. A ventricular septal defect murmur is normally heard only during the systolic phase. Tricuspid valve stenosis and patent ductus arteriosus murmurs are heard during diastole. However, a patent ductus arteriosus murmur is heard during systole as well.

Which of the following heart murmurs is only heard during diastole? a. Patent ductus arteriosus b. Mitral regurgitation c. Tricuspid valve stenosis d. Interventricular septal defect e. Aortic stenosis

d. Anatomical dead space Expanding the lungs expands the airways, which increases their volume and, therefore, the volume of the anatomical deadspace. Expanding the lungs lowers the intrathoracic pressure and the intrapleural pressure, which causes the air- ways to expand. Expanding the airways also causes a decrease in airway resistance. Lung compliance is decreased, that is, the lungs are more difficult to inflate (stiffer), at high lung volumes. Alveolar pressure is atmospheric at residual volume and at total lung capacity.

Which of the following is higher at total lung capacity than it is at residual volume? a. Airway resistance b. Lung compliance c. Alveolar pressure d. Anatomical dead space e. Intrapleural pressure

d. Transference for potassium During an action potential, the conductances for sodium and for potassium are higher than at rest. However, the conductance for sodium is higher than the conductance for potassium during the overshoot; as a result, the transference for potassium is less. Recall that transference is a measure of an ion's relative conductance: TNa = gNa/(gNa + gK) and TK = gK/(gNa + gK) where T = transference and g = conductance.

Which of the following is less during the overshoot of an action potential than during the resting state? a. Membrane conductance for sodium b. Membrane conductance for potassium c. Transference for sodium d. Transference for potassium e. Total membrane conductance

c. Efficiency of ventricular pumping is decreased 20-30 percent Atrial fibrillation occurs often with patients with an atrial enlargement. This causes an increased tendency for electrical circus movements to occur. The ventricular beat is irregular because impulses are rapidly arriving at the AV node; however, many times the AV node is in a refractory period. Therefore, the AV node will not pass a second impulse until about 0.35 second elapses after the previous one. There is also a variable interval between when the atrial impulses reach the AV node. This results in a very irregular heartbeat, but one that is very rapid with a rate of 125150 bpm.

Which of the following is most characteristic of atrial fibrillation? a. Occurs less frequently in patients with atrial enlargement b. Ventricular heart rate is about 40 beats per minute c. Efficiency of ventricular pumping is decreased 20-30 percent d. Ventricular beat is regular e. Atrial P wave is easily seen. f. Electrical impulses pass directly from the SA node to the AV node

a. Entire heart is depolarized At the J point, the entire heart is depolarized in a patient with a damaged cardiac muscle or a patient with a normal cardiac muscle. The area of the heart that is damaged will not repolarize, but remains depolarized at all times.

Which of the following is most likely at the "J point" in an EKG of a patient with a damaged cardiac muscle? a. Entire heart is depolarized b. All the heart is depolarized except for the damaged cardiac muscle c. About half the heart is depolarized d. All of the heart is repolarized e. All of the heart is repolarized except for the damaged cardiac muscle f. About half the heart is repolarized

f. Dilation of the afferent arterioles The GFR is proportional to the glomerular capillary hydrostatic pressure, the renal plasma flow, and the surface area and hydraulic conductivity of the diffusion barrier between the glomerular capillary and Bowman's space. Dilating the afferent arteriole causes an increase in glomerular capillary pres- sure and, therefore, an increase in GFR. Volume depletion causes a release of renin from the juxtaglomerular cells, leading to an increase in angiotensin II (AII) that causes constriction of the glomerular capillaries and contraction of the mesangial cells. Contraction of the mesangial cells causes a decrease in the surface area of the diffusion barrier between the glomerular capillary and Bowman's space. Blockage of the ureter causes an increase in tubular pressure that retards the filtration of water from the capillary to the nephron.

Which of the following is most likely to cause an increase in the glomerular filtration rate? a. Contraction of mesangial cells b. Blockage of the ureter c. Release of renin from the juxtaglomerular apparatus d. Dilation of the efferent arterioles e. Volume depletion f. Dilation of the afferent arterioles

g. Carnitine In TAG metabolism, after the breakdown to FAs, the second step is the β-oxidation of FAs, which takes place in the mitochondrial matrix. To deliver acyl CoA to the mitochondrial matrix, the cell uses carnitine acyltransferase I (CAT I) on the cytosolic side of the mitochondrial outer membrane to transfer the acyl group to carnitine. The resulting acylcarnitine moves through a porin in the mitochondrial outer membrane to enter the intermembrane space.

Which of the following is required to transport fatty acids across the inner mitochondrial membrane? a. Acyl carrier protein b. Albumin c. Acetyl CoA d. Chylomicrons e. Creatinine f. Lecithin-cholesterol acyltransferase g. Carnitine

f. Placenta Early in the first trimester, hCG that is manufactured by the syncytiotrophoblast rescues the corpus luteum, which is the major source of progesterone and estrogens. This function of the corpus luteum in the ovary continues well into early pregnancy. However, by itself, the corpus luteum is not adequate to generate the very high steroid levels that are characteristic of late pregnancy. The developing placenta itself augments its production of progesterone and estrogens, so by 8 weeks of gestation the placenta has become the major source of these steroids—the lutealplacental shift. The placenta continues to produce large quantities of estrogens, progestins, and other hormones throughout gestation.

Which of the following is the source of estrogen and progesterone during the last 7 months of pregnancy? a. Maternal anterior pituitary b. Corpus luteum c. Ovary d. Fetal anterior pituitary e. Posterior pituitary f. Placenta

c. it is used as a source of energy for the transport of other ions The sodium-potassium pump uses the energy contained in ATP to maintain the sodium gradient across the membrane. The sodium gradient, in turn, is used to transport other substances across the membrane. For example, the Na/Ca exchanger uses the energy in the sodium gradient to help maintain the low intracellular calcium required for normal cell function. Although sodium enters the cell during an action potential, the quantity of sodium is so small that no significant change in intracellular sodium concentration occurs. Because the sodium transference is so low, the sodium equilibrium potential is not an important determinant of the resting membrane potential.

Which of the following is true regarding the sodium gradient across the nerve cell membrane? a. It is a result of the Donnan equilibrium b. It is significantly changed during an action potential c. it is used as a source of energy for the transport of other ions d. It is an important determinant of the resting membrane potential e. It is maintained by a Na+/Ca2+exchanger

c. C Trace C shows a basal subatmospheric pressure with a positive pressure wave caused the passage of the food bolus. Trace A does not correspond to any normal event in the esophagus. Trace B could represent the LES in a patient with achalasia. Trace D depicts normal operation of the LES. Trace E shows a basal positive pressure trace, which does not occur where the esophagus passes through the chest cavity.

Which of the following manometric recordings illustrates normal function of the esophagus at mid-thoracic level before and after swallowing (indicated by the arrow)? The dashed lines represent a pressure of 0 mm Hg. a. A b. B c. C d. D e. E

c. T tubule When the sarcolemma of the myocyte depolarizes, the depolarization travels down the T tubule of the cell into its interior. This depolarization causes the release of calcium from the sarcoplasmic reticulum. Calcium associates with troponin C which induces a conformational change in tropomyosin. The active site of actin is then free to associate with the myosin head allowing excitation-contraction coupling to occur.

Which of the following muscle cell components helps spread the depolarization of the muscle cell membranes throughout the interior of muscle cells? a. Actin b. Myosin c. T tubule d. Tropomyosin e. Troponin f. Z disk

a. The Ib afferents from the Golgi tendon organ increase their rate of firing The Ib afferents innervating the quadriceps muscles are activated when the quadriceps contracted in response to tapping the patella tendon. Stretching the patella tendon stretches the intrafusal muscle fibers within the quadriceps muscle and causes an increase in Ia afferent activity. The increase in Ia afferent activity causes an increase in a motoneuron activity, which results in contraction of the quadriceps muscle. When the muscle contracts, the intrafusal muscle fibers are unloaded and the Ia afferent activity is reduced.

Which of the following occurs during the contraction of the quadriceps muscle? a. The Ib afferents from the Golgi tendon organ increase their rate of firing b. The Ia afferents from the muscle spindle increase their rate of firing c. The alpha motoneurons innervating the extrafusal muscle fibers decrease their rate of firing d. The gamma motoneurons innervating the intrafusal muscle fibers increase their rate of firing e. The alpha motoneurons to the antagonistic muscles increase their rate of firing

a. Blockade of voltage-gated Na+ channels In the absence of hyperpolarization, the absence of an excitatory stimulus is most likely due to the blockade of the voltage-gated channels responsible for the generation of the all-or-none depolarization. In nerve cells, tehse are the voltage-gated Na+ channels.

Which of the following perturbations of the nerve cell would account for the failure of the same stimulus to elicit an action potential in trace C when compared to trace A? a. Blockade of voltage-gated Na+ channels b. Blockade of voltage-gated K+ channels c. Blockade of Na-K leak channels d. Replacement of the voltage-gated K+ channels with "slow" Ca2+ channels e. Replacement of the voltage-gated Na+ channels with "slow" Ca2+ channels f. Hyperactivity of the Na+/K+-ATPase g. Complete inhibition of the Na+/K+-ATPase

c. Hypersecretion of vasopressin Hypersecretion of vasopressin (ADH) causes an increase in water reabsorption from the distal kidney tubule. An increase in the amount of free water reabsorbed dilutes the blood reducing the concentration of sodium in the blood. Though aldosterone does promote some water reabsorption, its main action is to increase sodium reabsorption from the kidney tubule. Angiotensin II is a powerful vasoconstrictor, but does not alter free water reabsorption. Renin is released to hypotension and reduced effective circulating volume. It promotes the conversion of angiotensinogen to ANG I, the inactive form of ANG II.

Which of the following physiological abnormalities would you expect to cause dilutional hyponatremia? a. Hyposecretion of aldosterone b. Inability to form angiotensin II c. Hypersecretion of vasopressin d. Hypersecretion of renin e. A and B only f. All of the above g. None of the above

a. A As barometric pressure decreases with ascent to high altitude, the partial pressure of inspired oxygen decreases below normal (21% of lower barometric pressure). The resultant decrease in alveolar PO2 leads to a decrease in arterial PO2, a condition classified as hypoxic hypoxia or hypoxemia. Hypoxemia stimulates the peripheral chemoreceptors to increase ventilation, causing arterial PCO2 to decrease and arterial pH to rise (respiratory alkalosis). Renal compensation for the respiratory alkalosis is a decrease in renal H+ excretion, which lowers plasma bicarbonate concentration.

Which of the following points on the graph below represents the blood gas values obtained from a patient who has ascended to high altitude? a. A b. B c. C d. D e. E

c. 1 L of 3% sodium chloride solution A 3% sodium chloride solution is hypertonic and when infused intravenously would increase extracellular fluid volume and osmolarity, thereby causing water to flow out of the cell. This would decrease intracellular fluid volume and further increase extracellular fluid volume. The 0.9% NaCl solution and 5% dextrose solution are isotonic, and therefore would not reduce intracellular fluid volume. Pure water and the 0.45% NaCl solution are hypotonic and when infused would increase both intracellular and extracellular fluid volumes.

Which of the following solutions, when infused intravenously would result in an increase in extracellular fluid volume, a decrease in intracellular fluid volume, and an increase in total body water after osmotic equilibrium? a. 1 L of 0.9% sodium chloride solution b. 1 L of 0.45% sodium chloride solution c. 1 L of 3% sodium chloride solution d. 1 L of 0.9% dextrose solution e. 1 L of 5% dextrose solution f. 1 L of distilled water g. 1 L of pure water

d. It converts angiotensinogen to angiotensin I Renin is secreted by the cells of the JGA in response to a drop in the MAP of the renal artery. Its secretion stimulates the conversion of angiotensinogen to ANG I. ANG I is converted to ANG II by ACE. The result is vasoconstriction of the efferent arteriole and increased sodium and water reabsorption from the proximal tubule.

Which of the following statements about renin is true? a. It is secreted by cells of the proximal tubule b. Its secretion leads to a loss of sodium and water from plasma c. Its secretion is stimulated by increased mean renal arterial pressure d. It converts angiotensinogen to angiotensin I e. It converts angiotensin I to angiotensin II f. It inhibits the activity of the juxtaglomerular apparatus

a. It is regulated by the activity of lipoprotein lipase Triglyceride uptake into adipocytes is regulated by lipoprotein lipase. The uptake of triglycerides into adipose tissue and other tissues from plasma lipoproteins requires hydrolysis of triglyceride to fatty acids and glycerol by an enzyme bound to the endothelial surface, lipoprotein lipase. The activity of this enzyme varies in reciprocal fashion with that of cytoplasmic hormone-sensitive lipase; that is, its activity is enhanced by insulin and glucose and decreased by catecholamines. Lipoprotein lipase is present in nearly every tissue and acts at the capillary surface as it does in adipose tissue. Receptor-mediated endocytosis is important in the turnover of the protein portion of plasma lipoproteins.

Which of the following statements about the uptake of triglycerides into adipose tissue from plasma lipoproteins is correct? a. It is regulated by the activity of lipoprotein lipase b. It is increased by catecholamines c. It is decreased by glucose d. It is decreased by insulin e. It requires receptor-mediated endocytosis f. It is regulated by the activity of the gallbladder

d. The majority of absorption occurs in the jejunum The bulk of water and electrolyte absorption occurs in the jejunum. This is because the transepithelial permeability of the jejunum is considerably greater than that of the ileum or colon, as evidenced by its lower spontaneous transepithelial voltage difference (Vte), higher passive movement of NaCl, and larger apparent pore size. The small intestine primarily absorbs Na+, Cl-, and K+.

Which of the following statements best describes water and electrolyte absorption in the GI tract? a. Most water and electrolytes come from ingested fluids b. The small intestine and colon have similar absorptive capacities c. Osmotic equilibrium of chyme occurs in the stomach d. The majority of absorption occurs in the jejunum e. The majority of absorption occurs in the large intestine f. Most of the dietary fluid ingested is reabsorbed in the duodenum g. The small intestine primarily secretes Na+, Cl-, and K+

c. The ratio of Na+ to K+ channels in astrocytes is high in comparison to neurons, making them incapable of producing an action potential. Because the ratio of Na+ to K+channels is low in adult astrocytes, these cells are not capable of regenerative electrical responses such as the action potential.

Which of the following statements is incorrect? a. Astrocytes are predominantly permeable to K+ therefore they help to regulate extracellular K+. b. Inwardly rectifying K+ channels are important in setting the resting potential in glial cells. c. The ratio of Na+ to K+ channels in astrocytes is high in comparison to neurons, making them incapable of producing an action potential. d. Astrocytes diminish neuronal excitability via influx of HCO3- via an electrogenic Na+/HCO3 cotransporter which causes a fall in extracellular pH. e. Glial membranes have a resting potential more negative than neurons, which makes them more selective to K+.

a. Creatinine The proximal tubule reabsorbs approximately twothirds of the filtered water and twothirds of the filtered Na+, Cl−, and K+. Therefore, the concentration of these substances is the same at the beginning and end of the proximal tubule. Because creatinine is not reabsorbed, its concentration increases from the proximal to distal ends of the proximal tubule. Phosphate, however, is almost completely reabsorbed in the proximal tubule, so its concentration decreases along the length of the tubule.

Which of the following substances is less concentrated at the end of the proximal tubule than at the beginning of the proximal tubule? a. Creatinine b. Hydrogen c. Chloride d. Phosphate e. Sodium

b. Mass action peristalsis The proximal colon has two types of motor activity, nonpropulsive segmentation and mass peristalsis. One to three times a day, a so-called mass peristalsis occurs in which a portion of the colonic contents is propelled distally 20 cm or more. Such mass peristaltic contractions are the primary form of propulsive motility in the colon and may be initiated by eating. During mass peristalsis, the haustra disappear; they reappear after the completion of mass peristalsis. The rectum itself is kept nearly empty by nonpropulsive segmentation until it is filled by mass peristalsis of the distal end of the colon.

Which of the following types of movements occurs in the colon and not in the small intestine? a. Segmental contractions b. Mass action peristalsis c. 5 cm/sec peristaltic waves d. 20 cm/sec peristaltic waves e. Peristaltic rushes f. Spincteric contraction g. Migrating motor complexes

e. Folic acid Folic acid is both produced in the intestine and absorbed from the intestine. In humans, several vitamins, including vitamin K, several of the B complex, and folic acid, are synthesized by intestinal bacteria. However, only folic acid is absorbed by the host. Dietary intake of the other vitamins is necessary.

Which of the following vitamins is both synthesized by intestinal bacteria and absorbed in significant quantities? a. Vitamin B6 b. Vitamin K c. Thiamine d. Riboflavin e. Folic acid f. Vitamin C

C. C In aplastic anemia, the patient has minimal or no red blood cell production. Hematocrit and hemoglobin would be low. EPO (erythropoietin) levels would be elevated to stimulate red blood cell production. Mean corpuscular volume (MCV) is the average volume of the red blood cells. Though there is reduced production of red blood cells, the cells themselves are normal.

Which of the following would describe the condition of a patient with aplastic anemia A. A B. B C. C D. D E. E F. F G. G

e. It is the only hypotonic gastrointestinal secretion The cells lining the salivary ducts are almost completely impermeable to water and water cannot diffuse from the ducts to blood along an osmotic gradient. Pancreatic amylase is essential for complete digestion of starch, but salivary secretion plays only a minor role in starch digestion. Salivary secretion is entirely controlled by the nervous system. There is no hormonal control of salivary secretion. The alkalinity of saliva helps prevent loss of calcium from teeth. Salivary secretion is alkaline because of its high bicarbonate content and this prevents acid damage to enamel as well as providing an optimum pH for salivary amylase activity.

Which one of the following statements is correct regarding salivary secretion? a. The secretion of saliva is essential for the complete digestion of starch b. It is primarily under hormonal control c. It does not prevent decalcification of the teeth d. It is an acidic solution that begins the digestion of protein in the mouth e. It is the only hypotonic gastrointestinal secretion

c. Epinephrine Epinephrine activates alpha and beta adrenergic receptors equally well. Norepinephrine excites both types of receptors, but has a markedly greater effect on alpha receptors.

Which substance activates adrenergic alpha and beta receptors equally well? a. Acetylcholine b. Norepinephrine c. Epinephrine d. Serotonin e. Dopamine f. Glutamate

b. Nicotinic Nicotinic cholinergic receptors are found at the synapses between preganglionic and postganglionic sympathetic neurons

Which type of cholinergic receptor is found at synapses between preganglionic and postganglionic neurons of the sympathetic nervous system? a. Muscarinic b. Nicotinic c. Alpha d. Beta1 e. Beta2 f. Adrenergic g. Gamma1

a. Urine with a positive reaction for conjugated bilirubin In obstructive jaundice, conjugated bilirubin imparts a dark yellow color to the urine. Measurement of free and conjugated bilirubin in serum serves as a sensitive test for detecting liver disease. In obstructive jaundice, no bilirubin reaches the intestine for conversion into urobilinogen, and therefore no urobilinogen appears in the blood for excretion by the kidney. As a result, tests for urobilinogen in urine are negative in obstructive jaundice. Because of the lack of stercobilin and other bile pigments in obstructive jaundice, the stool becomes clay colored.

While on a group expedition in tropical Africa, a 29-year-old woman becomes fatigued and jaundiced. Due to her isolated location, her medical condition is assessed by an analysis of urine and stool samples. Based on normal bilirubin metabolism, which of the following findings is most consistent with obstructive jaundice in this patient? a. Urine with a positive reaction for conjugated bilirubin b. Presence of occult blood in the feces c. White stools positive for stercobilin and bile pigment d. Urine with a positive reaction for indirect bilirubin e. Increased concentrations of urobilinogens in the urine and feces f. Presence of albumin in the urine g. Elevated levels of circulating coagulation factors and free heme

f. Sodium and glucose Sodium and glucose are crucial ingredients of hydrating solutions because glucose and sodium share the same cotransporter. Thus, one molecule of sodium cotransports one molecule of glucose. The cotransport of sodium and glucose sets up an osmotic gradient that stimulates rapid water absorption by solvent drag in the gut.

While on a rotation in India, you encounter a 10-year-old patient who is suffering cholera and is severely dehydrated. You are asked to prepare an oral rehydration solution. What, in addition to water, would the most important components of the solution be? a. Bicarbonate and potassium b. Chloride and potassium c. Lactate and potassium d. Sodium and bicarbonate e. Sodium and chloride f. Sodium and glucose

c. ACTH This patient is exhibiting some of the signs of Cushing's syndrome. The hormone most likely being secreted is ATCH which stimulates an excess of glucocorticoids leading to the above symptoms and insulin resistance.

You are presented with a 27yearold patient manifesting the following symptoms: polyuria, polydipsia, elevated urine osmolality, history of increased bone fragility, hyperpigmentation, easily bruises. On the basis of these observations, you requested the following laboratory tests: Thyroxine - Normal, Fasting insulin 50% higher than expected for normal individuals. The gland having the primary malfunction in this case is most likely secreting nonnormal levels of which hormone? a. Insulin b. ADH c. ACTH d. Thyroxine e. Cortisol f. Glucagon

a. Increased bone resorption Increased parathyroid hormone (PTH) levels associated with primary hyperparathyroidism stimulate osteoclastic activity in bone to increase bone resorption.

You determine that a 45-year-old female patient with kidney stones is suffering from primary hyperparathyroidism. What is the most likely reason for an increase in her plasma ionized calcium? a. Increased bone resorption b. Enhanced 24,25(OH)2-vitamin D3 production c. Increased kidney tubule reabsorption of phosphate d. Decreased osteoclast formation e. Decreased urinary phosphate f. Increased exposure to UV sunlight

a. Serum blood glucose The patient has acanthosis nigricans, a skin response to chronic insulin resistance and high blood sugars (such as occurs in type II diabetes mellitus). Darkening of the skin, particularly along skin folds and around the base and back of the neck, occurs after a prolonged period of significantly elevated glucose levels. In addition to this skin finding, the patient's obesity makes diabetes a more likely diagnosis. A simple fingerstick glucose check or serum glucose level could confirm insulin resistance; further workup may include fasting glucose, insulin levels, and hemoglobin A1c.

You see a 52yearold markedly obese man in clinic who complains of arthritic pain in the knees, but otherwise reports no significant health problems. On physical exam you notice a darkening of his skin along the back of his neck, at the folds of his axillary skin, and faintly in the groin area and behind his knees. He states that he has had these darker areas for 'quite a long time'. What laboratory test would be most helpful in confirming your suspicions of his metabolic disorder? a. Serum blood glucose b. Thyroid stimulating hormone (TSH) c. Parathyroid hormone d. Uric acid e. Urine vanillylmandelic acid (VMA) f. Serum aldosterone

e. Uniformly smooth prostatic enlargement A normal prostate should feel firm with a smooth surface to the examiner's gloved finger. The patient clearly has a urinary outlet obstruction, probably caused by benign prostatic hypertrophy (BPH). It is not uncommon for men to develop this condition as they age, but it should not be confused with malignant processes, particularly with normal PSA levels. BPH may be treated with antiandrogens such as finasteride, herbal remedies (such as saw palmetto), or with a surgical procedure (transurethral resection of prostate [TUR]). The condition may progress to complete outlet obstruction with urinary retention and kidney failure without treatment.

You see a 78yearold man in clinic for his annual physical. He reports excellent health. During your review of systems, he admits to having difficulty initiating his urine stream, nocturia, and occasionally feels as if he does not completely empty his bladder. Routine lab tests show a normal level of prostatespecific antigen (PSA). Analysis shows no hematuria or crystalluria. What related finding might you anticipate when you move on to his physical exam? a. Testicular atrophy b. Urinary calculi (stones) blocking the urethra c. Hypospadias d. Irregular prostatic nodularity e. Uniformly smooth prostatic enlargement f. Urethral stricture

b. Prolactin TRH is well known for stimulating prolactin release as well as TSH

You wish to determine the ability of thyrotropin-releasing hormone (TRH) to stimulate the release of pituitary hormones in a group of volunteer subjects. In addition to measuring thyroid stimulating hormone (TSH), which of the following hormones could you also measure? a. Growth hormone b. Prolactin c. Luteinizing hormone d. Adrenocorticotropic hormone (ACTH) e. Antidiuretic hormone (ADH) f. Parathyroid hormone

b. Partially compensated metabolic acidosis In metabolic acidosis, there is a decrease in plasma HCO3−, caused by reduced filtration of HCO3− ions due to a decrease in the extracellular concentration of bicarbonate. In addition, there is a rise in extracellular fluid (ECF) H+ ion concentration and hence a decrease in pH. Compensation mechanisms include an increased respiration rate, which reduces PCO2, and increased renal HCO3− reabsorption and H+ secretion, which help to minimize the initial fall in extracellular HCO3− concentration.

Your 15-year-old dizzy patient arrives at the emergency department after winning a breath holding contest. Her laboratory values are as follows: pH 7.10; HCO3−4 mEq/L; PCO217 mmHg. What is the acid-base status of this patient? a. Partially compensated metabolic alkalosis b. Partially compensated metabolic acidosis c. Partially compensated respiratory alkalosis d. Partially compensated respiratory acidosis e. Uncompensated respiratory alkalosis f. Fully compensated respiratory

b. 11β-hydroxylase deficiency A deficiency in 11β-hydroxylase prevents formation of cortisol, corticosterone, and aldosterone. Failure to hydroxylate deoxycorticosterone (DOC) results in an overproduction of DOC, causing a pattern of hypermineralocorticoidism with hypertension, increased extracellular volume, potassium depletion, sodium retention, and masculinization.

Your 21-year-old female patient presents with a history of secondary amenorrhea, acne, hyperpigmentation, clitoral enlargement, and physical and mental fatigue. Her blood pressure is 160/105 mmHg and laboratory values indicate hypokalemia, low fasting blood glucose, and hypernatremia. What, in the light of these findings, is the most likely cause of her symptoms? a. An adrenocorticotropic hormone (ACTH)-secreting tumor b. 11β-hydroxylase deficiency c. Absence of androgen receptors d. 21-hydroxylase deficiency e. Primary hypercortisolism f. A tumor in the anterior pituitary causing hypersecretion of FSH

a. Ventricular repolarization occurring from endocardium to epicardium Ventricular repolarization from epicardium to endocardium results in the T wave on the electrocardiogram. In individuals with an inverted T wave repolarization of the ventricle proceeds in the opposite direction from endocardium to epicardium. An inverted T wave is associated with the after effects of a myocardial infarction. The T wave reflects ventricular repolarization, not ventricular depolarization. Ventricular repolarization occurs from the epicardium to endocardium in normal individuals, not those that have an inverted T wave.

Your 49-year-old male patient is admitted to the heart hospital suffering the classic symptoms of myocardial ischemia. His electrocardiogram (ECG) at admission shows an elevated S-T segment with an inverted T wave. What is the most likely cause of the inverted T wave? a. Ventricular repolarization occurring from endocardium to epicardium b. Ventricular repolarization occurring from epicardium to endocardium c. Ventricular depolarization occurring from epicardium to endocardium d. Ventricular depolarization occurring from endocardium to epicardium e. An abnormal delay in atrioventricular (AV) nodal conduction f. Elevated conduction through atrioventricular node

e. Decreased heart rate and contractility of the myocardium β-blockers reduce the rate and force of myocardial contraction

Your 54-year-old female patient is suffering from myocardial ischemia. One drug that you suggest to improve her cardiac function is a β-blocker. What would be the myocardial effect of a β-blocker? a. Dilation of peripheral blood vessels, particularly the veins b. Prevention of pulmonary conversion of angiotensin I to angiotensin II c. Increased L-type calcium channel activity in ventricular muscle d. Increase in the dP/dt during ventricular isovolumetric contraction e. Decreased heart rate and contractility of the myocardium f. Increase tensile strength of the myocardial fibers

b. Sarcoplasmic reticulum Ca2+ release channel In malignant hyperthermia there is a mutation of the gene coding for the ryanodine receptor, which is the Ca++ channel in the sarcoplasmic reticulum. In malignant hyperthermia, there is an excess Ca2+ release during muscle contraction leading to increased muscle metabolism and heat production, which is fatal if not treated.

Your patient has been diagnosed with malignant hyperthermia after a surgical procedure. The most common cause of malignant hyperthermia in humans is an abnormal response to general anesthetics involving the: a. Na+-Ca2+ exchanger b. Sarcoplasmic reticulum Ca2+ release channel c. IP3 receptor d. Na+K+-ATPase e. Central nervous system (CNS) f. Supraoptic nucleus


संबंधित स्टडी सेट्स

Week 7 Check Your Understanding Chapters 36, 37, 38, 40, and 41

View Set

Social Media Marketing Chapters 1 - 10

View Set

Missouri Compromise, Compromise of 1850 and The Fugitive Slave Law

View Set